221 CS Corrige 332p1 [PDF]

  • 0 0 0
  • Gefällt Ihnen dieses papier und der download? Sie können Ihre eigene PDF-Datei in wenigen Minuten kostenlos online veröffentlichen! Anmelden
Datei wird geladen, bitte warten...
Zitiervorschau

Chimie des solutions 3e édition

CORRIGÉ DÉTAILLÉ Thomas J. Hummel Susan A. Zumdahl Steven S. Zumdahl

8101, boul. Métropolitain Est, Anjou (Québec) Canada H1J 1J9 Téléphone : 514-351-6010 • Télécopieur : 514- 351-3534

Chimie des solutions, 3e édition Corrigé détaillé

Direction de l’édition Services d’édition Danielle Guy Charge de projet Sébastien Grandmont Consultation et adaptation Ghislin Chabot Jean-Luc Riendeau Traduction Jean-Luc Riendeau

La Loi sur le droit d’auteur interdit la reproduction d’œuvres sans l’autorisation des titulaires des droits. Or, la photocopie non autorisée — le photocopillage — a pris une ampleur telle que l’édition d’oeuvres nouvelles est mise en péril. Nous rappelons donc que toute reproduction, partielle ou totale, du présent ouvrage est interdite sans l’autorisation écrite de l’Éditeur. Gouvernement du Québec – Programme de crédit d’impôt pour l’édition de livres Gestion SODEC. Les Éditions CEC remercient le gouvernement du Québec pour l’aide financière accordée à l’édition de cet ouvrage par l’entremise du Programme de crédit d’impôt pour l’édition de livres, administré par la SODEC. Chimie des solutions, 3e édition Corrigé détaillé © 2007 Les Éditions CEC inc. 8101, boul. Métropolitain Est Anjou (Québec) H1J 1J9 Traduction de: Complete Solutions Guide, Chemistry 7th edition, Steven S. Zumdahl et Susan A. Zumdahl Copyright: © 2007 by Houghton Mifflin Company All rights reserved Tous droits réservés. Il est interdit de reproduire, d’adapter ou de traduire l’ensemble ou toute partie de cet ouvrage sans l’autorisation écrite du propriétaire du copyright. Dépôt légal : 2007 Bibliothèque et Archives nationales du Québec Bibliothèque et Archives Canada ISBN 978-2-7617-2534-7 Imprimé au Canada 1 2 3 4 5 11 10 09 08 07

PRÉSENTATION Cet ouvrage est le corrigé complet, détaillé et annoté du manuel de S. Zumdahl Chimie des solutions, 3e édition. Ce corrigé comprend tous les tableaux, les équations, les graphiques et les illustrations nécessaires à la résolution intégrale de tous les exercices de fin de chapitre du manuel: les exercices numérotés en bleu et dont seule la réponse est fournie à la fin du manuel ainsi que les exercices numérotés en noir pour lesquels aucune réponse n’est fournie dans le manuel. Seules les Questions de révision et les Questions à discuter en classe ont été omises, car elles visent soit à servir d’introduction au chapitre, soit à permettre une synthèse. Un soin particulier a été apporté dans ce corrigé pour conserver à la fois la démarche pédagogique et les niveaux croissants de difficulté des exercices, ainsi que la nomenclature et les notations mathématiques de l’ouvrage. De plus, nous avons toujours tenté de présenter les démonstrations et méthodes de résolution les plus simples et les plus claires pour l’étudiant(e) tout en demeurant conscient que parfois d’autres méthodes de résolution étaient aussi possibles. Vous trouverez en annexe la liste de l’ensemble des tableaux du manuel ainsi qu’une liste non exhaustive des sites traitant de chimie sur internet. Ce corrigé s’adresse avant tout aux professeurs. Les Éditions CEC inc. en effectuent la diffusion et la vente en en gardant confidentiel le contenu. Il est cependant possible, en s’adressant à l’éditeur, d’obtenir la permission écrite de reproduire pour les étudiant(e)s le contenu de ce corrigé. Nous espérons que ce corrigé favorisera l’apprentissage et facilitera l’enseignement de la chimie dans vos salles de cours.

TABLE DES MATIÈRES

Chapitre 1

Types de réactions chimiques et stœchiométrie en solution

1

Chapitre 2

Propriétés des solutions

35

Chapitre 3

Cinétique chimique

71

Chapitre 4

Équilibre chimique

103

Chapitre 5

Acides et bases I

135

Chapitre 6

Applications de l’équilibre en milieux aqueux

189

Chapitre 7

Thermodynamique chimique

253

Chapitre 8

Électrochimie

283

Liste des tableaux du manuel

325

La chimie sur Internet

326

CHAPITRE 1 TYPES DE RÉACTIONS CHIMIQUES ET STŒCHIOMÉTRIE EN SOLUTION QUESTIONS 6.

Une mole de NaOH dissoute dans 1,00 L de solution donne NaOH 1,00 mol/L. Premièrement, peser 40,00 g de NaOH (1,000 mol). Ajouter ensuite un peu d’eau dans une fiole jaugée de 1 L (un instrument qui est précis à 1,000 L). Dissoudre le NaOH dans la fiole, ajouter un peu plus d’eau, mélanger, ajouter encore plus d’eau, mélanger, etc., jusqu’à ce que l’eau ajoutée atteigne la marque de jauge de la fiole volumétrique. Le résultat est 1,000 L d’une solution de NaOH 1,000 mol/L. Étant donné que l’on connaît le volume, de même que la masse, à quatre chiffres significatifs, la concentration molaire volumique sera connue à quatre chiffres significatifs. En pratique, si on veut une concentration molaire volumique à trois chiffres significatifs, nos mesures devraient être prises à quatre chiffres significatifs. Quand on a besoin de diluer une solution concentrée avec de l’eau pour préparer une solution, il faut faire toutes les mesures à quatre chiffres significatifs afin de s’assurer que la concentration molaire volumique soit à trois chiffres significatifs. Dans le cas présent, il faut couper la concentration molaire volumique en deux à partir de 2,00 mol/L à 1,00 mol/L. On commencerait avec une mole de NaOH de la solution concentrée, c’est-à-dire 500,0 mL de NaOH 2,00 mol/L. On ajoute ce volume dans une fiole jaugée de 1 L en ajoutant de l’eau et en mélangeant jusqu’à ce qu’on atteigne la marque de 1,000 L. La solution finale serait à 1,00 mol/L.

7.

Bromures solubles : NaBr, KBr et NH4Br (et d’autres) ; bromures insolubles : AgBr, PbBr2 et Hg2Br2 ; sulfates solubles : Na2SO4, K2SO4 et (NH4)2SO4 (et d’autres) ; sulfates insolubles : BaSO4, CaSO4 et PbSO4 ; hydroxydes solubles : NaOH, KOH et Ca(OH)2 ; hydroxydes insolubles : Al(OH)3, Fe(OH)3 et Cu(OH)2 ; phosphates solubles : Na3PO4, K3PO4 et (NH4)3PO4 ; phosphates insolubles : Ag3PO4, Ca3(PO4)2 et FePO4 ; plomb (insolubles) : PbCl2, PbBr2, PbI2, Pb(OH)2, PbSO4 et PbS (et d’autres) ; (soluble) : Pb(NO3). On utilise les règles de solubilité du tableau 1.1. Quelques bromures solubles selon la règle 2 sont NaBr, KBr et NH4Br (il y en a d’autres). Les bromures insolubles selon la règle 3 sont AgBr, PbBr2 et Hg2Br2. On utilise un raisonnement semblable pour les autres parties du problème. Sulfates : Na2SO4, K2SO4 et (NH4)2SO4 (et d’autres) sont solubles et BaSO4, CaSO4 et PbSO4 (ou Hg2SO4) sont insolubles. Hydroxydes : NaOH, KOH, Ca(OH)2 (et d’autres) sont solubles et Al(OH)3, Fe(OH)3, et Cu(OH)2 (et d’autres) sont insolubles. Phosphates : Na3PO4, K3PO4, (NH4)3PO4 (et d’autres) sont solubles et Ag3PO4, Ca3(PO4)2 et FePO4 (et d’autres) sont insolubles.

Chapitre 1 Types de réactions chimiques et stœchiométrie en solution

2

Plomb : PbCl2, PbBr2, PbI2, Pb(OH)2, PbSO4 et PbS (et d’autres) sont insolubles. Pb(NO3)2 est un sel de Pb2+ soluble. 8.

Pour nos besoins, la définition de Brønsted-Lowry est préférable. Un acide est un donneur de proton et une base est un accepteur de proton. Un proton est un ion H+. L’hydrogène neutre possède 1 électron et 1 proton de sorte qu’un ion H+ est tout simplement un proton. Une réaction acide-base est le transfert d’un ion H+ (un proton) d’un acide à une base.

9.

a) L’espèce réduite est l’élément qui gagne des électrons. L’agent réducteur permet à la réduction de se produire en étant lui-même oxydé. L’agent réducteur réfère généralement à la formule entière du composé ou de l’ion qui contient l’élément oxydé ; b) L’espèce oxydée est l’élément qui perd des électrons. L’agent oxydant permet à l’oxydation de se produire en étant lui-même réduit. L’agent oxydant réfère généralement à la formule entière du composé ou de l’ion qui contient l’élément réduit ; c) Pour les composés ioniques binaires simples, la charge réelle portée par les ions est l’état d’oxydation. Pour les composés covalents, les états d’oxydation différents de zéro sont des charges imaginaires que les éléments auraient s’ils étaient unis par des liaisons ioniques (en supposant que la liaison se trouve entre deux non-métaux différents). Les états d’oxydation différents de zéro pour les éléments dans des composés covalents ne sont pas des charges réelles.

EXERCICES Solutions aqueuses : électrolytes forts et électrolytes faibles 10.

a) NaBr(s) → Na+(aq) + Br–(aq)

b) MgCl2(s) → Mg2+(aq) + 2Cl–(aq)

c) Al(NO3)3(s) → Al3+(aq) + 3 NO3–(aq)

d) (NH4)2SO4(s) → 2NH4+(aq) + SO42–(aq)

e) HI(g) → H+(aq) + I–(aq)

f) FeSO4(s) → Fe2+(aq) + SO42–(aq)

g) KMnO4(s) → K+(aq) + MnO4–(aq)

h) HClO4(s) → H+(aq) + ClO4–(aq)

i) NH4CH3CO2(s) → NH4+(aq) + CH3CO2–(aq) 11.

MgSO4(s) → Mg2+(aq) + SO42-(aq) ; NH4NO3(s) → NH4+(aq) + NO3-(aq).

Concentration molaire volumique 12.

a) 5,623 g NaHCO3 ×

c=

1 mol NaHCO3 = 6,693 × 10–2 mol NaHCO3 84,01 g NaHCO3

6,693 × 10_ 2 mol 1000 mL × = 0,2677 mol NaHCO3/L 250,0 mL L

© 2007 Les Éditions CEC inc. Toute reproduction interdite

Chapitre 1 Types de réactions chimiques et stœchiométrie en solution

b) 0,1846 g K2Cr2O7 ×

c=

13.

1 mol K 2 Cr 2 O7 = 6,275 × 10–4 mol K2Cr2O7 294,20 g K 2 Cr 2 O7

6,275 × 10_ 4 mol = 1,255 × 10–3 mol K2Cr2O7/L _3 500,0 × 10 L

c) 0,1025 g Cu ×

c=

3

1 mol Cu = 1,613 × 10–3 mol Cu = 1,613 × 10–3 mol Cu2+ 63,55 g Cu

1,613 × 10_ 3 mol Cu 2+ 1000 mL × = 8,065 × 10–3 mol Cu2+/L 200,0 mL L

a) CaCl2(s) → Ca2+(aq) + 2Cl–(aq) :

cCa2+ = 0,15 mol/L : cCl_ = 2(0,15) = 0,30 mol/L b) Al(NO3)3(s) → Al3+(aq) + 3NO3–(aq) :

c Al3+ = 0,26 mol/L : c NO3_ = 3(0,26) = 0,78 mol/L c) K2Cr2O7(s) → 2K+(aq) + Cr2O72–(aq) :

c K+ = 2(0,25) = 0,50 mol/L : cCr2O72 _ = 0,25 mol/L d) Al2(SO4)3(s) → 2Al3+(aq) + 3SO42–(aq)

c Al

3+

=

2, 0 × 10−3 mol Al 2(SO 4) 3 2 mol Al3+ × = 4,0 × 10–3 mol/L L mol Al 2(SO 4)3 mol/L

2, 0 × 10−3 mol Al 2(SO 4) 3 3 mol SO 24 = 6,0 × 10–3 mol/L = × L mol Al 2(SO 4) 3 _

c 14.

_ SO 24

3+ – n solut C = Vc ; AlCl3(s) → Al (aq) + 3Cl (aq)

mol Cl– = 0,1000 L ×

0,30 mol AlCl3 3 mol Cl _ × = 9,0 × 10–2 mol Cl– mol AlCl3 L

MgCl2(s) → Mg2+(aq) + 2Cl–(aq)

© 2007 Les Éditions CEC inc. Toute reproduction interdite

Chapitre 1 Types de réactions chimiques et stœchiométrie en solution

4

mol Cl– = 0,0500 L ×

0,60 mol MgCl2 3 mol Cl _ × = 6,0 × 10–2 mol Cl– L mol MgCl2

NaCl(s) → Na+(aq) + Cl–(aq) mol Cl– = 0,2000 L ×

0,40 mol NaCl 1 mol Cl_ × = 8,0 × 10–2 mol Cl– L mol NaCl

100,0 mL de AlCl3 0,30 mol/L contiennent le plus grand nombre de moles de Cl–. 15.

Masse molaire de NaOH = 22,99 + 16,00 + 1,008 = 40,00 g/mol Masse NaOH = 0,2500 L ×

16.

10 g AgNO3 ×

17.

a) 2,00 L ×

0, 400 mol NaOH 40, 00 g NaOH × = 4,00 g NaOH L mol NaOH

1 mol AgNO3 1L = 0,235 L = 235 mL × 169,9 g 0,25 mol AgNO3

0,250 mol NaOH 40,00 g NaOH × = 20,0 g NaOH L mol NaOH

Placez 20,0 g de NaOH dans un ballon jaugé de 2 L ; ajoutez de l’eau pour dissoudre le NaOH et remplissez le ballon avec de l’eau jusqu’à la marque de jauge, en mélangeant plusieurs fois au cours de la préparation ; b) 2,00 L ×

0,250 mol NaOH 1 L solution-mère × = 0,500 L L 1,00 mol NaOH

Placez 500,0 mL de la solution-mère de NaOH 1,99 mol/L dans un ballon jaugé de 2 L ; remplir le ballon avec de l’eau jusqu’à la marque de jauge, en mélangeant plusieurs fois au cours de la préparation ; c) 2,00 L ×

0,100 mol K 2 CrO4 194,20 g K 2 CrO 4 × = 38,8 g K2CrO4 L mol K 2 CrO 4

méthode semblable à la préparation de la solution en a, mais en utilisant 38,8 g de K2CrO4 ; d) 2,00 L ×

0,100 mol K 2 CrO 4 1 L solution-mère × = 0,114 L L 1,75 mol K 2 CrO 4

méthode semblable à la préparation de la solution en b, mais en utilisant 114 mL de la solution-mère de K2CrO4 1,75 mol/L.

© 2007 Les Éditions CEC inc. Toute reproduction interdite

Chapitre 1 Types de réactions chimiques et stœchiométrie en solution 18.

5

Placer la quantité calculée dans une fiole jaugée de 1 L et compléter à la ligne de jauge avec de l’eau. S’il s’agit d’un solide, il faut le dissoudre avant de jauger. a) 1,00 L solution ×

1L 0,50 mol H 2 SO4 × = 2,8 × 10–2 L H2SO4 conc. 18 mol H 2 SO4 L

ou 28 mL b) 1,00 L ×

0,50 mol HCl 1L × = 4,2 × 10–2 L = 42 mL L 12 mol HCl

c) 1,00 L ×

237, 69 g NiCl2 × 6 H 2 O 0,50 mol NiCl2 1 mol NiCl2 × 6 H 2 O × × mol NiCl2 mol NiCl2 × 6 H 2 O L = 118,8 g NiCl2⋅6H2O ≈ 120 g

19.

d) 1,00 L ×

0,50 mol HNO3 1L × = 0,031 L = 31 mL 16 mol HNO3 L

e) 1,00 L ×

0,50 mol Na 2 CO3 105,99 g Na 2 CO3 × = 53 g Na2CO3 L mol

10,8 g (NH4)2SO4 ×

concentration =

1 mol = 8,17 × 10–2 mol (NH4)2SO4 132,15 g

8,17 × 10−2 mol 1000 mL × = 0,817 mol (NH4)2SO4/L 100, 0 mL L

Moles de (NH4)2SO4 dans la solution finale : 10,00 × 10–3 L ×

0,817 mol = 8,17 × 10–3 mol L

concentration finale =

8,17 × 10−3 mol × 1000 mL = 0,136 mol (NH4)2SO4/L (100, 00 + 50, 00) mL

(NH4)2SO4(s) → 2NH4+(aq) + SO42–(aq) : c NH4 + = 2(0,136) = 0,272 mol/L : cSO24 _ = 0,136 mol/L

© 2007 Les Éditions CEC inc. Toute reproduction interdite

Chapitre 1 Types de réactions chimiques et stœchiométrie en solution

6 20.

Solution-mère : 2+ 1,584 g Mn mol Mn 1 mol Mn × × = 0,02883 mol Mn2+/L 1,00 L solution 1 mol Mn 54,94 Mn 2+

0,02883 mol Mn a) L solution - mère

×

L solution − mère 50,00 mL solution − mère × 1000 mL solution − mère 1000,0 mL solution A ×

0,001442 mol Mn b) L solution A

2+

×

L solution A 10,00 mL solution A × 1000 mL solution A 250,0 mL solution B ×

5,766 x 10_ 5 mol Mn c) L solution B

1000 mL solution A = 0,001442 mol Mn2+/L L solution A

2+

×

1000 mL solution B = 5,766 × 10–5 mol Mn2+/L L solution B

L solution B 10,00 mL solution B × 1000 mL solution B 500,0 mL solution C ×

1000 mL solution C = 1,153 × 10–6 mol Mn2+/L L solution C

Réactions de précipitation 21.

Les règles de solubilités citées dans les réponsses suivantes sont décrites au tableau 1.1 du manuel. a) Soluble : la plupart des nitrates sont solubles (règle 1) b) Soluble : la plupart des chlorures sont solubles excepté les sels contenant: Ag+, Pb2+ et Hg22+ (Règle 3). c) Soluble : la plupart des sulfates sont solubles excepté BaSO4, PbSO4, Hg2SO4 et CaSO4 (Règle 4.) d) Insoluble : la plupart des hydroxydes ne sont que partiellement solubles (règle 4). Note : l’expression « partiellement soluble » signifie que la solution est insoluble et l’expression « à solubilité limitée » signifie soluble. Ainsi, on suppose que les hydroxydes Ba(OH)2, Sr(OH)2 et Ca(OH)2 sont solubles sauf indication contraire. e) Insoluble : la plupart des sulfures ne sont que partiellement solubles (règle 6). Encore une fois, l’expression « partiellement soluble » signifie insolubles.

© 2007 Les Éditions CEC inc. Toute reproduction interdite

Chapitre 1 Types de réactions chimiques et stœchiométrie en solution

7

f) Insoluble : Règle 5 (voir réponse d). g) Insoluble : la plupart des sulfates ne sont que partiellement solubles (règle 6). 22.

Dans ces réactions, les composés ioniques solubles sont mélangés ensemble. Pour prédire quel précipité se formera, permutez les anions et les cations dans les deux réactifs afin de prédire les produits possibles; puis, à l’aide des règles de solubilité du tableau 1.1, prédisez si l’un ou l’autre de ces produits possibles est insoluble (précipitent). Remarquez que l’expression « partiellement soluble » du tableau 1.1 signifie que le sel est insoluble et l’expression « à solubilité limitée » signifie que le sel est soluble. a) Produits possibles = FeCl2 et K2SO4; les deux sels sont solubles donc, aucun précipité ne se forme. b) Produits possibles = Al(OH)3 et Ba(NO3)2; précipité = Al(OH)3(s) c) Produits possibles = CaSO4 et NaCl; précipité = CaSO4(s) d) Produits possibles = KNO3 et NiS; précipité = NiS(s)

23.

Sont fournies dans l’ordre l’équation moléculaire, l’équation ionique complète et l’équation ionique nette. a) BaCl2(aq) + Na2SO4(aq) → BaSO4(s) + 2NaCl(aq) Ba2+(aq) + 2Cl–(aq) + 2Na+(aq) + SO42–(aq) → BaSO4(s) + 2Na+(aq) + 2Cl–(aq) Ba2+(aq) + SO42–(aq) → BaSO4(s) b) Pb(NO3)2(aq) + 2KCl(aq) → PbCl2(s) + 2KNO3(aq) Pb2+(aq) + 2NO3–(aq) + 2K+(aq) + 2Cl–(aq) → PbCl2(s) + 2K+(aq) + 2NO3–(aq) Pb2+(aq) + 2Cl–(aq) → PbCl2(s) c) 3AgNO3(aq) + Na3PO4(aq) → Ag3PO4(s) + 3NaNO3(aq) 3Ag+(aq) + 3NO3–(aq) + 3Na+(aq) + PO43–(aq) → Ag3PO4(s) + 3Na+(aq) + 3NO3(aq) 3Ag+(aq) + PO43–(aq) → Ag3PO4(s) d) 3NaOH(aq) + Fe(NO3)3(aq) → Fe(OH)3(s) + 3NaNO3(aq) 3Na+(aq) + 3OH–(aq) + Fe3+(aq) + 3NO3–(aq) → Fe(OH)3(s) + 3Na+(aq) + 3NO3–(aq) Fe3+(aq) + 3OH–(aq) → Fe(OH)3(s)

24.

a) Quand on ajoute CuSO4(aq) à Na2S(aq), le précipité qui se forme est CuS(s). Par conséquent, Na+ (les sphères grises) et SO42− (les sphères bleu-vert) sont les ions inertes. CuSO4(aq) + Na2S(aq) → CuS(s) + Na2SO4(aq); Cu2+(aq) + S2−(aq) → CuS(s) b) Quand on ajoute CoCl2(aq) à NaOH(aq), le précipité qui se forme est Co(OH)2(s). Par conséquent, Na+ (les sphères grises) et Cl- (les sphères vertes) sont les ions inertes. CoCl2(aq) + 2 NaOH(aq) → Co(OH)2(s) + 2 NaCl(aq) Co2+(aq) + 2 OH−(aq) → Co(OH)2(s) © 2007 Les Éditions CEC inc. Toute reproduction interdite

Chapitre 1 Types de réactions chimiques et stœchiométrie en solution

8

c) Quand on ajoute AgNO3(aq) à KI(aq), le précipité qui se forme est AgI(s). Par conséquent, K+ (les sphères rouges) et NO3− (les sphères bleues) sont les ions inertes. AgNO3(aq) + KI(aq) → AgI(s) + KNO3(aq); Ag+(aq) + I−(aq) → AgI(s) 25.

Il y a plusieurs choix acceptables pour les ions inertes. On choisit généralement Na+ et NO3− comme ions inertes parce que les sels de sodium et les nitrates sont habituellement solubles dans l’eau. a) Fe(NO3)3(aq) + 3NaOH(aq) → Fe(OH)3(s) + 3NaNO3(aq) b) Hg2(NO3)2(aq) + 2NaCl(aq) → Hg2Cl2(s) + 2NaNO3(aq) c) Pb(NO3)2(aq) + Na2SO4(aq) → PbSO4(s) + 2NaNO3(aq) d) BaCl2(aq) + Na2CrO4(aq) → BaCrO4(s) + 2 NaCl(aq)

26.

a) (NH4)2SO4(aq) + Ba(NO3)2(aq) → 2NH4NO3(aq) + BaSO4(s) Ba2+(aq) + SO42–(aq) → BaSO4(s) b) Pb(NO3)2(aq) + 2NaCl(aq) → PbCl2(s) + 2NaNO3(aq) Pb2+(aq) + 2Cl–(aq) → PbCl2(s) c) Aucune réaction, les deux produits possibles étant solubles. d) Aucune réaction, les deux produits possibles étant solubles. e) CuCl2(aq) + 2NaOH(aq) → Cu(OH)2(s) + 2NaCl(aq) Cu2+(aq) + 2OH–(aq) → Cu(OH)2(s)

27.

Il n’y a pas de Hg22+, il précipiterait sous forme de Hg2Cl2 en présence de NaCl. Il n’y a pas de Ba2+, il précipiterait sous forme de BaSO4 en présence de Na2SO4. Le précipité formé en présence de NaOH est Mn(OH)2. Le seul ion présent est donc Mn2+.

28.

2 AgNO3(aq) + Na2CrO4(aq) → Ag2CrO4(s) + 2 NaNO3(aq) 0,0750 L ×

0,100 mol AgNO3 1 mol Na 2 CrO 4 161,98 g Na 2 CrO 4 × × L 2 mol AgNO3 mol Na 2 CrO 4 = 0,607 g Na2CrO4

© 2007 Les Éditions CEC inc. Toute reproduction interdite

Chapitre 1 Types de réactions chimiques et stœchiométrie en solution

29.

9

3BaCl2(aq) + Fe2(SO4)3(aq) → 3BaSO4(s) + 2FeCl3(aq) 100,0 mL solution ×

0,100 mol BaCl2 L solution × = 0,0100 mol BaCl2 L solution 1000 mL solution

100,0 mL solution ×

0,100 mol Fe2 (SO4 )3 L solution × L solution 1000 mL solution = 0,0100 mol Fe2(SO4)3

3 mol BaCl2 × 0,0100 mol Fe2(SO4)3 = 0,0300 mol BaCl2 1 mol Fe2 (SO4 )3 BaCl2 est le réactif limitant. 0,0100 mol BaCl2 × 30.

3 mol BaSO4 233,4 g BaSO4 × = 2,33 g BaSO4 3 mol BaCl2 mol BaSO4

M2SO4(aq) + BaCl2(aq) → BaSO4(s) + 2MCl(aq) 2,33 g BaSO4 ×

1 mol BaSO4 1 mol M 2 SO4 × = 9,98 × 10–3 M2SO4 233,4 g BaSO4 mol BaSO4

Selon les données, 1,42 g M2SO4 a réagi, donc : 1,42 g M2SO4 = 9,98 × 10–3 mol M2SO4, masse molaire =

1,42 g M 2 SO4 9,98 × 10_ 3 mol M 2 SO4 = 142 g/mol

142 g/mol = 2 (masse atomique M) + 32,07 + 4(16,00), masse atomique M = 23 g/mol Donc, M est le sodium (Na). 31.

a) Les ions Na+, NO3−, Cl− et Ag+ sont présent avant qu’une réaction se produise. Les ions Ag+ ajoutés en excès enlèvent tous les ions Cl- présents. Par conséquent, les ions Na+, NO3− et les ions Ag+ en excès seront présents une fois la précipitation terminée. b) Ag+(aq) + Cl−(aq) → AgCl(s) c) masse de NaCl _

1 mol AgCl 1 mol Cl 1 mol NaCl 58,44 g = 0,641 g AgCl × × × × _ 143,4 g mol AgCl mol NaCl mol Cl = 0,261 g NaCl % massique de NaCl =

0,261 g NaCl × 100 = 17,4 % NaCl 1,50 g mixture

© 2007 Les Éditions CEC inc. Toute reproduction interdite

Chapitre 1 Types de réactions chimiques et stœchiométrie en solution

10 Réactions acide-base 32.

Toutes les bases des exercices 41 à 44 sont des composés ioniques solubles contenant l’ion OH–. Les acides sont soit des électrolytes forts, soit des électrolytes faibles. Il faut connaître par cœur les électrolytes forts (acides forts) : HCl, HBr, HI, HNO3, HClO4 et H2SO4. Tous les autres acides sont donc des électrolytes faibles. Les équations sont présentées dans l’ordre suivant : moléculaire, ionique complète, ionique nette. a) 2HClO4(aq) + Mg(OH)2(s) → 2H2O(l) + Mg(ClO4)2(aq) 2H+(aq) + 2ClO4–(aq) + Mg(OH)2(s) → 2H2O(l) + Mg2+(aq) + 2ClO4–(aq) 2H+(aq) + Mg(OH)2(s) → 2H2O(l) + Mg2+(aq) b) HCN(aq) + NaOH(aq) → H2O(l) + NaCN(aq) HCN(aq) + Na+(aq) + OH–(aq) → H2O(l) + Na+(aq) + CN–(aq) HCN(aq) + OH–(aq) → H2O(l) + CN–-(aq) c) HCl(aq) + NaOH(aq) → H2O(l) + NaCl(aq) H+(aq) + Cl–(aq) + Na+(aq) + OH–(aq) → H2O(l) + Na+(aq) + Cl–(aq) H+(aq) + OH–(aq) → H2O(l)

33.

a) KOH(aq) + HNO3(aq) → H2O(l) + KNO3(aq) K+(aq) + OH–(aq) + H+(aq) + NO3–(aq) → H2O(l) + K+(aq) + NO3–(aq) OH–(aq) + H+(aq) → H2O(l) b) Ba(OH)2(aq) + 2HCl(aq) → 2H2O(l) + BaCl2(aq) Ba2+(aq) + 2OH–(aq) + 2H+(aq) + 2Cl–(aq) → 2H2O(l) + Ba2+(aq) + 2Cl–(aq) 2OH–(aq) + 2H+(aq) → 2H2O(l) ou OH–(aq) + H+(aq) → H2O(l) c) 3HClO4(aq) + Fe(OH)3(s) → 3H2O(l) + Fe(ClO4)3(aq) 3H+(aq) + 3ClO4–(aq) + Fe(OH)3(s) → 3H2O(l) + Fe3+(aq) + 3ClO4–(aq) 3H+(aq) + Fe(OH)3(s) → 3H2O(l) + Fe3+(aq)

34.

On doit neutraliser 500 mL solution ×

0,200 mol NaOH 1 L solution × = 0,0100 mol NaOH L solution 1000 mL solution

a) HCl + NaOH → NaCl + H2O 0,0100 mol NaOH ×

1 mol HCl L solution × = 0,100 L (100 mL) 1 mol NaOH 0,100 mol HCl

b) H2SO3 + 2NaOH → Na2SO3 + 2H2O 0,0100 mol NaOH ×

1 mol H 2 SO3 L solution × = 0,0500 L (50,0 mL) 2 mol NaOH 0,100 mol H 2 SO3

© 2007 Les Éditions CEC inc. Toute reproduction interdite

Chapitre 1 Types de réactions chimiques et stœchiométrie en solution

11

c) H3PO4 + 3NaOH → Na3PO4 + 3H2O 0,0100 mol NaOH ×

35.

1 mol H3 PO4 L solution × = 0,0167 L (16,7 mL) 3 mol NaOH 0,200 mol H3 PO4

Ba(OH)2(aq) + 2HCl(aq) → BaCl2(aq) + 2H2O(l) ; H+(aq) + OH–(aq) → H2O(l) 75,0 × 10–3 L ×

0,250 mol HCl = 1,88 × 10–2 mol HCl = 1,88 × 10–2 mol H+ L

+ 1,88 × 10–2 mol Cl–

225,0 × 10–3 L ×

0,0550 mol Ba(OH )2 = 1,24 × 10–2 mol Ba(OH)2 L

= 1,24 × 10–2 mol Ba2+ + 2,48 × 10–2 mol OH–

L’équation ionique nette demande un rapport molaire 1:1 pour OH– et H+. Le rapport des ions en présence montre donc un excès d’ions OH–. Puisque 1,88 × 10–2 mol OH– sont neutralisées par H+, il demeure donc (2,48 – 1,88) × 10–2 = 0,60 × 10–2 mol OH– en excès.

cOH- = 36.

mol OH - en excès 6,0 × 10-3 mol OH = = 2,0 × 10–2 mol OH–/L volume total 0,0750 L + 0,2250 L

HCl et HNO3 sont des acides forts; Ca(OH)2 et RbOH sont des bases fortes. L’équation ionique nette est H+(aq) + OH−(aq) → H2O(l). mol H+ = 0,0500 L × ×

0,100 mol HCl 1 mol H + × + 0,1000 L L mol HCl

0,200 mol HNO3 1 mol H + × = 0,00500 + 0,0200 = 0,0250 mol H+ L mol HNO 3 _

0,0100 mol Ca(OH) 2 2 mol OH × mol OH = 0,5000 L × + 0,2000 L L mol Ca(OH)2 −

_

0,100 mol RbOH 1 mol OH = 0,0100 + 0,0200 = 0,0300 mol OH− × × L mol RbOH Il y a un excès de OH- de sorte que la solution est basique (non neutre). Le nombre de mol de OH- en excès est = 0,0300 mol OH− au départ − 0,0250 mol OH− qui a réagi (avec H+) = 0,0050 mol OH− en excès.

© 2007 Les Éditions CEC inc. Toute reproduction interdite

Chapitre 1 Types de réactions chimiques et stœchiométrie en solution

12 37.

HCl(aq) + NaOH(aq) → H2O(l) + NaCl(aq) 24,16 × 10 −3 L NaOH ×

0,106 mol NaOH 1 mol HCl = 2,56 × 10 −3 mol HCl × L NaOH mol NaOH

Concentration molaire volumique de HCl =

38.

2,56 ×10-3 mol = 0,102 mol/L HCl 25,00 ×10-3 L

KHP étant un monoacide, la réaction est : NaOH(aq) + KHP(aq) → H2O(l) + NaKP(aq) masse KHP = 0,02046 L NaOH ×

0,1000 mol NaOH 1 mol KHP 204,22 g KHP × × = 0,4178 g KHP L NaOH mol NaOH mol KHP

Réactions d’oxydoréduction 39.

Appliquons les règles du tableau 1.2. a) KMnO4 est composé d’ions K+ et MnO4–. L’oxygène étant –2, Mn est donc +7 pour que la charge globale de MnO4– soit –1. K, +1 ; O, –2 ; Mn, +7. b) Ni, +4 ; O, –2. c) K4Fe(CN)6 est composé de cations K+ et d’anions Fe(CN)64–. Fe(CN)64– est composé de fer et d’anions CN–. Pour que la charge globale soit –4, le fer doit être +2. d) (NH4)2HPO4 est composé de cations NH4+ et d’anions HPO42–. En donnant l’état d’oxydation +1 à H, N est donc –3 dans NH4+. Si H = +1 et O = –2, alors P = +5 dans HPO42–. e) O, –2 ; P, +3 f) O, –2 ; Fe, +8/3 g) O, –2 ; F, –1 ; Xe, +6 h) F, –1 ; S, +4 i) O, –2 ; C, +2 j) Na, +1 ; O, –2 ; C, +3

© 2007 Les Éditions CEC inc. Toute reproduction interdite

Chapitre 1 Types de réactions chimiques et stœchiométrie en solution

13

40. a) –3

b) –3

c) –2

d) +2

e) +1

f) +4

g) +3

h) +5

i) 0

41. Pour déterminer s’il s’agit d’une réaction rédox, il faut attribuer des nombres d’oxydation aux atomes. Si certains atomes changent de nombre d’oxydation des réactifs aux produits, il s’agit d’une réaction d’oxydoréduction. Dans ces réactions, l’espèce oxydée (ou agent réducteur) voit son nombre d’oxydation augmenter, et l’espèce réduite (agent oxydant) est celle dont le nombre d’oxydation diminue. Rédox a) Oui

42.

Substance Oxydant Réducteur O2 CH4

Substance oxydée réduite CH4 (C) O2 (O)

b) Oui

HCl

Zn

Zn

HCl (H)

c) non









d) Oui

O3

NO

NO (N)

O3 (O)

e) Oui

H2O2

H2O2

H2O2 (O)

H2O2 (O)

f) Oui

CuCl

CuCl

CuCl (Cu)

CuCl (Cu)

Utilisez la méthode des demi-réactions de la section 1.10 pour balancer ces réactions. Il faut d’abord séparer la réaction en deux demi-réactions, puis les balancer séparément : a) Zn → Zn2+ + 2e–

2e– + 2HCl → H2 + 2Cl–

On additionne ensuite les deux demi-réactions en éliminant les électrons : Zn(s) + 2HCl(aq) → H2(g) + Zn2+(aq) + 2Cl–(ag) b) 3I– → I3– + 2e– 2e– + 2H+ + ClO– → Cl– + H2O Addition : 2I–(aq) + 2H+(aq) + ClO–(aq) → I3–(aq) + Cl–(aq) + H2O(l) c) As2O3 → H3AsO4 As2O3 → 2H3AsO4 3 « O » à gauche, 8 « O » à droite 5 « O » de plus à droite.

NO3– → NO + 2H2O 4H+ + NO3– → NO + 2H2O (3e– + 4H+ + NO3– → NO + 2H2O) × 4

On balance d’abord les O en ajoutant 5H2O à gauche, puis les H en ajoutant 4H+ à droite, et finalement la charge globale avec 4e– à droite.

© 2007 Les Éditions CEC inc. Toute reproduction interdite

Chapitre 1 Types de réactions chimiques et stœchiométrie en solution

14

(5H2O + As2O3 → 2H3AsO4 + 4H+ + 4e–) × 3 Pour annuler les électrons, il faut multiplier par 3 et 4 (12e– à éliminer). 12e– + 16H+ + 4NO3– → 4NO + 8H2O 15H2O + 3As2O3 → 6H3AsO4 + 12H+ + 12e– 7H2O(l) + 4H+(aq) + 3As2O3(s) + 4NO3–(aq) → 4NO(g) + 6H3AsO4(aq) d) (2Br– → Br2 + 2e–) × 5

MnO4– → Mn2+ + 4H2O (5e– + 8H+ + MnO4– → Mn2+ + 4H2O) × 2

Il y a échange global de 10 e–. 10Br– → 5Br2 + 10e– 10e + 16H + 2MnO4– → 2Mn2+ + 8H2O –

+

16H+(aq) + 2MnO4–(aq) + 10Br–(aq) e) CH3OH → CH2O (CH3OH → CH2O + 2H+ + 2e–) × 3 L’échange comprend 6 électrons : 3CH3OH 6e + 14H + Cr2O72– –

+

→ 5Br2(l) + 2Mn2+(aq) + 8H2O(l) Cr2O72– → Cr3+ 14H+ + Cr2O72– → 2Cr3+ + 7H2O – 6e + 14H+ + Cr2O72– → 2Cr3+ + 7H2O → 3CH2O + 6H+ + 6e– → 2Cr3+ + 7H2O

8H+(aq) + 3CH3OH(aq) + Cr2O72–(aq) → 2Cr3+(aq) + 3CH2O(aq) + 7H2O(l) 43.

Utilisez la même méthode que pour les solutions acides puis, dans l’équation finale, convertissez les H+ en OH– par la méthode décrite à la section 1.10. a)

Al → Al(OH)4– 4H2O + Al → Al(OH)4– + 4H+ 4H2O + Al → Al(OH)4– + 4H+ + 3e–

MnO4– → MnO2 3e + 4H + MnO4– → MnO2 + 2H2O –

+

4H2O + Al → Al(OH)4– + 4H+ + 3e– 3e + 4H+ + MnO4– → MnO2 + 2H2O –

2H2O(l) + Al(s) + MnO4–(aq) → Al(OH)4–(aq) + MnO2(s) Comme H+ n’apparaît pas dans l’équation balancée, la tâche est terminée.

© 2007 Les Éditions CEC inc. Toute reproduction interdite

Chapitre 1 Types de réactions chimiques et stœchiométrie en solution

b)

Cl2 → Cl– 2e– + Cl2 → 2Cl–

15

Cl2 → ClO– 2H2O + Cl2 → 2ClO– + 4H+ + 2e– 2e– + Cl2 → 2Cl– 2H2O + Cl2 → 2ClO– + 4H+ + 2e– 2H2O + 2Cl2 → 2Cl– + 2ClO– + 4H+

Il faut maintenant convertir cette équation pour une solution basique en ajoutant 4OH– de chaque côté de l’équation. À droite, 4OH– + 4H+ deviennent 4H2O. On soustrait ensuite 2H2O de chaque côté de l’équation pour obtenir 4OH– + 2Cl2 → 2Cl– + 2ClO– + 2H2O, qui se simplifie en : 2OH–(aq) + Cl2(aq) → Cl–(aq) + ClO–(aq) + H2O(l) c)

NO2– → NH3 6e + 7H + NO2– → NH3 + 2H2O –

+

Al → AlO2– (2H2O + Al → AlO2– + 4H+ + 3e–) × 2

Échange de 6 électrons : 6e– + 7H+ + NO2– → NH3 + 2H2O 4H2O + 2Al → 2AlO2– + 8H+ + 6e– OH– + 2H2O + NO2– + 2Al → NH3 + 2AlO2– + H+ + OH– Après addition d’un OH– de chaque côté et soustraction d’un H2O, on obtient : H–(aq) + H2O(l) + NO2–(aq) + 2Al(s) → NH3(g) + 2AlO2–(aq).

44.

NaCl + H2SO4 + MnO2 → Na2SO4 + MnCl2 + Cl2 + H2O Équilibrons cette réaction par inspection. Pour équilibrer les Cl–, il faut 4NaCl : NaCl + H2SO4 + MnO2 → Na2SO4 + MnCl2 + Cl2 + H2O Équilibrons maintenant les ions Na+ et SO42–. NaCl + 2H2SO4 + MnO2 → 2Na2SO4 + MnCl2 + Cl2 + H2O À gauche, 4H et 10 « O » ; à droite, 8 « O » sans compter H2O. On a donc besoin de 2H2O à droite pour équilibrer les H et les O. NaCl(aq) + 2H2SO4(aq) + MnO2(s) → 2Na2SO4(aq) + MnCl2(aq) + Cl2(g) + 2H2O(l)

© 2007 Les Éditions CEC inc. Toute reproduction interdite

Chapitre 1 Types de réactions chimiques et stœchiométrie en solution

16 45.

Au + HNO3 + HCl → AuCl4– + NO Travaillons sans les H+ : Au + NO3– + Cl– → AuCl4– + NO Demi-réactions équilibrées : Au + 4Cl– → AuCl4– + 3e–

3e– + 4H+ + NO3– → NO + 2H2O

Équation globale : Au(s) + 4Cl–(aq) + 4H+(aq) + NO3–(aq) → AuCl4– + NO(g) + 2H2O(l)

EXERCICES SUPPLÉMENTAIRES 46.

Seul l’énoncé b est vrai. Une solution concentrée peut contenir un non électrolyte dissous dans l’eau, p. ex., une solution concentrée de sucre dans l’eau. Les acides sont des électrolytes forts ou faibles. Certains composés ioniques ne sont pas solubles dans l’eau de sorte qu’ils ne sont pas identifiés comme type spécifique d’électrolyte.

47.

mol CaCl2 présentes = 0,230 L ×

0,275 mol CaCl2 = 6,33 × 10–2 mol CaCl2 L

Le volume de la solution de CaCl2 après évaporation est : 6,33 × 10–2 mol CaCl2 ×

1L = 5,75 × 10–2 L = 57,5 mL 1,10 mol CaCl2

volume H2O évaporé = 230 mL – 57,5 mL = 173 mL 48.

a) MgCl2(aq) + 2 AgNO3(aq) → 2 AgCl(s) + Mg(NO3)2(aq) 0,641 g AgCl ×

1 mol AgCl 1 mol MgCl2 95,21 g MgCl2 = 0.213 g MgCl2 × × 143,4 g AgCl 2 mol AgCl mol MgCl2

0,213 g MgCl2 × 100 = 14,2 % MgCl2 1,50 g mélange b) 0,213 g MgCl2 ×

49.

2 mol AgNO3 1 mol MgCl2 1L 1000 mL × × × 95,21 g MgCl2 mol MgCl 2 0,500 mol AgNO3 1L

XCl2(aq) + 2 AgNO3(aq) → 2 AgCl(s) + X(NO3)2(aq) 1,38 g AgCl ×

1 mol XCl2 1 mol = 4,81 × 10 −3 mol XCl2 × 143,4 g 2 mol AgCl

© 2007 Les Éditions CEC inc. Toute reproduction interdite

= 8,95 mL AgNO3

Chapitre 1 Types de réactions chimiques et stœchiométrie en solution

1,00 g XCl2 = 208 g/mol; 4,91×10-3 mol XCl2

17

x + 2(35,45) = 208 x = 137 g/mol

Le métal X est le baryum (Ba). 50.

Tout le soufre du BaSO4 provient de la saccharine. 0,5032 g BaSO4 ×

32,07 g S 183,19 g saccharine × = 0,3939 g saccharine 233,4 g BaSO4 32,07 g S

masse moyenne de saccharine par comprimé =

% saccharine = 51.

0,3949 g = 0,03949 g/comprimé 10 comprimés

0,3949 g saccharine × 100 % = 67,00 % 0,5894 g

Cr(NO3)3(aq) + 3 NaOH(aq) → Cr(OH)3(s) + 3 NaNO3(aq) 2,06 g Cr(OH)3 ×

1mol Cr(OH)3 3 mol NaOH × 103,02 g 1 mol Cr(OH)3 = 6,00 × 10 −2 mol NaOH pour former un précipité

NaOH(aq) + HCl(aq) → NaCl(aq) + H2O(l) 0,1000 L ×

0,400 mol HCl 1 mol NaOH × L mol HCl = 4,00 × 10 −2 mol NaOH pour réagir avec HCl

6,00 ×10-2 mol + 4,00×10-2 mol cNaOH = = 2,00 mol/L NaOH 0,0500 L 52.

CH3CO2H(aq) + NaOH(aq) → H2O(l) + NaCH3CO2(aq) a) 16,58 × 10 −3 L soln ×

0,5062 mol NaOH 1 mol acide acétique × L soln mol NaOH = 8,393 × 10 −3 mol d’acide acétique

Concentration d’acide acétique =

8,393 ×10-3 mol = 0,8393 mol/L 0,01000 L

© 2007 Les Éditions CEC inc. Toute reproduction interdite

Chapitre 1 Types de réactions chimiques et stœchiométrie en solution

18

b) Si on a 1,000 L de solution : masse totale = 1000 mL ×

Masse de CH3CO2H = 0,8393 mol ×

% massique d’acide acétique =

53.

1,006 g = 1006 g mL

60,05 g = 50,40 g mol

50,40 g × 100 = 5,010 % 1006 g

Soit HA = acide inconnu; HA(aq) + NaOH(aq) → NaA(aq) + H2O(l) mol HA présentes = 0,0250 L ×

0,500 mol NaOH 1 mol HA × L 1 mol NaOH

x g HA 2,20 g HA = mol HA 0,0125 mol HA

= 0,0125 mol HA

x = masse molaire de HA = 176 g/mol

Poids de la formule empirique ≈ 3(12) + 4(1) + 3(16) = 88 g/mol 176/88 = 2,0, donc, la formule moléculaire est (C3H4O3)2 = C6H8O6. 54.

On obtient la formule empirique à partir de l’analyse élémentaire. Dans 100,00 g d’acide carminique, il y a: 53,66 g C ×

1 mol C 1 mol H = 4,468 mol C; 4,09 g H × = 4,06 mol H 12,01 g C 1,008 g H

42,25 g O ×

1 mol O = 2,641 mol O 16,00 g C

En divisant le nombre de moles par le plus petit nombre trouvé, on obtient:

4, 468 4, 06 = 1, 692; = 1,54 2, 641 2, 641 Ces nombres ne sont pas des rapports molaires évidents. Déterminons le rapport de mol C à mol H.

4, 468 11 = 1,10 = 4, 06 10

© 2007 Les Éditions CEC inc. Toute reproduction interdite

Chapitre 1 Types de réactions chimiques et stœchiométrie en solution

Essayons

19

4, 06 = 0,406 comme facteur commun: 10

4, 468 4, 06 2, 641 = 11, 0; = 10, 0; = 6,50 0, 406 0, 406 0, 406 Par conséquent, C22H20O13 est la formule empirique. On peut obtenir la masse molaire à partir des données du titrage. 18,02 × 10-3 L ×

0, 0406 mol NaOH 1 mol acide carminique × L mol NaOH = 7,32 × 10 −4 mol d’acide carminique

Masse molaire =

0,3602 g 492 g = -4 7,32 ×10 mol mol

La masse de la formule empirique de C22H20O13 ≈ 22(12) + 20(1) + 13(16) = 492 g. Donc, la formule moléculaire de l’acide carminique est aussi C22H20O13. 55.

a) Al(s) + 3HCl(aq) → AlCl3(aq) + 3/2H2(g) ou 2Al(s) + 6HCl(aq) → 2AlCl3(aq) + 3H2(g)

L’hydrogène est réduit de +1 à 0 ; Al est oxydé de 0 à +3. b) Équilibrez C et H en premier, puis S.

CH4(g) + 4S(s) → CS2(l) + 2H2S(g) S est réduit (0 → –2) et C est oxydé (–4 → +4). c) Équilibrez C et H en premier, puis O.

C3H8(g) + 5O2(g) → 3CO2(g) + 4H2O(l) O est réduit (0 → –2) et C est oxydé (–8/3 → +4). d) 2 moles d’Ag sont requises pour équilibrer les charges.

Cu(s) + 2Ag+(aq) → 2Ag(s) + Cu2+(aq) Ag est réduit (+1 → 0) et Cu est oxydé (0 → +2).

© 2007 Les Éditions CEC inc. Toute reproduction interdite

Chapitre 1 Types de réactions chimiques et stœchiométrie en solution

20

56.

Mn + HNO3 → Mn2+ + NO2 Mn → Mn2+ + 2 e–

HNO3 → NO2 HNO3 → NO2 + H2O (e– + H+ + HNO3 → NO2 + H2O) × 2 Mn → Mn2+ + 2e– 2e + 2H + 2HNO3 → 2NO2 + 2H2O –

+

2H+(aq) + Mn(s) + 2HNO3(aq) → Mn2+(aq) + 2NO2(g) + 2H2O(l) Mn2+ + IO4– → MnO4– + IO3– (4H2O + Mn2+ → MnO4– + 8H+ + 5e–) × 2

(2e– + 2H+ + IO4– → IO3– + H2O) × 5

8H2O + 2Mn2+ → 2MnO4– + 16H+ + 10e– 10e–- + 10H+ + 5IO4– → 5IO3– + 5H2O 3H2O(l) + 2Mn2+(aq) + 5IO4–(aq) → 2MnO4–(aq) + 5IO3–(aq) + 6H+(aq) PROBLÈMES DÉFIS 57.

a) 0,308 g AgCl ×

35,45 g Cl 0,0761 g = 0,0761 g Cl ; % Cl = × 100 % = 29,7 % 143,4 g AgCl 0,256 g

Oxyde de cobalt(III), Co2O3 : 2(58,93) + 3(16,00) = 165,86 g/mol 0,145 g Co2O3 ×

% Co =

117,86 g Co = 0,103 g Co ; 165,86 g Co2 O3

0,103 g × 100 % = 24,8 % 0,416 g

Le reste, 100,0 – (29,7 + 24,8) = 45,5 %, est de l’eau. Supposons 100,0 g de composé : 45,5 g H2O ×

2,016 g H 5,09 g H = 5,09 g H ; % H = × 100 % 18,02 g H 2 O 100,0 g composé = 5,09 %

16,00 g O 40,4 g O 45,5 g H2O × = 40,4 g O ; % O = × 100 % 18,02 g H 2 O 100,0 g composé = 40,4 % Ce composé contient donc 24,8 % Co, 29,7 % Cl, 5,09 % H et 40,4 % O.

© 2007 Les Éditions CEC inc. Toute reproduction interdite

Chapitre 1 Types de réactions chimiques et stœchiométrie en solution

21

b) Dans 100,0 g de composé, il y a :

24,8 g Co ×

5,09 g H ×

1 mol 1 mol = 0,421 mol Co ; 29,7 g Cl × = 0,838 mol Cl 58,93 g Co 35,45 g Cl

1 mol 1 mol = 5,05 mol H ; 40,4 g O × = 2,53 mol O 1,008 g H 16,00 g O

En divisant tous ces résultats par 0,421, on obtient CoCl2⋅6H2O. c) CoCl2⋅6H2O(aq) + 2AgNO3(aq) → 2AgCl(s) + Co(NO3)2(aq) + 6H2O(l)

CoCl2⋅6H2O(aq) + 2NaOH(aq) → Co(OH)2(s) + 2NaCl(aq) + 6H2O(l) Co(OH)2 → Co2O3 Le cobalt étant oxydé, on a besoin d’un oxydant. Le choix évident est O2. 4Co(OH)2(s) + O2(g) → 2Co2O3(s) + 4H2O(l) 58.

a) 2 AgNO3(aq) + K2CrO4(aq) → Ag2CrO4(s) + 2 KNO3(aq)

Masse molaire: 169,9 g/mol

194,20 g/mol

331,8 g/mol

La masse molaire de Ag2CrO4 est 331,8 g/mol, donc il s’est formé une mole de précipité. On a des masses égales de AgNO3 et de K2CrO4. Étant donné que la masse molaire de AgNO3 est inférieure à celle de K2CrO4, il y a plus de moles de AgNO3 présentes. Cependant, on n’aura pas deux fois le nombre de moles de AgNO3 présentes par rapport à K2CrO4 comme l’exige l’équation équilibrée;c’est parce que la masse molaire de AgNO3 n’est pas, tant s’en faut, la moitié de la masse molaire de K2CrO4. Donc, AgNO3 est limitant. masse de AgNO3 = 1,000 mol Ag2CrO4 ×

2 mol AgNO3 169,9 g × mol Ag 2 CrO 4 mol AgNO3 = 339,8 g AgNO3

Étant donné que des masses égales de réactifs sont présentes, il y avait au départ 339,8 g de K2CrO4.

cK + =

mol K + = volume total

339,8 g K 2 CrO 4 ×

1 mol K 2 CrO 4 2 mol K + × 194,20 g mol K 2 CrO 4 0,5000 L = 7,000 mol/L K+

© 2007 Les Éditions CEC inc. Toute reproduction interdite

Chapitre 1 Types de réactions chimiques et stœchiométrie en solution

22

b) mol CrO42− = 339,8 g K2CrO4 ×

1 mol K 2 CrO 4 1 mol CrO 2-4 × 194,20 g mol K 2 CrO 4 = 1,750 mol CrO42− présentes au départ

mol CrO42− = 1,000 mol Ag2CrO4 ×

cCrO24

1 mol CrO 2-4 = 1,000 mol CrO42− précipite mol Ag 2 CrO 4

mol CrO 2-4 en excès 1,750 mol -1,000 mol 0,750 mol = = = 0,5000 L + 0,5000 L 1,0000 L volume total = 0,750 mol/L

59.

Masses molaires : KCl, 39,10 + 35,45 = 74,55 g/mol ; KBr, 39,10 + 79,90 = 119,00 g/mol AgCl, 107,9 + 35,45 = 143,4 g/mol ; AgBr, 107,9 + 79,90 = 187,8 g/mol Soit x = nombre de moles de KCl dans le mélange et y = nombre de moles de KBr. Puisque Ag+ + Cl– → AgCl et Ag+ + Br– → AgBr, alors x = moles AgCl et y = moles AgBr. On peut établir deux équations : 0,1024 g = 74,55 x + 119,0 y et 0,1889 g = 143,4 x + 187,8 y Multiplions la première par

187,8 , et soustrayons-la de la deuxième. 119,0

0,1889 = 143,4 x + 187,8 y –0,1616 = –117,7 x – 187,8 y 0,0273 =

25,7 x,

1,06 × 10–3 mol KCl ×

% KCl = 60.

x = 1,06 × 10–3 mol KCl

74,55 g KCl = 0,0790 g KCl mol KCl

0,0790 g × 100 % = 77,1 %, % KBr = 100,0 – 77,1 = 22,9 % 0,1024 g

a) Soit x = masse de Mg, de sorte que 10,00 - x = masse de Zn. Ag+(aq) + Cl−(aq) → AgCl(s).

D’après les équations équilibrées fournies, il y a un rapport molaire 2:1 entre le nombre de moles de Mg et le nombre de moles de Cl-. La même chose est vraie pour Zn. Étant donné que mol Ag+ = mol Cl− présentes, on peut établir une équation qui met en relation le nombre de moles de Cl- présentes au nombre de moles de Ag+ ajoutées.

© 2007 Les Éditions CEC inc. Toute reproduction interdite

Chapitre 1 Types de réactions chimiques et stœchiométrie en solution

x g Mg ×

23

1 mol Mg 2 mol Cl1 mol Zn 2 mol Cl× + (10,00-x) g Zn × × 24,31 g Mg mol Mg 65,38 g Zn mol Zn = 0,156 L ×

3,00 mol Ag + 1 mol Cl= 0,468 mol Cl− × + L mol Ag

2x 2(10,00 - x) 20,00 - 2x ⎛ 2x ⎞ = 0,468, 24,31 × 65,38 ⎜ + + = 0,468 ⎟ 24,31 65,38 65,38 ⎝ 24,31 ⎠ 130,8 x + 486,2 – 48,62 x = 743,8 (portant un C. S. supplémentaire) 82,2 x = 257,6; x = 3,13 g Mg;

% Mg =

3,13g Mg × 100 = 31,3 % Mg 10,00 g mixture

3,00 mol Ag + 1 mol Clb) 0,156 L × = 0,468 mol Cl− = 0,468 mol HCl ajouté × + L mol Ag cHCl =

61.

0,648 mol = 6,00 mol/L HCl 0,0780 L

Pb2+(aq) + 2 Cl−(aq) → PbCl2(s) 3,407 g 3,407 g PbCl2 ×

1 mol PbCl2 278,1 g PbCl2

×

1 mol Pb 2+ = 0,01225 mol Pb2+ 1 mol PbCl2

0,01225 mol = 6,13 mol/L Pb2+ (concentration évaporée) 2,00 ×10-3 L

62.

concentration originale =

0,0800 L × 6,13 mol/L = 4,90 mol/L 0,100 L

moles CuSO4 = 87,7 mL ×

1L 0,500 mol = 0,0439 mol × 1000 mL L

moles Fe = 2,00 g ×

1 mol Fe = 0,0358 mol 55,85 g

Les deux réactions possibles sont : I.

CuSO4(aq) + Fe(s) → Cu(s) + FeSO4(aq)

© 2007 Les Éditions CEC inc. Toute reproduction interdite

Chapitre 1 Types de réactions chimiques et stœchiométrie en solution

24 II.

3 CuSO4(aq) + 2 Fe(s) → 3 Cu(s) + Fe2(SO4)3(aq)

Si la réaction I se produit, Fe est limitant et on peut former : 0,0358 mol Fe ×

1 mol Cu 63,55 g Cu = 2,28 g Cu × mol Fe mol Cu

Si la réaction II se produit, CuSO4 est limitant et on peut former : 0,0439 mol CuSO4 ×

3 mol Cu 63,55 g Cu × = 2,79 g Cu 3 mol CuSO 4 mol Cu

En supposant un rendement de 100 %, c’est la réaction I qui se produit parce qu’elle est plus conforme aux données. 63.

0,2750 L × 0,300 mol/L = 0,0825 mol H+; Soit y = volume (L) fourni par Y et z = volume (L) fourni par Z. H+(aq) + OH−(aq) → H2O(l);

, , y(0.150 mol/L) + z(0.250 mol/L) = 0,9725 mol H+ mol OH

0,2750 L + y + z = 0,655 L; y + z = 0,380; z = 0,380 - y y(0,150) + (0,380 - y)(0,250) = 0,0825; en résolvant : y = 0,125 L; z = 0,255 L débits : Y →

64.

125 mL 255 mL = 2,06 mL/min et Z → = 4,20 mL/min 60,65 min 60,65 min

a) H3PO4(aq) + 3 NaOH(aq) → 3 H2O(l) + Na3PO4(aq) b) 3 H2SO4(aq) + 2 Al(OH)3(s) → 6 H2O(l) + Al2(SO4)3(aq) c) H2Se(aq) + Ba(OH)2(aq) → 2 H2O(l) + BaSe(s) d) H2C2O4(aq) + 2 NaOH(aq) → 2 H2O(l) + Na2C2O4(aq)

65.

a) MgO + 2HCl(aq) → MgCl2(aq) + H2O(l)

Mg(OH)2 + 2HCl(aq) → MgCl2(aq) + 2H2O(l) Al(OH)3 + 2HCl(aq) → AlCl3(aq) + 3H2O(l)

© 2007 Les Éditions CEC inc. Toute reproduction interdite

Chapitre 1 Types de réactions chimiques et stœchiométrie en solution

b)

25

mol MgO L solution 2 mol HCl × × = 0,5 L solution/g MgO 40,31 g MgO 0,1 mol HCl mol MgO mol Mg(OH )2 2 mol HCl L solution × × mol Mg(OH )2 0,1 mol HCl 58,33 g Mg(OH )2 = 0,3 L solution/g Mg(OH)2

mol Al(OH )3 3 mol HCl L solution × × = 0,4 L solution/g Al(OH)3 1 mol Al(OH )3 0,1 mol HCl 78,00 g Al(OH )3 MgO neutralise le plus grand volume. 66.

Soit H2A = formule de l’acide diprotique H2A(aq) + 2 NaOH(aq) → 2 H2O(l) + Na2A(aq) mol H2A = 0,1375 L ×

1 mol H 2 A 0,750 mol NaOH × L 2 mol NaOH

masse molaire de H2A =

67.

= 0,0516 mol

6,50 g = 126 g/mol 0,0516 mol

mol C6H8O7 = 0,250 g C6H8O7 ×

1 mol C6 H8O7 = 1,30 × 10 −3 mol C6H8O7 192,12 g C6 H8O7

Soit HxA qui représente l’acide citrique où x est le nombre d’hydrogènes acides. L’équation de neutralisation équilibrée est : HxA(aq) + xOH−(aq) → xH2O(l) + Ax−(aq)

0,105 mol OH = 3,91 × 10 −3 mol OH− L mol OH 3,91×10-3 mol x= = = 3,01 mol acide citrique 1,30 ×10-3 mol

mol OH− qui ont réagi = 0,0372 L ×

Par conséquent, la formule générale pour l’acide citrique est H3A, ce qui signifie que l’acide citrique a trois hydrogènes acides par molécule (l’acide citrique est un acide triprotique).

© 2007 Les Éditions CEC inc. Toute reproduction interdite

Chapitre 1 Types de réactions chimiques et stœchiométrie en solution

26 68.

a) HCl(aq) se dissocie en H+(aq) + Cl–(aq). Pour simplifier, traitons H+ et Cl– séparément.

H+ → H2 (2H+ + 2e– → H2) × 3

Fe → HFeCl4 (H+ + 4Cl– + Fe → HFeCl4 + 3e–) × 2

6H+ + 6e– → 3H2 2H+ + 8Cl– + 2Fe → 2HFeCl4 + 6e– 8H+ + 8Cl– + 2Fe → 2HFeCl4 + 3H2

ou

8HCl(aq) + 2Fe(s) → 2HFeCl4(aq) + 3H2(g)

b)

IO3– → I3– 3IO3– → I3– 3IO3– → I3– + 9H2O 16e– + 18H+ + 3IO3– → I3– + 9H2O

I– → I3– (3I– → I3– + 2e–) × 8

16e– + 18H+ + 3IO3– → I3– + 9H2O 24I– → 8I3– + 16e– 18H+ + 24I– + 3IO3– → 9I3– + 9H2O Simplification : 6H+(aq) + 8I–(aq) + IO3–(aq) → 3I3–(aq) + 3H2O(l) c) (Ce4+ + e– → Ce3+) × 97

Cr(NCS)64– → Cr3+ + NO3– + CO2 + SO42– 54H2O + Cr(NCS)64– → Cr3+ + 6NO3– + 6CO2 + 6SO42– + 108H+ Charge à gauche –4. Charge à droite = +3 + 6(–1) + 6(–2) + 108(+1) = +93. Ajoutons 97e– à droite, puis additionnons les deux demi-réactions : 54H2O + Cr(NCS)64– → Cr3+ + 6NO3– + 6CO2 + 6SO42– + 108H+ + 97e– 97e– + 97Ce4+ → 97Ce3+ 97 Ce4+(aq) + 54H2O(l) + Cr(NCS)64–(aq) → 97Ce3+(aq) + Cr3+(aq) + 6NO3–(aq) + 6CO2(g) + 6SO42–(aq) + 108H+(aq) Cela vaut la peine de vérifier les charges. À gauche : charge = 97(+4) – 4 = +384 À droite : charge = 97(+3) + 3 + 6(–1) + 6(–2) + 108(+1) = +384

© 2007 Les Éditions CEC inc. Toute reproduction interdite

Chapitre 1 Types de réactions chimiques et stœchiométrie en solution

d)

CrI3 → CrO42– + IO4– (16H2O + CrI3 → CrO42– + 3IO4– + 32H+ + 27e–) × 2

27

Cl2 → Cl– (2e– + Cl2 → 2Cl–) × 27

Il y a échange de 54 électrons : 54e– + 27Cl2 → 54Cl– 32H2O + 2CrI3 → 2CrO42– + 6IO4– + 64H+ + 54e– 32H2O + 2CrI3 + 27Cl2 → 54Cl– + 2CrO42– + 6IO4– + 64H+ Ajoutons 64OH– aux deux côtés pour convertir 64H+ en 64H2O 64OH– + 32H2O + 2CrI3 + 27Cl2 → 54Cl– + 2CrO42– + 6IO4– + 64H2O Après simplification : 64OH–(aq) + 2CrI3(s) + 27Cl2(g) → 54Cl–(aq) + 2CrO42(aq) + 6IO4–(aq) + 32H2O(l) e)

Ce4+ → Ce(OH)3 (e + 3H2O + Ce4+ → Ce(OH)3 + 3H+) × 61 –

Fe(CN)64– → Fe(OH)3 + CO32– + NO3– Fe(CN)64– → Fe(OH)3 + 6CO32– + 6NO3– Il y a 39 atomes O de plus à droite. Ajoutons 39H2O à gauche, puis 75H+ à droite pour équilibrer H+. 39H2O + Fe(CN)64– → Fe(OH)3 + 6CO32– + 6NO3– + 75H+ charge nette = –4 charge nette = +57 Ajoutons enfin 61e– à droite, puis additionnons les deux demi-réactions. 39H2O + Fe(CN)64– → Fe(OH)3 + 6CO32– + 6NO3– + 75H+ + 61e– 61e + 183H2O + 61Ce4+ → 61Ce(OH)3 + 183H+ –

222H2O + Fe(CN)64– + 61Ce4+ → 61Ce(OH)3 + Fe(OH)3 + 6CO32– + 6NO3– + 258H+ Ajoutons 258OH– de chaque côté, puis réduisons à : 258OH–(aq) + Fe(CN)64–(aq) + 61Ce4+(aq) → 61Ce(OH)3(s) + Fe(OH)3(s) + 6CO32–(aq) + 6NO3–(aq) + 36H2O(l)

© 2007 Les Éditions CEC inc. Toute reproduction interdite

Chapitre 1 Types de réactions chimiques et stœchiométrie en solution

28 f)

Fe(OH)2 → Fe(OH)3 (H2O + Fe(OH)2 → Fe(OH)3 + H+ + e–) × 2

H2O2 → H2O 2e + 2H + H2O2 → 2H2O –

+

2H2O + 2Fe(OH)2 → 2Fe(OH)3 + 2H+ + 2e– 2e– + 2H+ + H2O2 → 2H2O 2H2O + 2H+ + 2Fe(OH)2 + H2O2 → 2Fe(OH)3 + 2H2O + 2H+ Après simplification : 2Fe(OH)2(s) + H2O2(aq) → 2Fe(OH)3(s) PROBLÈMES D’INTÉGRATION 69.

a) Supposons 100,00 g de substance.

42,23 g C ×

2,11 g B ×

1 mol C 1 mol F = 3,516 mol C; 55,66 g F × = 2,929 mol F 12,01 g C 19,00 g F

1 mol B = 0,195 mol B 10,81 g B

En divisant par le plus petit nombre :

3,516 2,929 = 18,0; = 15,0 0,195 0,195

La formule empirique est C18F15B. b) 0,3470 L ×

0,01267 mol = 4,396 × 10 −3 mol BARF L

masse molaire du BARF =

2,251 g = 512,1 g/mol 4,396 × 10-3 mol

La masse de la formule empirique du BARF est 511,99 g. Par conséquent, la formule moléculaire est la même que la formule empirique, C18F15B. 70.

3 (NH4)2CrO4(aq) + 2 Cr(NO2)3(aq) → 6 NH4NO2(aq) + Cr2(CrO4)3(s) 0,203 L ×

0,307 mol = 6,23 × 10 −2 mol (NH4)2CrO4 L

0,137 L ×

0,269 mol = 3,69 × 10 −2 mol Cr(NO2)3 L

© 2007 Les Éditions CEC inc. Toute reproduction interdite

Chapitre 1 Types de réactions chimiques et stœchiométrie en solution

29

0,0623 mol = 1,69 (réel); d’après l’équation équilibrée, il faut un rapport molaire 3/2 0,0369 mol = 1,5 entre (NH4)2CrO4 et Cr(NO2)3. Réel > requis, de sorte que Cr(NO2)3 (le dénominateur) est limitant. 3,69 × 10 −2 mol Cr(NO2)3 ×

1 mol Cr2 (CrO 4 )3 452,00 g Cr2 (CrO 4 )3 × 2 mol Cr(NO 2 )3 1 mol Cr2 (CrO 4 )3 = 8,34 g Cr2(CrO4)3

rendement réel 0,880 = , rendement réel = (8,34 g)(0,880) = 7,34 g Cr2(CrO4)3 isolé 8,34 g 71.

L’équation non équilibrée est : VO2+ + MnO4− → V(OH)4+ + Mn2+ C’est une réaction rédox en solution acide et il faut l’équilibrer en conséquence. Les deux demi-réactions à équilibrer sont : VO2+ → V(OH)4+ et MnO4− → Mn2+ On équilibre par la méthode des demi-réactions, ce qui donne : MnO4−(aq) + 5 VO2+(aq) + 11 H2O(l) → 5 V(OH)4+(aq) + Mn2+(aq) + 2 H+(aq) 0,02645 L ×

0,581 =

0,02250 mol MnO-4 5 mol VO 2+ 1 mol V 50,94 g V = 0,1516 g V × × × 2+ L mol MnO 4 mol VO mol V

0.1516 g V , 0,1516/0,581 masse de l'échantillon de minerai = 0,261 g de l’échantillon de minerai

Les états d’oxydation des éléments dans les divers ions sont : VO2+ : O, -2; V, x + (-2) = + 2, x = +4 MnO4− : O, -2; Mn, x + 4(-2) = -1, x = +7 V(OH)4+ : O, -2, H, +1; V, x + 4(-2) + 4(+1) = +1, x = +5 Mn2+ : Mn, +2 MnO4- comporte le métal de transition qui a le degré d’oxydation le plus élevé (+7). 72.

X2− contient 36 électrons, de sorte que X2− a 34 protons qui permet d’identifier X comme étant le sélénium (Se). Le nom de H2Se serait l’acide séléhydrique, selon les conventions décrites au chapitre 2 de Chimie générale. H2Se(aq) + 2 OH−(aq) → Se2−(aq) + 2 H2O(l) 0,0356 L ×

0,175 mol OH - 1 mol H 2Se 80,98 g H 2Se = 0,252 g H2Se × × L 2 mol OH mol H 2Se

© 2007 Les Éditions CEC inc. Toute reproduction interdite

30

Chapitre 1 Types de réactions chimiques et stœchiométrie en solution

PROBLÈMES DE SYNTHÈSE 73.

Patrick : TiSO4 ; Christian : Na2SO4 ; Paul : Ga2(SO4)3. Selon les manuels de référence, le sulfate de sodium est un cristal blanc de forme orthorhombique, alors que le sulfate de gallium est une poudre blanche. Le sulfate de titane se présente sous forme de poudre verte, mais sa formule est Ti2(SO4)3. Comme il a la même formule que le sulfate de gallium, sa masse molaire calculée devrait se situer autour de 443 g/mol. Cependant, la masse molaire de Ti2(SO4)3 est 383,97 g/mol. Il est donc peu plausible que le sel soit le sulfate de titane. Pour distinguer entre Na2SO4 et Ga2(SO4)3, on peut dissoudre le sel dans l’eau et ajouter NaOH. Ga3+ forme un précipité avec l’hydroxyde, alors que Na2SO4 ne le fait pas. Les manuels de référence confirment que l’hydroxyde de gallium est insoluble dans l’eau. moles BaSO4 = 0,2327 g ×

1 mol = 9,970 × 10 −4 mol BaSO4 233,4 g

Le nombre de moles du sulfate dépend de la formule du sel. L’équation générale est : Mx(SO4)y(aq) + y Ba2+(aq) → y BaSO4(s) + x Mz+ Selon la valeur de y, le rapport molaire entre le sulfate inconnu et BaSO4 varie. Par exemple, si Patrick pense que la formule est TiSO4, l’équation devient : TiSO4(aq) + Ba2+(aq) → BaSO4(s) + Ti2+(aq) Étant donné qu’il y a un rapport molaire 1:1 entre le nombre de moles de BaSO4 et le nombre de moles de TiSO4, il faut 9,970 × 10 −4 mol de TiSO4. Étant donné que 0,1472 g de sel a été utilisé, le composé aurait une masse molaire de (en supposant que la formule est TiSO4) : 0,1472 g/9,970 × 10 −4 mol = 147,6 g/mol À partir des masses atomiques du tableau périodique, la masse molaire de TiSO4 est 143,95 g/mol. En se basant uniquement sur ces données, la formule TiSO4 semble logique. Christian pense que le sel est le sulfate de sodium dont la formule est Na2SO4. L’équation est : Na2SO4(aq) + Ba2+(aq) → BaSO4(s) + 2 Na+(aq) Comme dans le cas de TiSO4, il y a un rapport molaire 1:1 entre le nombre de moles de BaSO4 et le nombre de moles de Na2SO4. Pour que le sulfate de sodium soit un choix logique, il faut que sa masse molaire soit 147,6 g/mol. En utilisant les masses atomiques, la masse molaire de Na2SO4 est 142,05 g/mol. Donc, Na2SO4 est également logique. Paul, qui a choisi le gallium, déduit que le gallium doit avoir une charge de +3 (étant donné que l’élément se situe dans la colonne 13 ou 3A) et que la formule du sulfate doit être Ga2(SO4)3. L’équation serait : Ga2(SO4)3(aq) + 3 Ba2+(aq) → 3 BaSO4(s) + 2 Ga3+(aq)

© 2007 Les Éditions CEC inc. Toute reproduction interdite

Chapitre 1 Types de réactions chimiques et stœchiométrie en solution

31

La masse molaire calculée de Ga2(SO4)3 serait :

0,1472 g Ga 2 (SO 4 )3 3 mol BaSO 4 = 442,9 g/mol × -4 9,970 ×10 mol BaSO 4 mol Ga 2 (SO 4 )3 En utilisant les masses atomiques, la masse molaire de Ga2(SO4)3 est 427,65 g/mol. Donc, Ga2(SO4)3 est également logique. En consultant les renseignements dans les livres de référence, le sulfate de sodium (Na2SO4) existe sous forme de solide blanc cristallisant dans le système orthorhombique, alors que le sulfate de gallium Ga2(SO4)3 est une poudre blanche. Le sulfate de titane existe sous forme de poudre verte, mais sa formule est Ti2(SO4)3. Étant donné que sa formule est la même que celle du sulfate de gallium, la masse molaire calculée devrait être autour de 443 g/mol. Cependant, la masse molaire de Ti2(SO4)3 est 383,97 g/mol. Il est donc peu probable que le sel soit le sulfate de titane. Afin de faire la distinction entre Na2SO4 et Ga2(SO4)3, on peut dissoudre le sulfate dans l’eau et ajouter du NaOH. Ga3+ formerait un précipité avec l’hydroxyde alors que Na2SO4 ne le ferait pas. Les ouvrages de référence confirment que l’hydroxyde de gallium est insoluble dans l’eau. 74.

a) Composé A = M(NO3)x; dans 100,00 g de composé : 8,246 g N ×

48,00 g O 14,01 g N = 28,25 g O

Donc, la masse de nitrate dans le composé = 8,246 + 28,25 g = 36,50 g si x = 1. si x = 1 : la masse de M = 100,00 – 36,50 g = 63,60 g mol M = mol N =

8,246 g = 0,5886 mol 14,01 g/mol

masse molaire du métal M =

63,50 g = 107,9 g/mol (C’est l’argent, Ag.) 0,5886 mol

Si x = 2 : la masse de M = 100,00 - 2(36,50) = 27,00 g mol M = ½ mol N =

0,5886 mol = 0,2943 mol 2

masse molaire du métal M =

27,00 g = 91,74 g/mol 0,2943 mol

C’est près de Zr, mais Zr ne forme pas d’ions +2 stables en solution; il forme des ions +4 stables. Étant donné qu’on ne peut pas avoir x = 3 ou plus de nitrates (3 nitrates auraient une masse supérieure à 100,00 g), le composé A doit être AgNO3.

© 2007 Les Éditions CEC inc. Toute reproduction interdite

Chapitre 1 Types de réactions chimiques et stœchiométrie en solution

32

Le composé B : K2CrOx est la formule. Ce sel est composé d’ions K+ et CrOx2−. En utilisant les états d’oxydation, 6 + x(-2) = -2, x = 4. Le composé B est K2CrO4 (chromate de potassium). b) La réaction est :

2 AgNO3(aq) + K2CrO4(aq) → Ag2CrO4(s) + 2 KNO3(aq) Le précipité rouge sang est Ag2CrO4(s). c) 331,8 g de Ag2CrO4 se forment; c’est égal à la masse molaire de Ag2CrO4, donc 1 mol de précipité se forme. D’après l’équation équilibrée, il faut que 2 mol de AgNO3 réagissent avec 1 mol de K2CrO4 pour produire 1 mol (331,8 g) de Ag2CrO4.

2,000 mol AgNO3 ×

169,9 g = 339,8 g AgNO3 mol

1,000 mol K2CrO4 ×

194,2 g = 194,2 g K2CrO4 mol

Le problème mentionne qu’il y a des masses égales de réactifs. Nos deux choix sont 339,8 g AgNO3 + 339,8 g K2CrO4 ou 194,2 g AgNO3 + 194,2 g K2CrO4. Si l’on suppose que la quantité de 194,2 g est correcte, alors quand 194,2 g de K2CrO4 (1 mol) réagissent, 339,8 g de AgNO3 (2,0 mol) doivent être présents pour réagir avec tout le K2CrO4. Il n’y a que 194,2 g de AgNO3 présents; cela ne peut pas être correct. À la place de K2CrO4, c’est AgNO3 qui doit être limitant et 339,8 g AgNO3 et 339,8 g de K2CrO4 ont réagi. Solution A :

2,000 mol NO32,000 mol Ag + = 4,000 mol/L Ag+; 0,5000 L 0,5000 L = 4,000 mol/L NO3−.

Solution B : 339,8 g K2CrO4 ×

1 mol = 1,750 mol K2CrO4 194,2 g

1,750 mol CrO 4 22 ×1,750 mol K + + = 7,000 mol/L K ; = 3,500 mol/L CrO42− 0,5000 L 0,5000 L d) Une fois la réaction terminée, le nombre de moles de K+ et le nombre de moles de NO3− restent inchangés parce que ce sont des ions inertes. Étant donné que Ag+ est limitant, sa concentration est de 0 mol/L une fois que la précipitation est complétée.

2 Ag+(aq) + CrO42−(aq) → Ag2CrO4(s) Initialement Changement Après la réaction

2,000 mol -2,000 mol 0

1,750 mol -1,000 mol 0,750 mol

© 2007 Les Éditions CEC inc. Toute reproduction interdite

0 +1,000 mol 1,000 mol

Chapitre 1 Types de réactions chimiques et stœchiométrie en solution

33

2 ×1,750 mol 2,000 mol = 3,500 mol/L K+; cNO- = = 2,000 mol/L NO3− 3 1,0000 L 1,0000 L 0,750 mol cCrO2- = = 0,750 mol/L CrO42−; cAg+ = 0 mol/L (le réactif limitant) 4 1,0000 L

cK + =

© 2007 Les Éditions CEC inc. Toute reproduction interdite

CHAPITRE 2 PROPRIÉTÉS DES SOLUTIONS RÉVISION DU CHAPITRE 1

1 mol 0,365 mol 0,365 mol saccharose = 0,365 mol ; c = = 342,30 g 1,00 L L

9.

125 g saccharose ×

10.

1,28 g CaCl2 ×

11.

mol Na2CO3 = 0,0700 L ×

1 mol CaCl2 1L 1000 mL = 19,9 mL × × 110,98 g CaCl2 0,580 mol CaCl2 L

3,0 mol Na 2 CO3 = 0,21 mol Na2CO3 L

Na2CO3(s) → 2 Na+(aq) + CO32−(aq); mol Na+ = 2(0.21) = 0.42 mol mol NaHCO3 = 0,0300 L ×

1,0 mol NaHCO3 = 0.030 mol NaHCO3 L

NaHCO3(s) → Na+(aq) + HCO3−(aq); mol Na+ = 0,030 mol

cNa + = 12.

mol total Na + 0,42 mol + 0,030 mol 0,45 mol = = = 4,5 mol/L Na+ volume total 0,0700 L + 0,030 L 0,1000 L

a) HNO3(l) → H+(aq) + NO3–(aq)

b) Na2SO4(s) → 2Na+(aq) + SO42–(aq)

c) AlCl3(s) → Al3+(aq) + 3Cl–(aq)

d) SrBr2(s) → Sr2+(aq) + 2Br–(aq)

e) KClO4(s) → K+(aq) + ClO4–(aq)

f) NH4Br(s) → NH4+(aq) + Br–(aq)

g) NH4NO3(s) → NH4+(aq) + NO3–(aq)

h) CuSO4(s) → Cu2+(aq) + SO42–(aq)

i) NaOH(s) → Na+(aq) + OH–(aq)

QUESTIONS 13.

À mesure que la température augmente, les molécules de gaz acquièrent une énergie cinétique moyenne plus grande. Une plus grande proportion des molécules de gaz en solution aura une énergie cinétique plus grande que les forces d’attraction entre les molécules de gaz et les molécules de solvant. Un plus grand nombre de molécules de gaz s’échapperont dans la phase vapeur et la solubilité du gaz diminuera.

36

Chapitre 2 Propriétés des solutions

14.

C’est dans le cas des solutions diluées de gaz qui ne réagissent pas avec le solvant que la loi de Henry s’applique le mieux. NH3 est une base faible et réagit avec l’eau selon la réaction suivante : NH3(aq) + H2O(l) → NH4+(aq) + OH−(aq) O2 se lie à l’hémoglobine dans le sang. À cause de ces réactions avec le solvant, NH3(g) dans l’eau et O2(g) dans le sang ne respectent pas la loi de Henry.

15.

Puisque le soluté est moins volatil que l’eau, on s’attend à ce qu’il y ait un transfert net de molécules d’eau dans le bécher droit plus important que le transfert net de molécules de soluté dans le bécher gauche. Il s’ensuit qu’il y aura un volume de solution plus grand dans le bécher droit lorsque l’équilibre sera atteint, c.-à-d. lorsque la concentration des solutés sera identique dans chaque bécher.

16.

Les solutions de A et de B ont des pressions de vapeur inférieures aux pressions de vapeur d’une solution idéale (voir la figure 2.13 dans le manuel); donc ce graphique présente des déviations négatives par rapport à la loi de Raoult. Les déviations négatives se produisent lorsque les forces intermoléculaires sont plus importantes en solution que dans le solvant et le soluté purs. Il en résulte une enthalpie de dissolution exothermique. Le seul énoncé qui est faux est e. Une substance entre en ébullition quand la pression de vapeur est égale à la pression externe. Étant donné que la solution dont χB = 0,6 a une pression de vapeur inférieure à celle de A pure ou de B pure, à la température du graphique, on s’attendrait à ce que cette solution nécessite la température la plus élevée pour que la pression de vapeur atteigne la pression externe. Par conséquent, la solution dont χB = 0,6 aura une température d’ébullition plus élevée que celle de A pure ou de B pure. (On remarque que parce que P°B > P°A, B est plus volatile que A, et B aura une température d’ébullition inférieure à celle de A). B

B

B

17.

La solution n’est pas idéale, car ΔH ≠ 0. Dans une solution idéale, la force des attractions soluté-solvant est identique à celle combinée des attractions solvant-solvant et solutésoluté, ce qui donne une ΔHsolution = 0. Puisque dans le cas présent ΔHsolution < 0, cette solution présente un écart négatif par rapport à la loi de Raoult et n’est pas idéale.

18.

La partie ionique –SO4– est orientée vers l’eau qui est polaire et la longue chaîne non polaire s’oriente vers les molécules d’hydrocarbures non polaires de l’huile. HHOHHOHHOHHO

© 2007 Les Éditions CEC inc. Toute reproduction interdite

Chapitre 2 Propriétés des solutions

37

Les micelles se forment de sorte que les extrémités ioniques des molécules de détergent, les extrémités SO4−, sont exposées vers l’extérieur aux molécules d’eau polaires, alors que les chaînes d’hydrocabures non polaires des molécules de détergent sont protégées de l’eau en pointant vers l’intérieur de la micelle. La saleté, qui est fondamentalement non polaire, est stabilisée à l’intérieur non polaire de la micelle et emportée au lavage. 19.

La normalité est le nombre d’équivalents par litre de solution. Pour un acide ou une base, un équivalent est la masse de l’acide ou de la base qui peut fournir 1 mol de protons (si c’est un acide) ou accepter 1 mol de protons (si c’est une base). Un proton est un ion H+. La concentration molaire volumique est le nombre de moles de soluté par litre de solution. Lorsque le nombre d’équivalents est égal au nombre de moles de soluté, alors à normalité = la concentration molaire volumique. C’est vrai pour les acides qui ne possèdent qu’un seul proton acide et pour les bases qui n’acceptent qu’un seul proton par unité de formule. Exemples d’acides pour lesquels le nombre d’équivalents = nombre de moles de soluté : HCl, HNO3, HF et CH3CO2H. Exemples de bases pour lesquelles le nombre d’équivalents = nombre de moles de soluté : NaOH, KOH et NH3. Lorsque équivalents ≠ moles de soluté, alors la normalité ≠ de la concentration molaire volumique.C’est vrai pour les acides qui donnent plus d’un proton (H2SO4, H3PO4, H2CO3, etc.) et pour les bases qui réagissent avec plus d’un proton par unité de formule [Ca(OH)2, Ba(OH)2, Sr(OH)2, etc.].

20.

C’est vrai qu’il faut briser le réseau de chlorure de sodium pour qu’il se dissolve dans l’eau, mais beaucoup d’énergie est libérée lorsque les molécules d’eau hydratent les ions Na+ et Cl−. Ces deux processus ont des valeurs relativement élevées d’énergie qui leur sont associées, mais elles sont de signes opposés. Le résultat final est qu’ils s’annulent l’un l’autre, ce qui donne une valeur de ΔHsoln ≈ 0. Donc, la considération d’ordre énergétique n’est pas la seule raison pour laquelle les solides ioniques comme NaCl sont si solubles dans l’eau. La réponse se trouve dans la tendance de la nature à rechercher le désordre dans l’état du mélange. Les processus qui, en général, sont favorisés sont ceux qui aboutissent à une augmentation du désordre parce que l’état désordonné est l’état le plus facile (le plus probable) à atteindre. La tendance des processus à augmenter le désordre sera traitée au chapitre 7 lorsque l’entropie, S, sera abordée.

21.

Seul l’énoncé b est vrai. Lorsqu’on ajoute un soluté à l’eau, la pression de vapeur de la solution à 0 °C est inférieure à la pression de vapeur du solide, et le résultat net est que la glace présente se transforme en liquide afin d’égaliser les pressions de vapeur (ce qui ne peut pas se produire à 0 °C). Il faut une température plus basse pour égaliser la pression de vapeur de l’eau et celle de la glace, d’où l’abaissement du point de congélation. Pour l’énoncé a, la pression de vapeur d’une solution est directement reliée à la fraction molaire du solvant (pas du soluté) par la loi de Raoult. Pour l’énoncé c, les propriétés colligatives dépendent du nombre de particules de soluté présentes et non de la nature du soluté. Pour l’énoncé d, le point d’ébullition de l’eau augmente parce que le soluté (sucre) diminue la pression de vapeur de l’eau ; une température plus élevée est nécessaire pour que la pression de vapeur de la solution soit égale à la pression externe, afin que l’ébullition se produise.

22.

C’est vrai si le soluté se dissout dans le camphre. Le camphre possède les constantes kéb et kcong les plus élevées. Cela signifie que le camphre présente les variations de point d’ébullition et de point de congélation les plus grandes quand on ajoute un soluté. Plus la ΔT est grande, plus la mesure est précise et plus la masse molaire calculée est précise.

© 2007 Les Éditions CEC inc. Toute reproduction interdite

Chapitre 2 Propriétés des solutions

38

Cependant, si le soluté ne se dissout pas dans le camphre, alors ce dernier n’est pas un bon solvant et il faut en choisir un autre qui pourra dissoudre le soluté. 23.

Les solutions isotoniques sont celles qui ont des pressions osmotiques identiques. Crénelure et hémolyse désignent des phénomènes qui se produisent lorsque les globules rouges du sang baignent dans des solutions dont les pressions osmotiques ne sont pas égales à celle de l’intérieur de la cellule. Lorsqu’un globule rouge est dans une solution qui a une pression osmotique supérieure à celle de la cellule, la cellule se contracte parce qu’il y a un net transfert d’eau qui sort de la cellule. C’est la crénelure. L’hémolyse a lieu lorsqu’un globule rouge baigne dans une solution qui a une pression osmotique inférieure à celle de l’intérieur de la cellule. La cellule se rompt alors parce qu’il y a un net transfert d’eau qui entre dans le globule rouge.

24.

La formation de paires d’ions est un phénomène qui se produit en solution lorsque des ions de charges opposées s’agrègent et se comportent comme une particule unique. Par exemple, lorsque NaCl se dissout dans l’eau, on s’attend à ce que le chlorure de sodium existe sous forme d’ions Na+ et Cl− hydratés. Quelques ions, toutefois, restent ensemble sous forme de NaCl dans une solution et se comportent comme une seule particule. La formation de paires d’ions augmente dans une solution à mesure que la concentration des ions augmente (à mesure que la concentration molaire volumique augmente).

EXERCICES Composition des solutions 25.

Étant donné que la masse volumique de l’eau est 1,00 g/mL, 100,0 mL d’eau ont une masse de 100 g. masse volumique =

10,0 g H 3PO 4 + 100 g H 2O masse = = 106 g/mL = 1,06 g/cm3 volume 104 mL

mol H3PO4 = 10,0 g ×

mol H2O = 100 g ×

1 mol = 0,102 mol H3PO4 97,99 g

1 mol = 5,55 mol H2O 18,02 g

fraction molaire de H3PO4 =

0,102 mol H3 PO 4 = 0,0180 (0,102 + 5,55) mol

χ H 2O = 1,000 – 0,0180 = 0,9820

© 2007 Les Éditions CEC inc. Toute reproduction interdite

Chapitre 2 Propriétés des solutions

39

concentration molaire volumique =

molalité =

26.

0,102 mol H3 PO 4 = 0,981 mol/L 0,104 L

0,102 mol H3 PO 4 = 1,02 mol/kg 0,100 kg

C2H6O2 40 % ; r = 1,05 g/mL

m=

mol C 2 H 6O 2 40 g C 2 H 6O 2 mol C 2 H 6O 2 1000 g H 2 O = × × kg H 2 O (100 × 40) g H 2 O 62 g C 2 H 6O 2 kg H 2 O = 11 mol/kg

mol C 2 H 6O 2 = L solution

c=

40 g C 2H 6O 2 1, 05 g solution 1000 mL solution mol C 2 H 6O 2 × × × 100 g solution mL solution L solution 62 g C 2 H 6O 2 c = 6,8 mol/L

molC2 H6O2 =

40 g C 2 H 6O 2 x mol C 2 H 6O 2 = 0,645 mol C2H6O2 62 g C 2 H 6O 2

mol H2O = molH2O =

χ =

27.

(100 × 40)g H 2 O × mol H2O = 3,33 mol H2O 18g H 2 O

0,645 = 0,16 0,645 + 3,33

Acide chlorhydrique : c=

m=

38 g HCl 1,19 g soln 1000 cm3 1 mol HCl × × × = 12 mol/L 3 100 g soln L 36,46 g cm soln 38 g HCl 1000 g 1 mol HCl × × = 17 mol/kg 62 g solvant kg 36,46 g

38 g HCl ×

1 mol 1 mol = 1,0 mol HCl ; 62 g H2O × = 3,4 mol H2O 36,46 g 18,02 g © 2007 Les Éditions CEC inc. Toute reproduction interdite

Chapitre 2 Propriétés des solutions

40

χ HCl =

1,0 = 0,23 3,4 + 1,0

Acide nitrique :

70 g HNO3 1,42 g soln 1000 cm3 1 mol HNO3 c= × × × = 16 mol/L 3 100 g soln L 63,02 g cm soln m=

70 g HNO3 1000 g 1 mol HNO3 × × = 37 mol/kg 30 g solvant kg 63,02 g

70 g HNO3 ×

χ HNO = 3

1 mol 1 mol = 1,1 mol HNO3 ; 30 g H2O × = 1,7 mol H2O 63,02 g 18,02 g

1,1 = 0,39 1,7 + 1,1

Acide sulfurique : c=

95 g H 2 SO4 1,84 g soln 1000 cm3 1 mol H 2 SO4 × × × = 18 mol/L 3 100 g soln L 98,09 g H 2 SO4 cm soln

m=

95 g H 2 SO4 1000 g 1 mol × × = 194 mol/kg ≈ 200 mol/kg 5 g H2 O kg 98,09 g

95 g H2SO4 ×

χ H SO = 2

4

1 mol 1 mol = 0,97 mol H2SO4 ; 5 g H2O × = 0,3 mol H2O 98,09 g 18,02 g

0,97 = 0,76 0,97 + 0,3

Acide acétique : c=

99 g CH3 CO2 H 1,05 g soln 1000 cm3 1 mol × × × = 17 mol/L 3 100 g soln L 60,05 g cm soln

m=

99 g CH3 Cl2 H 1000 g 1 mol × × = 1600 mol/kg ≈ 2000 mol/kg 1 g H2 O kg 60,05 g

© 2007 Les Éditions CEC inc. Toute reproduction interdite

Chapitre 2 Propriétés des solutions

99 g CH3Cl2H ×

1 mol 1 mol = 1,6 mol CH3Cl2H ; 1 g H2O × = 0,06 mol H2O 60,05 g 18,02 g

1,6 = 0,96 1,6 + 0,06

χ CH Cl H = 3

41

2

Ammoniac : c=

m=

28 g NH3 0,90 g 1000 cm3 1 mol × × × = 15 mol/L 3 100 g soln L 17,03 g cm 28 g NH3 1000 g 1 mol × × = 23 mol/kg 72 g H 2 O kg 17,03 g

28 g NH3 ×

χ NH = 3

28.

1 mol 1 mol = 1,6 mol NH3 ; 72 g H2O × = 4,0 mol H2O 1,03 g 18,02 g

1,6 = 0,29 4,0 + 1,6

a) Si on suppose Vsolution ≈ VH2O

masse KCl =

kg H 2 O 1 mL H 2O 2 mol KCl 1, 0 g H 2 O 74,5 g KCl × × × × kg H 2 O mL H 2 O 1000 g H 2 O mL solution mol KCl × 100 mL soln = 15 g

Dissoudre 15 g KCl dans 100 mL d’eau. b) Si on suppose Vsolution ≈ V H 2O

masse NaOH =

15 g NaOH 1 g H 2O 1 mL H 2 O × × × 100 mL solution (100 × 15) g H 2O mL H 2 O mL solution = 18 g NaOH

Dissoudre 18 g NaOH dans 100 mL d’eau.

© 2007 Les Éditions CEC inc. Toute reproduction interdite

Chapitre 2 Propriétés des solutions

42 c) Si on suppose Vsolution ≈ VCH3OH

masse NaOH =

0, 79 g CH 3OH 1 mL CH 3OH 25 g NaOH × × (100 × 25) g CH 3OH mL CH 3OH mL solution × 100 mL solution = 26 g

Dissoudre 26 g NaOH dans 100 mL de CH3OH. d) Utilisons 100 mL (100 g) de H2O. Soit x = mol C6H12O6.

100 g H2O ×

χC H 6

12 O6

1 mol H 2 O = 5,55 mol H2O 18, 02 g

= 0,10 =

0,62 mol C6H12O6 ×

x ; 0,10 x + 0,56 = x, x = 0,62 mol C6H12O6 x + 5,55 180,16 g = 110 g C6H12O6 mol

Dissoudre 110 g C6H12O6 dans 100 mL H2O. 29.

Pour 100,0 mL de vin : 12,5 mL C2H5OH ×

% éthanol =

m= 30.

0,789 g 1,00 g = 9,86 g C2H5OH et 87,5 mL H2O × = 87,5 g H2O mL mL

9,86 g × 100 % = 10,1 % en masse 87,5 g + 9,86 g

9,86 g C2 H5 OH 1 mol × = 2,45 mol/kg 0,0875 kg H 2 O 46,07 g

Pour 1,00 L de solution : 1,37 mol ac. citrique ×

192,12 g = 263 g ac. citrique mol

1,00 × 103 mL solution ×

% ac. citrique =

1,10 g = 1,10 × 103 g solution mL

263 g × 100 % = 23,9% 1,10 × 103 g

Dans 1,00 L de solution, il y a 263 g ac. citrique et (1,10 × 103 – 263) = 840 g H2O. © 2007 Les Éditions CEC inc. Toute reproduction interdite

Chapitre 2 Propriétés des solutions

m=

1,37 mol ac . citrique = 1,6 mol/kg 0,84 kg H 2 O

840 g H2O ×

31.

43

1 mol 1,37 mol = 47 mol H2O ; χac. citrique = = 0,028 18,02 g 47 mol + 1,37 mol

1,00 mol acétone 1 mol = 1,00 mol/kg ; 1,00 × 103 g C2H5OH × 1,00 kg éthanol 46,07 g = 21,7 mol C2H5OH χacétone =

1,00 mol = 0,0441 1,00 mol + 21,7 mol

1 mol CH3COCH3 ×

58,08 g CH3 COCH3 1 mL _ = 73,7 mL CH3COCH3 mol CH3 COCH3 0,788 g

1,00 × 103 g éthanol ×

c=

1 mL = 1270 mL ; volume total = 1270 + 73,7 = 1340 mL 0,789 g

1,00 mol = 0,746 mol/L 1,34 L

© 2007 Les Éditions CEC inc. Toute reproduction interdite

Chapitre 2 Propriétés des solutions

44 Aspects énergétiques des solutions et solubilité 32.

Selon la loi de Hess : KCl(s) → K+(g) + Cl–(g) K (g) + Cl–(g) → K+(aq) + Cl–(aq)

ΔH = –ΔHréseau = –(–715 kJ/mol) ΔH = –ΔHhyd = –684 kJ/mol

+

KCl(s) → K+(aq) + Cl–(aq)

ΔHdiss = 31 kJ/mol

ΔHdiss = chaleur émise ou absorbée quand un soluté se dissout dans un solvant. 33.

a)

CsI(s) → Cs+(g) + I–(g) Cs+(g) + I–(g) → Cs+(aq) + I–(aq)

ΔH = –ΔHréseau = –(–604 kJ) ΔH = ΔHhyd

CsI(s) → Cs(aq) + I–(aq)

ΔHdiss = 33 kJ

33 kJ = 604 kJ + ΔHhyd, ΔHhyd = –571 kJ CsOH(s) → Cs+(g) + OH–(g) Cs+(g) + OH–(g) → Cs+(aq) + OH–(aq) CsOH(s) → Cs+(aq) + OH–(aq)

ΔH = –ΔHréseau = –(–724 kJ) ΔH = ΔHhyd ΔHdiss = –72 kJ

–72 kJ = 724 kJ + ΔHhyd, ΔHhyd = –796 kJ b) L’enthalpie d’hydratation de CsOH étant plus exothermique que celle de CsI, on en déduit que l’anion OH– est plus fortement attiré (hydraté) par l’eau. 34.

Les deux composés, NaOH et Al(OH)3, sont ioniques. Puisque l’énergie de réseau est proportionnelle à la charge, l’énergie de réseau de Al(OH)3 est supérieure à celle de NaOH. L’attraction des molécules d’eau par les ions Al3+ et OH– n’est pas assez grande pour s’opposer à l’énergie de réseau supplémentaire : Al(OH)3 est insoluble.

35.

L’énergie de dissolution d’un solide ionique dans l’eau peut être représentée par l’addition de deux autres énergies, l’énergie de réseau qui correspond à la séparation du composé ionique en ions gazeux (énergie très positive, processus défavorable) et l’énergie d’hydratation des ions gazeux par des molécules d’eau (valeur très négative, processus favorable). Ces deux processus tendant à s’annuler, la chaleur de dissolution d’un solide ionique est soit faiblement négative, soit faiblement positive, et la variation de température est donc minime.

36.

Comme solvant, l’eau apprécie les solutés ioniques et ceux qui sont aptes à faire des liaisons hydrogène. CCl4 est non polaire et dissout de préférence les solutés de même nature. a) Eau ; Cu(NO3)2 est un solide ionique. b) CCl4 : CS2 est une substance non polaire.

© 2007 Les Éditions CEC inc. Toute reproduction interdite

Chapitre 2 Propriétés des solutions

45

c) Eau ; CH3CO2H fait des liaisons hydrogène. d) CCl4 ; bien que le groupe OH puisse faire des liaisons hydrogène, c’est la longue chaîne hydrocarbonée non polaire qui prédomine. e) Eau : HCl réagit avec H2O pour donner les ions H3O+ et Cl– très solubles. f) CCl4 ; C6H6 est non polaire. 37.

À l’état pur, les molécules d’eau polaires s’attirent par la formation de liaisons hydrogène. Comme solvant, l’eau préfère donc les solutés porteurs de charges (ioniques) et ceux qui peuvent faire des liaisons hydrogène. a) CH3CH2OH parce qu’il peut faire des liaisons hydrogène avec l’eau. b) CHCl3 parce qu’il est polaire ; cependant CHCl3 n’est que faiblement soluble dans l’eau parce qu’il n’est pas ionique et ne peut faire de liaisons hydrogène. c) CH3CH2OH parce que CH3(CH2)14CH2OH est en majeure partie non polaire.

38.

Pour les composés ioniques, à mesure que la charge des ions augmente et/ou que la taille des ions diminue, l’attraction de l’eau (hydratation) augmente. a) Mg2+; taille plus petite, charge plus élevée

b) Be2+; plus petit

c) Fe3+; taille plus petite, charge plus élevée

d) F-; plus petit

e) Cl-; plus petit 39.

f) SO42-; charge plus élevée

À mesure que la chaîne hydrocarbonée s’allonge, la solubilité diminue. Le groupement –OH peut former des liens hydrogène avec l’eau. La chaîne hydrocarbonée, elle, est non polaire et ne forme pas de lien avec l’eau. À mesure que la chaîne hydrocarbonée s’allonge, l’importance de la partie non polaire devient de plus en plus grande, et l’effet du groupement –OH diminue. La solubilité diminue.

40.

L’acide benzoïque peut former des liaisons hydrogène. Dans le benzène, il forme un dimère grâce à des liaisons hydrogène avec lui-même. O

H

O

C

C O

H

O

Le dimère est non polaire et, par conséquent, il est plus soluble dans le benzène non polaire que dans l’eau. L’acide benzoïque est plus soluble dans NaOH 0,1 mol/L que dans l’eau parce que OH– réagit ainsi :

© 2007 Les Éditions CEC inc. Toute reproduction interdite

Chapitre 2 Propriétés des solutions

46 C6H5CO2H + OH– → C6H5CO2– + H2O

En devenant l’anion C6H5CO2–, l’acide benzoïque est plus soluble dans l’eau.

41.

c = kP,

8,21×10-4 mol = k × 0,790 atm, k = 1,04 × 10 −3 mol/L atm L

c = kP, c =

1,04 ×10-4 mol × 1,10 atm = 1.14 × 10 −3 mol/L L atm

Pressions de vapeur des solutions 42.

mol C3H8O3 = 164 g ×

mol H2O = 338 mL ×

PH2O = χ H2O PHo2O = 43.

1 mol = 1,78 mol C3H8O3 92,09 g 0,992 g 1 mol × = 18,6 mol H2O mL 18,02 g

18,6 mol × 54,74 torr = 0,913 × 54,74 torr = 50,0 torr (1,78 + 18,6) mol

PB = χ B PBo , χ B = PB / PBo = 0,900 atm/0,930 atm = 0,968 0,968 =

mol benz è ne 1 mol ; mol benzène = 78,11 g C6H6 × = 1,000 mol mol totales 78,11 g

Soit x = mol soluté, alors : χB = 0,968 = B

1,000 mol , 0,968 + 0,968 x = 1,000, 1,000 + x x = 0,033 mol

masse molaire =

44.

10,0 g = 303 g/mol ≈ 3,0 × 102 g/mol 0,033 mol

19,6 torr = χ H2 O (23,8 torr), χ H2 O = 0,824 ; χsoluté = 1,000 – 0,824 = 0,176 0,176 est la fraction molaire de toutes les particules de soluté présentes, c’est-à-dire des ions Na+ et Cl– présents. La fraction molaire de NaCl est 1/2 (0,176) = 0,088. À 45 °C, P H2O = 0,824 (71,9 torr) = 59,2 torr

© 2007 Les Éditions CEC inc. Toute reproduction interdite

Chapitre 2 Propriétés des solutions

45.

a) 25 mL C5H12 ×

45 mL C6H14 ×

47

0,63 g 1 mol = 0,22 mol C5H12 × mL 72,15 g 0,66 g 1 mol = 0,34 mol C6H14 ; total mol = 0,22 + 0,34 × mL 86,17 g = 0,56 mol

χ Lpen =

mol pentane en solution 0,22 mol L = 0,39, χ hex = 1,00 – 0,39 = 0,61 = mol totales en solution 0,56 mol

L o Ppen = χ pen Ppen = 0,39(511 torr) = 2,0 × 102 torr ; P hex = 0,61(150, torr) = 92 torr

2 P totale = P pen + P hex = 2,0 × 10 + 92 = 292 torr = 290 torr

b) Selon la loi des pressions partielles de Dalton, la pression d’un gaz est proportionnelle au nombre de moles de ce gaz en présence.

χ Vpen =

mol pentane dans la vapeur 2, 0 × 10 2 torr P = pen = = 0,69 mol totales (vapeur) 290 torr P totale

Remarque : on ajoute le symbole V ou L au symbole χ quand on doit traiter des deux phases. Si V ou L sont absents, c’est qu’il s’agit de la seule phase liquide.

46.

Ptotale = Pméthanol + Ppropanol ; P = χ P°;

χ méthanol = χ propanol =

1, 00 mol = 0,500 2, 00 mol

Ptotale = 0,500 (303 torr) + 0,500 (44,6 torr) = 152 + 22,3 = 174 torr V Dans la vapeur : χ méthanol =

Pméthanol P totale

=

152 torr V = 0,874 ; χ propanol = 1,000 – 0,874 174 torr = 0,126

47.

o ; pour la vapeur, χ A = PA/Ptotale. Ptol = χ tol Ptolo ; Pben = χ ben Ppen

L

L

V

Comme χ ben = χ tol , alors Ptol = Pben. V

V

L L L o o = (1,00 – χ tol ) Ppen , χ tol (28 torr) = (1,00 – χ tol ) 95 torr χ Ltol Ptolo = χ Lben Ppen

123 χ tol = 95, χ tol = 0,77 ; χ ben = 1,00 – 0,77 = 0,23 L

L

L

© 2007 Les Éditions CEC inc. Toute reproduction interdite

Chapitre 2 Propriétés des solutions

48 48.

La pression de vapeur de la solution d est supérieure à celle de l’eau pure puisque le méthanol est plus volatil que l’eau : " " " " P solution = PH2O χ H2O + PCH3OH χ CH3OH où PCH3OH > PH2O

Les solutions b et c ont des pressions de vapeur inférieures à celle de l’eau pure. " P solution = PH2O χ H2O où χ H2O < 1

Toutefois, le glucose est un non-électrolyte, et NaCl est dissous pour donner Na+ et Cl–. Puisqu’il y a davantage de particules, la pression de vapeur de la solution c est plus petite que celle de la solution b. 49.

La solution d), (voir la réponse de l’exercice 48).

50.

50,0 g CH3COCH3 ×

50,0 g CH3OH ×

L = χ acétone

1 mol = 0,861 mol acétone 58,08 g

1 mol = 1,56 mol méthanol 32,04 g

0,861 L L = 0,356 ; χ méthanol = 1, 000 × χ acétone = 0,644 0,861 + 1,56

Ptotale = Pméthanol + Pacétone = 0,644(19,1 kPa) + 0,356(36,1 kPa) = 12,3 kPa + 12,9 kPa = 25,2 kPa

En phase vapeur : V = χ acétone

Pacétone P totale

=

96,5 torr V = 0,512 ; χ méthanol = 1,000 – 0,512 = 0,488 188, 6 torr

La pression de vapeur réelle de la solution (21,5 kPa) étant moindre que celle calculée théoriquement (25,2 kPa), il y a donc un écart négatif par rapport à la loi de Raoult impliquant que les molécules de méthanol et d’acétone s’attirent davantage en solution que séparément à l’état pur.

© 2007 Les Éditions CEC inc. Toute reproduction interdite

Chapitre 2 Propriétés des solutions 51.

49

a) La pression de vapeur d’une solution idéale à n’importe quelle fraction molaire de H2O se situerait entre celle du propanol pur et celle de l’eau pure (entre 74,0 torr et 71,9 torr). Les pressions de vapeur des solutions ne se situent pas entre ces limites, donc l’eau et le propanol ne forment pas des solutions idéales. b) À partir des données, les pressions de vapeur des diverses solutions sont supérieures à celle de la solution idéale (déviation positive à la loi de Raoult). Cela se produit lorsque les forces intermoléculaires dans la solution sont plus faibles que les forces intermoléculaires dans le solvant pur et le soluté pur. Cela donne naissance à des valeurs de ΔHdiss endothermiques (positives). c) Les forces d’interaction entre les molécules de propanol et les molécules d’eau sont plus faibles que celles entre les substances pures étant donné que cette solution présente une déviation positive par rapport à la loi de Raoult. d) À χ H2O = 0,54, la pression de vapeur est la plus élevée en comparaison des autres

solutions. Étant donné qu’une solution bout lorsque sa pression de vapeur est égale à la pression externe, la solution dont χ H 2O = 0,54 devrait avoir le point d’ébullition normal le plus bas; cette solution aura une pression de vapeur égale à 1 atm à une température plus faible que les autres solutions.

Propriétés colligatives 52.

molalité = m =

mol soluté 27, 0 g N 2 H 4 CO 1000 g 1 mol N 2 H 4 CO = × × kg solvant 150, 0 g H 2 O kg 60, 06 g N 2 H 4 CO = 3,00 mol/kg

ΔTéb = kébm = 0,51 °C·kg/mol × 3,00 mol/kg = 1,5 °C Le point d’ébullition passe de 100,0 °C à 101,5 °C (en supposant que P = 1 atm).

53.

ΔTéb = 77,85 °C − 76,50 °C = 1,35 °C; m =

mol de biomolécule = 0,0150 kg solvant ×

ΔTéb 1,35 °C = = 0,268 mol/kg 5,03 °C ⋅ kg/mol kéb

0, 268 mol hydrocarbure = 4,02 × 10 −3 mol kg solvant

D’après les données du problème, 2,00 g de biomolécule utilisés contiennent 4,02 × 10 −3 mol de biomolécule. La masse molaire de la biomolécule est :

2,00 g = 498 g/mol 4,02 ×10-3 mol

© 2007 Les Éditions CEC inc. Toute reproduction interdite

Chapitre 2 Propriétés des solutions

50

54.

ΔT = 25,50 °C – 24,59 °C = 0,91 °C = kcm, m =

0,91 ° C = 0,10 mol/kg 9,1 ° C⋅ kg/mol

⎛ 0,10 mol H 2 O ⎞ ⎛ 18,02 g H 2 O ⎞ ⎟ = 0,018 g H2O ⎟ ⎜ ⎝ kg t-butanol ⎠ ⎝ mol H 2 O ⎠

masse H2O = 0,0100 kg t-butanol ⎜

55.

m=

40,0 g C2 H6 O2 1000 g 1 mol × × = 10,7 mol/kg 60,0 g H 2 O kg 62,07 g

ΔTc = kcm = 1,86 °C·kg/mol × 10,7 mol/kg = 19,9 °C ; Tc = 0,0 °C – 19,9 °C = –19,9 °C ΔTéb = kébm = 0,51 °C·kg/mol × 10,7 mol/kg = 5,5 °C ; Téb = 100,0 °C + 5,5 °C = 105,5 °C

56.

m=

ΔTcong kcong

=

30, 0 ° C = 16,1 mol C2H6O2/kg 1,86 ° C⋅ kg/mol

mol C2H6O2 = 15,0 L H2O ×

1, 00 kg H 2 O 16,1 mol C 2 H 6O 2 = 242 mol C2H6O2 × L H 2O kg H 2 O

volume C2H6O2 = 242 mol C2H6O2 ×

ΔTéb = kébm =

57.

62,07 g 1 cm3 × = 13,500 cm3 = 13,5 L mol C2 H6 O2 1,11 g

0,51 ° C⋅ kg × 16,1 mol/kg = 8,2 °C ; Téb = 100,0 °C + 8,2 °C = 108,2 °C mol

ΔTcong = kcongm, m =

ΔTcong kcong

=

0,300°C 5,86 × 10-2 mol thyroxine = kg benzène 5,12 °C ⋅ kg/mol

Le nombre de moles de thyroxine présentes est : 5,86 × 10-2 mol thyroxine = 5,86 × 10 −4 mol thyroxine 0,0100 kg benzène × kg benzène D’après les données du problème, 0,455 g de thyroxine a été utilisé , ce qui correspond à 5,86 × 10 −4 mol thyroxine. La masse molaire de la thyroxine est : masse molaire =

0,455 g

5,86 ×10-4 mol

= 776 g/mol

© 2007 Les Éditions CEC inc. Toute reproduction interdite

Chapitre 2 Propriétés des solutions 58.

51

Masse de la formule empirique ≈ 7(12) + 4(1) + 16 = 104 g/mol ΔTcong = kcongm, m =

ΔTcong k cong

=

22,3 ° C = 0,56 mol/kg 40 ° C⋅ kg/mol

mol anthraquinone = 0,0114 kg solvant ×

masse molaire =

0,56 mol anthraquinone = 6,4 × 10–3 mol kg solvant

1,32 g = 210 g/mol 6,4 × 10_ 3 mol

masse molaire 210 = 2,0 ; formule moléculaire = C14H8O2 = masse formule empirique 104 59.

a) Protéine : prot Solution : sln m=

1,0 g prot L sln

L sln 1000 mL sln

mL sln 1,0 g sln

mol prot 90 000 g prot 1000 g sln (1000 - 1) g H 2 O

1000 g H 2 O kg H 2 O

m = 1,1 × 10–5 mol/kg

ΔTcong =

1,86 ° Ckg 1,1 x 10-5 mol − = 2,1 × 10–5 °C mol kg

–π = cRT =

1, 0 g prot mol prot 8,315 L⋅ kPa × × 298 K − L sln 90 000 g prot K ⋅ mol

π = 0,028 kPa = 28 Pa b) Un changement de 2,1 × 10-5 °C est difficile à mesurer. Un changement de 28 Pa est plus facile à mesurer avec précision. Il est donc préférable de déterminer la masse molaire des grosses molécules à l’aide de la pression osmotique.

© 2007 Les Éditions CEC inc. Toute reproduction interdite

Chapitre 2 Propriétés des solutions

52

1 atm π 760 torr c= = 4,0 × 10 −5 mol/L = 0,08206 L ⋅ atm RT × 300 K mol ⋅ K 0,74 torr ×

60.

4,0 ×10-5 mol 1,00 L × = 4,0 × 10 −5 mol catalase L masse molaire =

61.

π = cRT, c =

62.

c=

π RT

π RT

10,00 g = 2,5 × 105 g/mol -5 4,0 ×10 mol =

8,00 atm = 0,327 mol/L 0,08206 L ⋅ atm × 298 K mol ⋅ K

= 1520 kPa ×

K⋅ mol 1 = 0,62 mol/L × 8,315 Pa⋅ L 295 K

Cette concentration représente le total des moles de particules présentes en solution. Comme NaCl se dissocie en deux ions, Na+ et Cl–, la concentration en NaCl sera

0,62 2

mol/L = 0,31 mol/L. masse NaCl = 1,0 L ×

0,31 mol NaCl 58,44 g NaCl = 18 g NaCl × mol NaCl L

Dissoudre 18 g de NaCl dans une fiole jaugée de 1 litre et compléter avec de l’eau.

Propriétés des solutions d’électrolytes 63.

2+ − Na3PO4(s) → 3 Na+(aq) + PO34 (aq ), i = 4,0; CaBr2(s) → Ca (aq) + 2 Br (aq), i = 3,0

KCl(s) → K+(aq) + Cl−(aq), i = 2,0. Les concentrations de particules effectives des solutions sont : 4,0(0,010 mol/kg) = 0,040 mol/kg pour la solution de Na3PO4; 3,0(0,020 mol/kg) = 0,060 mol/kg pour la solution de CaBr2; 2,0(0,020 mol/kg) = 0,040 mol/kg pour la solution de KCl; légèrement supérieure à 0,020 mol/kg pour la solution de HF étant donné que HF ne se dissocie que partiellement dans l’eau (c’est un acide faible). a) La solution de Na3PO4 0,010 mol/kg et la solution de KCl 0,020 mol/kg ont toutes les deux des concentrations de particules effectives de 0,040 mol/kg (en supposant une dissociation

© 2007 Les Éditions CEC inc. Toute reproduction interdite

Chapitre 2 Propriétés des solutions

53

complète), de sorte que ces deux solutions doivent avoir la même température d’ébullition que la solution de C6H12O6 0,040 mol/kg (un non-électrolyte). b) P = χP°; à mesure que la concentration du soluté diminue, la pression de vapeur du solvant augmente étant donné que χ augmente. Par conséquent, la solution de HF 0,020 mol/kg aura la pression de vapeur la plus élevée étant donné que cette solution possède la concentration de particules effective la plus petite. c) ΔT = kcongm; la solution de CaBr2 0,020 mol/kg possède la concentration de particules effective la plus grande de sorte qu’elle aura l’abaissement du point de congélation le plus grand (ΔT plus grande). 64.

En nombre de particules de soluté par kg de solvant, ces systèmes se classent ainsi : eau < solution de C6H12O6 < solution de NaCl < solution de CaCl2 a) Eau pure. b) Solution de CaCl2. c) Solution de CaCl2. d) Eau pure. e) Solution de CaCl2.

65.

Tcong = Tcong (solvant) – ΔTcong = Tcong (solvant) – (kcongm) a) Tcong = 0,00 °C –

1000g H 2 O 1,86 °C ⋅ kg 5,0 g NaCl mol NaCl 2 mol ions × × × × mol 25 g H 2 O 58,5 g NaCl mol NaCl kg H 2 O Tcong = 0,00 – (13) = –13 °C

b) Tcong = 0,00 °C –

2,0 g Al(NO3 ) 2 1,86 °C×kg × mol 15 g H 2 O ×

mol Al(NO3 )3 1000 g H 2 O 4 mol ions × × 213 g Al(NO3 )3 mol Al(NO3 )3 kg H 2 O

Tcong = 0,00 – (4,7) = –4,7 °C

© 2007 Les Éditions CEC inc. Toute reproduction interdite

Chapitre 2 Propriétés des solutions

54 66.

La pression exercée doit être plus grande que la pression osmotique de l’eau de mer. π (eau de mer) = cRT π=

67.

0,5 mol NaCl 2 mol ions 8,315 kPa ⋅ L × × × 298 K = 2,5 × 103 kPa L solution mol NaCl K ⋅ mol

i = 2,63 ; i = 2,60 et i = 2,57.

ΔTcong = ikcongm, i =

ΔTcong

kcong m

=

0,110 ° C = 2,63 pour CaCl2 1,86 ° C⋅ kg/mol × 0, 0225 mol/kg

0,0225 mol/kg

68.

i=

0,440 = 2,60 pour CaCl2 0,0910 mol/kg 1,86 × 0,0910

i=

1,330 = 2,57 pour CaCl2 0,278 mol/kg 1,86 × 0,278

a) En supposant que la solubilité de CaCl2 soit indépendante de la température, la molalité d’une solution saturée est :

1 mol CaCl2 6, 71 mol CaCl2 74,5 g CaCl 2 1000 g × × = 100, 0 g H 2 O kg 110,98 g CaCl2 kg H 2 O ΔTcong = ikcongm = 3,00 × 1,86 °C·kg/mol × 6,71 mol/kg = 37,4 °C En supposant que i = 3,00, une solution saturée de CaCl2 a un point de fusion de – 37,4 °C. CaCl2 peut donc faire fondre de la glace à –34 °C. b) Selon l’exercice 67, i ≈ 2,6; ΔTcong = ikcongm = 2,6 × 1,86 × 6,71 = 32 °C ; Tcong = –32 °C

Si i = 2,6, CaCl2 ne peut faire fondre de la glace à –34 °C.

© 2007 Les Éditions CEC inc. Toute reproduction interdite

Chapitre 2 Propriétés des solutions

55

EXERCICES SUPPLÉMENTAIRES 69.

a) NH4NO3(s) → NH4+(aq) + NO3−(aq) ΔHsoln = ?

Gain de chaleur dû au processus de dissolution = perte de chaleur par la solution; on garde toutes les quantités positives afin d’éviter les erreurs de signes. Étant donné que la température de l’eau a diminué, la dissolution de NH4NO3 est endothermique (ΔH est positif). Masse de solution = 1.60 + 75.0 = 76.6 g. perte de chaleur par la solution =

ΔHsoln =

4,18 J × 76,6 g × (25,00°C – 23,34°C) = 532 J g °C

80,05g NH 4 NO3 532 J = 2,66 × 104 J/mol = 26,6 kJ/mol × 1,60g NH 4 NO3 mol NH 4 NO3

b) Nous utiliserons la loi de Hess pour trouver l’énergie de réseau. L’équation pour l’énergie de réseau est :

NH4+(g) + NO3−(g) → NH4NO3(s)

ΔH = énergie de réseau

NH4+(g) + NO3−(g) → NH4+(aq) + NO3−(aq) ΔH = ΔHhyd = -630 kJ/mol + − NH4 (aq) + NO3 (aq) → NH4NO3(s) ΔH = -ΔHsoln = -26,6 kJ/mol ____________________________________________________________________ NH4+(g) + NO3−(g) → NH4NO3(s) ΔH = ΔHhyd - ΔHsoln = -657 kJ/mol 70.

Les forces intermoléculaires augmentent graduellement : des forces de London pour l’hexane non polaire aux ponts hydrogène pour le méthanol et l’eau. De plus, deux solvants qui se suivent sont miscibles l’un dans l’autre puisqu’ils ne sont pas très différents.

71.

a) L’eau bout lorsque la pression de vapeur est égale à la pression atmosphérique au-dessus de l’eau. Dans une casserole ouverte Patm ≈ 1,0 atm. Dans une marmite à pression, Pint > 1,0 atm, et l’eau bout à une température plus élevée. Plus la température de cuisson est élevée, plus la cuisson est rapide; b) Le sel se dissout dans l’eau et forme une solution dont le point de fusion est plus bas que celui de l’eau pure (ΔTcong = kcongm). Cela se produit dans l’eau à la surface de la glace. Si la température n’est pas trop froide, la glace fond; c) Lorsque l’eau gèle à partir d’une solution, elle gèle sous forme d’eau pure, ce qui laisse une solution de sel plus concentrée; d) Le point triple est supérieur à 1 atm de sorte que CO2(g) est la phase stable à 1 atm et à la température ambiante. CO2(l) ne peut pas exister à la pression atmosphérique normale. Par conséquent, la glace sèche sublime au lieu de bouillir. Dans un extincteur, P > 1 atm et CO2(l) peuvent exister. Lorsque CO2 sort d’un extincteur, CO2(g) se forme comme prévu d’après le diagramme de phase;

© 2007 Les Éditions CEC inc. Toute reproduction interdite

Chapitre 2 Propriétés des solutions

56

e) L’ajout d’un soluté à un solvant augmente le point d’ébullition et diminue le point de congélation du solvant. Par conséquent, le solvant est liquide sur une plus grande gamme de températures quand un soluté y est dissous. 72.

Parce que les pressions partielles sont proportionnelles au nombre de moles de gaz V présentes, alors χ CS = PCS2 / Ptot . 2 V PCS2 = χ CS Ptot = 0,855 (263 torr) = 225 torr 2

L L = PCS2 = χ CS P o , χ CS 2 CS2 2

73.

π = cRT =

0,1mol L

×

PCS2 o CS2

P

=

225 torr = 0,600 375 torr

0,08206 L ⋅ atm × 298 K = 2,45 atm ≈ 2 atm mol ⋅ K

760 mm Hg ≈ 2000 mm ≈ 2 m atm

π = 2 atm ×

La pression osmotique pourrait supporter une colonne de mercure ≈ 2 m. La hauteur d’une colonne de liquide dans un arbre sera plus élevée parce que Hg est plus dense que la sève d’un arbre. Si on suppose que le liquide dans un arbre est surtout de l’eau, alors le liquide a une masse volumique ≈1,0 g/cm3. La masse volumique de Hg est de 13,6 g/cm3. Hauteur du liquide ≈ 2 m × 13,6 ≈ 30 m 74.

Dans 100,00 g de produit, il y a : 31,57 g C ×

5,30 g H ×

1 mol C = 2,629 mol C ; 12,01 g

2,629 = 1,000 2,629

1 mol H 5,26 = 5,26 mol H ; = 2,00 mol H/mol C 1,008 g 2,629

63,13 g O ×

1 mol O = 3,946 mol O ; 16,00 g

3,946 = 1,501 mol O/mol C 2,629

Formule empirique : C2H4O3 ; utilisons l’abaissement du point de congélation pour calculer la masse molaire. m=

ΔTcong

k cong

=

5, 20 ° C = 2,80 mol/kg 1,86 ° C⋅ kg/mol

© 2007 Les Éditions CEC inc. Toute reproduction interdite

Chapitre 2 Propriétés des solutions

mol soluté = 0,0250 kg ×

masse molaire =

57

2,80 mol soluté = 0,0700 mol soluté kg

10,56 g = 151 g/mol 0,0700 mol

Masse de la formule empirique C2H4O3 = 76,05 g/mol. Comme la masse molaire est environ le double de celle de la formule empirique, alors la formule moléculaire est C4H8O6 et sa masse molaire est 152,10 g/mol. 75.

Le système tend à équilibrer les concentrations. a) L’eau passe dans le compartiment de gauche : le niveau de liquide monte dans la branche de gauche et baisse dans la branche de droite ; b) Les ions Na+ et Cl- peuvent traverser la membrane : après un certain temps, la concentration devient la même dans les deux compartiments, et les niveaux demeurent égaux.

76.

Dans le cas idéal, NaCl se dissocie totalement et i = 2,00. ΔTcong = ikcongm; 1,28 °C = 2 × 1,86 °C⋅kg/mol × m, m = 0,344 mol NaCl/kg H2O Supposons 1,00 kg d’eau : 0,344 mol NaCl ×

20,1 g 58,44 g = 20,1 g NaCl ; % NaCl = × 100 % 1, 00 × 103g + 20,1 g mol = 1,97

77.

ΔT 2,79o C ΔT = kcongm, m = = 1,50 mol/kg = kcong 1,86o C ⋅ kg/mol a) ΔT = kébm, ΔT = (0,51

o b) Peau = χ eau Peau , χ eau =

°C ⋅ kg °C/mol/kg)(1,50 mol/kg) = 0,77 °C, Téb = 100,77 °C mol mol H 2 O mol H 2 O + mol soluté

En supposant 1,00 kg d’eau, on a 1,50 mol de soluté et : mol H2O = 1,00 × 103 g H2O ×

1 mol H 2 O = 55,5 mol H2O 18,02 g H 2 O

© 2007 Les Éditions CEC inc. Toute reproduction interdite

Chapitre 2 Propriétés des solutions

58

χeau =

55,5 mol = 0,974; Peau = (0,974)(23,76 mm Hg) = 23,1 mm Hg 1,50 + 55,5

c) On suppose un comportement idéal dans la formation de la solution et on suppose i = 1 (il ne se forme pas d’ions).

PROBLÈMES DÉFIS 78.

Pour la deuxième vapeur recueillie, χ B,V 2 = 0,714 et χ T,V 2 = 0,286, où χ B,L 2 = fraction molaire du benzène dans la deuxième solution et χ T,L 2 = fraction molaire du toluène dans la deuxième solution. χ B,L 2 + χ T,L 2 = 1,000

χ

V B, 2

χ B,L 2 (750,0 torr) PB PB = = 0,714 = = L PTOT PB + PT χ B, 2 (750,0 torr) + (1,000 − χ B,L 2 )(300, 0 torr)

En résolvant : χ B,L 2 = 0,500 = χ T,L 2 Cette deuxième solution vient de la vapeur recueillie à partir de la première solution (initiale). Donc, χ B,V 1 = χ T,V 1 = 0,500. Soit χ B,L 1 = fraction molaire du benzène dans la première solution et χ T,L 1 = fraction molaire du toluène dans la première solution. L χ B,L 1 + χ T,1 = 1,000

χ B,V 1 = 0,500 =

χ B,L 1 (750,0 torr) PB PB = = L L PTOT PB + PT )(300, 0 torr) χ B, 1 (750,0 torr) + (1, 000 − χ B,1

En résolvant : χ B,L 1 = 0,286 Dans la solution de départ, χB = 0,286 et χT = 0,714. B

79.

Pour 30 % de A en moles dans la vapeur, 30 =

0,30 =

PA × 100 : PA + PB

χA x χA x , 0,30 = χA x + χB y χ A x + (1, 00 − χ A ) y

χAx = 0,30(χAx) + 0,30y – 0,30χAy, χAx – 0,30χAx + 0,30χAy = 0,30y χA(x – 0,30x + 0,30y) = 0,30y, χA =

0,30 y ; χB = 1,00 - χA 0, 70 x + 0,30 y

© 2007 Les Éditions CEC inc. Toute reproduction interdite

B

Chapitre 2 Propriétés des solutions

59

De la même manière, si la vapeur au-dessus de la solution est à 50 % de A :

χA =

y y ; χ B = 1, 00 − x+ y x+ y

Si la vapeur est à 80 % de A : χA =

0,80 y ; χB = 1,00 - χA 0, 20 x + 0,80 y B

Si la solution liquide est à 30 % de A en moles, χA = 0,30. Donc, χ AV =

80.

PA 0,30 x 0,30 x et χ BV = 1, 00 − = PA + PB 0,30 x + 0, 70 y 0,30 x + 0, 70 y

Si la solution est à 50 % de A : χ AV =

x et χ BV = 1, 00 − χ AV x+ y

Si la solution est à 80 % de A : χ AV =

0,80 x et χ BV = 1, 00 − χ AV 0,80 x + 0, 20 y

m=

ΔT 0,406 o C = 0,218 mol/kg = 1,86 o C ⋅ kg/mol kcong

π = cRT où c = mol/L; on doit supposer que la concentration molaire volumique = la molalité afin de calculer la pression osmotique. C’est une hypothèse logique pour les solutions diluées lorsque 1,00 kg d’eau ≈ 1,00 L de solution. En supposant que la dissociation de NaCl est complète, une solution 0,218 mol/kg correspond à 6,37 g de NaCl dissous dans 1,00 kg d’eau. Le volume de la solution peut être un peu supérieur à 1,00 L, mais pas beaucoup (à trois C.S.). La supposition selon laquelle la concentration molaire volumique = molalité est acceptable ici.

π = (0,218 mol/L)(0,08206 L·atm/K·mol)(298 K) = 5,33 atm 81.

ΔT 0,426 o C = m= = 0,229 mol/kg 1,86 o C ⋅ kg/mol kcong Soit x = mol NaCl dans le mélange et y = mol C12H22O11. NaCl(aq) → Na+(aq) + Cl−(aq); en solution, NaCl existe sous forme d’ions Na+ et Cl− séparés. 0,229 mol = mol Na+ + mol Cl− + mol C12H22O11 = x + x + y = 2x + y La masse molaire de NaCl est 58,44 g/mol et la masse molaire de C12H22O11 = 342,3 g/mol. On établit une autre équation pour la masse du mélange :

© 2007 Les Éditions CEC inc. Toute reproduction interdite

Chapitre 2 Propriétés des solutions

60

20,0 g = x(58,44) + y(342,3) On substitue : 20,0 = 58,44 x + (0,229 – 2x) 342,3 On résout :

x = mol NaCl = 0,0932 mol et y = mol C12H22O11 = 0,229 – 2(0,0932) = 0,043 mol

masse NaCl = 0,0932 mol × 58,44 g/mol = 5,45 g NaCl % en masse NaCl =

5,45 g × 100 = 27,3 % 20,0 g

% en masse C12H22O11 = 100,0 – 27,3 = 72,7 %

χsaccharose =

82.

0,043 mol = 0,32 (0,043 + 0,0932) mol

a) π = icRT, ic =

π RT

=

7,83 atm = 0,320 mol/L 0,08206 L ⋅ atm × 298 K mol ⋅ K

Soit 1,000 L de solution : nombre total de moles de particules de soluté = mol Na+ + mol Cl− + mol NaCl = 0,320 mol masse de solution = 1000 mL ×

1,071g = 1071 g de solution mL

masse de NaCl dans la solution = 0,0100 × 1071 g = 10,7 g NaCl mol NaCl ajoutées à la solution = 10,7 g ×

1 mol = 0,183 mol NaCl 58,44 g

Une partie de ce NaCl se dissocie en Na+ et en Cl− (deux moles d’ions par mole de NaCl) et une partie reste non dissociée. Soit x = mol de NaCl non dissocié = mol paires d’ions. mol de particules de soluté = 0,320 mol = 2(0,183 - x) + x 0,320 = 0,366 - x, x = 0,046 mol de paires d’ions fraction de paires d’ion =

0, 046 = 0,25, ou 25 % 0,183

© 2007 Les Éditions CEC inc. Toute reproduction interdite

Chapitre 2 Propriétés des solutions

61

b) ΔT = kcongm où kcong = 1,86 °C·kg/mol; d’après la partie a, 1,000 L de solution contient 0,320 mol de particules de soluté. Pour calculer la molalité de la solution, il faut connaître le nombre de kg de solvant présent dans 1,000 L de solution.

masse de 1,000 L de solution = 1071 g; masse de NaCl = 10,7 g masse de solvant dans 1,000 L de solution = 1071 g – 10,7 g = 1060 g ΔT = 1,86 °C·kg/mol ×

0,320 mol = 0,562 °C 1,060 kg

En supposant que l’eau gèle à 0,000 °C, alors Tcong = -0,562 °C.

83.

χ Vpen = 0,15 =

P pen L " ; P pen = χ pen Ppen = χ Lpen (511 torr) P totale

L L P totale = P pen + P hex = χ pen (511) + χ hex (150)

Puisque χ hex = 1,000 – χ pen , alors : P totale = L

L

χ Lpen (511) + (1, 000 − χ Lpen )(150) = 150 + 361 χ Lpen χ

V pen

χ Lpen (511) P pen L L = , 0,15 = , 0,15 (150 + 361 χ pen ) = 511 χ pen L 150 + 361 χ pen P totale

23 + 54 χ pen = 511 χ pen , χ pen = L

84.

a) 1,32 °C =

M=

L

L

23 = 0,050 457

5,12 ° C⋅ kg ⎛ 1,22 g X 1000 g benzène 1 ⎞ x x ⎜ ⎟ mol kg benzène M⎠ ⎝ 15,60 g benzène

303 g (0,04 x 303) ± = 303 ± 9 g/mol mol 1,32

b) Le résultat expérimental se situe entre 303 – 9 = 294 g/mol et 303 + 9 = 312 g/mol. On ne peut donc exclure la possibilité que la masse molaire soit de 299 g/mol et que ce soit de la codéine. c) En utilisant une concentration plus grande, l’abaissement du point de congélation sera plus grand et l’incertitude relative sur Δtc sera plus petite : une concentration 10 fois plus grande amènerait une incertitude d’environ 1 g/mol. On pourrait aussi utiliser un solvant

© 2007 Les Éditions CEC inc. Toute reproduction interdite

Chapitre 2 Propriétés des solutions

62

dont la constante cryoscopique est plus grande, l’abaissement du point de congélation serait plus grand pour une même concentration, et l’incertitude relative plus petite.

85.

ΔTcong = 5,51 °C – 2,81 °C = 2,70 °C; m =

ΔTcong kcong

=

2,70°C = 0,527 mol/kg 5,12 °C ⋅ kg/mol

Soit x = masse de naphthalène (masse molaire = 128,2 g/mol). Donc, 1,60 - x = masse d’anthracène (masse molaire = 178,2 g/mol).

x 1, 60 − x = mol napthalène et = mol anthracène 178, 2 128, 2 x 1, 60 − x + 0,527 mol soluté 128, 2 178, 2 = , 1,05 × 10 −2 0, 0200 kg solvant kg solvant =

178,2 x + 1,60(128,2) - 128,2 x 128,2 (178,2)

50,0 x + 205 = 240, 50,0 x = 35, x = 0,70 g naphthalène Donc le mélange est :

0,70 g × 100 = 44 % en masse de naphthalène et 56 % en masse 1,60 g

d’anthracène . 86.

ic =

π RT

=

0,3950 atm = 0,01614 mol/L = concentration totale des ions 0,08206 L ⋅ atm × 298,2 K mol ⋅ K

0,01614 mol/L = cMg2+ + cNa + + cCl- ; cCl- = 2 cMg 2+ + cNa + (charges équilibrées) En combinant : 0,01614 = 3 cMg2+ + 2 cNa + Soit x = masse de MgCl2 et y = masse de NaCl, alors x + y = 0,5000 g.

cMg2+ =

x y et cNa + = (Parce que V = 1,000 L.) 95, 21 58, 44

Concentration totale des ions =

3x 2y + = 0,01614 mol/L; 95, 21 58, 44

on réarrange : 3 x + 3,258 y = 1,537

© 2007 Les Éditions CEC inc. Toute reproduction interdite

Chapitre 2 Propriétés des solutions

63

En résolvant le système d’équations : 3 x + 3,258 y = 1,537 + y) = -3(0,5000) -3 (x __________________________________________ 0,258 y = 0,037, y = 0,14 g NaCl masse de MgCl2 = 0,5000 g – 0,14 g = 0,36 g; % en masse MgCl2 =

0,36 g × 100 0,5000 g = 72 %

87.

HCO2H → H+ + HCO2-; l’ionisation à 4,2 % produit les quantités suivantes de H+ et HCO2- :

4,2 × 0,10 mol/L = 0,0042 mol/L 100 La concentration totale des espèces présentes est donc : cHCO2 H + cH+ + c HCO −2 = 0,10 + 0,0042 + 0,0042 = 0,11 mol/L Puisque c ≈ m, alors : ΔTcong = kcongm = 1,86 °C·kg/mol × 0,11 mol/kg = 0,20 °C ; Tcong = –0,20 °C

ΔTcong = kcongm = 0,51 °C·kg/mol × 0,11 mol/kg = 0,056 °C ; Téb = 100,056 °C 88.

a) Les valeurs moyennes pour chaque ion sont :

300 mg Na+; 15,7 mg K+; 5,45 mg Ca2+; 388 mg Cl−; 246 mg lactate, C3H5O3− Remarque : Étant donné qu’il est possible de peser avec une précision de ± 0,1 mg sur une balance analytique, on gardera des chiffres sugnificatifs supplémentaires et on calculera les résultats à ± 0,1 mg. La seule source de lactate est NaC3H5O3. 246 mg C3H5O3− ×

112,06 mg NaC3 H 5O3 _

89,07 mg C3 H 5O3

= 309,5 mg lactate de sodium

La seule source de Ca2+ est CaCl2·2H2O. 5,45 mg Ca2+ ×

147,0 mg CaCl2 •2H 2 O = 19,99 ou 20,0 mg CaCl2·2H2O 40,08 mg Ca 2+

© 2007 Les Éditions CEC inc. Toute reproduction interdite

Chapitre 2 Propriétés des solutions

64

La seule source de K+ est KCl. 15,7 mg K+ ×

74,55 mg KCl = 29,9 mg KCl 39,10 mg K +

D’après ce qu’on a utilisé déjà, calculons la masse de Na+ ajoutée. 309,5 mg lactate de sodium = 246,0 mg lactate + 63,5 mg Na+ Donc, il faut ajouter encore 236,5 mg Na+ pour obtenir les 300 mg désirés. 236,5 mg Na+ ×

58,44 mg NaCl = 601,2 mg NaCl 22,99 mg Na +

Maintenant, vérifions la masse de Cl− ajoutée : 20,0 mg CaCl2·2H2O ×

70,90 mg Cl= 9,6 mg Cl− 147,0 mg CaCl2 •2H 2 O

20,0 mg CaCl2·2H2O = 9,6 mg Cl− 29,9 mg KCl – 15,7 mg K+ = 14,2 mg Cl− 601,2 mg NaCl – 236,5 mg Na+ = 364,7 mg Cl− _______________________________________ Cl− total = 388,5 mg Cl− C’est la quantité de Cl− qu’on veut (la quantité moyenne de Cl−). Une balance analytique permet de peser à 0,1 mg près. On utilise 309,5 mg de lactate de sodium, 20,0 mg de CaCl2·2H2O, 29,9 mg de KCl et 601,2 mg de NaCl. b) Pour obtenir la gamme de la pression osmotique, il faut calculer la concentration molaire de chaque ion à leurs valeurs maximales et minimales. Aux concentrations maximales, on a :

285 mg Na + 1 mmol 14,1 mg K + 1 mmol = 0,124 mol/L; = 0,00361 mol/L × × 100 mL 22,99 mg 100 mL 39,10 mg 4,9 mg Ca 2+ 1 mmol 368 mg Cl- 1 mmol = 0,0012 mol/L; = 0,104 mol/L × × 100 mL 40,08 mg 100 mL 35,45 mg 231 mg C3 H 5O3- 1 mmol = 0,0259 mol/L × 100 mL 89,07 mg (Remarque : concentration molaire volumique = mol/L = mmol/mL.) Concentration totale = 0,124 + 0,00361 + 0,0012 + 0,104 + 0,0259 = 0,259 mol/L © 2007 Les Éditions CEC inc. Toute reproduction interdite

Chapitre 2 Propriétés des solutions

π = cRT =

65

0,259 mol 0,08206 L ⋅ atm × 310 K = 6,59 atm × L mol ⋅ K

De la même manière, aux concentrations maximales, la concentration de chaque ion est : K+: 0,00442 mol/L Na+: 0.137 mol/L 2+ Cl−: 0,115 mol/L Ca : 0,0015 mol/L C3H5O3−: 0,0293 mol/L La concentration totale de tous les ions est la somme, soit 0,287 mol/L.

0,287 mol 0,08206 L ⋅ atm × 310 K = 7,30 atm × L mol ⋅ K

π=

La pression osmotique varie de 6,59 atm à 7,30 atm.

89.

a) En supposant que MgCO3 ne se dissocie pas, sa concentration est :

560 μg MgCO 3(s) 560 mg 560 × 10−3 g 1 mol MgCO 3 × = = mL L L 84,32 g = 6,6 × 10–3 mol MgCO3/L Une pression de 8,0 atm purifiera l’eau jusqu’à une concentration de : c=

π RT

=

8,0 atm 0,32 mol = 0,08206 L ⋅ atm/mol ⋅ K × 300 K L

Quand la concentration de MgCO3 atteint cette valeur, l’appareil d’osmose ne peut plus purifier l’eau. Soit V = volume d’eau demeurant après purification de 45 L d’eau. À ce moment, les particules de soluté seront : 6,6 × 10–3 mol/L × (45 L + V) = 0,32 mol/L × V En résolvant, on obtient V = 0,96 L Le volume minimal d’eau à traiter est 45 L + 0,96 L = 46 L. Remarque : si MgCO3 se dissocie en Mg2+ et CO32–, la concentration en particules devient 1,3 × 10–2 mol/L et au moins 47 L d’eau doivent être traités. b) Non, car cet appareil ne peut purifier par osmose inverse que des solutions contenant moins de 0,32 mol de particules par litre de solution. Or la concentration en particules de l’eau de mer



0,60 mol NaCl 2 mol ions × ≈ 1,20 mol ions/L L mol NaCl

© 2007 Les Éditions CEC inc. Toute reproduction interdite

Chapitre 2 Propriétés des solutions

66 PROBLÈMES D’INTÉGRATION 90.

10,0 mL ×

1L 10 dL 1,0 mg 1g 1 mol × × × × 1000 mL 1L 1 dL 1000 mg 113,14 g = 8,8 × 10 −7 mol C4H7N3O

masse de sang = 10,0 mL ×

molalité =

8,8 × 10-7 mol = 8,5 × 10 −5 mol/kg 0,0103 kg

π = cRT, c =

8,8 × 10-7 mol = 8,8 × 10 −5 mol/L 0,0100 L

π = 8,8 × 10 −5 mol/L ×

91.

1,025 g = 10,3 g mL

ΔT = imkcong, i =

0,08206 L ⋅ atm × 298 K = 2,2 × 10 −3 atm K ⋅ mol

2,79 o C ΔT = = 3,00 0,250 mol 1,86o C ⋅ kg mkcong × 0,500 kg mol

On a 3 ions dans les solutions et il y a deux fois plus d’anions que de cations. Par conséquent, la formule de Q est MCl2. Soit 100,00 g de composé : 38,68 g Cl ×

1 mol Cl = 1,091 mol Cl 35,45 g

mol M = 1,091 mol Cl ×

masse molaire de M =

92.

1 mol M = 0,5455 mol M 2 mol Cl

61,32 g M = 112,4 g/mol; M est Cd. Donc Q = CdCl2. 0,5455 mol M

masse de H2O = 160 mL ×

mol NaDTZ = 0,159 kg ×

0,995 g = 159 g = 0,159 kg mL 0,378 mol = 0,0601 mol kg

© 2007 Les Éditions CEC inc. Toute reproduction interdite

Chapitre 2 Propriétés des solutions

masse molaire de NaDTZ =

67

38,4 g = 639 g/mol 0,0601 mol

Psoln = χ H 2O PHo2O ; mol H2O = 159 g ×

1 mol = 8,82 mol 18,02 g

Le diatrizoate de sodium est un sel parce qu’il y a un métal (sodium) dans le composé. À partir de la notation en abrégé du diatrizoate de sodium, NaDTZ, on peut supposer que ce sel se sépare en ions Na+ et DTZ−. Le nombre de moles de particules de soluté est donc 2(0,0601) = 0,120 mol de particules de soluté.

χH O = 2

8,82 mol = 0,987; Psoln = 0,987 × 34,1 torr = 33,7 torr 0,120 mol + 8,82 mol

PROBLÈME DE SYNTHÈSE

93.

a) D’après les informations de la partie a, on peut calculer la masse molaire de NanA et en déduire la formule.

mol NanA = mol de l’agent réducteur = 0,01526 L ×

0,02313 mol L = 3,530 × 10 −4 mol NanA

30,0 × 10-3 g = 85,0 g/mol masse molaire de NanA = 3,530 × 10-4 mol Pour déduire la formule, on suppose différentes charges et différents nombres d’oxygène présents dans l’oxanion, puis, à l’aide du tableau périodique, on examine si un élément correspond aux données de la masse molaire. En supposant n = 1, la formule est donc NaA. La masse molaire de l’oxanion, A−, est 85,0 – 23,0 = 62,0 g/mol. La partie oxanion de la formule peut être EO− ou EO2− ou EO3− où E est un élément quelconque. Si l’oxanion est EO−, alors la masse molaire de E est 62,0 – 16,0 = 46,0 g/mol; aucun élément ne possède cette masse molaire. Si la formule de l’oxanion est EO2−, la masse molaire de E = 62,0 – 32,0 = 30,0 g/mol. La masse molaire du phosphore est près de cette valeur, mais les anions PO2− ne sont pas courants. Si la formule de l’oxanion est EO3−, la masse molaire de E est = 62,0 – 48,0 = 14,0. L’azote possède cette masse molaire et les anions NO3− sont très courants. Par conséquent, NO3− est une formule possible pour A−. Ensuite, on suppose les formules Na2A et Na3A et on procède de la même manière. Dans tous les cas, aucun élément du tableau périodique ne correspond aux données. Par conséquent, on suppose que l’oxanion est NO3− = A−.

© 2007 Les Éditions CEC inc. Toute reproduction interdite

Chapitre 2 Propriétés des solutions

68

b) Les données critallographiques de la partie b permettent de déterminer la nature du métal, M, dans la formule. Voir le chapitre 8 de Chimie générale pour une révision des relations dans les réseaux à mailles cubiques centrées. Dans une maille élémentaire bcc, il y a 2 atomes par maille élémentaire et la diagonale de la maille cubique est reliée au rayon du métal par l’équation 4r = 3 ℓ où ℓ = longueur de l’arête du cube.

ℓ=

4r 4(1,984 × 10−8 cm) = = 4,582 × 10 −8 cm 3 3

volume de la maille élémentaire = ℓ 3 = (4,582 × 10 −8 )3 = 9,620 × 10 −23 cm3 masse de M dans une maille élémentaire = 9,620 × 10 −23 cm3 ×

5,243 g cm3

= 5,044 × 10 −22 g M mol de M dans une maille élémentaire = 2 atomes ×

1 mol 6,022 ×1023 atomes = 3,321 × 10 −24 mol M

masse molaire de M =

5,044 ×10-22 g M = 151,9 g/mol 3,321×10-24 mol M

D’après le tableau périodique, M est l’europium, Eu. Étant donné que la charge de Eu est +3, la formule du sel est alors Eu(NO3)3·zH2O. c) Les données de la partie c permettent de déterminer la masse molaire de Eu(NO3)3·zH2O, à partir de laquelle on peut déterminer z, le nombre de molécules d’eau d’hydratation.

⎛ 1 atm ⎞ 558 torr ⎜ ⎟ π ⎝ 760 torr ⎠ π = icRT, ic = = = 0,0300 mol/L 0,08206 L ⋅ atm RT × 298 K mol ⋅ K La concentration molaire volumique totale des particules de soluté est 00300 mol/L. Les particules de soluté sont les ions Eu3+ et NO3− (les molécules d’eau d’hydratation ne sont pas des particules de soluté). Puisque chaque mol de Eu(NO3)3·zH2O qui se dissous forme 4 ions (Eu3+ + 3 NO3−), la concentration molaire volumique de Eu(NO3)3·zH2O est 0,0300/4 = 0,00750 mol/L. mol Eu(NO3)3·zH2O = 0,01000 L ×

0,00750 mol = 7,50 × 10 −5 mol L

© 2007 Les Éditions CEC inc. Toute reproduction interdite

Chapitre 2 Propriétés des solutions

69

masse molaire de Eu(NO3)3·zH2O =

33,45 ×10-3 g = 446 g/mol 7,50 ×10-5 mol

446 g/mol = 152,0 + 3(62,0) + z(18,0), z(18,0) = 108, z = 6,00 La formule de l’électrolyte fort est Eu(NO3)3·6H2O.

© 2007 Les Éditions CEC inc. Toute reproduction interdite

CHAPITRE 3 CINÉTIQUE CHIMIQUE QUESTIONS 9.

Étape monomoléculaire : réaction dans laquelle n’intervient qu’une seule molécule. Étape bimoléculaire : réaction dans laquelle intervient la collision de deux molécules. La probabilité qu’il y ait collision simultanée de trois molécules possédant l’énergie et l’orientation appropriées est excessivement faible.

10.

Il faut ajouter de l’énergie pour que la réaction puisse démarrer, c’est-à-dire pour vaincre la barrière de potentiel de l’énergie d’activation. À température ambiante, les molécules H2 et O2 n’ont pas une énergie cinétique suffisante pour initier la réaction.

11.

Tous ces choix influent sur la vitesse de réaction, mais seuls b et c influent sur la vitesse en changeant la valeur de k. Selon l’équation d’Arrhenius, la valeur de k dépend de la température et de l’énergie d’activation : k = Ae-Ea/RT. Or, un catalyseur change la valeur de E a.

12.

Une des méthodes expérimentales pour déterminer les équations de vitesse est la méthode des vitesses initiales. On effectue plusieurs expériences à des concentrations différentes de réactifs et on détermine la vitesse initiale dans chaque cas. On compare ensuite les résultats pour établir comment la vitesse initiale dépend des concentrations initiales. Cela permet de déterminer la forme de l’équation de vitesse. Une fois que les ordres sont connus, on peut déterminer la valeur de la constante de vitesse. La deuxième méthode expérimentale utilise le fait que les équations de vitesse intégrées peuvent être transformées en équation d’une droite. Les données de la concentration en fonction du temps sont recueillies pour un réactif pendant que la réaction a lieu. Ces données sont ensuite transformées et portées en graphique pour voir quelle transformation donne une droite. À partir de la droite, on obtient l’ordre par rapport aux réactifs et la pente de la droite est mathématiquement reliée à k, la constante de vitesse.

13.

La vitesse moyenne diminue avec le temps, parce que la réaction inverse se produit à une fréquence plus élevée à mesure que la concentration des produits augmente. Au départ, à défaut de produits, la vitesse de la réaction directe est à un maximum ; mais à mesure que le temps passe, la vitesse diminue de plus en plus étant donné que les produits sont reconvertis en réactifs. La seule vitesse qui est constante, c’est la vitesse initiale. Au temps t ≈ 0, la quantité de produits est négligeable et la vitesse de la réaction ne dépend que de la vitesse de la réaction directe.

14.

La méthode la plus courante pour déterminer expérimentalement la loi de vitesse différentielle est la méthode des vitesses initiales. Une fois que l’équation de vitesse différentielle est déterminée expérimentalement, on peut écrire l’équation de vitesse intégrée. Cependant, il est parfois plus commode et plus précis de recueillir les données de concentration en fonction du temps pour un réactif. Quand c’est le cas, on « met à

Chapitre 3 Cinétique chimique

72

l’épreuve » les courbes pour déterminer l’équation de vitesse intégrée. Une fois qu’elle est déterminée, on peut déterminer l’équation de vitesse différentielle. L’une ou l’autre des méthodes expérimentales permet la détermination de l’équation de vitesse intégrée et de l’équation de vitesse défférentielle; quelle équation de vitesse est déterminée expérimentalement et laquelle est dérivée de l’autre est habituellement décidé selon les données qui sont les plus faciles et les plus précises à recueillir.

15.

⎛ [A] ⎞ vitesse 2 k[A]2x = = ⎜ 2⎟ x k[A]1 vitesse1 ⎝ [A]1 ⎠

x

La vitesse double lorsque la concentration quadruple : 2 = (4)x, x = 1/2 L’ordre est de 1/2 (la racine carrée de la concentration du réactif). Pour un réactif dont l’ordre est -1 et dont la concentration est doublée:

vitesse 2 1 = (2)-1 = vitesse1 2 Pour une réaction d’ordre -1, la vitesse diminue d’un facteur ½ lorsque la concentration du réactif est doublée . 16.

Une réaction catalysée par un métal est dépendante du nombre de sites d’absorption à la surface de ce métal. Une fois que la surface du métal est saturée par le réactif, la vitesse de réaction devient indépendant de la concentration.

17.

La collision doit mettre en jeu suffisamment d’énergie pour que la réaction ait lieu, c.-à-d. que l’énergie de la collision doit être égale ou supérieure à l’énergie d’activation. L’orientation relative des réactifs lors de la collision doit permettre la formation des liaisons nouvelles nécessaires à la formation des produits.

18.

La pente de la droite ln k en fonction de 1/T(K) est égale à -Ea/R. Étant donné que Ea pour la réaction catalysée sera inférieure à Ea pour la réaction non catalysée, la pente de la droite catalysée sera moins négative.

EXERCICES Vitesses de réaction 19.

Les coefficients dans la réaction équilibrée relient la vitesse de disparition des réactifs à la vitesse de production des produits. D’après la réaction équilibrée, la vitesse de production de P4 sera égale à ¼ de la vitesse de disparition de PH3, et la vitesse de production de H2 sera égale à 6/4 de la vitesse de disparition de PH3. Par convention, toutes les vitesses sont données en valeurs positives.

© 2007 Les Éditions CEC inc. Toute reproduction interdite

Chapitre 3 Cinétique chimique

Vitesse = -

73

Δ [ PH 3 ] (0,0048 mol/2,0 L) = Δt s

= 2,4 ×

10 −3 mol/L·s

Δ[P4 ] 1 Δ[PH 3 ] = 2,4 × 10 −3 /4 = 6,0 × 10 −4 mol/L·s = Δt 4 Δt Δ[H 2 ] 6 Δ[PH 3 ] = 6(2,4 × 10 −3 )/4 = 3,6 × 10 −3 mol/L·s = Δt 4 Δt

20.

1 Δ[ NH 3] 1 Δ[H 2] =2 Δt 3 Δt Δ[ NH 3] 2 Δ[H 2] = Δt 3 Δt

21.

a) vitesse moyenne de décomposition de H2O2

=

-Δ[H 2O 2 ] -(0,500 mol/L - 1,000 mol/L) = = 2,31 × 10 −5 mol/L·s Δt (2,16 ×104 s - 0)

D’après les coefficients de l’équation équilibrée :

Δ[O 2 ] 1 Δ[H 2 O 2 ] = = 1,16 × 10 −5 mol/L·s Δt 2 Δt b)

-Δ[H 2 O 2 ] -(0,250 - 0,500) mol/L = = 1,16 × 10 −5 mol/L·s Δt (4,32 ×104 - 2,16 ×104 ) s Δ[O 2 ] = 1/2 (1,16 × 10 −5 ) = 5,80 × 10 −6 mol/L·s Δt On remarque que la vitesse moyenne diminue avec le temps dans une réaction.

22.

0,0120/0,0080 = 1,5; le réactif B est consommé 1,5 fois plus vite que le réactif A. Cela correspond à un rapport molaire de 3 pour 2 entre B et A dans l’équation équilibrée. 0,0160/0,0080 = 2; le produit C est formé 2 fois plus vite que le réactif A est consommé. Donc, le coefficient de C est le double du coefficient de A. Une équation équilibrée possible est : 2A + 3B → 4C

23.

a) mol/L⋅s b) v = k ; k a les mêmes unités que la vitesse, mol/L⋅s.

© 2007 Les Éditions CEC inc. Toute reproduction interdite

Chapitre 3 Cinétique chimique

74

2 mol L L x = d) v = k⋅[ ] ; k : 2 L⋅ s mol⋅ s mol

mol L 1 x = c) v = k [ ] ; k : L⋅ s mol s

e) v = k⋅[ ]3 ; k : 24.

2

3 2 mol x L 3 = L2 L⋅ s mol mol ⋅ s

v = k[Cl2]1/2 [CHCl3] k:

1/ 2 1/ 2 mol L x L 1/ 2 x = L1 / 2 L⋅ s mol mol mol ⋅ s

Équations de vitesse obtenues à partir de données expérimentales : méthode des vitesses initiales 25.

a) v = k [NO]x [Cl2]y y

0,18 k (0,10) x (0,10) y ⎛1⎞ , 0,50 = ⎜ ⎟ , y = 1 = x y 0,36 k (0,10) (0,20) ⎝2⎠ 0,36 k (0,10) x (0,20)1 , 0,25 = = 1,45 k (0,20) x (0,20)1

x

⎛1⎞ ⎜ ⎟ ,x=2 ⎝2⎠

v = k [NO]2 [Cl2] b) k =

2 v 1, 45 mol L L × × × 2 2 2 L⋅ min 0, 20 mol [NO] [Cl] (0, 20) mol

k = 1,8 × 102 L2/mol2⋅min

26.

a) v = k [I–]x [S2O82–]y

12,5 × 10 -6 k (0,080) x(0,040) y = , 2,00 = 2,0x, x = 1 6,25 × 10 -6 k (0,040) x(0,040) y 12,5 × 10 -6 k (0,080)1(0,040) y , 2,00 = (2)y, y = 1 = y 1 -6 6,25 × 10 k (0,080) (0,020) v = k [I–] [S2O82–]

© 2007 Les Éditions CEC inc. Toute reproduction interdite

Chapitre 3 Cinétique chimique

75

v 12,5 x 10 -6 mol L L b) k = = x x 2[I ] [S2O8 ] L⋅s 0,080 mol 0,040 mol k = 3,91 × 10–3 L/mol⋅s vinitiale (mol/L⋅s)

12,5 × 10–6 6,25 × 10–6 6,25 × 10–6 5,00 × 10–6 7,00 × 10–6

k (L/mol⋅s)

3,91 × 10–3 3,91 × 10–3 3,91 × 10–3 3,91 × 10–3 3,89 × 10–3

kmoyen = 3,91 × 10–3 L/mol⋅s

Si on garde le bon nombre de chiffres significatifs, 3,9 × 10–3 L/mol⋅s. 27.

a) v = k [NOCl]x

5,98 × 10 4 k (3,0 × 1016) x , 2,25 = (1,5)x = 4 16 x 2,66 × 10 k (2,0 × 10 ) log 2,25 = x log 1,5 x=2 v = k [NOCl]2 b) k =

5,98 × 104 molécules (cm 3) 2 v = x 2 2 [NOCl]2 (3,0 × 1016 ) (molécules) cm3 ⋅ s

k = 6,6 × 10–29 cm3/molécules⋅s

6,6 × 10-29 cm 3 6,02 × 1023 molécules L c) × × molécules ⋅ s mol 1000 cm3 k = 4,0 × 10–8 L/mol⋅s 28.

a) Vitesse = k[Hb]x[CO]y; en comparant les deux premières expériences, [CO] ne varie pas, [Hb] double et la vitesse double. Par conséquent, x = 1 et la réaction est d’ordre un par rapport à Hb. Si on compare la deuxième et la troisième expériences, [Hb] est constante, [CO] triple et la vitesse triple. Par conséquent, y = 1 et la réaction est d’ordre un par rapport à CO. b) Vitesse = k[Hb][CO]

© 2007 Les Éditions CEC inc. Toute reproduction interdite

Chapitre 3 Cinétique chimique

76 c) D’après la première expérience :

0,619 µmol/L·s = k(2,21 µmol/L)(1,00 µmol/L), k = 0,280 L/µmol·s La deuxième et la troisième expériences donnent des valeurs de k similaires, de sorte que kmoyen = 0,280 L/µmol·s. d) Vitesse = k[Hb][CO] =

0,280 L 3,36 μ mol 2,40 μ mol = 2.26 µmol/L·s × × μ mol ⋅ s L L

Équations de vitesse intégrées 29.

35 0 0 0

Supposons que la réaction soit d’ordre 1. Si tel est le cas, un graphique de ln[H2O2] en fonction du temps donnera une ligne droite. Si tel n’est pas le cas, alors la réaction n’est pas d’ordre 1. temps (s) 0 120 300 600 1200 1800 2400 3000 3600

[H2O2] (mol/L) 1,00 0,91 0,78 0,59 0,37 0,22 0,13 0,082 0,050

ln [H2O2] –0,000 –0,094 –0,25 –0,53 –0,99 –1,51 –2,04 –2,50 –3,00

Comme le graphique de ln[H2O2] en fonction du temps est linéaire, la réaction est d’ordre 1. Sa loi de vitesse est : v = k[H2O2] et sa loi de vitesse intérgrée : ln[H2O2] = –kt + ln[H2O2]. La valeur de k est fournie par la pente du graphique. pente = –k =

Δy 0 - (-3,00) = = –8,3 × 10–4 s–1, k = 8,3 × 10–4 s–1 0 - 3600 s Δx

Pour calculer [H2O2] à 4000 s, on utilise l’équation de vitesse intégrée où t = 0, [H2O2]0 = 1,00 mol/L ln[H2O2] = –kt + ln[H2O2]0 ou ln

ln

[ H 2 O2] = –kt [H 2 O2 ]0

[ H 2 O2] = –8,3 × 10–4 s–1 × 4000 s, ln[H2O2] = –3,3, [H2O2] = e–3,3 = 0,037 mol/L 1

© 2007 Les Éditions CEC inc. Toute reproduction interdite

Chapitre 3 Cinétique chimique 30.

77

a) Puisque le graphique de ln[A] en fonction du temps est linéaire, c’est que la réaction est d’ordre 1. La pente du graphique = –k. Donc, la loi de vitesse, la loi de vitesse intégrée et la valeur de la constante de vitesse sont : v = k[A] ; ln[A] = –kt + ln[A]0 ; k = 6,90 × 10–2 s–1 b) Temps de demi-réaction : t1/2 =

ln 2 0, 6931 0,6931 , t1/2 = = 10,0 s = 6,90 × 10-2s -1 k k

c) Quand une réaction est complétée à 87,5 % (ou qu’il en reste 12,5 % à faire), c’est qu’elle a duré trois demi-vies :

1,00 % → 50,0 % t1/2 t1/2



25 % t1/2



12,5 % ; t = 3 × t1/2 = 3 × 10,0 s = 30,0 s

On peut aussi utiliser la loi de vitesse intégrée, où [A] = 0,125 [A]0.

⎛ [A] ln ⎜⎜ ⎝ [A ]o

⎛ 0,125 [A ]o ⎞ ⎟ = –kt, ln ⎜ ⎜ [A ] ⎟ o ⎝ ⎠

⎞ ln (0,125) ⎟ = – (6,90 × 10–3 s–1) t, t = ⎟ - 6,90 × 10_ 3 s _1 ⎠ = 30,1 s

31.

Si la réaction est d’ordre 1, ln[NO2] = f(t) sera linéaire. Si elle est d’ordre 2,

1 [ NO2]

= f(t) sera linéaire.

temps

[NO2] (mol/L)

0 1,20 × 103 3,00 × 103 4,50 × 103 9,00 × 103 1,80 × 104

0,500 0,444 0,381 0,340 0,250 0,174

ln [NO2] –0,693 –0,812 –0,965 –1,079 –1,386 –1,749

1/[NO2] (L/mol) 2,00 2,25 2,62 2,94 4,00 5,75

© 2007 Les Éditions CEC inc. Toute reproduction interdite

Chapitre 3 Cinétique chimique

78

Le graphique 1/[NO2] en fonction du temps étant linéaire, la réaction est d’ordre 2. Ses équations de vitesse sont : v = k[NO2]2 et

1 [ NO 2]

= kt +

1 [ NO 2]o

La pente du graphique donne la valeur de k. pente = k =

Δy (5,75 × 2,00)L/mol = = 2,08 × 10–4 L/mol⋅s -4 (1,80 × 10 - 0)s Δx

Pour calculer [NO2] à 2,70 × 104 s, utilisons la loi de vitesse intégrée où 1/[NO2]0 = 1/0,500 mol/L = 2,00 L/mol.

1 [ NO 2] 1 [ NO2] 32.

= kt +

1 [ NO 2]o

,

1 [ NO 2]

=

2,08 × 10-4 L × 2,70 × 104 s + 2,00 L/mol mol ⋅ s

= 7,62, [NO2] = 0,131 mol/L

a) Étant donné que le graphique de [C2H5OH] en fonction du temps est linéaire, la réaction est d’ordre zéro par rapport à C2H5OH. La pente du graphique de [C2H5OH] en fonction du temps est égale à -k. Par conséquent, l’équation de vitesse, l’équation de vitesse intégrée et la valeur de la constante de vitesse sont: vitesse = k[C2H5OH]0 = k; [C2H5OH] = -kt + [C2H5OH]o; k = 4,00 × 10 −5 mol/L·s. b) L’expression de la demi-vie pour une réaction d’ordre zéro est: t1/2 = [A]o/2k.

[C2 H 5OH]o 1,25 ×10-2 mol/L t1/2 = = 156 s = 2k 2 × 4,00 ×10-5 mol/L ⋅ s Note : on aurait pu utiliser l’équation de vitesse intégrée pour trouver t1/2 où [C2H5OH] = (1,25 × 10 −2 /2) mol/L.

© 2007 Les Éditions CEC inc. Toute reproduction interdite

Chapitre 3 Cinétique chimique

79

c) [C2H5OH] = -kt + [C2H5OH]o , 0 mol/L = -(4,00 × 10 −5 mol/L·s)t + 1,25 × 10 −2 mol/L t=

33.

1,25 ×10-2 mol/L = 313 s 4,00 ×10-5 mol/L ⋅ s

D’après les données, la pression de C2H5OH diminue à une vitesse constante de 13 torr à chaque 100 s. Étant donné que la vitesse de disparition de C2H5OH ne dépend pas de la concentration, la réaction est d’ordre zéro par rapport à C2H5OH.

13 torr 1 atm = 1,7 × 10 −4 atm/s × 100 s 760 torr

k=

La loi de vitesse et la loi de vitesse intégrée sont :

⎛ 1 atm ⎞ ⎟ = -kt + 0,329 atm ⎝ 760 torr ⎠

Vitesse = k = 1,7 × 10 −4 atm/s; PC2 H5OH = -kt + 250 torr ⎜

À 900 s : PC2 H5OH = -1,7 × 10 −4 atm/s × 900 s + 0,329 atm = 0,176 atm = 0,18 atm = 130 torr 34.

Étant donné que 1/[A] en fonction du temps est linéaire avec une pente positive, la réaction est d’ordre deux par rapport à A. L’ordonnée à l’origine dans le graphique est égal à 1/[A]o. Si on prolonge la droite, l’ordonnée à l’origine sera environ 10, donc 1/10 = 0,1 mol/L = [A]o.

35.

La pente du graphique de 1/[A] en fonction du temps dans l’exercice 3.34 avec k égale à pente = k =

a)

(60 - 20) L/mol = 10 L/mol·s (5 - 1) s

1 1 10 L 1 = 100, [A] = 0,01 mol/L = kt + = × 9s+ [A] [A]o mol ⋅ s 0,1 M

b) Pour une réaction d’ordre 2, la demi-vie dépend de la concentration : t1/2 =

Première demi-vie : t1/2 =

1 10 L 0,1 mol × mol ⋅ s L

1 k [A]o

=1s

Deuxième demi-vie : ([A]o est maintenant 0,05 mol/L) : t1/2 = 1/(10 × 0.05) = 2 s Troisième demi-vie ([A]o est maintenant 0,025 mol/L): t1/2 = 1/(10 × 0,025) = 4 s

© 2007 Les Éditions CEC inc. Toute reproduction interdite

Chapitre 3 Cinétique chimique

80 36.

a) [A] = –kt + [A]0, [A] = –(5,0 × 10–2 mol/L⋅s)t + 1,0 × 10–3 mol/L b) La demi-vie est : t1/2 =

t1/2 =

[A]o 2k

1,0 ×10-3 mol/L = 1,0 × 10–2 s -2 2 × 5,0 × 10 mol/L ⋅ s

c) [A] = –5,0 × 10–2 mol/L⋅s × 5,0 × 10–3 s + 1,0 × 10–3 mol/L = 7,5 × 10–4 mol/L

S’il demeure 7,5 × 10–4 mol/L de A, alors 2,5 × 10–4 mol/L ont réagi et 2,5 × 10–4 mol/L de B ont été produits. 37.

0,693

k=

t 1/2

=

0,693 = 0,0485 jour–1 14,3 jours

⎛ [A ]o ⎞ ln ⎜ ⎟ = kt ⎝ [A] ⎠ Si l’échantillon est désintégré à 95 %, [A] = 0,05 [A]o. t=

⎛ [A ]o ⎞ [A]o 1 1 1 ⎛ 1 ⎞ × × = ln ⎜ = ln ⎜ ⎟× ⎟ −1 −1 [A] k ⎝ 0, 05 ⎠ 0, 0485 j ⎝ 0, 05[A ]o ⎠ 0, 0485 jour

t = 62 jours 39.

En comparant les expériences 1 et 2, si on double la concentration de AB, la vitesse initiale augmente d’un facteur 4. La réaction est d’ordre deux par rapport à AB. Vitesse = k[AB]2, 3,20 × 10−3 mol/L ⋅ s = k1(0,200 mol/L)2 k = 8,00 × 10-2 L/mol ⋅ s = kmoyen

Pour une réaction d’ordre deux : t1/2 = 40.

1 1 = = 12,5 s -2 k[AB]o 8,00 ×10 L/mol ⋅ s × 1,00 mol/L

Comme les temps de demi-réaction successifs augmentent avec le temps, on en déduit qu’il s’agit d’une réaction d’ordre 2. t1/2 =

1 1 1 = ,k= = 1,0 L/mol⋅min k[A]o 10, 0 min (0,10 mol/L) t 1/2[A]o

© 2007 Les Éditions CEC inc. Toute reproduction interdite

Chapitre 3 Cinétique chimique

a)

81

1 1 1,0 L 1 = kt + × 80,0 min + = 90 L/mol, [A] = [A] [A ]o mol⋅ min 0,10 mol/L = 1,1 × 10–2mol/L

b) 30,0 min = 2 temps de demi-réaction, alors il reste 25 % de [A]o.

[A] = 0,25 (0,10 mol/L) = 0,025 mol/L 41.

Étant donné que [B]o>>[A]o, la concentration de B est essentiellement constante au cours de l’expérience, de sorte que la vitesse = k′[A] où k′ = k[B]2. Pour cette expérience, la réaction est d’ordre un apparent par rapport à A.

⎛ 3,8 ×10-3 mol/L ⎞ ⎛ [A] ⎞ ⎟ ⎜ a) ln ⎜ = -k′t, ln ⎜ = -k′× 8,0 s, k′ = 0,12 s −1 ⎟ -2 ⎟ ⎝ [A ]o ⎠ ⎝ 1,0 ×10 mol/L ⎠ Pour la réaction : k′ = k[B]2, k = 0,12 s −1 /(3,0 mol/L)2 = 1,3 × 10 −2 L2/mol2·s b) t1/2 =

ln 2 0, 693 = = 5,8 s k ' 0,12 s-1



⎞ [A] [A] −1 = e −0,12(13,0) = 0,21 ⎟ = -0,12 s × 13,0 s, -2 -2 1,0 ×10 ⎝ 1,0 ×10 mol/L ⎠

c) ln ⎜

[A] = 2,1 × 10 −3 mol/L d) [A]réagi = 0,010 mol/L – 0,0021 mol/L = 0,008 mol/L

[C]réagi = 0,008 mol/L ×

2 mol C = 0,016 mol/L ≈ 0,02 mol/L 1 mol A

[C]restant = 2,0 mol/L – 0,02 mol/L = 2,0 mol/L; comme prévu, la concentration de C demeure fondamentalement constante au cours de l’expérience étant donné que [C]o>> [A]o.

Mécanismes réactionnels 42.

Pour une réaction élémentaire, les ordres correspondent aux coefficients de l’équation équilibrée. a) v = k[CH3NC]

b) v = k[O3][NO]

c) v = k[O3]

d) v = k[O3][O]

© 2007 Les Éditions CEC inc. Toute reproduction interdite

82

Chapitre 3 Cinétique chimique

43.

Selon la question 11, la loi de vitesse de cette réaction est v = k[NO2]2[H2]. Or la loi de vitesse est déterminée par l’étape lente du mécanisme, ce qui signifie que pour être acceptable, un mécanisme doit impliquer 2 molécules NO et 1 molécule H2 comme réactifs. Le mécanisme III est donc le seul acceptable.

44.

Un mécanisme consiste en une série de réactions élémentaires où l’équation de vitesse pour chaque étape peut être déterminée à l’aide des coefficients des équations équilibrées. Pour un mécanisme acceptable, l’équation de vitesse déterminée à partir d’un mécanisme doit correspondre à l’équation de vitesse déterminée expérimentalement. Pour déterminer l’équation de vitesse à partir du mécanisme, on suppose que la vitesse est égale à la vitesse de l’étape la plus lente du mécanisme. Comme l’étape 1 est l’étape déterminante, l’équation de vitesse pour ce mécanisme est : vitesse = [C4H9Br]. Pour obtenir l’équation globale, on additionne les étapes individuelles du mécanisme. La somme de toutes les étapes donne : C4H9Br → C4H9+ + Br− C4H9 + H2O → C4H9OH2+ C4H9OH2+ + H2O → C4H9OH + H3O+ ____________________________________ C4H9Br + 2 H2O → C4H9OH + Br− + H3O+ +

Dans un mécanisme, les intermédiaires sont les espèces qui ne sont ni des réactifs, ni des produits, mais qui sont formées au cours d’une séquence de réactions. Les intermédiaires pour ce mécanisme sont C4H9+ et C4H9OH2+. 45.

v = vitesse de l’étape lente = k[NO2]2

L’équation globale est obtenue en additionnant les deux étapes. NO2 + NO2 → NO3 + NO NO3 + CO → NO2 + CO2 NO2 + CO → NO + CO2

© 2007 Les Éditions CEC inc. Toute reproduction interdite

Chapitre 3 Cinétique chimique

83

Influence de la température sur les constantes de vitesse et théorie des collisions

46.

47.

Dans ce graphique, R = réactifs, P = produits, Ea = énergie d’activation. Remarquer que ΔE est positive puisque les produits sont de plus haute énergie que les réactifs.

a)

b)

© 2007 Les Éditions CEC inc. Toute reproduction interdite

Chapitre 3 Cinétique chimique

84 c)

La réaction a est la plus rapide puisque son énergie d’activation est la plus petite. 48. 125 kJ/mol

E

R E a, inverse

216 kJ/mol

P Progression de la réaction

L’énergie d’activation pour la réaction inverse est : Ea, inverse = 216 kJ/mol + 125 kJ/mol = 341 kJ/mol

49.

Lorsque Ead > Eai, ΔE est positive. Lorsque ΔE est négative, c’est que Ead < Eai.

50.

1/T (K–1) 2,96 × 10–3 3,14 × 10–3 3,36 × 10–3

ln k –5,32 –7,60 –10,26

pente = –10,26 – (–7,60) (3,36 × 10–3 – 3,14 × 10–3) K–1 pente = –12,1 × 103 K =

-E a R

© 2007 Les Éditions CEC inc. Toute reproduction interdite

Chapitre 3 Cinétique chimique

85

⎛ -12,1×103K × 8,315 J ⎞ ⎟ K ⋅ mol ⎝ ⎠

Ea = – ⎜

Ea = 51.

1, 01 × 105 J = 101 kJ/mol mol

Selon l’équation d’Arrhenius dans sa forme logarithmique (ln k = -Ea/RT + ln A), un graphique de ln k en fonction de 1/T devrait donner une droite de pente égale à -Ea/R et une ordonnée à l’origine égale à ln A. a) pente = -Ea/R, Ea = 1,10 × 104 K ×

8,3145 J = 9,15 × 104 J/mol = 91,5 kJ/mol K ⋅ mol

b) Les unités de A sont les mêmes que les unités de k( s −1 ).

ordonnée à l’origine = ln A, A = e33,5 = 3,54 × 1014 s −1 c) ln k = -Ea/RT + ln A ou k = A exp(-Ea/RT)



⎞ -9,15 ×10-4 J/mol −2 −1 ⎟ = 3,24 × 10 s ⎝ 8,3145 J/K ⋅ mol × 298 K ⎠

k = 3,54 × 1014 s −1 × exp ⎜

52.

⎛ k2 ⎞ Ea ⎛ 1 - 1 ⎞ ⎟ = R ⎜ ⎟ T2⎠ ⎝ T1 ⎝ k1 ⎠

Pour les deux conditions : ln ⎜

⎛ 8,1 × 10−2 s −1 ⎞ 1 ⎞ ⎛ 1 Ea = × ln ⎜ ⎜ ⎟ −2 −1 ⎟ 293 K ⎠ ⎝ 4,6 × 10 s ⎠ 8,315 J/mol⋅ K ⎝ 273 K 0,57 =

53.

E a (2,5 × 10–4), E = 1,9 × 104 J/mol = 19 kJ/mol a 8,315

⎛ k2 ⎞ E a ⎛ 1 − 1 ⎞ ; si la vitesse double, alors k = 2 k . 2 1 ⎟= ⎟ R ⎜⎝ T 1 T2⎠ ⎝ k1 ⎠

ln ⎜

ln 2,00 = 54.

Ea 8,315 J/mol⋅ K

1 ⎞ ⎛ 1 × ⎟ , Ea = 5,3 × 104 J/mol = 53 kJ/mol ⎜ 308 K ⎠ ⎝ 298 K

H3O+(aq) + OH-(aq) → 2 H2O(l) devrait avoir la plus grande vitesse. H3O+ et OH- sont liés l’un à l’autre de façon électrostatique; Ce4+ et Hg22+ se repoussent (de sorte que Ea est beaucoup plus élevée).

© 2007 Les Éditions CEC inc. Toute reproduction interdite

Chapitre 3 Cinétique chimique

86 55.

Le carbone ne peut pas former la cinquième liaison nécessaire pour l’état de transition en raison de sa petite taille atomique, et parce qu’il ne possède pas d’orbitales d de faible énergie disponibles pour agrandir l’octet.

Catalyse 56.

a) NO : il ne disparaît pas lors de la réaction. b) NO2 : il apparaît à l’étape 1 et disparaît à l’étape 2. c) k = Ae–Ea/RT, ln k = ln A –

Ea RT

ln k1 = ln A –

E a1 ; (1) réaction catalysée RT

ln k2 = ln A –

E a2 ; (2) réaction non catalysée RT

En supposant que A soit le même pour les réactions catalysée et non catalysée et en soustrayant (1) – (2) ln k1 – ln k2 =

(- E a 2) E a1 − RT RT

1 ⎛ k1 ⎞ ⎟ = − RT ( E a1 − E a 2) ⎝ k2 ⎠

ln ⎜

K⋅ mol ⎛ k1 ⎞ 3 ⎟ = − 8,315 J × (11,9 × 10 ) = 0,848 ⎝ k2 ⎠

ln ⎜

k1 = 2,33 k2 57.

L’énergie d’activation de la réaction catalysée par Cl est plus petite que celle de la réaction catalysée par NO. Cl est un meilleur catalyseur.

© 2007 Les Éditions CEC inc. Toute reproduction interdite

Chapitre 3 Cinétique chimique

87

58.

Surface du métal

La réaction à la surface du catalyseur se passe ainsi : CH2D–CH2D est donc le produit. Si le mécanisme de la section 3.8 est correct, alors la réaction devrait donner : C2H4 + D2 → CH2DCH2D Si la réaction donnait un autre produit, tel CH3CHD2, on devrait conclure que le mécanisme proposé est incorrect. Si la réaction donne CH2DCH2D tel que prédit, cela ne prouve pas que le mécanisme est correct : il est seulement plausible, car on pourrait concevoir un mécanisme différent donnant le même produit. 59.

a) La surface de tungstène (W) représente le meilleur catalyseur, son Ea étant la plus faible. b) kw = Aw exp[–Ea(W)/RT] ; knc = Anc exp[–Ea(nc)/RT] ; supposons que Aw = Anc ; nc = non catalysée

⎛ - E a (W ) E a (nc) ⎞ kw = exp ⎜ + ⎟ RT ⎠ k nc ⎝ RT ⎛ - 163,000 J/mol + 335,000 J/mol ⎞ kw = exp ⎜ ⎟ = 1,41 × 1030 8,315 J/mol⋅ K × 298 K k nc ⎠ ⎝ La réaction catalysée par W est environ 1030 plus rapide que la réaction non catalysée. c) Selon la loi de vitesse, H2 diminue la vitesse de la réaction. Pour que cette réaction ait lieu, les molécules NH3 doivent être adsorbées à la surface du catalyseur. Si le catalyseur adsorbe aussi les molécules H2, il reste moins de place pour l’adsorption des molécules NH3. Voilà pourquoi la vitesse de réaction diminue lorsque [H2] augmente. 60.

En supposant que les réactions catalysée et non catalysée aient la même forme et soient du même ordre, et étant donné qu’on suppose des concentrations égales, les vitesses sont égales lorsque les valeurs de k sont égales. k = A exp(-Ea/RT), kcat = knc quand Ea,cat/RTcat = Ea,nc/RTnc

4,20 ×104 J/mol 7,00 ×104 J/mol = , Tnc = 488 K = 215 °C 8,3145 J/mol ⋅ K × 293 K 8,3145 J/mol ⋅ K × Tnc

© 2007 Les Éditions CEC inc. Toute reproduction interdite

Chapitre 3 Cinétique chimique

88

61.

Vitesse =

-Δ[A] = k[A]x Δt

En supposant que les réactions catalysée et non catalysée aient la même forme et soient du même ordre et étant donné qu’on suppose des concentrations égales, la vitesse ∝

1 . Δt

vitessecat Δt vitessecat k 2400 ans = nc = = cat et vitesse nc Δtcat Δtcat vitesse nc knc A exp [ - Ea(cat) RT ] exp[− Ea(cat) + Ea(nc) ] vitessecat k = cat = = RT vitesse nc A exp [ - Ea(nc) RT ] knc ⎛ −5,90 ×104 J/mol + 1,84 ×105 J/mol ⎞ kcat 10 = exp ⎜ ⎟ = 7,62 × 10 knc 8,3145 J/mol ⋅ K × 600 K ⎝ ⎠

Δtnc vitessecat k = = cat , knc Δtcat vitesse nc

2400 ans = 7,62 × 1010, Δtcat = 3,15 × 10 −8 an ≈ 1 sec Δtcat

EXERCICES SUPPLÉMENTAIRES 62.

Vitesse = k[NO]x[O2]y; en comparant les deux premières expériences, on constate que [O2] reste constante, que [NO] a triplé et que la vitesse augmente d’un facteur 9. Par conséquent, la réaction est d’ordre deux par rapport à NO (32 = 9). L’ordre par rapport à O2 est plus difficile à déterminer. En comparant la deuxième et la troisième expérience;

3,13 ×1017 k (2,50 ×1018 ) 2 (2,50 ×1018 ) y = , 1,74 = 0,694 (2,50)y, 2,51 = 2,50y, y = 1 17 18 2 18 y k (3,00 ×10 ) (1,00 ×10 ) 1,80 ×10 Vitesse = k[NO]2[O2]; d’après l’expérience 1: 2,00 × 1016 molécules/cm3·s = k (1,00 × 1018 molécules/cm3)2 (1,00 × 1018 molécules/cm3) k = 2,00 × 10 −38 cm6/molecules2·s = kmoyen

2,00 ×10-38 cm 6 Vitesse = molécules 2 ⋅ s

2

⎛ 6,21×1018 molécules ⎞ 7,36 ×1018 molécules ×⎜ ⎟ × cm3 cm3 ⎝ ⎠ = 5,68 × 1018 molécules/cm3·s

© 2007 Les Éditions CEC inc. Toute reproduction interdite

Chapitre 3 Cinétique chimique 63.

89

La pression d’un gaz étant proportionnelle à sa concentration, on peut utiliser les données de pression pour représenter la vitesse : –Δ[SO2Cl2]/Δt = – ΔPSO2Cl2 /Δt En supposant que la réaction soit d’ordre 1, on obtient le graphique suivant : temps (h) PSO2Cl2 (atm)

0,00 4,93

1,00 4,26

2,00 3,52

4,00 2,53

8,00 1,30

16,00 0,34

PSO2Cl2

1,595

1,449

1,258

0,928

0,262

–1,08

Puisque le graphique donne une droite, la réaction est d’ordre 1. a) k = –pente = 0,168 h–1 b) t1/2 =

ln 2 0,6931 = = 4,13 h 0,168 h −1 k

⎛ P SO 2 Cl 2 ⎞ ⎛ P SO 2 Cl2 ⎞ –3,36 –1 = 3,47 × 10–2 ⎟ = –kt = –0,168 h (20,0 h) = –3,36, ⎜ ⎟=e ⎝ Po ⎠ ⎝ Po ⎠

c) ln ⎜

fraction = 0,0347 = 3,47 % 64.

D’après les données à 338 K, le graphique de ln[N2O5] en fonction de t est linéaire et la pente = -4,86 × 10 −3 . Cela nous indique que la réaction est d’ordre un par rapport à N2O5 avec k = 4,86 × 10 −3 à 338 K.

© 2007 Les Éditions CEC inc. Toute reproduction interdite

Chapitre 3 Cinétique chimique

90

D’après les données à 318 K, la pente de ln[N2O5] en fonction de t est égale à -4,98 × 10 −4 , de sorte que k = 4,98 × 10 −4 à 318 K. On a maintenant deux valeurs de k à deux températures, et on peut alors résoudre pour trouver Ea.

⎛k ⎞ E ⎛ 1 1 ⎞ ⎛ 4,86 ×10-3 ⎞ ⎛ 1 Ea 1 ⎞ − ln ⎜ 2 ⎟ = a ⎜ - ⎟ , ln ⎜ = ⎜ ⎟ -4 ⎟ ⎝ 4,98 ×10 ⎠ 8,3145 J/mol ⋅ K ⎝ 318 K 338 K ⎠ ⎝ k1 ⎠ R ⎝ T1 T2 ⎠ Ea = 1,0 × 105 J/mol = 1,0 × 102 kJ/mol 65.

L’équation d’Arrhenius est : k = A exp (-Ea/RT) ou dans sa forme logarithmique, ln k = Ea/RT + ln A. D’où, un graphique de ln k en fonction de 1/T doit donner une droite de pente égale à -Ea/R étant donné que la forme logarithmique de l’équation d’Arrhenius est une équation de la forme d’une droite, y = mx + b. Note : on garde un chiffre significatif supplémentaire dans les valeurs suivantes de ln k afin de réduire les erreurs d’arrondissement. T (K)

195 230 260 298 369

1/T (K −1 )

5,13 × 10 −3 4,35 × 10 −3 3,85 × 10 −3 3.36 × 10 −3 2,71 × 10 −3

k (L/mol·s)

ln k

1,08 × 109 2,95 × 109 5,42 × 109 12,0 × 109 35,5 × 109

20,80 21,81 22,41 23,21 24,29

D’après un « examen rapide » de la droite sur le graphique :

pente =

20,95 − 23,65 = (5,00 × 10-3 − 3,00 ×10-3 ) K −1

−E a −2,70 = − 1,35 ×103 K= -3 2,00 ×10 R

© 2007 Les Éditions CEC inc. Toute reproduction interdite

Chapitre 3 Cinétique chimique

Ea = 1,35 × 103 K ×

91

8,3145 J = 1,12 × 104 J/mol = 11,2 kJ/mol K ⋅ mol

La pente calculée est : pente = -1,43 × 103 K et Ea = 11,9 kJ/mol 66.

À [S] élevée, l’enzyme est complètement saturée par le substrat. Une fois l’enzyme complètement saturée, la vitesse de décomposition de ES ne peut plus augmenter, et la vitesse globale reste constante.

67.

k = A exp (-Ea/RT);

⎛ Ea,cat + Ea, nc ⎞ kcat A cat exp(-Ea,cat /RT ) = = exp ⎜ ⎟ knc Anc exp(-Ea,nc /RT ) RT ⎝ ⎠

⎛ - Ea,cat + 5,00 ×104 J/mol ⎞ kcat = exp ⎜ 2,50 × 10 = ⎜ 8,3145 J/mol ⋅ K × 310 K ⎟⎟ knc ⎝ ⎠ 3

ln (2,50 × 103 ) × 2,58 × 103 J/mol = -Ea,cat + 5,00 × 104 J/mol Ea, cat = 5,00 × 104 J/mol – 2,02 × 104 J/mol = 2,98 × 104 J/mol = 29,8 kJ/mol 68.

a) Étant donné que [A]o < < [B]o ou [C]o, les concentrations de B et de C demeurent constantes à 1,00 mol/L pour cette expérience. Donc : vitesse = k[A]2[B][C] = k′[A]2 où k′ = k[B][C]

Pour cette réaction d’ordre deux apparent :

1 1 1 1 , = k′t + = k′(3,00 min) + -5 [A] [A] 0 3,26 ×10 mol/L 1,00 ×10-4 mol/L k′ = 6890 L/mol·min = 115 L/mol·s k′ = k[B][C], k =

k' 115 L/mol ⋅ s = = 115 L3 /mol3 ⋅ s [B][C] (1,00 mol/L) (1,00 mol/L)

b) Pour cette réaction d’ordre deux apparent :

vitesse = k′[A]2, t1/2 =

c)

1 1 = = 87,0 s 115 L/mol ⋅ s (1,00 ×10-4 mol/L) k '[A]0

1 1 1 = k′t + = 7,90 × 104 L/mol, = 115 L/mol·s × 600 s + [A] 0 [A] 1, 00 × 10−4 mol/L [A] =

1 = 1,27 × 10–5 mol/L −4 7.90 × 10 L / mol

D’après la stoechiométrie de la réaction équilibrée, 1 mol de B réagit avec 3 mol de A.

© 2007 Les Éditions CEC inc. Toute reproduction interdite

Chapitre 3 Cinétique chimique

92

quantité de A qui a réagi = 1,00 × 10–4 mol/L – 1,27 × 10–5 mol/L = 8,7 × 10–5 mol/L quantité de B qui a réagi = 8,7 × 10–5 mol/L × 1 mol B/3 mol A = 2,9 × 10–5 mol/L [B] = 1,00 mol/L – 2,9 × 10–5 mol/L = 1,00 mol/L Comme il a été mentionné dans la partie a, la concentration de B (et de C) demeure constante étant donné que la concentration de A est très petite.

PROBLÈMES DÉFIS 69.

-d[A] = k[A]3, dt n ∫ x dx =

[ A ]t

d [A]

∫[ ] [ A ] A

0

3

t

= - ∫ kdt 0

xn + 1 1 ; Donc : − 2[A]2 n +1

[A]t [A]0

= − kt , −

1 1 + = − kt 2 2[A]t 2[A]o2

Pour l’équation de demi-vie, [A]t = 1/2[A]0 :



1 ⎛1 ⎞ 2⎜⎜ [A ]0 ⎟⎟ ⎝2 ⎠

-

3 2[A]02

2

+

1 = − kt1/ 2 , 2[A ]02

= -kt1/2 , t1/2 =

-

4 1 + = -kt1/2 2 2[A]0 2[A]02

3 2[A]02 k

La première demi-vie est t1/2 = 40 s et correspond à la variation de [A]0 à 1/2[A]0. La deuxième demi-vie correspond à la variation de 1/2[A]0 à 1/ 4[A]0 . Première demi-vie =

3 ; deuxième demi-vie = 2[A]02 k

3 2

⎛1 ⎞ 2 ⎜ [A]0 ⎟ k ⎝2 ⎠

=

6 [A]02 k

3 2[A]02 k Première demi-vie = = 3/12 = 1/4 6 Deuxième demi-vie [A]02 k Étant donné que la première demi-vie est de 40 s, la deuxième demi-vie sera un quart de cette valeur, soit 10 s.

© 2007 Les Éditions CEC inc. Toute reproduction interdite

Chapitre 3 Cinétique chimique 70.

93

Si la vitesse dépend de [OH–], v = k [I–]x [OCl–]y [OH–]z x y z −3 v 1 = k (0,0013) (0,012) (0,10) = 9,4 × 10 x y z 18,7 × 10−3 v 2 k (0,0026) (0,012) (0,10)

x

⎛1⎞ ⎜ ⎟ = 0,503, x = 1 ⎝2⎠ v1 k (0,0013)1(0,012) y (0,10) z 9,4 × 10−3 = = v3 4,7 × 10−3 k (0,0013)1(0,06) y (0,10) z (2)y = 2, y = 1 1 1 z 9,4 × 10−3 v1 k (0,0013) (0,012) (0,10) = = z 1 1 18,7 × 10−3 v5 k (0,0013) (0,012) (0,05)

(2)z = 0,50 ; z = –1 v=

k[I _ ][OCl _ ] [OH _ ]

k=

9,4 × 10−3 0,10 mol L L × × × L⋅s L 0, 0013 mol 0, 012 mol

k = 60 s–1 (6,0 × 10 s–1) 71.

Pour une réaction d’ordre 2 :

a)

1 1 1 − = kt et t 1/ 2 = [A] [A]o k[A ]o

1 1 1 1 = (0,250 L/mol⋅s)t + , = 0,250 L/mol⋅s × 180 s + [A ]o [A] [A] 1,00 × 10−2 mol/L 1 = 145 L/mol, [A] = 6,90 × 10–3 mol/L [A] Quantité de A ayant réagi = 0,0100 – 0,00690 = 0,0031 mol/L [A2] =

1 (3,1 × 10–3 mol/L) = 1,6 × 10–3 mol/L 2

© 2007 Les Éditions CEC inc. Toute reproduction interdite

Chapitre 3 Cinétique chimique

94

b) Après 3,00 min (180 s) : [A] = 3,00 [B], 6,90 × 10–3 mol/L = 3,00 [B], [B] = 2,30 × 10–3 mol/L

1 1 = k2t + , [B] [B]o

1 1 , = k2 (180 s) + −3 2,30 × 10 mol/L 2,50 × 10−2 mol/L

k2 = 2,19 L/mol⋅s c) t1/2 =

72.

1 1 = 4,00 × 102 s = k[A]o 0,250 L/mol ⋅ s × 1,00 × 10−2 mol/L

a) Note : [O3] constante à 1 × 1014 molécules/cm3 pour la première partie. t (ms)

0 100 500 700 1000

[NO] (molécules/cm3) 6,0 × 108 5,0 × 108 2,4 × 108 1,7 × 108 9,9 × 107

ln[NO] 20,21 20,03 19,30 18,95 18,41

1/[NO] (cm3/molécule) 1,7 × 10–9 2,0 × 10–9 4,2 × 10–9 5,9 × 10–9 1,0 × 10–8

Le graphique ln[NO] en fonction du temps donne une droite, la réaction est d’ordre 1 par rapport à NO.

© 2007 Les Éditions CEC inc. Toute reproduction interdite

Chapitre 3 Cinétique chimique

t(ms)

0 50 100 200 300

95

[O3] (molécules/cm3) 1,0 × 1010 8,4 × 109 7,0 × 109 4,9 × 109 3,4 × 109

ln[O3] 23,03 22,85 22,67 22,31 21,95

1/[O3] (cm3/molécule) 1,0 × 10–10 1,2 × 10–10 1,4 × 10–10 2,0 × 10–10 2,9 × 10–10

Le graphique ln[O3] en fonction du temps donne une droite, la réaction est d’ordre 1 par rapport à O3.

© 2007 Les Éditions CEC inc. Toute reproduction interdite

Chapitre 3 Cinétique chimique

96 b) v = k [NO][O3] c) 1. v = k1[NO] où k' = k[O3] k' = – pente du graphique ln[NO] en fonction du temps k' = –

18,41 - 20,21 = 1,8 × 10–3 ms–1 = 1,8 s–1 (1000 - 0) ms

2. v = k''[O3] où k'' = k[NO] k'' = – pente du graphique ln[O3] en fonction du temps k'' = –

d) k =

k'

[ O 3]

ou k = 73.

(21,95 − 23,03) = 3,6 × 10–3 ms–1 = 3,6 s–1 (300 - 0) ms = 1,8 s −1 ×

3

cm = 1,8 × 10–14 cm3/molécules·s 1 × 1014 molécules

3 k" cm = 1,8 × 10–14 cm3/molécules⋅s = 3,6 s -1 × [NO] 2 × 1014 molécules

Dans ce diagramme, R = réactifs, P = produits, Ea = énergie d’activation, ΔE = variation globale d’énergie et I = espèce intermédiaire

a) à d)

Voir le diagramme.

e) Ce diagramme ayant deux barrières d’énergie, il s’agit donc d’une réaction à deux étapes :

1) 2)

R→I I →P R→P

© 2007 Les Éditions CEC inc. Toute reproduction interdite

Chapitre 3 Cinétique chimique

97

Puisque la barrière d’énergie de la deuxième étape est plus élevée que celle de la première, son énergie d’activation est plus importante et cette étape est la plus lente, c’est-à-dire l’étape déterminante. 74.

a) Si la fréquence entre deux éclairs est de 16,3 sec, la vitesse est :

1 éclair/16,3 s = 6,13 × 10 −2 s −1 = k Fréquence 16.3 s 13,0 s

k −2

−1

6.13 × 10 s 7,69 × 10 −2 s −1

T 21,0 °C (294,2 K) 27,8 °C (301,0 K)

⎛k ⎞ E ⎛ 1 1 ⎞ ln ⎜ 2 ⎟ = a ⎜ - ⎟ ; En résolvant : Ea = 2,5 × 104 J/mol = 25 kJ/mol ⎝ k1 ⎠ R ⎝ T1 T2 ⎠ ⎛

⎞ 2,5 × 104 J/mol ⎛ ⎞ k 1 1 = − ⎜ ⎟ = 0.30 -2 ⎟ ⎝ 6,13 ×10 ⎠ 8,3145 J/mol ⋅ K ⎝ 294,2 K 303,2 K ⎠

b) ln ⎜

k = e0,30 × 6,13 × 10 −2 = 8,3 × 10 −2 s −1 ; Fréquence = 1/k = 12 secondes c)

T 21,0 °C 27,8 °C 30,0 °C

Fréquence 16,3 s 13,0 s 12 s

54-2(Fréquences) 21 °C 28 °C 30 °C

Cette règle empirique concorde bien à deux chiffres significatifs. 75.

a) Vitesse = k[H2]x[NO]y; en examinant les données de l’expérience 2, on remarque que la concentration de H2 diminue de moitié toutes les 10 secondes. Seules les réactions d’ordre un ont une demi-vie indépendante de la concentration. La réaction est d’ordre un par rapport à H2. Dans les données de l’expérience 1, on remarque que la demi-vie est de 40 s. Cela indique qu’en passant de l’expérience 1 à l’expérience 2 où la concentration de NO est doublée, la vitesse de réaction augmente d’un facteur quatre. Cela indique que la réaction est d’ordre deux par rapport à NO.

Vitesse = k[H2][NO]2 b) Cette réaction est d’ordre un apparent par rapport à [H2] alors que la concentration de NO est si élevée, qu’elle est fondamentalement constante.

Vitesse = k[NO]2[H2] = k′[H2] où k′ = k[NO]2 Pour une réaction d’ordre un, l’équation de vitesse intégrée est : ⎛ [H 2 ] ⎞ ⎟⎟ = -k′t ln⎜⎜ ⎝ [H 2 ]0 ⎠

© 2007 Les Éditions CEC inc. Toute reproduction interdite

Chapitre 3 Cinétique chimique

98

Calculer k′ à l’aide de n’importe quelle série de données. Par exemple, dans l’expérience 1, de 0 à 20 s, la concentration de H2 diminue de 0,010 mol/L à 0,0071 mol/L.

⎛ 0, 0071 ⎞ = -k′(20 s), k′ = 1,7 × 10 −2 s −1 ln ⎜ ⎟ ⎝ 0, 010 ⎠ k′ = k[NO]2, 1,7 × 10 −2 s −1 = k(10,0 mol/L)2 k = 1,7 × 10-4 L2 /mol2 ⋅ s

On obtient des résultats similaires pour les valeurs de k en utilisant d’autres données soit de l’expérience 1, soit de l’expérience 2.



⎞ [H 2 ] -2 2 2 −3 ⎟ = -k′t = -(1,7 × 10 L /mol ⋅ s ) × 30 s, [H2] = 60 × 10 mol/L ⎝ 0,010 mol/L ⎠

c) ln ⎜ 76.

a) Vitesse = (k1 + k2[H+])[I−]m[H2O2]n

Dans toutes les expériences, la concentration de H2O2 est faible comparée aux concentrations de I− et de H+. Par conséquent, les concentrations de I− et de H+ sont effectivement constantes et l’équation de vitesse se réduit à : Vitesse = kobs[H2O2]n où kobs = (k1 + k2[H+])[I−]m Étant donné que tous les graphiques de ln[H2O2] en fonction du temps sont linéaires, la réaction est d’ordre un par rapport à H2O2 (n = 1). Les pentes des graphiques de ln[H2O2] en fonction du temps sont égales à -kobs qui est égale à -(k1 + k2[H+])[I−]m. Pour déterminer l’ordre par rapport à I−, comparer les pentes de deux expériences où I− change et H+ est constant. En comparant les deux premières expériences :

-[k1 + k2 (0,0400 mol/L)][0,3000 mol/L) m pente (exp. 2) -0.360 , = = pente (exp.1) -0.120 -[k1 + k2 (0,0400 mol/L)][0,1000 mol/L)m m

⎛ 0,3000 ⎞ m 3,00 = ⎜ ⎟ = (3,000) , m = 1 0,1000 ⎝ ⎠

La réaction est également d’ordre un par rapport à I−. b) L’équation de la pente comporte deux inconnues, k1 et k2. Pour trouver les valeurs de k1 et de k2, on a besoin de deux équations. Il faut prendre une de la première série des trois expériences et une de la seconde série de trois expérience pour générer les deux équations en k1 et en k2.

Expérience 1 : pente = -(k1 + k2[H+])[I−] -0,120 min −1 = -[k1 + k2 (0,0400 mol/L)] 0,1000 mol/L ou 1,20 = k1 + k2 (0,0400)

© 2007 Les Éditions CEC inc. Toute reproduction interdite

Chapitre 3 Cinétique chimique

99

Expérience 4 : -0, 0760 min −1 = -[k1 + k2 (0,0200 mol/L)] 0,0750 mol/L ou 1,01 = k1 + k2 (0,0200) On soustrait 4 de 1 : 1,20 = k1 + k2(0,0400) -1,01 = -k1 - k2(0,0200) _____________________ k2 (0,0200), k2 = 9,5 L2/mol2·min 0,19 = 1,20 = k1 + 9,5(0,0400), k1 = 0,82 L/ mol·min c) Il y a deux voies, une qui implique H+ avec vitesse = k2[H+][I−][H2O2] et l’autre voie qui ne fait pas intervenir H+ avec vitesse = k1[I−][H2O2]. La vitesse globale de la réaction dépend de la voie qui domine, ce qui dépend de la concentration de H+.

PROBLÈMES D’INTÉGRATION 77.

8,75 h ×

ln 2 ln 2 3600 s = 3,15 × 104 s; k = = 2,20 × 10 −5 s −1 = h t1/2 3,15 ×104 s

La pression partielle d’un gaz est directement proportionnelle à la concentration en mol/L. Alors, au lieu d’utiliser les mol/L comme unités de concentration dans l’équation de vitesse intégrée d’ordre un, on peut utiliser les pressions partielles de SO2Cl2.

⎛P⎞ ⎛ P ⎞ 3600 s −5 -1 ) ln ⎜ ⎟ = -kt, ln ⎜ ⎟ = -(2,20 × 10 s )(12,5 h × h ⎝ 791 torr ⎠ ⎝ P0 ⎠ 1 atm = 0,387 atm PSO2Cl2 = 294 torr × 760 torr n=

0,387 atm ×1,25 L PV = 9,94 × 10 −3 mol SO2Cl2 = 0,08206 L atm ⋅ RT × 593 K mol ⋅ K

9,94 × 10 −3 mol ×

6,022 ×1023 molécules = 5,99 × 1021 molécules SO2Cl2 mol

© 2007 Les Éditions CEC inc. Toute reproduction interdite

Chapitre 3 Cinétique chimique

100

78.

k=

ln 2 ln 2 = 1,04 × 10 −3 s −1 = t1/2 667 s 2,38 g InCl ×

[In+]0 =

1 mol InCl 1 mol In + × 150,3 g mol InCl = 0,0317 mol/L 0,500 L

⎛ [In + ] ⎞ ⎛ ⎞ 3600 s [In + ] −3 −1 ln ⎜ + ⎟ = -kt , ln ⎜ ⎟ = -(1,04 × 10 s ) × 1,25 h × h ⎝ 0,0317 mol/L ⎠ ⎝ [In ]0 ⎠ [In+] = 2,94 × 10 −4 mol/L L’équation redox équilibrée est : 3 In+(aq) → 2 In(s) + In3+(aq)

0,0317 mol 2,94 ×10-4 mol − 0,500 L × mol In qui ont réagi = 0,500 L × L L +

= 1,57 × 10 −2 mol In+ 1,57 × 10 −2 mol In+ ×

79.

2 mol In 114,8 g In × = 1,20 g In + 3 mol In mol In

⎛k ⎞ E ⎛1 1⎞ ln ⎜ 2 ⎟ = a ⎜ − ⎟ ; R ⎝ T1 T2 ⎠ ⎝ k1 ⎠ ⎛ 1,7 ×10-2 s -1 ⎞ ⎛ 1 Ea 1 ⎞ − ln ⎜ = ⎜ ⎟ -4 -1 ⎟ ⎝ 7,2 ×10 s ⎠ 8,3145 J/K ⋅ mol ⎝ 660 K 720 K ⎠ Ea = 2,1 × 105 J/mol

Pour k à 325 °C (598 K):

⎛ 1,7 ×10-2 s-1 ⎞ 2,1×105 J/mol ⎛ 1 1 ⎞ −5 −1 − ln ⎜ = ⎟ ⎜ ⎟ , k = 1,3 × 10 s k ⎝ ⎠ 8,3145 J/K ⋅ mol ⎝ 598 K 720 K ⎠ Pour trois demi-vies, on passe de 100 % → 50 % → 25 % → 12,5 %. Après trois demivies, il reste 12,5 % de la quantité initiale de C2H5I. Les pressions partielles sont directement proportionnelles aux concentration de gaz en mol/L :

PC2 H5 I = 894 torr × 0,125 = 112 torr après trois demi-vies

© 2007 Les Éditions CEC inc. Toute reproduction interdite

Chapitre 3 Cinétique chimique

101

PROBLÈME DE SYNTHÈSE 80.

a) Vitesse = k[CH3X]x[Y]y; pour l’expérience 1, [Y] est constante de sorte que vitesse = k′[CH3X]x où k′ = k(3,0 mol/L)y.

Un graphique (non inclus) de ln[CH3X] en fonction de t donne une droite (x = 1). L’équation de vitesse intégrée est : ln[CH3X] = -0,93t – 3,99; k′ = 0,93 h −1 Pour l’expérience 2, [Y] est encore constante avec vitesse = k′′[CH3X]x où k′′ = k(4,5 mol/L)y. Le graphique du ln est linéaire avec une équation de vitesse intégrée : ln[CH3X] = -0,93t – 5,40; k′′ = 0,93 h −1 En divisant les valeurs de la constante de vitesse :

k' 0,93 k (3,0) y = = , 1,0 = (0,67)y, y = 0 k'' 0,93 k (4,5) y La réaction est d’ordre un par rapport à CH3X et d’ordre zéro par rapport Y. L’équation de vitesse globale est : Vitesse = k[CH3X] où k = 0,93 h −1 à 25 °C. b) t1/2 = (ln 2)/k = 0,6931/(7,88 × 108 h −1 ) = 8,80 × 10 −10 h c)

ln

⎛ 7,88 ×108 ⎞ ⎛ 1 E ⎛1 1⎞ Ea k2 1 ⎞ = a ⎜ - ⎟ , ln ⎜ ⎟ = ⎜ ⎟ k1 R ⎝ T1 T2 ⎠ 8,3145 J/K ⋅ mol ⎝ 298 K 358 K ⎠ ⎝ 0,93 ⎠

Ea = 3,0 × 105 J/mol = 3,0 × 102 kJ/mol d) D’après la partie a, la réaction est d’ordre un par rapport à CH3X et d’ordre zéro par rapport à Y. D’après la partie c, l’énergie d’activation est près de la valeur de l’énergie de liaison de C-X. Un mécanisme plausible qui explique les résultats des parties a et c est :

CH3X → CH3 + X CH3 + Y → CH3Y

(lente) (rapide)

Note : C’est un mécanisme possible étant donné que l’équation de vitesse déterminée est la même que l’équation de vitesse expérimentale et la somme des étapes donne l’équation globale équilibrée.

© 2007 Les Éditions CEC inc. Toute reproduction interdite

CHAPITRE 4 ÉQUILIBRE CHIMIQUE QUESTIONS 9.

Non. L’équilibre étant un processus dynamique, les réactions directe et inverse vont distribuer les atomes 14C entre les espèces CO et CO2.

10.

Non, la façon dont l’équilibre est atteint n’a pas d’importance, les deux mélanges atteindront la même position d’équilibre, puisqu’ils ont tous deux un état initial débutant avec des quantités stœchiométriques de réactifs ou de produits. (Il s’agit toutefois d’un cas particulier, toutes les positions d’équilibre n’auraient pas la même composition.)

11.

H2O(g) + CO(g) ⇌ H2(g) + CO2(g)

K=

[H 2 ][CO 2 ] = 2,0 [H 2 O][CO]

K est un nombre sans unité étant donné qu’il y a un nombre égal de moles de produits gazeux et de moles de réactifs gazeux dans l’équation équilibrée. Par conséquent, on peut utiliser des unités de molécules par litre au lieu de moles par litre pour déterminer K. Il faut commencer quelque part. Alors, supposons que 3 molécules CO réagissent. Si trois molécules CO réagissent, alors 3 molécules H2O doivent réagir, et 3 molécules H2 et CO2 chacun seront formées. On aura 6 – 3 = 3 molécules CO, 8 - 3 = 5 molécules H2O, 0 + 3 = 3 molécules H2 et 0 + 3 = 3 molécules CO2 présentes. Ce mélange sera à l’équilibre si K = 2,0 :

⎛ 3 molécules H 2 ⎞⎛ 3 molécules CO 2 ⎞ ⎜ ⎟⎜ ⎟ L L ⎠⎝ ⎠ =3 K= ⎝ ⎛ 5 molécules H 2 O ⎞ ⎛ 3 molécules CO ⎞ 5 ⎜ ⎟⎜ ⎟ L L ⎠ ⎝ ⎠⎝ Étant donné que ce mélange ne donne pas une valeur de K = 2,0, il n’est pas à l’équilibre. Essayons 4 molécules CO qui réagissent pour atteindre l’équilibre. molécules CO restantes = 6 - 4 = 2 molécules CO; molécules H2O restantes = 8 - 4 = 4 molécules H2O; molécules H2 présentes = 0 + 4 = 4 molécules H2; molécules CO2 présentes = 0 + 4 = 4 molécules CO2

© 2007 Les Éditions CEC inc. Toute reproduction interdite

Chapitre 4 Équilibre chimique

104

⎛ 4 molécules H 2 ⎞ ⎛ 4 molécules CO 2 ⎞ ⎜ ⎟⎜ ⎟ L L ⎝ ⎠ ⎝ ⎠ = 2,0 K= ⎛ 4 molécules H 2O ⎞ ⎛ 2 molécules CO ⎞ ⎜ ⎟⎜ ⎟ L L ⎠ ⎝ ⎠⎝ Étant donné que K = 2,0, ce mélange réactionnel est à l’équilibre. 12.

K et Kp sont des constantes d’équilibre tellles que déterminées par la loi d’action de masse. Pour K, les unités de concentration généralement utilisées sont mol/L et pour Kp, les pressions partielles en atm. Q s’appelle quotient réactionnel. Q a exactement la même forme que K et Kp, mais au lieu d’utiliser les concentrations à l’équilibre, on se sert des concentrations initiales pour calculer la valeur de Q. On se sert de Q pour comparer avec la valeur de K. Quand Q = K (ou quand Qp = Kp), la réaction est à l’équilibre. Quand Q ≠ K, la réaction n’est pas à l’équilibre et on peut déduire le changement global que doit subir le système pour atteindre l’équilibre.

13.

H2(g) + I2(g) → 2 HI(g) K =

[HI] 2 [H 2 ][I 2 ]

H2(g) + I2(s) → 2 HI(g) K =

[HI]2 (Les solides ne font pas partie des expressions de K.) [H 2 ]

Les réactions ont des expressions de K différentes ; pour la première réaction K = KP (étant donné que Δn = 0), et pour la deuxième réaction, K ≠ KP (étant donné que Δn ≠ 0).Une variation de volume du contenant n’aura aucun effet sur l’équilibre pour la réaction 1, alors qu’une variation de volume aura un effet sur l’équilibre pour la réaction 2. 15.

Seul l’énoncé e est vrai. L’addition d’un catalyseur n’a aucun effet sur la position d’équilibre ; la réaction ne fait qu’atteindre l’équilibre plus rapidement. L’énoncé a est faux pour les réactifs qui sont solides ou liquides (l’ajout de solide ou de liquide n’a aucun effet sur la position d’équilibre). L’énoncé b est toujours faux. Si la température demeure constante, alors la valeur de K est constante. L’énoncé c est faux pour les réactions exothermiques où une augmentation de température diminue la valeur de K. Pour l’énoncé d, seules les réactions qui ont plus de réactifs gazeux que de produits gazeux se déplacent vers la gauche sous l’influence d’une augmentation du volume du contenant. Si le nombre de moles est égal ou s’il y a plus de moles de produits gazeux que de réactifs gazeux, la réaction ne se déplacera pas vers la gauche avec une augmentation de volume.

© 2007 Les Éditions CEC inc. Toute reproduction interdite

Chapitre 4 Équilibre chimique

105

EXERCICES La constante d’équilibre 16.

17.

a) K =

[ NO2] [O2] [NO][O3]

b) K =

[O2][O] [O3]

c) K =

[ClO][O2] [Cl][O3]

d) K =

[O2 ]3 [O3 ]2

a) Kp =

P NO 2 × P O 2 =K P NO × P O3

b) Kp =

3

P ClO × P O 2 c) Kp = =K P Cl × P O3 18.

K = 278 =

P O2 × P O ≠K P O3

( P O2) d) Kp = ≠K 2 ( P O 3)

[SO3 ]2 pour 2SO2(g) + O2(g) ⇌ 2SO3(g) [SO2 ]2 [O2]

Si on inverse une réaction, alors Ki = 1/Kd. Si on la multiplie par une valeur n, alors Knouveau = (K)n. a) SO2(g) + 1/2O2(g) → SO2(g), K′ =

[SO3] = K1/2 = (278)1/2 = 16,7 1/ 2 [SO2][O2 ]

[SO 2]2[O 2] 1 1 b) 2SO3(g) → 2SO2(g) + O2(g), K″ = = 3,60 × 10–3 = = 2 278 K [SO 3] 1/ 2

c) SO3(g) → SO2(g) + 1/2O2(g), K′′′ =

[SO 2][O 2]1/2 ⎛1⎞ = ⎜ ⎟ [SO 3] ⎝K⎠

d) 4SO2(g) + 2O2(g) → 4SO3(g), K′′′′ =

= 6,00 × 10–2

[SO3 ]4 = K2 = 7,73 × 104 4 2 [SO2 ] [O2 ]

2

19.

Kp =

P HBr = 3,5 × 104 ( P H 2) ( P Br 2)

a) Kp′ =

( P Br 2)1/ 2 ( P H 2)1/ 2 1 1 = 5,3 × 10–3 = = 1/ 2 4 1/ 2 ( K p) P HBr (3,5 × 10 ) © 2007 Les Éditions CEC inc. Toute reproduction interdite

Chapitre 4 Équilibre chimique

106

20.

b) Kp′′ =

( P H 2) ( P Br 2) 1 1 = = = 2,9 × 10–5 2 4 3,5 × 10 ( P HBr ) Kp

c) Kp′ =

P HBr = (Kp)1/2 = 190 1/ 2 ( P H 2) ( P Br 2) 1/ 2

2 NO(g) + 2 H2(g) ⇌ N2(g) + 2 H2O(g)

K=

[ N 2 ][H 2 O]2 [ NO]2 [H 2 ]2

K=

(5,3 × 10−2 )(2,9 × 10−3 ) 2 = 4,0 × 106 (8,1 × 10−3 ) 2 (4,1 × 10−5 ) 2

21.

K=

[ NCl3]2 (0,19 mol/L) 2 = 3,2 × 1011 = 3 3 -3 -4 [ N 2] [Cl 2] (1,4 × 10 mol/L)(4,3 × 10 mol/L)

22.

[N2O] =

2,00 ×10-2 mol 2,80 ×10-4 mol 2,5 ×10-5 mol ; [N2] = ; [O2] = 2,00 L 2,00 L 2,00 L 2

⎛ 2,00×10-2 ⎞ ⎜ ⎟ 2,00 ⎠ [N 2 O]2 (1,00×10-2 ) 2 ⎝ = = K= = 4,08 × 108 2 2 -4 2 -5 -4 -5 [N 2 ] [O 2 ] ⎛ 2,80×10 ⎞ ⎛ 2,50×10 ⎞ (1,40×10 ) (1,25×10 ) ⎜ ⎟ ⎜ ⎟ ⎝ 2,00 ⎠ ⎝ 2,00 ⎠ Si les concentrations données représentent les concentrations à l’équilibre, alors elles devraient donner une valeur de K = 4,08 × 108.

(0, 200) 2 = 4,08 × 108 −4 2 (2, 00 × 10 ) (0, 00245) Étant donné que les concentrations substituées dans l’expression de la constante d’équilibre donnent une valeur égale à celle de K (4,08 × 108), cet ensemble de concentrations constitue un système à l’équilibre.

23.

24.

Kp =

Kp =

2 PNO × PO2 2 PNO 2 2 PNH 2

PN2 × PH32

(6,5 × 10−5 ) 2 (4,5 × 10−5 ) = = 6,3 × 10 −13 2 (0,55)

=

(3,1 × 10−2 ) 2 = 3,8 × 104 (0,85)(3,1 × 10−3 )3

© 2007 Les Éditions CEC inc. Toute reproduction interdite

Chapitre 4 Équilibre chimique

107

(0,167) 2 = 1,21 × 103 (0,525) (0, 00761)3 Quand on substitue les valeurs des pressions partielles dans l’expression de Kp, la valeur obtenue n’est pas égale à la valeur de Kp de 3,8 × 104. Par conséquent, on peut conclure que l’ensemble donné de pressions partielles ne représente pas un système à l’équilibre. 25.

Kp = K(RT)Δn où Δn = somme des coefficients des produits gazeux – somme des coefficients des réactifs gazeux. Pour cette réaction, Δn = 3 - 1 = 2. K=

[CO][H 2 ]2 (0,24)(1,1) 2 = 1,9 = [CH3OH] ( 0,15 )

KP = K(RT)2 = 1,9(0,08206 L·atm/K·mol × 600 K)2 = 4,6 × 103

26.

Kp = K(RT)Δn, K =

K=

27.

Kp ; Δn = 2 – 3 = –1 ; Δn ( RT )

2,5 × 10-3 kPa -1 ⎛ 8,315 kPa ⋅ L ⎞ × 1100 K ⎟ ⎜ ⎝ K ⋅ mol ⎠

−1

= 23

Les solides et les liquides n’apparaissent pas dans les expressions des constantes d’équilibre. Seuls les gaz et les solutés en solution apparaissent dans les expressions des constantes d’équilibre. a) K =

PH O [ H 2 O] ; Kp = 2 2 2 [ NH 3 ] [CO 2 ] PNH3 × PCO2

c) K = [O2]3; Kp = PO32

b) K = [N2][Br2]3; Kp = PN 2 × PB3r2

d) K=

PH2O [ H 2 O] ; Kp = [H 2 ] PH 2

28.

Kp = K(RT)Δn où Δn égale la différence entre la somme des coefficients des produits gazeux et la somme des coefficients des réactifs gazeux (Δn = mol produits gazeux - mol réactifs gazeux). Quand Δn = 0, alors Kp = K. Dans l’exercice 27, Δn = 0 seulement pour la réaction d de sorte que Kp = K seulement pour la réaction d.

© 2007 Les Éditions CEC inc. Toute reproduction interdite

108

Chapitre 4 Équilibre chimique

29.

Étant donné que les solides n’apparaissent pas dans l’expression de la constante d’équilibre, K = 1/[O2]3.

1,0 ×10-3 mol 1 1 1 = ; K= = 8,0 × 109 = 3 -4 3 3 -3 [ O ] 2,0 L (5,0 ×10 ) 2 ⎛ 1,0 ×10 ⎞ ⎜ ⎟ ⎝ 2,0 ⎠

[O2] =

30.

Kp =

PH42 PH42O

; Ptot = PH 2O + PH 2 , 36,3 torr = 15,0 torr + PH 2 , PH 2 = 21,3 torr

(21,3/760) 4 Puisque l atm = 760 torr : KP = = 4,07 (15,0/760) 4

Calculs relatifs à l’équilibre 31.

[ N 2 ][O 2 ] = 2,4 × 103 [ NO]2 Utilisez le quotient réactionnel Q pour déterminer dans quel sens la réaction se déplace pour atteindre l’équilibre. Pour le quotient réactionnel, on utilise les concentrations initiales données dans le problème pour calculer la valeur de Q. Si Q < K, la réaction se déplace vers la droite pour atteindre l’équilibre. Si Q > K, la réaction se déplace vers la gauche pour atteindre l’équilibre. Si Q = K, la réaction ne se déplace ni d’un côté, ni de l’autre parce qu’elle est à l’équilibre. 2 NO(g) ⇌ N2(g) + O2(g)

a) [N2]=

K=

2,0 mol 2,6 mol 0,024 mol = 2,0 mol/L; [O2] = = 2,6 mol/L; [NO] = 1,0 L 1,0 L 1,0 L = 0,024 mol/L

Q=

[N 2 ]o [O 2 ]o (2,0)(2,6) = 9,0 × 103 = 2 2 [NO]o (0,024)

Q > K alors la réaction se déplace vers la gauche pour atteindre l’équilibre b) [N2]=

0,62 mol 4,0 mol 0,032 mol = 0,31 mol/L; [O2] = = 2,0 mol/L; [NO] = 2,0 L 2,0 L 2,0 L = 0,016 mol/L

Q=

(0,31)(2, 0) = 2,4 × 103 = K; à l’équilibre 2 (0, 016)

© 2007 Les Éditions CEC inc. Toute reproduction interdite

Chapitre 4 Équilibre chimique

c) [N2] =

109

2,4 mol 1,7 mol 0,060 mol = 0,80 mol/L; [O2] = = 0,57 mol/L; [NO] = 3,0 L 3,0 L 3,0 L = 0,020 mol/L

Q=

(0,80)(0,57) = 1,1 × 103 < K; 2 (0, 020)

La réaction se déplace vers la droite pour atteindre l’équilibre. 32.

Comme dans l’exercice 4.31, déterminez Q pour chaque réaction et comparez sa valeur à celle de Kp (2,4 × 103) pour déterminer dans quel sens la réaction se déplace pour atteindre l’équilibre. Notez que pour cette réaction, K = Kp étant donné que Δn = 0. a) Q =

PN2 × PO2 2 NO

P

=

(0,11)(2,0) = 2,2 × 103 (0,010) 2

Q < Kp alors la réaction se déplace vers la droite pour atteindre l’équilibre. b) Q =

(0,36)(0, 67) = 4,0 × 103 > Kp (0, 078) 2

La réaction se déplace vers la gauche pour atteindre l’équilibre. c) Q =

33.

(0,51)(0,18) = 2,4 × 103 = Kp; à l’équilibre 2 (0, 0062)

CaCO3(s) ⇌ CaO(s) + CO2(g) Kp = P CO 2 = 105 kPa La pression de CO2 à l’équilibre est 105 kPa. Il s’agit donc de calculer la pression de CO2 ; si celle-ci est plus grande que 105 kPa, le système se déplacera vers la gauche, et la quantité d’oxyde de calcium diminuera. Si la pression de CO2 est plus petite que 105 kPa, le système se déplacera vers la droite et la quantité d’oxyde de calcium augmentera. Si la pression de CO2 est égale à 105 kPa, le système est à l’équilibre, et la quantité d’oxyde de calcium demeure la même. PV = nRT V = 50,0 L T = 1173 K a) P CO 2 =

58, 4 g CO 2 mol CO 2 8,315 kPa⋅ L × 1173 K × × = 259 kPa 50, 0 L 44, 01 g CO 2 K⋅ mol

Q > KP la quantité d’oxyde de calcium diminue.

© 2007 Les Éditions CEC inc. Toute reproduction interdite

Chapitre 4 Équilibre chimique

110

b) P CO 2 =

23,76 g CO2 mol CO2 8,315 kPa⋅ L × 1173 K × × = 105 kPa 50,0 L 44,01 g CO2 K⋅ mol

Le système est à l’équilibre, la quantité de CaO demeure la même. c) P CO 2 = 105 kPa (comme en b)), le système est à l’équilibre, la quantité de CaO demeure la même. d) P CO 2 =

4,82 g CO2 mol CO2 8,315 kPa⋅ L × 1173 K × × = 21,4 kPa 50,0 L 44,01 g CO2 K⋅ mol

Q < KP, la quantité de CaO augmente puisque 715 g de CaCO3 peuvent se décomposer pour former CaO et CO2. 34.

CH3COOH + C2H5OH ⇌ CH3COOC2H5 + H2O K =

[CH 3COOC 2 H 5] [H 2 O] = 2,2 [CH 3COOH] [C2 H5OH ]

a) Q =

(0,22) (0,10) = 220 > K ; déplacement vers la gauche, [H2O] diminuera. (0,010) (0,010)

b) Q =

(0,22) (0,0020) = 2,2 = K ; à l’équilibre, [H2O] demeure constante. (0,0020) (0,10)

c) Q =

(0,88) (0,12) = 0,40 < K ; déplacement vers la droite, [H2O] augmentera. (0,044) (6,0)

d) Q =

(4,4) (4,4) = 2,2 = K ; [H2O] demeure constante. (0,88) (10,0)

e) K = 2,2 =

2, 0 x [H 2O] , 0,10 x 5, 0

[H2O] = 0,55 mol/L f) Il s’agit d’un équilibre homogène, l’eau n’est pas le solvant mais un réactif. L’eau apparaît donc dans l’expression de la constante d’équilibre.

35.

K=

[H 2 ]2 [O 2 ] (1,9 ×10-2 ) 2 [O 2 ] -3 , [O2] = 0,080 mol/L , 2,4 ×10 = [H 2 O]2 (0,11) 2

© 2007 Les Éditions CEC inc. Toute reproduction interdite

Chapitre 4 Équilibre chimique

111

36.

2 PNOBr (0,768) 2 , 109 = 2 KP = 2 , PNO = 0,0583 atm PNO × PBr2 PNO × 0,0159

37.

SO2(g) + NO2(g) ⇌ SO3(g) + NO(g) K =

[SO3] [NO] [SO2] [ NO2]

Pour déterminer K, il faut calculer les concentrations à l’équilibre. Initialement, [SO3]o = [NO]o = 0 ; [SO2]o = [NO2]o =

2,00 mol = 2,00 mol/L 1,00 L

À l’équilibre, [NO] = 1,30 mol/L, donc : [SO3] = [NO] = 0 + 1,30 mol/L = 1,30 mol/L ; [SO2] = [NO2] = 2,00 mol/L – 1,30 mol/L = 0,70 mol/L Donc, K=

38.

[SO3] [NO] (1,30 mol/L) (1,30 mol/L) = 3,4 = [SO2] [ NO2] (0,70 mol/L) (0,70 mol/L)

S8(g) ⇌ 4 S2(g)

KP =

PS42 PS8

Initiale : PS8 = 1,00 atm et PS2 = 0 atm Changement : étant donné qu’il reste 0,25 atm de S8 à l’équilibre, 1,00 atm – 0,25 atm = 0,75 atm de S8 doit avoir réagi afin d’atteindre l’équilibre. Étant donné qu’il y a un rapport molaire de 4:1 entre S2 et S8 (selon l’équation équilibrée), 4(0,75 atm) = 3,0 atm de S2 doivent avoir été produites lorsque la réaction a atteint l’équilibre (les moles et les pressions sont directement proportionnelles, à T et V constants). Équilibre : PS8 = 0,25 atm, PS2 = 0 + 3,0 atm = 3,0 atm; en résolvant pour trouver Kp : KP =

(3, 0) 4 = 3,2 × 102 0, 25

39. Initiale Changement Équilibre

2 NH3(g)



N2(g)

4,0 mol/2,0 L –2x 2,0 – 2x



0 +x x

+

3 H2(g)

K=

[ N 2] [H 2 ]3 [ NH3 ]2

0 +3x 3x

© 2007 Les Éditions CEC inc. Toute reproduction interdite

Chapitre 4 Équilibre chimique

112

Selon les données : [NH3]é = 2,0 mol/2,0 L = 1,0 mol/L = 2,0 – 2x, x = 0,50 mol/L [N2] = x = 0,50 mol/L ; [H2] = 3x = 3(0,50 mol/L) = 1,5 mol/L K=

[ N 2] [H 2 ]3 (0,50 mol/L) (1,5 mol/L )3 = 1,7 mol2/L2 = 2 2 [ NH3 ] (1,0 mol/L )

3 H2(g)

40.

+

N2(g)



2 NH3 (g)

Initiale

[H2]o [N2]o 0 x mol/L N2 réagit pour atteindre l’équilibre Changement -3x -x → +2x Équilibre [H2]o - 3x [N2]o - x 2x Selon le problème: [NH3]é = 4,0 mol/L = 2x, x = 2,0 mol/L; [H2]é = 5,0 mol/L = [H2]o -3x; [N2]é = 8,0 mol/L = [N2]o -x 5,0 mol/L = [H2]o - 3(2,0 mol/L), [H2]o = 11,0 mol/L; 8,0 mol/L = [N2]o – 2,0 mol/L, [N2]o = 10,0 mol/L

N2(g)

41.

+

3 H2(g)



2 NH3(g)

Initiale

1,00 atm 2,00 atm 0 x atm N2 réagit pour atteindre l’équilibre Changement -x -3x → +2x Équilibre 1,00 - x 2,00 - 3x 2x D’après le tableau : PTOT = 2,00 atm = PN2 + PH2 + PNH3 2,00 atm = (1,00 – x) + (2,00 – 3x) + 2x = 3,00 – 2x x = 0,500 atm; PH2 = 2,00 – 3x = 2,00 – 3(0,500) = 0,50 atm

© 2007 Les Éditions CEC inc. Toute reproduction interdite

Chapitre 4 Équilibre chimique 42.

113

Q = 1,00 < K. Il y aura déplacement vers la droite.

SO2(g) Initiale Changement Équilibre K=

+

NO2(g)

1,00 mol/L –x 1,00 – x



SO3(g)

1,00 mol/L –x → 1,00 – x

+

1,00 mol/L +x 1,00 + x

NO(g)

K = 2,50

1,00 mol/L +x 1,00 + x

(1,00 + x) 2 [SO3] [NO] = 2,50 = ; en prenant la racine carrée des deux côtés : [SO2] [ NO2] (1,00 - x) 2

1,00 + x = 1,58, 1,00 + x = 1,58 – 1,58 x, 2,58 x = 0,58, x = 0,22 mol/L 1,00 - x Les concentrations à l’équilibre deviennent : [SO3] = [NO] =1,00 + x = 1,00 + 0,22 = 1,22 mol/L ; [SO2] = [NO2] = 1,00 – x = 0,78 mol/L 43.

Q = 1,00 < K ; déplacement vers la droite.

H2(g) Initiale Changement Équilibre

+

1,00 mol/L –x 1,00 – x

I2(g)



1,00 mol/L –x → 1,00 – x

2 HI(g)

K=

[HI ]2 = 100 [ H 2] [ I2]

1,00 mol/L +2x 1,00 + 2x

(1,00 + 2x) 2 K = 100 = ; en prenant la racine carrée des deux côtés : (1,00 - x) 2 10,0 =

1,00 + 2x , 10,0 – 10,0 x = 1,00 + 2x, 12,0 x = 9,0, x = 0,75 mol/L 1,00 - x

[H2] = [I2] = 1,00 – 0,75 = 0,25 = 0,25 mol/L ; [HI] = 1,00 + 2(0,75) = 2,50 mol/L

44.

Puisqu’il n’y a que des réactifs à l’état initial, la réaction doit se déplacer vers la droite. N2(g) Initiale Changement Équilibre

0,80 atm –x 0,80 – x

+



2 NO(g)

0,20 atm –x → 0,20 – x

0 +2x 2x

O2(g)

Kp = 0,050

© 2007 Les Éditions CEC inc. Toute reproduction interdite

Chapitre 4 Équilibre chimique

114 2

2

(2 x ) P NO = Kp = 0,050 = , 0,050(0,16 – 1,00 x + x2) = 4x2 (0,80 − x)(0, 20 − x) P N2 × P O2 4x2 = 8,0 × 10–3 – 0,050x + 0,050x2, 3,95x2 + 0,050x – 8,0 × 10–3 = 0 x = 3,9 × 10–2 atm ou x = –5,2 × 10–2 atm ; Seul x = 3,9 × 10–2 atm est plausible (x ne peut être négatif), alors PNO à l’équilibre est : –2 –2 P NO = 2x = 2(3,9 × 10 atm) = 10 atm

2SO2(g)

45. Initiale Changement Équilibre

50,7 –2x 50,7 – 2x



2SO3(g)

50,7 –x → 50,7 – x

0 +2x 2x

+ O2(g)

Kp = 0,25

kPa kPa kPa

2 2 ( P SO3) 2, 47 x 10-3 (2 x) 2 (2x) Kp = = ≈ = 2 2 2 kPa ( P SO 2) × ( P O 2) (50,7 - 2x) (50,7 - x) (50, 7) (50, 7)

x = 8,97 et n’est pas négligeable par rapport à 50,7

8,97/50,7 > 5 % Procédons par approximations successives. 2,47 × 10–3 =

(2x) 2 , x = 5,26 (50,7 - 17,96) 2(50,7 - 8,97)

2,47 × 10–3 =

(2x) 2 , x = 6,73 (50,7 - 10,52) 2(50,7 - 5,26)

(2x) 2 2,47 × 10 = , x = 6,14 (50,7 - 13,46) 2(50,7 - 6,73) –3

(2x) 2 2,47 × 10 = , x = 6,37 (50,7 - 12,28) 2(50,7 - 6,14) –3

2,47 × 10–3 =

(2x) 2 , x = 6,28 (50,7 - 12,74) 2(50,7 - 6,37)

© 2007 Les Éditions CEC inc. Toute reproduction interdite

Chapitre 4 Équilibre chimique

115

(2x) 2 2,47 × 10 = , x = 6,31 (50,7 - 12,56) 2(50,7 - 6,28) –3

x = 6,3 kPa

P SO 2 = 50,7 – 2x = 50,7 – (2 × 6,3) = 38,1 kPa P O 2 = 50,7 – x = 50,7 – 6,3 = 44,4 kPa P SO3 = 2x = 2 × 6,3 = 12,6 kPa 46.

a) La réaction doit se déplacer vers les produits pour atteindre l’équilibre. Si on résume le problème dans un tableau où x atm N2O4 réagit pour atteindre l’équilibre :

N2O4(g) Initiale Changement Équilibre Kp =

2 PNO 2

PN2O4

=

4.5 atm -x 4.5 - x

⇌ →

2 NO2(g)

Kp = 0,25

0 +2x 2x

(2x) 2 , 4x2 = 1,125 – 0,25 x, 4x2 + 0,25 x – 1,125 = 0 4,5 - x

Dans cette expression, on garde des chiffres significatifs supplémentaires (comme c’est l’habitude dans la résolution d’une expression à l’aide de la formule quadratique). Si l’on utilise l’équation quadratique, on obtient (Annexe A1.4 du manuel) :

−0, 25 ± [(0, 25) 2 − 4(4)(−1,125)]1/ 2 −0, 25 ± 4, 25 = x= , x = 0,50 2(4) 8 (L’autre valeur est négative.)

PNO2 = 2x = 1,0 atm; PN2O4 = 4,5 - x = 4,0 atm b) La réaction doit se déplacer vers les réactifs (vers la gauche) pour atteindre l’équilibre.

Initiale Changement Équilibre Kp =

N2O4(g)



2 NO2(g)

0 +x x



9,0 atm -2x 9,0 - 2x

(9, 0 − 2 x) 2 = 0,25, 4x2 – 36,25 x + 81 = 0 (chiffres significatifs suppl.) x

À l’aide de la formule quadratique, on obtient :

© 2007 Les Éditions CEC inc. Toute reproduction interdite

Chapitre 4 Équilibre chimique

116

x=

−(−36, 25) ± [(−36, 25) 2 − 4(4)(−81)]1/ 2 , x = 4,0 atm 2(4)

L’autre valeur, 5,1, est impossible. PN 2O4 = x = 4,0 atm; PNO2 = 9,0 - 2x = 1,0 atm c) Non, on obtient la même position d’équilibre en commençant soit avec N2O4 pur ou NO2 pur en quantités stoechiométriques. 47.

2NOCl(g)

a)



2NO(g)

+

Cl2(g)

K = 1,6 × 10–5

Initiale

2,0 2,0

0

0

mol/L

Initiale

1,0

0

0

mol/L

Changement

–2x

+2x

+x

mol/L

2x

x

mol/L

Équilibre K=



1,0 mol/L – 2x

[NO ]2 [Cl2] [NOCl ]2

K = 1,6 × 10–5 K étant petit, x est petit et on suppose que (1,0 – 2x) ≈ 1,0.

1,6 × 10–5 =

(2x) 2 (x) (1,0) 2

x = 0,0159 ≈ 0,016

Voyons si l’approximation est valable. 1,0 – 2x = 0,968, la différence entre 1,0 et 0,968 e st plus petite que 5 % et on considère l’approximation comme valable. [NOCl] = 1,0 – 2x = 0,968 → 1,0 mol/L [NO] = 2x = 2 × 0,016 = 0,032 mol/L [Cl2] = x = 0,016 mol/L 2NOCl(g)

b) Initiale Changement Équilibre

1,0 mol/L –2x 1,0 – 2x



2NO(g)



1,0 mol/L +2x 1,0 + 2x

© 2007 Les Éditions CEC inc. Toute reproduction interdite

+

Cl2(g) 0 +x x

Chapitre 4 Équilibre chimique

117

(1,0 + 2x) 2(x) (1,0) 2(x) 1,6 × 10 = ≈ (en supposant que 2x > 1 et les acides faibles ont une Ka < 1. Le tableau 5.2 dans le manuel donne la liste des valeurs de quelques Ka pour les acides faibles. Les valeurs de Ka pour les acides forts sont diffiles à déterminer de sorte qu’elles ne sont pas données dans le manuel. Cependant, il n’y a que quelques acides forts courants et si vous les mémorisez, tous les autres acides seront des acides faibles. Les acides forts qu’il faut mémoriser sont: HCl, HBr, HI, HNO3, HClO4 et H2SO4. a) HClO4 est un acide fort. b) HOCl est un acide faible (Ka = 3,5 × 10 −8 ). c) H2SO4 est un acide fort. d) H2SO3 est un acide faible dont les valeurs de Ka1 et de Ka2 sont inférieures à l’unité.

27.

Le bécher à gauche représente un acide fort en solution ; l’acide, HA, est dissocié à 100 % en ions H+ et A-. Le bécher à droite représente un acide faible en solution ; seulement une petite partie de l’acide, HB, se dissocie en ions, de sorte que l’acide existe surtout sous forme de molécules non dissociées HB dans l’eau. a) HNO2, bécher acide faible ; b) HNO3, bécher acide fort ; c) HCl, bécher acide fort ; d) HF, bécher acide faible ; CH3COOH, bécher acide faible.

© 2007 Les Éditions CEC inc. Toute reproduction interdite

Chapitre 5 Acides et bases

138 28.

L’acidité est directement proportionnelle à la valeur de Ka. HNO3 > HOCl > NH4+ > H2O

↑ plus grande Ka 29.

↑ plus faible Ka

Plus un acide est fort, plus sa base conjuguée est faible. Ainsi, la base conjuguée NO3– de l’acide fort HNO3 est tellement faible qu’elle n’est pas basique dans l’eau. NH3 > OCl- > H2O > NO3-.

30.

a) HClO4 > H2O b) HClO2 > H2O c) HF > HCN

31.

a) H2O > ClO4– b) ClO2– > H2O c) CN– > F–

Auto-ionisation de l’eau et échelle de pH 32.

Si [H+] = [OH–] = 1,0 × 10–7 mol/L, la solution est neutre. Si [H+] > 1,0 × 10–7 mol/L et [OH–] < 1,0 × 10–7 mol/L, la solution est acide. Si [H+] < 1,0 × 10–7 mol/L et [OH–] > 1,0 × 10–7 mol/L, la solution est basique. a) Neutre. b) Acide. c) Basique. d) Acide.

33.

a) Basique; pOH = 1,08; pH = 12,9 b) acide; pOH = 10,09; pH = 3,9 c) Neutre; pOH = 7; pH = 7 d) Basique.

34.

a) Puisque la valeur de la constante d’équilibre augmente lorsque la température augmente, c’est donc qu’une augmentation de la température favorise la réaction vers la droite et que la réaction est endothermique.

© 2007 Les Éditions CEC inc. Toute reproduction interdite

Chapitre 5 Acides et bases

139

b) [H+][OH–] = 5,47 × 10–14 et [H+] = [OH–] [H+]2 = 5,47 × 10–14 → [H+] = 2,34 × 10–7 mol/L 35.

a) H2O(l) ⇌ H+(aq) + OH− (aq)

Ke = 2.92 × 10 −14 = [H+][OH−]

Dans l’eau pure [H+] = [OH−], donc 2,92 × 10 −14 = [H+]2, [H+] = 1,71 × 10 −7 mol/L = [OH−] b) pH = -log [H+] = -log (1,71 × 10 −7 ) = 6,767 c) [H+] = Ke/[OH−] = 2,92 × 10 −14 /0,10 = 2,9 × 10 −13 mol/L; pH = -log (2,9 × 10 −13 ) =12,54

36.

pH = –log [H] ; pOH = –log [OH–] ; à 25 °C, pH + pOH = 14,00 a) pH = –log [H+] = –log (1,0 × 10–7) = 7,00 ; pOH = 14,00 – pH = 14,00 – 7,00 = 7,00 b) pH = –log (1,4 × 10–3) = 2,85 ; pOH = 14,00 – 2,85 = 11,15 c) pH = –log (2,5 × 10–10) = 9,60 ; pOH = 14,00 – 9,60 = 4,40 d) pH = –log (6,1) = –0,79 ; pOH = 14,00 – (–0,79) = 14,79

On remarque que le pH est inférieur à zéro dans des solutions très acides.

37.

a) [H+] = 3,9 × 10-8 mol/L; [OH-] = 2,6 × 10-7 mol/L; b) [H+] = 5 × 10-16 mol/L; [OH-] = 20 mol/L; c) [H+] = 10 mol/L; [OH-] = 1 × 10-15 mol/L; d) [H+] = 6 × 10-4 mol/L; [OH-] = 1,7 × 10-11 mol/L; e) [H+] = 1 × 10-9 mol/L; [OH-] = 1 × 10-5 mol/L; f) [OH-] = 2,5 × 10-10 mol/L; [H+] = 4 × 10-5 mol/L.

38.

a) pOH = 14,00 – 6,88 = 7,12; [H+] = 10−6,88 = 1,3 × 10 −7 mol/L

[OH−] = 10−7,12 = 7,6 × 10 −8 mol/L; acide

1, 0 × 10−14 b) [H ] = = 0,12 mol/L; pH = -log(0,12) = 0,92 8, 4 × 10−14 +

pOH = 14,00 – 0,92 = 13,08; acide

© 2007 Les Éditions CEC inc. Toute reproduction interdite

Chapitre 5 Acides et bases

140

c) pH = 14,00 – 3,11 = 10,89; [H+] = 10−10,89 = 1,3 × 10 −11 mol/L

[OH−] = 10−3,11 = 7,8 × 10 −4 mol/L; basique −7 d) pH = -log (1,0 × 10 ) = 7,00; pOH = 14,00 – 7,00 = 7,00

[OH−] = 10−7,00 = 1,0 × 10 −7 mol/L; neutre

39.

pH = 14,00 – pOH = 14,00 – 5,74 = 8,26 ; [H+] = 10–pH = 10–8,26 = 5,5 × 10–9 mol/L [OH–] =

-14 K eau = 1,0 × 10 = 1,8 × 10–6 mol/L où [H +] 5,5 × 10-9

[OH–] = 10–pOH = 10–5,74 = 1,8 × 10–6 mol/L La solution est basique puisque son pH est supérieur à 7,00.

Solutions d’acides 40.

HClO4 et HNO3 sont des acides forts complètement dissociés dans l’eau. a) [H+] = 0,250 mol/L ; [ClO4–] = 0,250 mol/L ; pH = 0,602. b) [H+] = 0,250 mol/L ; [NO3–] = 0,250 mol/L ; pH = 0,602.

41.

[HCl] =

0,050 mol mol 50,0 mL × = 0,0125 (0,013) 200,0 mL L L

[HNO3] =

0,10 mol 150,0 mL mol × = 0,075 200,0 mL L L

HCl(aq) → H+(aq) + Cl–(aq) HNO3(aq) → H+(aq) + NO3–(aq)

[Cl–] =

0,0125 mol HCl 1 mol Cl_ mol × = 0,0125 (0,013) mol HCl L L

© 2007 Les Éditions CEC inc. Toute reproduction interdite

Chapitre 5 Acides et bases

141

0,075 mol HNO3 1 mol NO3mol [NO3 ] = × = 0,075 L mol HNO3 L –

⎛ 0,0125 mol HCl 1 mol H + ⎞ ⎛ 0,075 mol HNO3 1 mol H + ⎞ + × × ⎟ ⎟ ⎜ L mol HNO3 ⎠ L mol HCl ⎠ ⎝ ⎝ mol = 0,088 L

[H+] = ⎜

42.

90,0 × 10–3 L ×

5,00 mol = 0,450 mol H+ de HCl L

30,0 × 10–3 L ×

8,00 mol = 0,240 mol H+ de HNO3 L

[H+] = 0,450 mol + 0,240 mol = 0,690 mol/L ; pH = –log (0,690) = 0,161 pOH = 14,000 – 0,161 = 13,839 ; [OH–] = 10–13,839 = 1,45 × 10–14 mol/L 43.

[H+] = 10-1,50 = 3,16 10-2 mol/L (avec un C.S. supplémentaire) ; c1V1 = c2V2

V1 =

c2V2 3,16 × 10-2 mol/L × 1,6 L = = 4,2 × 10-3 L 12 mol/L c1

Ajouter 4,2 mL de HCl 12 mol/L à suffisamment d’eau pour obtenir 1600 mL de solution. La solution résultante aura une [H+] = 3,2 × 10-4 et un pH de 1,50. 44.

[H+] = 10–pH = 10–4,25 = 5,6 × 10–5 mol/L. Une solution de HNO3 5,6 × 10–5 mol/L est requise pour produire un pH de 4,25.

45.

a) HNO2 (Ka = 4,0 × 10–4) et H2O (Ka = Keau = 1,0 × 10–14) sont les espèces majeures. Comme HNO2 est un acide faible, il faut faire un tableau de réaction en considérant qu’il est la principale source de H+.

HNO2 Initiale Changement Équilibre Ka =

0,25 mol/L –x 0,250 –x



H+



~0 –x – x

+

NO2– 0 –x x

2 [H + ][ NO 2−] x = 4,0 × 10–4 = ; supposons que x H2O > NO3− (Étant la base conjuguée d’un acide fort, NO3− n’a aucune capacité basique dans l’eau.)

61.

HNO3 > NH4+ > CH3NH3+ > H2O.

62.

b) NH3 c) OH– d) CH3NH2 a) NH3 La base la plus forte est celle qui a le plus grand Kb. OH– est la base la plus forte en milieu aqueux.

63.

a) HNO3 ; b) NH4+ ; c) NH4+.

Pour calculer Ka, on utilise l’égalité Ka Kb = Keau. 64.

NaOH(aq) → Na+(aq) + OH–(aq) ; a) [OH–] = 0,10 mol/L ; pOH = – log [OH–] = – log (0,10) = 1,00

pH = 14,00 – pOH = 14,00 – 1,00 = 13,00 Bien que l’eau soit présente, la quantité de OH– produite par H2O est insignifiante par rapport à la quantité produite par NaOH. b) La concentration de OH– produite par NaOH = 1,0 × 10–10 mol/L; comme l’eau en produit

1,0 × 10–7 mol/L, [OH–]totale = 1,0 × 10–7 mol/L. pOH = –log (1,0 × 10–7) = 7,00; pH = 14,00 – 7,00 = 7,00 c) [OH–] = 2,0 mol/L; pOH = –log (2,0) = –0,30 ; pH = 14,00 – (–0,30) = 14,30 65.

a) Ca(OH)2 → Ca2+ + 2 OH− ; Ca(OH)2 est une base forte qui se dissocie complètement.

[OH−] = 2(0,00040) = 8,0 × 10 −4 mol/L; pOH = -log [OH−] = 3,10; pH = 14,00 - pOH = 10,90 b)

25 g KOH 1 mol KOH × = 0,45 mol KOH/L L 56,11 g KOH

© 2007 Les Éditions CEC inc. Toute reproduction interdite

Chapitre 5 Acides et bases

151

KOH est une base forte, donc [OH−] = 0,45 mol/L; pOH = -log (0,45) = 0,35; pH = 13,65

c)

150,0 g NaOH 1 mol × = 3,750 mol/L; NaOH est une base forte, L 40,00 g donc [OH−] = 3,750 mol/L. pOH = -log (3,750) = -0,5740 et pH = 14,0000 - (-0,5740) = 14,5740 Bien qu’il soit justifié de calculer la réponse à quatre décimales, en réalité, les valeurs de pH sont généralement mesurées à deux décimales, parfois trois.

66.

a) Principales espèces : Na+, Li+, OH–; H2O (NaOH et LiOH sont deux bases fortes).

[OH–] = 0,050 + 0,050 = 0,100 mol/L; pOH = 1,000; pH = 13,000 b) Principales espèces : Ba2+, Rb+, OH–, H2O; Ba(OH)2 et RbOH sont des bases fortes et Ba(OH)2 produit 2 mol OH– par mol Ba(OH)2.

[OH–] = 2(0,0010) + 0,020 = 0,022 mol/L; pOH = –log (0,022) = 1,66; pH = 12,34 67.

pOH = 14,00 – 11,56 = 2,44; [OH−] = [KOH] = 10−2,44 = 3,6 × 10 −3 mol/L 0,8000 L ×

68.

3,6 ×10−3 mol KOH 56,11 g KOH × = 0,16 g KOH L mol KOH

pH = 13,00 ; pOH = 14,00 – 13,00 = 1,00 ; [OH–] = 10–1,00 = 0,10 mol/L Ca(OH)2(aq) → Ca2+(aq) + 2OH–(aq) ; [Ca(OH)2] =

69.

0,10 mol OH − 1 mol Ca(OH) 2 × = 0,050 mol Ca(OH)2/L 2 mol OH − L

Les principales espèces sont NH3 et H2O, et NH3 est le principal producteur de OH–. NH3(aq) + H2O(l) Initiale 0,150 mol/L Changement –x Équilibre 0,150 – x

Kb = 1,8 × 10–5 =



NH4+(aq)



0 +x x

+

OH–(aq) ~0 +x x

2 2 [ NH +4 ] [OH − ] x x ≈ (en supposant que x NH4Cl > KCl > KCN > KOH 82.

Voir l’exercice 5.81 pour les principes de base. Ca(NO3)2 donne Ca2+ (neutre, d) et 2NO3– (neutre, c) : solution neutre. NaNO2 donne Na+ (neutre, d) et NO2–, base conjuguée de l’acide HNO2 : solution basique. HNO3 : acide fort. NH4NO3 donne NH4+ (acide conjugué de NH3) et NO3– (neutre, c) : solution acide.

© 2007 Les Éditions CEC inc. Toute reproduction interdite

Chapitre 5 Acides et bases

159

Ca(OH)2 : base forte. Du plus acide au plus basique, on a donc : HNO3 > NH4NO3 > Ca(NO3)2 > NaNO2 > Ca(OH)2 83.

1× 10−14 = 1× 10-14 = 3,3 × 10–7 K b, OCl_ = K a, HOCl 3 × 10-8

K b, CH3COO



=

1×10−14 = 1×10−14 = 5,6 × 10–10 −5 K a, CH 3COOH 1,8 ×10

OCl– est une base plus forte que CH3COO–. L’acide le plus faible (HOCl) possède la base conjuguée la plus forte. 84.

Comme NH3 est une base plus faible que CH3NH2, son acide conjugué NH4+ est plus fort que CH3NH3+.

85.

NaN3 → Na+ + N3– ; N3– est la base conjuguée de l’acide faible HN3, Na+ est neutre. N3– + H2O Initiale 0,010 mol/L Changement –x Équilibre 0,010 – x



HN3 + OH– Kb =



0 +x x

−14 K eau = 1,0 × 10 = 5,3 × 10–10 −5 1,9 × 10 Ka

~0 +x x

2 2 [HN 3][OH − ] x x –10 Kb = = 5,3 × 10 = ≈ (en supposant que x K a, NH+4

Relations entre la structure et la force des acides et des bases 93.

a) HBrO < HBrO2 < HBrO3 ; l’acidité augmente avec le nombre d’oxygène ; b) HIO2 < HBrO2 < HClO2 ; la force de l’acide est directement proportionnelle à l’électronégativité de l’atome central ; c) HBrO3 < HClO3 ; même raisonnement qu’en b ; d) H2SO3 < H2SO4 ; même raisonnement qu’en a.

94.

a) BrO3– < BrO2– < BrO– ; ces bases conjuguées des acides de l’exercice 93 a) sont d’autant plus faibles que leur acide conjugué est fort.

© 2007 Les Éditions CEC inc. Toute reproduction interdite

Chapitre 5 Acides et bases

164

b) ClO2– < BrO2– < IO2– ; même raisonnement pour ces bases conjuguées des acides de l’exercice 93 b). 95.

a) H2O < H2S < H2Se ; la force de l’acide augmente en fonction inverse de l’énergie de liaison ; b) CH3COOH < FCH2COOH < F2CHCOOH < F3CCOOH ; la force de l’acide augmente en fonction du nombre d’atomes électronégatifs voisins ; c) NH4+ < HONH3+ ; même raisonnement qu’en b ; d) NH4+ < PH4+ ; même raisonnement qu’en a.

96.

Plus un acide est fort, plus sa base conjuguée est faible. a) OH– > SH– > HSe– ; voir ex. 95 a. b) PH3 < NH3 ; voir ex. 95 b. c) HONH2 < NH3 ; voir ex. 95 c.

97.

a) Basique ; CaO(s) + H2O(l) → Ca(OH)2(aq) ; b) Acide ; SO2(g) + H2O(l) → H2SO3 (aq) ; c) Acide ; Cl2O(g) + H2O(l) → 2 HOCl(aq).

Acides et bases de Lewis 98.

99.

Acide de Lewis Base de Lewis a) B(OH)3

H2O

b) Ag+

NH3

c) BF3

NH3

a) acide : I2, base : I- ; b) acide : Zn(OH)2, base : OH- ; c) acide : Fe3+, base : SCN-.

100. Al(OH)3 comme base :

Al(OH)3 + 3H+ → Al3+ + 3H2O Al(OH)3 comme acide Al(OH)3 + OH– → Al(OH)4– 101. Zn(OH)2(s) + 2H+(aq) → Zn2+(aq) + 2H2O(l) (Zn(OH)2 = base de Lewis) ;

Zn(OH)2(s) + 2OH-(aq) → Zn(OH)42-(aq) (Zn(OH)2 = acide de Lewis). © 2007 Les Éditions CEC inc. Toute reproduction interdite

Chapitre 5 Acides et bases

165

102. Étant plus petit et portant une charge plus grande, Fe3+ devait être l’acide le plus fort, celui le plus apte à attirer fortement des doublets d’électrons de bases de Lewis. 103. Structures de Lewis :

La base de Lewis H2O cède un doublet d’électrons au carbone de l’acide de Lewis CO2, puis un proton H+ est déplacé pour former l’acide H2CO3. EXERCICES SUPPLÉMETAIRES 104. À pH = 2,000, [H+] = 10−2,000 = 1,00 × 10 −2 mol/L;

à pH = 4,000, [H+] = 10−4,000 = 1,00 × 10 −4 mol/L mol H+ présentes = 0,0100 L ×

0,0100 mol H + = 1,00 × 10 −4 mol H+ L

Soit V = volume total de solution à pH = 4,000 : 1,00 × 10 −4 mol/L =

1,00 ×10−4 mol H + , V = 1,00 L V

Volume d’eau ajoutée = 1,00 L - 0,0100 L = 0,99 L = 990 mL 105. Les paires des parties a, c et d sont des couples acide-base conjugués. À la partie b, HSO4est la base conjuguée de H2SO4. De plus, HSO4- est l’acide conjugué de SO42-. 106. Soit HSac = saccharine et c0 = [HSac]o.

HSac Initiale Équilibre

c0 c0 - x

Ka = 2,0 × 10

−12



H+ ~0 x

+

− S sac

Ka = 10−11,70 = 2,0 × 10 −12

0 x

x2 = ; x = [H+] = 10−5,75 = 1,8 × 10 −6 mol/L c0 − x

© 2007 Les Éditions CEC inc. Toute reproduction interdite

Chapitre 5 Acides et bases

166

2,0 × 10 −12 =

(1,8 ×10−6 )2 , c0 = 1,6 mol/L = [HSac]o. c0 − 1,8 ×10−6

100,0 g HC7H4NSO3 ×

1 mol 1L 1000 mL = 340 mL × × 183,19 g 1,6 mol L

107. L’ampoule brille fortement à cause de la présence d’un électrolyte fort, c.-à-d., un soluté qui se dissout pour produire beaucoup d’ions en solution. La valeur de 4,6 au pH-mètre indique la présence d’un acide faible. (Si un acide fort était présent, le pH serait près de zéro.) Parmi les substances possibles, seulement HCl (acide fort), NaOH (base forte) et NH4Cl sont des électrolytes forts. De ces trois substances, seul NH4Cl contient un acide faible (la solution de HCl aurait un pH près de zéro et la solution de NaOH, un pH près de 14,0). NH4Cl se dissocie en ions NH4+ et en ions Cl- lorsqu’il est dissous dans l’eau. Cl- est la base conjuguée de l’acide fort, de sorte qu’il ne présente aucune propriété basique (ni acide) dans l’eau. NH4+, par contre, est l’acide conjugué de la base faible NH3, de sorte que NH4+ est un acide faible et produirait une solution de pH = 4,6 lorsque sa concentration est ~ 1 mol/L. 108. HBz = C6H5COOH

c0 = [HBz]o = concentration de HBz qui se dissout pour donner une solution saturée. HBz



H+

+

Bz−

c0 ~0 0 x mol/L HBz se dissocie pour atteindre l’équilibre. Changement -x → +x +x Équilibre c0 - x x x Initiale

2

x [H + ][Bz − ] Ka = = 6,4 × 10 −5 = , où x = [H+] [HBz] c0 − x 6,4 × 10 −5 =

[ H + ]2 ; pH = 2,80; [H+] = 10−2,80 = 1,6 × 10 −3 mol/L + c0 − [ H ]

c0 - 1,6 × 10 −3 =

(1, 6 × 10−3 ) 2 = 4,0 × 10 −2 , c0 = 4,0 × 10 −2 + 1,6 × 10 −3 6.4 × 10−5 = 4,2 × 10 −2 mol/L

La solubilité molaire de C6H5COOH is 4,2 × 10 −2 mol/L.

© 2007 Les Éditions CEC inc. Toute reproduction interdite

Chapitre 5 Acides et bases

167

109. Parce que K a 2 pour H2S est très petit, on peut négliger la contribution en H+ de la réaction

Ka2 .



H2S Initiale Équilibre

0,10 mol/L 0,10 - x

K a1 = 1,0 × 10 −7 =

H+

HS−

~0 x

0 x

K a1 = 1,0 × 10 −7

x2 x2 ≈ , x = [H+] = 1,0 × 10 −4 ; approximation valide. 0,10 − x 0,10

pH = -log (1,0 × 10 −4 ) = 4,00 À l’aide de la réaction de K a 2 , on détermine [S2−].

Initiale Équilibre

⇌ H+ + HS− −4 −4 1,0 × 10 mol/L 1,0 × 10 mol /L 1,0 × 10 −4 - x 1,0 × 10 −4 + x

K a2 = 1,0 × 10 −19 =

S2− 0 x

(1, 0 × 10−4 + x) x 1, 0 × 10−4 x ≈ (1, 0 × 10−4 − x) 1, 0 × 10−4

x = [S2−] = 1,0 × 10 −19 mol/l; approximation valide. 110. Pour H2C6H6O6. K a1 = 7,9 × 10 −5 et K a 2 = 1,6 × 10 −12 . Étant donné que K a1 >> K a 2 , la

quantité de H+ produite par la réaction de K a 2 est négligeable.

1 mol H 2 C6 H 6 O6 176,12 g = 0,0142 mol/L 0,2000 L

0,500 g × [H2C6H6O6]o =

H2C6H6O6(aq) Initiale Équilibre



0,0142 mol/L 0,0142 - x

HC6H6O6−(aq) 0 x

+ H+(aq)

K a1 = 7,9 × 10 −5

~0 x

x2 x2 K a1 = 7,9 × 10 = ≈ , x = 1,1 × 10 −3 ; approximation non valide, 0, 0142 − x 0, 0142 −5

ne respecte pas la règle des 5 %. Résolvons par la méthode des approximations successives :

© 2007 Les Éditions CEC inc. Toute reproduction interdite

Chapitre 5 Acides et bases

168

7,9 × 10 −5 =

x2 , x = 1,0 × 10 −3 mol/L (réponse logique) 0, 0142 − 1,1 × 10−3

Étant donné que H+ produit par la réaction de K a 2 est négligeable, [H+] = 1,0 × 10 −3 et pH = 3,00. 111. [H+]o = 1,0 × 10 −2 + 1,0 × 10 −2 = 2,0 × 10 −2 mol/L provenant des acides forts HCl et H2SO4.

HSO4− est un bon acide faible (Ka = 0,012). Cependant, HCN est un acide très faible (Ka = 6,2 × 10 −10 ) et on peut l’ignorer. Si on calcule la contribution de HSO4− en H+ :



HSO4− Initiale Équilibre

Ka =

0,010 mol/L 0,010 - x

H+

SO42−

+

0,020 mol/L 0,020 + x

Ka = 0,012

0 x

x (0, 020 + x) x (0, 020) = 0,012 ≈ , x = 0,0060; (0, 010 − x) (0, 010)

Approximation non valide (60 % d’erreur). À l’aide de la formule quadratique : x2 + 0,032 x - 1,2 × 10 −4 = 0, x = 3,4 × 10 −3 mol/L [H+] = 0,020 + x = 0,020 + 3,4 × 10 −3 = 0,023 mol/L; pH = 1,64 112. Dans ce problème on utilise l’abréviation Hacr pour CH2=CHCOOH et CH2=CHCOO−. a) Résolvons le problème de l’acide faible :



Hacr Initiale Équilibre

x2 0,10 − x

0,10 mol/L 0,10 - x = 5,6 × 10

−5

H+

+

~0 x

acr−

Ka = 5,6 × 10 −5

0 x

x2 ≈ , x = [H+] = 2,4 × 10 −3 M; pH = 2,62; 0,10

approximation valide. b) % de dissociation =

2, 4 × 10−3 [H + ] × 100 = × 100 = 2,4 % [Hacr]0 0,10

© 2007 Les Éditions CEC inc. Toute reproduction interdite

acr− pour

Chapitre 5 Acides et bases

169

c) acr− est une base faible et la principale source de OH − dans la solution.

acr−

Initiale Équilibre

+ H2O



Hacr

0,050 mol/L 0,050 - x

+

0 x

Kb =

~0 x

Kb = 1,8 × 10 −10

2

Kb =

Ke 1, 0 × 10−14 = 5, 6 × 10−5 Ka

OH−

2

x x [Hacr][OH − ] = 1,8 × 10 −10 = ≈ − [acr ] 0, 050 − x 0, 050

x = [OH-] = 3,0 × 10 −6 mol/L; pOH = 5,52; pH = 8,48

Approximation valide.

113. D’après le pH, C7H4ClO2− est une base faible. Utilisez les données de la base faible pour déterminer Kb pour C7H4ClO2− (qui est représenté par l’abréviation CB−).

CB− Initiale Équilibre

+

H2O



0,20 mol/L 0,20 - x

HCB

+

0 x

OH− ~0 x

Étant donné que pH = 8,65, pOH = 5,35 et [OH−] = 10−5,35 = 4,5 × 10 −6 mol/L = x. Kb =

[HCB][OH − ] (4,5 × 10−6 ) 2 x2 = = = 1,0 × 10 −10 − −6 [CB ] 0, 20 − x 0, 20 − 4,5 × 10

Étant donné que CB− est une base faible, HCB, l’acide chlorobenzoïque, est un acide faible. Résolvons le problème de l’acide faible : HCB Initiale Équilibre

Ka =



0,20 mol/L 0,20 - x

H+ ~0 x

0 x

Ke 1, 0 × 10−14 x2 x2 −4 = = 1,0 × 10 = ≈ 5, 6 × 10−10 Kb 0, 20 − x 0, 20

x = [H+] = 4,5 × 10 −3 mol/L; pH = 2,35 114. a)

CB−

+

Fe(H2O)63+ + H2O Initiale 0,10 mol/L Équilibre 0,10 - x



approximation valide. Fe(H2O)5(OH)2+ 0 x

+

H3O+ ~0 x

© 2007 Les Éditions CEC inc. Toute reproduction interdite

Chapitre 5 Acides et bases

170

[Fe(H 2 O)5 (OH) 2+ ][H 3O + ] x2 x2 −3 Ka = = 6,0 × 10 = ≈ [Fe(H 2 O)3+ 0,10 − x 0,10 6 ] x = 2,4 × 10 −2 ; approximation non valide (x = 24 % de 0,10). En résolvant le problème par approximations successives :

x2 = 6,0 × 10 −3 , x = 0,021 0,10 − 0, 024 x2 x2 = 6,0 × 10 −3 , x = 0,022; = 6,0 × 10 −3 , x = 0,022 0,10 − 0, 021 0,10 − 0, 022 x = [H+] = 0,022 mol/L; pH = 1,66 b) Puisque Fe2+(aq) a une charge plus faible que Fe3+(aq), une solution de nitrate de fer(II) (Fe(NO3)2) sera moins acide qu’une solution de nitrate de fer(III) (Fe(NO3)3. 115. Voir la réponse à l’exercice 81. a) HI :

acide fort; HF: acide faible (Ka = 7,2 × 10 −4 )

NaF : F- est la base conjuguée de l’acide faible HF; donc F- est une base faible. La valeur de Kb pour F- = Ke/Ka, HF = 1,4 × 10 −11 . Na+ est neutre. NaI : neutre (pH = 7,0); Na+ et I- sont neutres. Classement en ordre croissant de pH : on place les composés du plus acide (pH le plus bas) au plus basique (pH le plus élevé). Ordre croissant de pH : HI < HF < NaI < NaF. b) NH4Br :NH4+ est un acide faible (Ka = 5,6 × 10 −10 ) and Br- est neutre.

HBr : acide fort KBr : neutre; K+ et Br- sont neutres NH3 : base faible, Kb = 1,8 × 10 −5 Ordre croissant de pH : HBr < NH4Br < KBr < NH3 plus plus acide basique c) NH4NO3 : NH4+ est un acide faible (Ka = 5,6 × 10-10) et NO3- est neutre.

NaNO3:

neutre; Na+ et NO3− sont des espèces neutres.

NaOH:

base forte

© 2007 Les Éditions CEC inc. Toute reproduction interdite

Chapitre 5 Acides et bases

171

HF :

acide faible (Ka = 7,2 × 10-4.

KF :

F- est une base faible (Kb = 1,4 × 10-11) et K+ est neutre.

NH3 :

base faible, Kb = 1,8 × 10 −5

HNO3 :

acide fort.

Ordre croissant de pH: HNO3 < HF < NH4NO3 < NaNO3 < KF < NH3 < NaOH plus plus acide basique 116. Na+ est neutre, mais HSO4− est un acide faible

HSO4− Initiale Équilibre

1,2 × 10

−2



0,10 mol/L ,10 - x

H+ ~0 x

+

SO42−

Ka = 1,2 × 10-2

0 x

[H + ][SO 24− ] x2 x2 = ≈ , x = 0,035 = [HSO 4− ] 0,10 − x 0,10

Approximation non valide (x = 35 % de 0,10). En résolvant l’équation par approximations successives :

x2 x2 = = 1,2 × 10 −2 , x = 0,028 0,10 − x 0,10 − 0, 035 x2 x2 = 1,2 × 10 −2 , x = 0,029; = 1,2 × 10 −2 , x = 0,029 0,10 − 0, 028 0,10 − 0, 029 x = [H+] = 0,029 mol/L; pH = 1,54 117. Les réactions en jeu sont :

H2CO3 ⇌ H+ + HCO3−

K a1 = 4,3 × 10 −7 ;

HCO3− ⇌ H+ + CO32−

K a 2 = 5,6 × 10 −11

Pour commencer, on ne s’occupe que de la première réaction (parce que K a1 >> K a 2 ) et les résultats obtenus contrôlent les valeurs des concentrations dans la deuxième réaction.

© 2007 Les Éditions CEC inc. Toute reproduction interdite

Chapitre 5 Acides et bases

172

H2CO3 Initiale Équilibre



0,010 mol/L 0,010 - x

K a1 = 4,3 × 10 −7 =

H+

HCO3−

~0 x

0 x

[H + ][HCO3− ] x2 x2 = ≈ [H 2 CO3 ] 0, 010 − x 0, 010

x = 6,6 × 10 −5 mol/L = [H+] = [HCO3−] ;

HCO3Initiale Équilibre

+



6,6 × 10 −5 mol/L 6,6 × 10 −5 - y

approximation valide. H+

6,6 × 10 −5 mol/L 6,6 × 10 −5 + y

Si y est petit, alors [H+] = [HCO3−] et K a 2 = 5,6 × 10 −11 = y = [CO32−] = 5,6 × 10 −11 mol/L;

CO32−

+

0 y

[H + ][CO32− ] ≈y [HCO3− ]

Approximation valide.

La quantité de H+ provenant de la deuxième dissociation est 5,6 × 10 −11 mol/L ou :

5, 6 × 10−11 × 100 = 8,5 × 10-5 % de H+ provenant de la deuxième dissociation. −5 6, 6 × 10 Ce résultat justifie de traiter les équilibres séparément. Si la deuxième dissociation contribuait une quantité non négligeable de H+, il faudrait alors traiter les deux équilibres simultanément. La réaction qui se produit quand on ajoute l’acide à la solution de HCO3− est : HCO3-(aq) + H+(aq) → H2CO3(aq) → H2O(l) + CO2(g) Le bouillonnement sont des bulles de CO2(g) qui se forment par la décomposition des molécules H2CO3 instables. Il faudrait écrire H2O(l) + CO2(aq) ou CO2(aq) pour ce qu’on appelle l’acide carbonique. C’est par commodité, toutefois, qu’on écrit H2CO3(aq). 118. HbH44+ + 4O2 ⇌ Hb(O2)4 + 4H+ a) Au niveau des poumons, il y a une grande quantité de O2, la réaction vers la droite est favorisée et la forme Hb(O2)4 est favorisée. b) CO2 est un acide faible, CO2 + H2O ⇌ HCO3− + H+. Si on l’élimine, la concentration de H+ diminue. La forme Hb(O2)4 est favorisée et l’oxygène n’est pas libéré au niveau des cellules.

© 2007 Les Éditions CEC inc. Toute reproduction interdite

Chapitre 5 Acides et bases

173

Respirer dans un sac de papier fait augmenter la quantité de CO2 dans le sang, qui augmente alors la quantité de H+, ce qui déplace la réaction vers la gauche. c) CO2 s’accumule dans le sang qui devient trop acide; l’équilibre est déplacé vers la gauche et l’hémoglobine ne fixe pas O2 aussi efficacement au niveau des poumons. L’ion HCO3- est une base qui peut neutraliser l’acidité en excès. 119. a) H2SO3; acide sulfureux. b) HClO3; acide chlorique c) H3PO3; acide phosphoreux.

NaOH et KOH sont des composés ioniques formés de cations Na+ et K+ et d’anions OH— basiques. Les oxacides comme H2SO3, HClO4 et H3PO3 sont des composés covalents ; quand on les dissout dans l’eau, la liaison covalente entre O et H se brise pour former des ions H+.

PROBLÈMES DÉFIS 120. Le pH de la solution n’est pas 8,00 parce que l’eau donne une quantité importante de H+ par auto-ionisation. On ne peut pas ajouter un acide à de l’eau et obtenir un pH basique. Les principales réactions sont :

H2O ⇌ H+ + OH−

Ke = [H+] [OH−] = 1,0 × 10 −14

HCl → H+ + Cl−

Ka est très grand, de sorte qu’on suppose que seule la réaction directe se produit.

Si la solution est électriquement neutre, alors [charge positive] = [charge négative ], alors [H+] = [OH−] + [Cl−] À partir de Ke, [OH−] = Ke/[H+], et si [HCl] = 1,0 × 10 −8 mol/L, [Cl−] = 1,0 × 10 −8 mol/L. En substituant ces valeurs dans l’égalité précédente :

1,0 ×10−14 + 1,0 × 10 −8 , [H+]2 - 1,0 × 10 −8 [H+] - 1,0 × 10 −14 = 0 [H ] = + [H ] +

© 2007 Les Éditions CEC inc. Toute reproduction interdite

Chapitre 5 Acides et bases

174 Résolvons à l’aide de la formule quadratique : [H+]

−(−1, 0 × 10−8 ) ± [(−1, 0 × 10−8 ) 2 − 4(1)(−1, 0 × 10−14 )]1/ 2 , [H+] = 1,1 × 10 −7 mol/L 2(1)

pH = -log (1,1 × 10 −7 ) = 6,96 121. Comme cette solution est très diluée, il faut tenir compte de deux sources d’ions OH-.

NaOH → Na+ + OH− H2O ⇌ H+ + OH−

Ke = [H+] [OH−] = 1,0 × 10 −14

Comme cette solution doit être électriquement neutre, alors [charge positive] = [charge négative]. [Na+] + [H+] = [OH−], où [Na+] = 1,0 × 10 −7 mol/L et [H+] =

Ke [OH − ]

En substituant les valeurs, on obtient :

1,0 ×10−14 1,0 × 10 + = [OH−], [OH−]2 - 1,0 × 10 −7 [OH−] - 1,0 × 10 −14 = 0 − [OH ] −7

On résout par la formule quadratique : [OH−] =

−(−1, 0 × 10−7 ) ± [(−1, 0 × 10−7 ) 2 − 4(1)(−1, 0 × 10−14 )]1/ 2 2(1)

[OH−] = 1,6 × 10 −7 mol/L; pOH = -log (1,6 × 10 −7 ) = 6,80; pH = 7,20

HA

122. Initiale Équilibre

Ka =



c0 c0 - 1,00 × 10 −4

H+ ~0 1,00 × 10 −4

+

A− 0 1,00 × 10 −4

Ka = 1,00 × 10 −6 c0 = [HA]o; pour pH = 4,000, x = [H+] = 1,00 × 10 −4 mol/L

(1,00 × 10−4 ) 2 = 1,00 × 10 −6 ; c0 = 0,0101 mol/L c0 - 1,00 × 10−4

La solution contient au départ 50,0 × 10 −3 L × 0,0101 mol/L = 5,05 × 10 −4 mol HA. On dilue alors à un volume total, V, en litres. Le pH résultant est 5,000, alors [H+] = 1,00 × 10 −5 . C’est un problème typique d’acide faible, x = [H+], alors :

© 2007 Les Éditions CEC inc. Toute reproduction interdite

Chapitre 5 Acides et bases

175



HA

5,05 × 10 −4 mol/V 5,05 × 10 −4 /V - 1,00 × 10 −5

Initiale Équilibre

Ka =

H+

A−

+

~0 1,00 × 10 −5

0 1,00 × 10 −5

(1, 00 × 10−5 ) 2 = 1,00 × 10 −6 , 1,00 × 10 −4 −4 −5 5, 05 × 10 / V − 1, 00 × 10

= 5,05 × 10 −4 /V - 1,00 × 10 −5 V = 4,59 L; 50,0 mL sont déjà présents, il faut donc ajouter 4540 mL d’eau.



HBrO

123.

1,0 × 10 −6 mol/L

Initiale

H+

+

~0

BrO−

Ka = 2 × 10 −9

0

x mol/L HBrO se dissocie pour atteindre l’équilibre Changement -x Équilibre 1,0 × 10 −6 - x

Ka = 2 × 10

−9



+x x

x2 x2 = ≈ ; 1, 0 × 10−6 − x 1, 0 × 10−6

+x x x = [H+] = 4 × 10 −8 mol/L; pH = 7,4

Vérifions l’approximation. La réponse est impossible! On ne peut pas ajouter une petite quantité d’un acide à l’eau et obtenir une solution basique. Le pH le plus élevé pour un acide dans l’eau est 7,0. Il faut tenir compte de H+ qui provient de la dissociation de l’eau. 124. Les principales espèces présentes sont H2O, C5H5NH+ (Ka = Ke/Kb(C5H5N) = 1,0 × 10 −14 /1,7 × 10 −9 = 5,9 × 10 −6 ) et F− (Kb = Ke/Ka(HF) = 1,0 × 10 −14 /7,2 × 10 −4 = 1,4 × 10 −11 ).

La réaction à prendre en compte est celle du meilleur acide présent (C5H5NH+) qui réagit avec la meilleure base présente (F−). En résolvant pour trouver les concentrations à l’équilibre : C5H5NH+(aq) Initiale 0,200 mol/L Changement -x Équilibre 0,200 - x

+

F−(aq)



0,200 mol/L -x → 0,200 - x

C5H5N(aq) + HF(aq) 0 +x x

0 +x x

© 2007 Les Éditions CEC inc. Toute reproduction interdite

Chapitre 5 Acides et bases

176

K = K a, C H NH + × 5

5

1

K a, HF

= 5,9 × 10 −6 (1/7,2 × 10 −4 ) = 8,2 × 10 −3

2

K=

x [C 5 H 5 N][HF] = 8,2 × 10 −3 = ; en prenant la racine carrée des + − [C5 H 5 NH ][F ] (0, 200 − x) 2

deux membres : 0,091 =

x , x = 0,018 - 0,091 x, x = 0,016 mol/L 0, 200 − x

D’après le tableau de réaction du problème, x = [C5H5N] = [HF] = 0,016 mol/L et 0,200 - x = 0,200 - 0,016 = 0,184 mol/L = [C5H5NH+] = [F−]. Pour résoudre et trouver [H+], on peut soit utiliser la constante d’équilibre Ka pour C5H5NH+ ou Ka pour HF. Si on utilise les données pour C5H5NH+ :

K a, C H NH+ = 5,9 × 10 −6 = 5

5

[C5 H 5 N][H + ] (0,016)[H + ] , [H+] = 6,8 × 10 −5 mol/L = [C5 H 5 NH + ] (0,184)

pH = -log(6,8 × 10 −5 ) = 4,17 Comme on s’y attend, étant donné que Ka pour l’acide faible est plus élevée que Kb pour la base faible, une solution de ce sel doit être acide. 125. Comme NH3 est très concentré, il faut calculer la contribution en OH- de la base faible NH3.

NH3 + Initiale



NH4+

15,0 mol/L

0

+

OH−

Kb = 1,8 × 10 −5

0,0100 mol/L

(On suppose aucune variation de volume.) Équilibre

15,0 - x

Kb = 1,8 × 10 −5 =

x

0,0100 + x

x (0, 0100 + x) x (0, 0100) . , x = 0,027; approximation non valide 15, 0 − x 15, 0 (x = 270% de 0,0100),

En utilisant la formule quadratique : 1,8 × 10 −5 (15,0 - x) = 0,0100 x + x2, x2 + 0,0100 x - 2,7 × 10 −4 = 0 x = 1,2 × 10 −2 , [OH−] = 1,2 × 10 −2 + 0,0100 = 0,022 mol/L

© 2007 Les Éditions CEC inc. Toute reproduction interdite

Chapitre 5 Acides et bases

177

126. Pour une dissociation de 0,0010% : [NH4+] = 1,0 × 10 −5 (0,050) = 5,0 × 10 −7 mol/L

NH3 + H2O

NH4+ + OH−

Kb =

(5,0 × 10−7 )[OH −] = 1,8 × 10 −5 0,050 - 5,0 × 10−7

Résolvons : [OH−] = 1,8 mol/L; en supposant aucun changement de volume : 1,0 L ×

1,8 mol NaOH 40,00 g NaOH = 72 g de NaOH × L mol NaOH

5,11 g 0,08206 L ⋅ atm × × 298 K ρ RT L mol ⋅ K 127. Masse molaire = = = 125 g/mol P 1,00 atm 1 mol 125 g = 0,120 mol/L; pH = 1,80, [H+] = 10−1,80 = 1,6 × 10 −2 mol/L 0,100 L

1,50 g × [HA]o =

HA Équilibre



H+ + A−

0,120 - x

x

x = [H+] = 1,6 × 10 −2 mol/L

x

−2 2

Ka =

(1, 6 × 10 ) [H + ][A − ] = = 2,5 × 10 −3 [HA] 0,120 − 0, 016 CH3COOH

128. Initiale Équilibre

1,8 × 10

−5

1,00 mol/L 1,00 − x



H+ + 0 x

CH3COO−

Ka = 1,8 × 10 −5

0 x

x2 x2 = ≈ , x = [H+] = 4,24 × 10 −3 mol/L 1, 00 − x 1, 00

(on utilise un C.S. supplémentaire) pH = -log (4,24 × 10 −3 ) = 2,37 approximation valide. On veut doubler le pH à 2(2,37) = 4,74 en ajoutant la base forte NaOH. Le meilleur acide présent réagira avec la base forte. C’est CH3COOH. La réaction initiale qui se produit quand une base forte est ajoutée est : CH3COOH + OH− → CH3COO− + H2O

© 2007 Les Éditions CEC inc. Toute reproduction interdite

Chapitre 5 Acides et bases

178

On remarque que cette réaction a comme effet global de convertir CH3COOH en sa base conjugué, CH3COO−. Pour un pH = 4,74, calculons le rapport entre [CH3COO−]/ CH3COOH] nécessaire pour obtenir ce pH. CH3COOH

⇌ H+

+ CH3COO−



Ka =

[H + ][C 2 H 3 O 2 ] [ HC 2 H 3 O 2 ]

Quand pH = 4,74, [H+] = 10−4,74 = 1,8 × 10 −5 .

Ka = 1,8 × 10 −5 =

1,8 ×10−5 [C2 H 3O −2 ] [C2 H 3O −2 ] = 1,0 , [HC2 H 3O 2 ] [HC2 H 3O 2 ]

Pour une solution de pH = 4,74, il faut des concentrations égales (nombre de moles égal) de CH3COO− et de CH3COOH. Par conséquent, il faut ajouter une quantité de NaOH qui convertira la moité de CH3COOH en CH3COO-. Cette quantité est 0,50 mol/L de NaOH. Initiale Changement Après réaction

CH3COOH 1,00 mol/L -0,50 0,50 mol/L

+ OH− → 0,50 mol/L -0,50 0

CH3COO− + 0 +0,50 0,50 mol/L

H2O

Selon le problème de stoechiométrie précédent, en ajoutant suffisamment de NaOH(s) pour produire une solution de OH- à 0,50 mol/L on convertira la moitié de CH3COOH en CH3COO−, ce qui donne une solution de pH = 4,74. masse de NaOH = 1,0 L ×

0,500 mol NaOH 40,00 g NaOH = 20,0 g NaOH × L L

129. PO43− est la base conjuguée de HPO42−. La valeur de Ka pour HPO42− est K a 3 = 4,8 × 10-13.

PO43−(aq) + H2O(l) ⇌ HPO42−(aq) + OH−(aq)

Kb =

Ke 1, 0 × 10−14 = = 0,021 4,8 × 10−13 K a3

HPO42− est la base conjuguée de H2PO4− ( K a 2 = 6,2 × 10 −8 ). HPO42−+ H2O

⇌ H2PO4− + OH−

Kb =

Ke 1, 0 × 10−14 = = 1,6 × 10 −7 6, 2 × 10−8 K a1

H2PO4− est la base conjuguée de H3PO4 ( K a1 = 7,5 × 10 −3 ).

© 2007 Les Éditions CEC inc. Toute reproduction interdite

Chapitre 5 Acides et bases

H2PO4− + H2O



179

H3PO4 + OH−

Kb =

Ke 1, 0 × 10−14 = = 1,3 × 10 −12 7,5 × 10−3 K a1

Selon les valeurs de Kb, PO43− est la base la plus forte. Il fallait s’y attendre puisque PO43− est la base conjuguée de l’acide le plus faible (HPO42−). 130. Principales espèces : Na+, PO43− (une base faible), H2O; d’après les valeurs de Kb calculées dans l’exercice 5.129, le plus important producteur de OH− est la réaction Kb pour PO43−. On peut ignorer la contribution de OH− provenant des réactions Kb pour HPO42− et H2PO4−. D’après l’exercice 5.129, Kb pour PO43− = 0,021.

PO43− +

H2O



HPO42− + OH− 0 x

Kb = 0,021

Initiale Équilibre

0,10 mol/L 0,10 - x

~0 x

Kb = 0,021 =

x2 ; comme Kb est très grande, l’approximation n’est pas valide. 0,10 − x

Résolvons à l’aide de l’équation quadratique :

x2 + 0,021 x – 0,0021 = 0, x = [OH−] = 3,7 × 10 −2 mol/L, pOH = 1,43, pH = 12,57 131. a) NH4(HCO3) → NH4+ + HCO3−

Ka pour NH4+ =

K e 1, 0 × 10−14 1, 0 × 10−14 − −10 = 5,6 × 10 ; K pour HCO = = b 3 1,8 × 10−5 K a1 4,3 × 10−7 = 2,3 × 10 −8

La solution est basique parce que la force de la base HCO3− est plus grande que la force de l’acide NH4+. Les propriétés acides de HCO3− ont été ignorées parce que K a 2 est très petite (5,6 × 10 −11 ). b) NaH2PO4 → Na+ + H2PO4−; on ignore Na+.

K a 2 pour H 2 PO −4 = 6,2 × 10 −8 ; Kb pour H2PO4− =

K e 1, 0 × 10−14 = = 1,3 × 10 −12 K a1 7,5 × 10−3

La solution est acide parce que Ka > Kb.

© 2007 Les Éditions CEC inc. Toute reproduction interdite

Chapitre 5 Acides et bases

180 c) Na2HPO4 → 2 Na+ + HPO42−; on ignore Na+.

K a3 pour HPO 24− = 4,8 × 10 −13 ; Kb pour HPO42− =

K e 1, 0 × 10−14 = = 1,6 × 10 −7 −8 K a 2 6, 2 × 10

La solution est basique parce que Kb > Ka. d) NH4(H2PO4) → NH4+ + H2PO4−

NH4+ est un acide faible et H2PO4− est également acide (voir b). La solution comportant les deux ions est acide. e) NH4(HCO2) → NH4+ + HCO2−;

d’après l’annexe 5, Ka pour HCOOH = 1,8 × 10 −4 .

Ka pour NH4+ = 5,6 × 10 −10 ; Kb pour HCO2− =

K e 1, 0 × 10−14 = = 5,6 × 10 −11 −4 K a 1,8 × 10

La solution est acide parce que la force de l’acide NH4+ est plus grande que la force de la base HCO2−. 132. a) HCO3− + HCO3− ⇌ H2CO3 + CO32−

[H 2 CO3 ][CO32− ] [H + ] K a 2 5, 6 × 10−11 Kéq = = = = 1,3 × 10 −4 × 4,3 × 10−7 K a1 [HCO3− ][HCO3− ] [H + ] b) [H2CO3] = [CO32−] étant donné que la réaction de la partie a est la principale réaction d’équilibre. c) H2CO3

⇌ 2 H+ + CO32−

Kéq =

[H + ]2 [CO32− ] = K a1 × K a 2 [H 2 CO3 ]

Puisque, [H2CO3] = [CO32−] d’après la partie b, [H+]2 = K a1 × K a 2 . [H+] = (K a1 × K a 2 )1/ 2 ou pH =

pK a1 + pK a 2 2

d) [H+] = [(4,3 × 10 −7 ) × (5,6 × 10 −11 )]1/2, [H+] = 4,9 × 10 −9 mol/L; pH = 8,31

© 2007 Les Éditions CEC inc. Toute reproduction interdite

Chapitre 5 Acides et bases

181

1 mol 100,0 g = 2,00 × 10 −3 mol/kg ≈ 2,00 × mol/L 0,5000 kg

0,100 g × 133. Molalité = m =

(solution diluée) ΔTf = iKfm, 0,0056 °C = i(1,86°C·kg/mol)(2,00 × 10 −3 mol/kg), i = 1,5 Si i = 1,0, le pourcentage de dissociation de l’acide = 0 % et si i = 2,0, le pourcentage de dissociation = 100 %. Puisque i = 1,5, l’acide faible est dissocié à 50 %. HA



H+ + A−

Ka =

[H + ][A − ] [HA]

Puisque l’acide faible est dissocié à 50 % : [H+] = [A−] = [HA]o × 0,50 = 2,00 × 10 −3 mol/L × 0,50 = 1,0 × 10 −3 mol/L [HA] = [HA]o – quantité de HA qui a réagi = 2,00 × 10 −3 mol/L - 1,0 × 10 −3 mol/L = 1,0 × 10 −3 mol/L

(1, 0 × 10−3 ) (1, 0 × 10−3 ) [H + ][A − ] Ka = = = 1,0 × 10 −3 −3 [HA] 1, 0 × 10 134. a) En supposant qu’il n’y a pas association entre les ions SO42−(aq) et Fe3+(aq), alors i = 5 pour Fe2(SO4)3.

π = icRT = 5(0,0500 mol/L)(0,08206 L·atm/K·mol)(298 K) = 6,11 atm b) Fe2(SO4)3(aq) → 2 Fe3+(aq) + 3 SO42−(aq)

Dans des conditions idéales, 2/5 de π calculée ci-dessus est dû à Fe3+ et 3/5 est dû à SO42−. La contribution de SO42− à π est 3/5 × 6,11 atm = 3,67 atm. Étant donné la supposition que SO42− ne change pas dans la solution, la contribution de SO42− dans la solution réelle sera également de 3,67 atm. La contribution de la réaction de dissociation de Fe(H2O)63+ à la pression osmotique réelle est de 6,73 - 3,67 = 3,06 atm. La concentration initiale de Fe(H2O)62+ est 2(0,0500) = 0,100 mol/L. Le tableau de réaction pour le problème de l’acide faible est : Fe(H2O)63+ Initiale

0,100 mol/L



H+ ~0

+ Fe(OH)(H2O)52+ Ka =

[H + ][Fe(OH)(H 2 O)52+ ] [Fe(H 2 O)3+ 6 ]

0

x mol/L de Fe(H2O)63+ réagit pour atteindre l’équilibre Équilibre 0,100 - x

x

x

© 2007 Les Éditions CEC inc. Toute reproduction interdite

Chapitre 5 Acides et bases

182

π = icRT; concentration totale des ions = ic =

π RT

=

3,06 atm 0,8206 L ⋅ atm/ K ⋅ mol (298) = 0,125 mol/L

0,125 mol/L = 0,100 - x + x + x = 0,100 + x, x = 0,025 mol/L

Ka =

[H + ][Fe(OH)(H 2 O)52+ ] x2 (0, 025) 2 (0, 025) 2 = = = , 0,100 − x (0,100 − 0, 025) 0, 075 [Fe(H 2 O)3+ 6 ]

Ka = 8,3 × 10 −3

PROBLÈMES D’INTÉGRATION

2,14 g NaIO × 135. [IO−] =

IO− Initiale Équilibre

Kb =

1 mol NaIO 1 mol IO − × 165,89 g mol NaIO = 1,03 × 10 −2 mol/L IO− 1,25 L

+

H2O



HIO

1,03 × 10 −2 mol/L 1,03 × 10 −2 - x

+

0 x

OH−

Kb =

[HIO][OH − ] [IO − ]

~0 x

x2 ; d’après les données du problème, pOH = 14,00 – 11,32 = 2,68 1, 03 × 10−2 − x

[OH−] = 10−2,68 = 2,1 × 10 −3 mol/L = x; Kb =

136. 10,0 g NaOCN ×

10,0 g H2C2O4 ×

(2,1 × 10−3 ) 2 = 5,4 × 10 −4 1, 03 × 10−2 − 2,1 × 10−3

1 mol = 0,154 mol NaOCN 65,01 g 1 mol = 0,111 mol H2C2O4 90,04 g

0,154 mol mol NaOCN (réel) = = 1,39 mol H 2SO 4 0,111 mol L’équation équilibrée nécessite un rapport molaire plus grand de 2:1. Par conséquent, NaOCN au numérateur est limitant. Étant donné qu’il y a une correspondance en moles de

© 2007 Les Éditions CEC inc. Toute reproduction interdite

Chapitre 5 Acides et bases

183

2:2 entre les moles NaOCN qui ont réagi et les moles de HNCO produites, 0,154 mol HNCO seront produites. HNCO Initiale Équilibre

Ka = 1,2 × 10 −4 =



H+

0,154 mol/0,100 L 1,54 - x

+

~0 x

NCO−

Ka = 1,2 × 10 −4

0 x

x2 x2 ≈ , x = [H+] = 1,4 × 10 −2 mol/L 1,54 1,54 − x

pH = –log(1,4 × 10 −2 ) = 1,85; approximation valide.

137.

30,0 mg papH + Cl− 1000 mL 1g 1 mol papH + Cl− 1 mol papH + × × × × mL soln L 1000 mg 378,85 g mol papH + Cl− = 0,0792 mol/L



papH+ Initiale Équilibre

Ka =

0,0792 mol/L 0,0792 - x

pap

+

0 x

H+ ~0 x

Ke 2,1 × 10−14 = = 2,5 × 10 −6 −9 Kb, pap 8,33 × 10

Ka = 2,5 × 10 −6 =

x2 x2 ≈ , x = [H+] = 4,4 × 10 −4 mol/L 0, 0792 − x 0, 0792

pH = -log(4,4 × 10 −4 ) = 3,36; approximation valide.

© 2007 Les Éditions CEC inc. Toute reproduction interdite

Chapitre 5 Acides et bases

184 PROBLÈMES DE SYNTHÈSE

138. Pour déterminer le pH de la solution A, il faut calculer la valeur de Ka pour HX. Utilisons la solution B pour déterminer Kb pour X−, qui peut par la suite servir à calculer Ka pour HX (Ka = Ke/Kb).

Solution B : X− Initiale Changement Équilibre

Kb =

+

H2O

0,0500 mol/L -x 0,0500 - x



HX



0 +x x

+

OH−

Kb =

[HX][OH− ] [X − ]

~0 +x x

x2 ; 0, 0500 − x

d’après les données du problème, pH = 10,02, alors pOH = 3,98 et [OH−] = x = 10−3,98

Kb =

(10−3,98 ) 2 = 2,2 × 10 −7 0, 0500 − 10−3,98

Solution A :

H a,HX = K e /K b,X − = 1,0 × 10 −14 /2,2 × 10 −7 = 4,5 × 10 −8 HX



H+ + ~0 +x x

X−

Ka = 4,5 × 10 −8 =

Initiale Changement Équilibre

0,100 mol/L -x → 0,100 - x

Ka = 4,5 × 10 −8 =

x2 x2 ≈ , x = [H+] = 6,7 × 10 −5 mol/L 0,100 − x 0,100

[H + ][X − ] [HX]

0 +x x

Approximation valide (x est 6,7 × 10 −2 % de 0,100); pH = 4,17 Solution C : Principales espèces : HX (Ka = 4,5 × 10 −8 ), Na+, OH−; OH− provenant de la base forte accepte facilement des protons. Les ions OH- réagissent avec le meilleur acide présent (HX) et on peut supposer que OH− réagira complètement avec HX, c.-à-d., jusqu’à ce qu’un (ou les deux) réactif s’épuise. Étant donné que les deux substances ont été mélangées, les concentrations initiales des deux solutions ont été diluées. Ce qui n’a pas changé, c’est le nombre de moles de chaque réactif. Alors, travaillons avec le nombre de moles initial de chaque réactif.

© 2007 Les Éditions CEC inc. Toute reproduction interdite

Chapitre 5 Acides et bases

mol HX = 0,0500 L ×

185

0,100 mol HX = 5,00 × 10 −3 mol HX L

0,250 mol NaOH 1 mol OH − × = 3,75 × 10 −3 mol OH− mol OH = 0,0150 L × L mol NaOH −

Maintenant, déterminons ce qui reste dans la solution après que les ions OH- ont réagi complètement avec HX. On remarque que OH- est le réactif limitant. HX Initiale Changement Après réaction

OH−

+

5,00 × 10 −3 mol -3,75 × 10 −3 1,25 × 10 −3 mol

X−



3,75 × 10 −3 mol -3,75 × 10 −3 → 0

+

H2O

0 − −3 +3,75 × 10 +3,75 × 10 −3 − 3,75 × 10 −3 mol

Après réaction, la solution contient HX, X−, Na+ et H2O. L’ion Na+ (comme la plupart des ions métalliques de charge +1) n’a aucun effet sur le pH de l’eau. Cependant, HX est un acide faible et sa base conjuguée, X−, est une base faible. Étant donné que les réactions de Ka et de Kb se rapportent à ces espèces, on peut utiliser l’une ou l’autre de ces réactions pour trouver le pH; utilisons la réaction de Kb. Pour résoudre le problème d’équilibre en utilisant la réaction de Kb, il faut convertir les unités de concentration étant donné que Kb est exprimée en mol/L.

1,25 × 10−3 mol 3,75 × 10−3 mol − [HX] = = 0,0192 mol/L; [X ] = = 0,0577 mol/L (0,0500 + 0,0150) L 0,0650 L [OH−] = 0 (La réaction est complète.) X− Initiale Changement Équilibre

Kb = 2,2 × 10 −7 =

+

H2O



HX

+

OH−

Kb = 2,2 × 10 −7

0,0577 mol/L 0,0192 mol/L 0 − x mol/L de X réagit pour atteindre l’équilibre -x → +x +x 0,0577 - x 0,0192 + x x

(0, 0192 + x )( x) (0, 0192) x ≈ (en supposant que x [base], alors < 1 et log ⎜ ⎟ < 0. [acide] [acide] ⎝ [acide] ⎠

Selon l’équation de Henderson-Hasselbalch, si le terme du logarithme est négatif, alors pH < pKa. S’il y a plus d’acide que de base dans le tampon, le pH sera du côté acide de la valeur de pKa, c.-à-d., le pH est à une valeur inférieure à celle de pKa. S’il y a plus de base que d’acide dans un tampon ([base conjuguée]) > [acide faible]), alors le terme du logarithme dans l’équation de Henderson-Hasselbalch est positif ce qui donne pH > pKa. S’il y a plus de base que d’acide dans un tampon, le pH est du côté basique de la valeur de pKa, c.-à-d. que le pH est à une valeur supérieure à celle de pKa. L’autre possibilité rencontrée dans une solution tampon c’est quand [acide] = [base]. Dans ce cas, le terme du logarithme est égal à zéro et pH = pKa. 15.

Plus il y a d’acide faible et de sa base conjuguée dans la solution, plus il y aura d’ions H+ ou OH- qui seront absorbés par le tampon, sans changement notable du pH. Lorsque les concentrations de l’acide faible et de la base conjuguée sont égales, (de sorte que pH = pKa), le système tampon est aussi efficace pour absorber soit H+, soit OH-.

16.

Dans la question i), il s’agit du titrage d’un acide fort par une base forte. Le pH est très acide jusqu’avant le point d’équivalence ; au point d’équivalence, pH = 7,00; après le point d’équivalence, le pH est très basique. Dans la question ii), c’est le titrage d’une base forte par un acide fort. Dans ce cas, le pH est très basique jusqu’avant le point d’équivalence; au point d’équivalence, pH = 7,00; après le point d’équivalence, le pH est très acide. Dans la question iii), c’est le titrage d’une base faible par un acide fort. Le pH au départ est basique à cause de la présence d’une base faible. Cependant, le pH n’est pas aussi basique que dans le titrage ii) où une base forte est titrée. Le pH diminue quand HCl est ajouté, puis au point de demi-neutralisation, pH = pKa. Étant donné que Kb = 4,4 × 10 −4 pour CH3NH2, CH3NH3+ a Ka = Ke/Kb = 2,3 × 10 −11 et pKa = 10,64. Donc, au point de demi-neutralisation pour ce titrage d’une base faible par un acide fort, pH = 10,64. Le pH continue à diminuer à mesure que HCl est ajouté; puis au point d’équivalence, le pH est acide (pH < 7,00) parce que la seule espèce importante présente est un acide faible (l’acide conjugué de la base faible).

© 2007 Les Éditions CEC inc. Toute reproduction interdite

Chapitre 6 Applications de l’équilibre en milieu aqueux

190

Après le point d’équivalence, le pH devient plus acide à cause de l’excès de HCl ajouté. Dans la question iv), il s’agit du titrage d’un acide faible par une base forte. Le pH est acide au départ, mais pas autant que dans le cas du titrage i) d’un acide fort. Le pH augmente à mesure que NaOH est ajouté, puis au point de demi-neutralisation, pH = pKa pour HF = log(7,2 × 10 −4 ) = 3,14. Le pH continue à augmenter après le point de demi-neutralisation, puis au point d’équivalence le pH est basique (pH > 7,0) étant donné que la seule espèce importante est une base faible (la base conjuguée de l’acide faible). Après le point d’équivalence, le pH devient plus basique car un excès de NaOH est ajouté. a) Tous les titrages exigent le même volume de titrant pour atteindre le point d’équivalence. Au point d’équivalence, pour tous ces titrages, moles acide = moles base (cAVA = cBVB). Étant donné que toutes les concentrations molaires volumiques et les volumes sont les même dans les titrages, le volume de titrant sera le même (50,0 mL de titrant ajouté pour atteindre le point d’équivalence). B

B

b) Ordre croissant de pH initial : i < iv < iii < ii; le titrage de l’acide fort a le pH le plus bas, le titrage de l’acide faible est le suivant, suivi du titrage de la base faible, le titrage de la base forte ayant le pH le plus élevé. c) i < iv < iii < ii. Le titrage de l’acide fort a le pH au point de demi-neutralisation le plus bas et le titrage de la base forte a le pH au point de demi-neutralisation le plus élevé. Pour le titrage de l’acide faible, pH = pKa = 3,14, et pour le titrage de la base faible, pH = pKa = 10,64. d) pH au point d’équivalenve : iii < ii = i < iv; les titrages d’une espèce forte par une espèce forte ont un pH = 7,00 au point d’équivalence. Le titrage de la base faible a un pH acide au point d’équivalence et le titrage d’un acide faible a un pH basique au point d’équivalence.

La seule réponse différente quand l’acide faible et la base faible sont changés serait pour la partie c). C’est pour le point de demi-neutralisation où pH = pKa. HOC6H5; Ka = 1,6 × 10 −10 , pKa = -log(1,6 × 10 −10 ) = 9,80 C5H5NH+; Ka =

Ke K b , C5 H 5 N

=

1,0 × 10-14 = 5,9 × 10 −6 , pKa = 5,23 1,7 × 10-9

D’après les valeurs de pKa, le bon ordre au point de demi-neutralisation serait : i < iii < iv < ii. Remarquez que pour le titrage base faible-acide fort dans le cas de C5H5N, le pH est acide au point de demi-neutralisation, alors que pour le titrage acide faible-base forte dans le cas de HOC6H5, le pH est basique au point de demi-neutralisation. C’est ce qui se produit toujours lorsque la base faible titrée a Kb < 1 × 10 −7 (de sorte que Ka de l’acide conjugué est > 1 × 10 −7 ) et quand l’acide faible titré a Ka < 1 × 10 −7 (de sorte que Kb de la base conjuguée est > 1 × 10 −7 ). 17.

Les trois points importants à mettre en évidence dans le graphique sont le pH initial, le pH au point de demi-neutralisation et le pH au point d’équivalence. Pour toutes les bases faibles titrées, pH = pKa au point de demi-neutralisation (50,0 mL de HCl ajouté) parce que

© 2007 Les Éditions CEC inc. Toute reproduction interdite

Chapitre 6 Applications de l’équilibre en milieu aqueux

191

[base faible] = [acide conjugué] à ce point. Ici, la base faible dont Kb = 10-5 a un pH = 9,0 au point de demi-neutralisation et la base faible dont Kb = 10-10 a un pH = 4,0 au point de demi-neutralisation. Pour le pH initial, la base forte a le pH le plus élevé (le plus basique), alors que la base la plus faible a le pH le plus bas (le moins basique). Au point d’équivalence (100,0 mL de HCl ajouté), le titrage de la base forte a un pH = 7,0. Les bases faibles titrées ont des pH acides parce que les acides conjugués des bases faibles titrées sont les espèces principales présentes. La base la plus faible a l’acide conjugué le plus fort de sorte que son pH sera le plus faible (le plus acide) au point d’équivalence.

18.

HIn ⇌ H+ + In−

Ka =

[H + ][In − ] [HIn]

Les indicateurs sont eux-mêmes des acides faibles. Leur propriété spéciale est que la forme acide de l’indicateur (HIn) et la forme de la base conjuguée (In−) ont une couleur différente. C’est le pH qui détermine quelle forme domine et, par conséquent, qui détermine la couleur de la solution. On choisit un indicateur de façon à ce que le pH du changement de couleur corresponde à peu près au pH du point d’équivalence. 19.

i)

C’est le cas d’un sel qui se dissocie en deux ions. Exemples : AgCl, SrSO4, BaCrO4 et ZnCO3 ;

ii) Cas d’un sel qui se dissocie en trois ions, soit deux cations et un anion ou un cation et deux anions. Exemples : SrF2, Hg2I2 et Ag2SO4 ; iii) Cas d’un sel qui se dissocie en quatre ions, soit trois cations et un anion (Ag3PO4), soit un cation et trois anions (si on fait exception des hydroxydes, il n’y a aucun exemple de ce type au tableau 6.4) ;

20.

iv) Cas d’un sel qui se dissocie en cinq ions, soit trois cations et deux anions [Sr3(PO4)3], soit deux cations et trois anions (aucun exemple de ce type de sel dans le tableau 6.4). Certaines personnes croient qu’une augmentation de température augmente automatiquement la solubilité d’un sel. Ce n’est pas toujours le cas car certains sels présentent une diminution de leur solubilité lorsque la température augmente. Les deux principales méthodes qui permettent d’augmenter la solubilité d’un sel font intervenir, dans

© 2007 Les Éditions CEC inc. Toute reproduction interdite

Chapitre 6 Applications de l’équilibre en milieu aqueux

192

les deux cas, l’élimination d’un des ions du sel par une réaction. Si le sel possède un ion ayant des propriétés basiques, l’addition de H+ augmentera la solubilité du sel parce que H+ ajouté réagit avec l’ion basique, l’éliminant ainsi de la solution. Il se dissout plus de sel pour compenser l’ion perdu. Parmi les sels ayant des ions basiques, on trouve les exemples suivants : AgF, CaCO3 et Al(OH)3. L’autre façon d’enlever un ion consiste à former un ion complexe. Par exemple, l’ion Ag+ dans les sels d’argent forme l’ion complexe Ag(NH3)2+ quand on ajoute de l’ammoniac. Les sels d’argent augmentent leur solubilité à mesure qu’on ajoute NH3 parce que l’ion Ag+ est enlevé grâce à la formation d’un ion complexe. EXERCICES Tampons 21.

Quand on ajoute un acide fort ou une base faible au mélange bicarbonate/carbonate, l’acide fort ou la base forte sont neutralisés. La réaction est complète, ce qui entraîne le remplacement de l’acide fort ou de la base forte par un acide faible ou une base faible, formant ainsi une nouvelle solution tampon. Les réactions sont : H+(aq) + CO32-(aq) → HCO3-(aq) ; OH-(aq) + HCO3-(aq) → CO32-(aq) + H2O(l).

22.

CH3CH2COOH(aq)

a) Initiale Changement Équilibre



CH3CH2COO–(aq)

0,100 –x 0,100 – x

+

0 +x x

H+(aq) ≈0 mol/L +x mol/L x mol/L

2 2 [H +] [CH 3CH 2 COO − ] x x Ka = 1,3 × 10 = = ≈ [CH 3CH 2 COOH] 0,100 − x 0,100

–5

x = 1,1 × 10–3 ;

1,1 × 10−3 × 100 % = 1,1 %, l’approximation est valide. 0,0100

[H+] = x = 1,1 × 10–3 mol/L, pH = 2,96

b)

H2O

CH3CH2COONa

→ CH3CH2COO– + Na+

[CH3CH2COO–] =

0,10 mol CH 3CH 2 COONa 1 mol CH3CH 2 COO − × = 0,10 L 1 mol CH 3CH 2 COONa

mol/L

© 2007 Les Éditions CEC inc. Toute reproduction interdite

193

Chapitre 6 Applications de l’équilibre en milieu aqueux

CH3CH2COO–(aq) + Initiale Changement Équilibre

H2O(l)

0,100 –x 0,100 – x



CH3CH2COOH(aq) +

0 +x x

≈0 +x x

OH–(aq)

mol/L mol/L mol/L

Kb =

[OH − ][CH 3CH 2 COOH] −10 10−14 10−14 × = = 7,7 10 = K a (CH 3CH 2 COOH) 1,3 × 10−5 [CH 3CH 2 COO − ] 7,7 × 10–10 =

2

2

x x ≈ 0,100 - x 0,100

x = 8,8 × 10–6 ; l’approximation est valide.

[OH–] = x = 8,8 × 10–6 mol/L pOH = 5,06 pH = 8,94 H2O(l)

c) Initiale Changement Équilibre



H+(aq) + 0 +x x

OH–(aq) 0 mol/L +x mol/L x mol/L

Keau = 10–14 = [H+] [OH–] = x2 x = 1 × 10–7

[H+] = x = 1 × 10–7 mol/L, pH = 7,0 CH3CH2COOH(aq) ⇌ CH3CH2COO–(aq) +

d) Initiale Changement Équilibre

0,10 –x 0,10 – x

0,10 +x 0,10 + x

H+(aq) ≈0 +x x

mol/L mol/L mol/L

Il s’agit d’une solution tampon. Ka =

(0,100 + x)x (0,100)x ≈ (0,100 - x) (0,100)

© 2007 Les Éditions CEC inc. Toute reproduction interdite

Chapitre 6 Applications de l’équilibre en milieu aqueux

194

pH = pKa + log

[base] [acide]

pH = –log (1,3 × 10–5) + log

23.

(0,10) = 4,89 (0,10)

CH3CH2COOH 0,100 mol/L : % dissociation =

[H +] × 100 % = [ CH3CH 2COOH ]o

1,1 × 10−3 mol/L × 100 % = 1,1 % 0,100 mol/L CH3CH2COOH 0,100 mol/L + NaCH3CH2COO 0,100 mol/L : % dissociation =

1,3 × 10−5 × 100 % = 1,3 × 10–2 % 0,100

Le pourcentage de dissociation diminue de 1,1 % à 1,3 × 10–2 % à cause de la présence de l’ion commun CH3CH2COO– qui inhibe la réaction de dissociation de l’acide CH3CH2COOH. 24.

1,3 × 10−3 mol/L [OH _ ] NH3 0,100 mol/L : % ionisation = × 100 % = × 100 % = 1,3 0,100 mol/L [ NH3 ]o % NH3 0,100 mol/L + NH4Cl 0,100 mol/L : % ionisation =

1,8 × 10−5 × 100 % = 1,8 × 10–2 0,100

% Le pourcentage d’ionisation diminue d’un facteur d’environ 100 à cause de la présence de l’ion commun NH4+ qui inhibe la Changement de NH3 avec H2O.

25.

[OH–] = 0,020

mol NaOH 1 mol OH − × = 0,020 mol/L L mol NaOH CH3CH2COOH + OH–

a) Initiale Changement Équilibre

0,100 –0,020 0,080



0,020 –0,020 0

© 2007 Les Éditions CEC inc. Toute reproduction interdite

CH3CH2COO– + 0 +0,020 0,020

H2O

mol/L mol/L mol/L

195

Chapitre 6 Applications de l’équilibre en milieu aqueux _ + CH 3CH 2 COOH (aq ) CH 3CH 2COO (aq ) [H ](aq) ⇌ + x mol/L 0, 020 0,080

pH = pKa + log

0,020 = 4,89 + (–0,60) = 4,29 0,080

CH3CH2COO–(aq) +

b)

H2O(l)



CH3CH2COOH(aq) +

0 +x x

0,020 +x (0,020 + x)

OH–(aq)

Initiale Changement Équilibre

0,100 –x 0,100 – x

mol/L mol/L mol/L

Kb = 7,7 × 10–10 =

[CH 3CH 2 COOH ][OH _ ] x(0,020 + x) x(0,020) = ≈ − 0,100 [CH 3CH 2COO ] 0,100 − x

x = 3,9 × 10–9 ; l’approximation est valide.

[OH–] = 0,020 + x = (0,020 + 3,9 × 10–9) = 0,020 mol/L pOH = 1,70 pH = 12,30 c) [OH–] = 0,020 mol/L

pOH = 1,70 pH = 12,30 d) Initiale Changement Équilibre

CH3CH2COOH +

OH–

0,100 –0,020 0,080

0,020 –0,020 0



CH3CH2COO– + 0,100 +0,020 0,120

H2O

mol/L mol/L mol/L

CH CH COO − (aq ) [H +](aq) CH 3CH 2 COOH (aq ) ⇌ 3 2 + 0,120 x mol/L 0,080 pH = pKa + log

0,120 = 4,89 + (0,2) = 5,1 0,080

© 2007 Les Éditions CEC inc. Toute reproduction interdite

Chapitre 6 Applications de l’équilibre en milieu aqueux

196 26.

Reprenons les résultats des exercices 22, 24 et 25 : solution base a b c d

pH initial

pH après addition d’acide

2,96 8,94 7,00 4,89

pH après addition de

1,70 5,49 1,70 4,71

4,29 12,30 12,30 5,07

La solution de l’exercice 22 d) est une solution tampon qui contient à la fois un acide faible, CH3CH2COOH, et sa base conjuguée CH3CH2COO–. On voit que cette solution est celle qui démontre la plus grande capacité à résister aux changements de pH à la suite de l’addition d’acide ou de base. C’est là une caractéristique propre aux tampons. 27.

Principales espèces : HNO2, NO2– et Na+. Na+ est neutre. HNO2 et NO2 forment un couple tampon. HNO2 Initiale Changement Équilibre Ka = 4,0 × 10–4 =



NO2–



1,00 mol/L +x 1,00 + x

1,00 mol/L –x 1,00 – x

+

[ NO −2 ][H +] (1,00 + x)(x) 1,00(x) = ≈ [HNO 2] (1,00 − x) 1,00

H+ ~0 +x x

(en supposant que x 5 selon le vert de bromocrésol. pH < 8 selon le bleu de thymol. Le pH se situe entre 5 et 8.

60.

a) Jaune

b) Orange

c) Bleu

d) Bleu-vert

Équilibres de solubilité 61.

a) AgCH3COO(s) ⇌ Ag+(aq) + CH3COO-(aq) ; Kps = [Ag+][CH3COO-] ; b) Al(OH)3(s) ⇌ Al3+(aq) + 3OH-(aq) ; Kps = [Al3+][OH-]3 ; c) Ca3(PO4)2(s) ⇌ 3Ca2+(aq) + 2PO43-(aq) ; Kps = [Ca2+]3 [PO43-]2.

62.

Pour les prochaines questions, s = solubilité du solide ionique en mol/L = quantité maximale du sel qui peut se dissoudre dans un litre d’eau. Comme le solide n’apparaît pas dans l’expression du Kps, on peut omettre d’écrire les quantités initiales et à l’équilibre. CaC2O4(s)

a) Initiale Changement Équilibre

0 –s s



Ca2+(aq)



0 +s s

+

C2O42–(aq) 0 +s s

Selon les données, s = 4,8 × 10–5 mol/L. Kps = [Ca2+] (C2O42–] = (s)(s) = s2, Kps = (4,8 × 10–5)2 = 2,3 × 10–9 b) Initiale Changement Équilibre

BiI3(s)



Bi3+(aq)

–s



0 +s s

3I–(aq)

+

0 +3s 3s

Kps = [Bi3+][I–]3 = (s)(3s)3 = 27s4, Kps = 27(1,32 × 10–5)4 = 8,20 × 10–19

Ce(IO3)3(s)

63.

État initial À l’équilibre



Ce3+(aq)

+

0 s

© 2007 Les Éditions CEC inc. Toute reproduction interdite

3IO3–(aq) 0 3s

219

Chapitre 6 Applications de l’équilibre en milieu aqueux Selon les données, [IO3–] = 3s = 5,6 × 10–3, s = 1,9 × 10–3 mol/L Kps = [Ce3+] [IO3–]3 = s(3s)3 = 27s4 = 27(1,9 × 10–3)4 = 3,5 × 10–10

64.



Ag3PO4(s)

a) Initiale Changement Équilibre

3Ag+(aq) 0 +3s 3s



–s

+ PO43–(aq) 0 +s s

Kps = 1,8 × 10–18 = [Ag+]3[PO43–] = (3s)3(s) = 27s4

27s4 = 1,8 × 10–18, s = (6,7 × 10–20)1/4 = 1,6 × 10–5 mol/L CaCO2(s)

b)



Ca2+(aq)

Initiale Équilibre

+

0 s

CO32–(aq) 0 s

Kps = 8,7 × 10–9 = [Ca2+][CO32–] = s2, s = 9,3 × 10–5 mol/L

Hg2Cl2(s)

c)



Hg22+(aq)

Initiale Équilibre

0 s

+

2Cl–(aq) 0 2s

Kps = 1,1 × 10–18 = [Hg22+][Cl–]2 = (s)(2s)2 = 6,5 × 10–7 mol/L

65.

[M3+]2[X2−]3

M2X3(s)



2 M3+(aq) +

3 X2−(aq)

Kps =

Initiale s = solubilité (mol/L) 0 0 s mol/L de M2X3(s) se dissout pour atteindre l’équilibre Changement -s +2s +3s Équilibre 2s 3s Kps = (2s)2(3s)3 = 108 s5 ; s =

3,60 × 10-7 g 1 mol M 2 X 3 × = 1,25 × 10 −9 mol/L L 288 g

Kps = 108 (1,25 × 10 −9 )5 = 3,30 × 10 −43

© 2007 Les Éditions CEC inc. Toute reproduction interdite

Chapitre 6 Applications de l’équilibre en milieu aqueux

220

66.

mol Ag+ ajouté = 0,200 L ×

0,24 mol AgNO3 1 mol Ag + × = 0,048 mol Ag+ L mol AgNO3

Ag+ ajouté réagit avec les ions halogène pour former un précipité. Étant donné que les valeurs de Kps sont petites, on peut supposer que ces réactions de précipitation sont complètes. AgI(s) précipite en premier (le composé le moins soluble puisque sa Kps est la plus petite), suivi par AgBr(s), et AgCl(s) se formant le dernier [AgCl(s) est le composé le plus soluble de la liste étant donné qu’il a la Kps la plus élevée]. Soit la réaction complète de Ag+ avec I−. Ag+(aq) Initiale Changement Équilibre

I−(aq) →

+

0,048 mol -0,018 0,030 mol

AgI(s)

0,018 mol -0,018 0

0 +0,018 0,018 mol

K = 1/Kps >> 1

I− est limitant.

Puis, la réaction complète de Ag+ restant avec Br−. Ag+(aq)

+

Initiale 0,030 mol Changement -0,018 Équilibre 0,012 mol

Br−(aq) →

AgBr(s)

K = 1/Kps >> 1

0,018 mol -0,018 0

0 +0,018 0,018 mol

Br− est limitant.

Enfin, la réaction complète de Ag+ restant avec Cl−. Ag+(aq)

+

Initiale 0,012 mol Changement -0,012 Équilibre 0

Cl−(aq) →

AgCl(s)

K = 1/Kps >> 1

0,018 mol -0,012 0,006 mol

0 +0,012 0,012 mol

Ag+ est limitant.

Une partie du AgCl se redissout pour produire des ions Ag+; on ne peut pas avoir [Ag+] = 0 mol/L. Calculons combien de AgCl(s) se redissout : AgCl(s)



Ag+(aq) +

Cl−(aq)

Kps = 1,6 × 10 −10

Initiale

s = solubilité (mol/L) 0 0,006 mol/0,200 L = 0,03 mol/L s mol/L de AgCl se dissolvent pour atteindre l’équilibre Changement -s → +s +s Équilibre s 0,03 + s Kps = 1,6 × 10 −10 = [Ag+][Cl−] = s(0,03 + s) ≈ 0,03 s s = 5 × 10 −9 mol/L;

l’approximation de 0,03 + s ≈ 0,03 est valide.

mol AgCl présent = 0,012 mol – 5 × 10 −9 mol = 0,012 mol

© 2007 Les Éditions CEC inc. Toute reproduction interdite

221

Chapitre 6 Applications de l’équilibre en milieu aqueux

143,4 g = 1,7 g AgCl mol AgCl

masse AgCl présent = 0,012 mol AgCl × [Ag+] = s = 5 × 10 −9 mol/L Al(OH)3(s)

67. Initiale Changement Équilibre



Al3+(aq)



0 +s s

–s

3OH–(aq)

+

1,0 × 10–7 mol/L (de l’eau) +3s 1,0 × 10–7 + 3s

Kps = 2 × 10–32 = [Al3+][OH–]3 = (s)(1,0 × 10–7 + 3s)3 ≈ s(1,0 × 10–7)3

2 × 10-32 s= = 2 × 10–11 mol/L ; approximation valide (1,0 × 10–7 + 3s ≈ 1,0 × 10–7) -21 1,0 × 10 68.

a) Comme les solides CaF2 et BaF2 produisent tous deux 3 ions en solution, on peut affirmer que celui qui a le plus petit Kps (CaF2) est le moins soluble des deux. b) Ici, il faut calculer la solubilité de chaque sel, le nombre d’ions étant différent.



Ca3(PO4)2(s) Initiale Équilibre

3Ca2+(aq) +

2PO43–(aq)

0 3s

Kps = 1,3 × 10–32

0 2s

Kps = [Ca2+]3[PO43–]2 = (3s)3(2s)2 = 108s5, s = (1,3 × 10–32/108)1/5 = 1,6 × 10–7 mol/L

FePO4 (s)



Initiale Équilibre

Fe3+(aq)

+

PO43–(aq)

0 s

Kps = [Fe3–]3[PO43–]2 = s2, s =

Kps = 1,0 × 10–32

0 s

1,0 × 10-22 = 1,0 × 10–11 mol/L

FePO4 est donc le moins soluble des deux. 69.

Fe(OH)3(s)

a) Initiale Changement Équilibre



Fe3+(aq) 0 +s s

+

3OH–(aq) 10–7 +3s 10–7 + 3s

mol/L mol/L mol/L

Kps = 4 × 10–38 = [Fe3+][OH–]3 = s(3s + 10–7)3 ≈ x (10–7)3

© 2007 Les Éditions CEC inc. Toute reproduction interdite

Chapitre 6 Applications de l’équilibre en milieu aqueux

222

s = 4 × 10–17; 10–7 + (3 × 4 × 10–17) ≈ 10–7, l’approximation est valide.

Solubilité : 4 × 10–17 mol Fe(OH)3/L Fe(OH)3(s)

b)

Initiale Changement Équilibre



Fe3+(aq) + 3OH–(aq)



–s

pH = 5,0, [OH–] = 1 × 10–9 mol/L

1 × 10–9 ----1 × 10–9

0 +s s

mol/L (le pH du tampon ne change pas)

Kps = 4 × 10–38 = [Fe3+]3[OH–]3 = (s)(1 × 10–9)3, s = 4 × 10–11 mol/L c) La solution est tamponnée à un pH de 11,0. La concentration de OH– est donc 1,0 × 10–3 mol/L et demeure constante.

Fe(OH)3(s)



Fe3+(aq)

Initiale Changement Équilibre

3OH–(aq)

+

1,0 × 10–3 — 1,0 × 10–3

0 +s x

mol/L mol/L mol/L

Kps = 4 × 10–38 = [Fe3+] =[OH–]3 = s (1,0 × 10–3)3 x = 4 × 10–29

Solubilité : 4 × 10–29 mol Fe(OH)3/L Remarque : lorsque [OH–] augmente, s diminue. C’est l’effet d’ion commun.

Ca3(PO4)2(s)

70. Initiale Changement Équilibre

–s

⇌ →

3Ca2+(aq)

+

0 +3s 3s

2PO43–(aq) 0,20 mol/L +2s 0,20 + 2s

Kps = 1,3 × 10–32 = [Ca2+]3[PO43–]2 = (3s)3(0,20 + 2s)2

Supposons que 0,20 + 2s ≈ 0,20 ; 1,3 × 10–32 = (3s)2(0,20)2 = 27s3(0,040) s = 2,3 × 10–11 mol/L ; approximation valide.

Ce(IO3)3(s)

71. Initiale Équilibre



s = solubilité (mol/L)

Ce3+(aq)

+

0 s

© 2007 Les Éditions CEC inc. Toute reproduction interdite

3 IO3− (aq) 0,20 mol/L 0,20 + 3s

223

Chapitre 6 Applications de l’équilibre en milieu aqueux

Kps = [Ce3+][IO3−]3 = s(0,20 + 3s)3

D’après les données du problème, s = 4,4 × 10 −8 mol/L; résolvons pour Kps: Kps = (4,4 × 10 −8 ) × [0,20 + 3(4,4 × 10-8)]3 = 3,5 × 10 −10 ⇌

ZnS(s)

72. Initiale Équilibre

Zn2+

s = solubilité (mol/L)

+

0,050 mol/L 0,050 + s

S2 −

Kps = [Zn2+][S2−]

0 s

Kps = 2,5 × 10 −22 = (0,050 + s)(s) ≈ 0,050 s, s = 5,0 × 10 −21 mol/L; approximation valide.

masse ZnS qui se dissout = 0,3000 L ×

5,0 × 10-21 mol ZnS 97,45 g ZnS × L mol = 1,5 × 10 −19 g

73.

Pour enlever 99 % du Mg2+, il nous faut, à l’équilibre, [Mg2+] = 0,01(0,052 mol/L). À l’aide de la constante d’équilibre Kps, calculons [OH−] requis pour atteindre cette [Mg2+] réduite. Mg(OH)2(s)

⇌ Mg2+(aq)

+ 2 OH−(aq)

Kps = 8,9 × 10 −12

8,9 × 10 −12 = [Mg2+][OH−]2 = [0,01(0,052 mol/L)][OH−]2, [OH−] = 1,3 × 10 −4 mol/L pOH = -log(1,3 × 10 −4 ) = 3,89; pH = 10,11; à un pH = 10,1, 99 % des ions Mg2+ dans l’eau de mer précipent sous forme de Mg(OH)2(s). 74.

Les sels dont les anions peuvent agir comme une base sont plus solubles si le pH diminue. a) AgF

75.

b) Pb(OH)2

c) Sr(NO2)2

d) Ni(CN)2

BaSO4(s) se formera si Q > Kps. BaSO4(s) ⇌ Ba2+(aq) + SO42–(aq) Kps = 1,5 × 10–9 On calcule Q à partir des concentrations initiales de Ba2+ et SO42– :

0,020 mmol Ba 2+ 75,0 mL × mmol Ba 2+ mL = = 0,0075 mol/L [Ba2+]o = volume total 75,0 mL × 125 mL

[SO42–]o =

24

mmol SO = volume total

125 mL ×

0,040 mmol SO 2-4 mL = 0,025 mol/L 200 mL

© 2007 Les Éditions CEC inc. Toute reproduction interdite

Chapitre 6 Applications de l’équilibre en milieu aqueux

224

Q = [Ba2+]o[SO42–]o = (0,0075 mol/L) (0,025 mol/L) = 1,9 × 10–4 Q > Kps (1,5 × 10–9), donc BaSO4 sera formé. 76.

Le précipité possible est PbCl2(s). Après le mélange, [Pb2+]o = [Cl–]o = 0,010 mol/L. Q = [Pb2+]o[Cl–]02 = (0,010)(0,010)2 = 1,0 × 10–6 < Kps (1,6 × 10–5) ; il ne se formera pas de précipité.

77.

Étant donné que les concentrations initiales sont élevées, il est certain que BaC2O4(s) sera formé. Pour résoudre ce problème, supposons que la précipitation est totale, puis résolvons un problème d’équilibre pour obtenir les concentrations des ions. 100 mL ×

0,200 mmol K 2 C 2 O 4 = 20,0 mmol K2C2O4 mL

150 mL ×

0,250 mmol BaBr2 = 37,5 mmol BaBr2 mL Ba2+(aq)

État initial Réaction État final

+

37,5 mmol –20,0 17,5

C2O42–(aq)



BaC2O4(s)

20,0 mmol –20,0 0



0 +20,0 20,0

K = 1/Kps >> 1

réaction totale (K est élevé)

Nouvelles concentrations initiales (après la précipitation complète) : [Ba2+] =

[K+] =

17,5 mmol = 7,00 × 10–2 mol/L 250 mL

2(20,0 mmol) 2(37,5 mmol) = 0,160 mol/L ; [Br–] = = 0,300 mol/L 250 mL 250 mL

Pour K+ et Br–, ce sont les concentrations finales. Pour Ba2+ et C2O42–, il faut faire un calcul d’équilibre. BaC2O4(s) État initial À l’équilibre



Ba2+(aq)

+

0,0700 0,0700 + s

C2O42–(aq)

Kps = 2,3 × 10–8

0 s

Kps = 2,3 × 10–8 = [Ba2+] [C2O42–] = (0,0700 + s)(s) = 0,0700s s = [C2O42–] = 3,3 × 10–7 mol/L; [Ba2+] = 0,0700 mol/L approximation valide (s Kps. Comme nous connaissons [PO43–] = 1,0 × 10–5 mol/L, il suffit de calculer [Ag+] qui satisfera Kps. [Ag+]3 =

1,8 × 10-18 K ps = = 1,8 × 10–13 -5 3− [PO 4 ] 1,0 × 10

[Ag+] = 5,6 × 10–5 mol/L. Toute valeur de Ag+ supérieure à celle-ci causera la précipitation de Ag3PO4(s). 80.

D’après le tableau 6.4, Kps pour NiCO3 = 1,4 × 10 −7 et Kps pour CuCO3 = 2,5 × 10 −10 . Selon les valeurs de Kps, CuCO3 précipite en premier puisqu’il a un Kps plus petit et qu’il est le moins soluble. Pour CuCO3(s), le précipité commence à se former quand : [CO32−] =

K ps, CuCO3

=

[Cu 2+ ]

2,5 × 10-10 = 1,0 × 10 −9 mol/L CO32− 0,25 mol/L

Pour que NiCO3(s) précipite : [CO32−] =

K ps, NiCO3 [Ni 2+ ]

=

1,4 × 10-7 = 5,6 × 10 −7 mol/L CO32− 0,25 mol/L

Déterminons [Cu2+] quand NiCO3(s) commence à précipiter : [Cu2+] =

K ps, CuCO3 [CO32- ]

=

2,5 × 10-10 = 4,5 × 10 −4 mol/L Cu2+ 5,6 × 10-7 mol/L

Pour une bonne séparation, au plus 1 % de Cu2+ de la quantité initiale de Cu2+ (0,25 mol/L) doit être présent avant que NiCO3(s) commence à précipiter. Le pourcentage de Cu2+ présent quand NiCO3(s) commence à précipiter est : © 2007 Les Éditions CEC inc. Toute reproduction interdite

Chapitre 6 Applications de l’équilibre en milieu aqueux

226

4,5 × 10-4 mol/L × 100 = 0,18 % Cu2+ 0,25 mol/L Étant donné que moins de 1 % de la quantité initiale de Cu2+ reste, on peut séparer les métaux par addition lente de Na2CO3(aq).

Équilibres des ions complexes 81.

+NH3 ⇌ CoNH32+ K1

a) Co2+

CoNH32+

+NH3 ⇌ Co(NH3)22+K2

Co(NH3)22+

+NH3 ⇌ Co(NH3)32+K3

Co(NH3)32+

+NH3 ⇌ Co(NH3)42+K4

Co(NH3)42+

+NH3 ⇌ Co(NH3)52+K5

Co(NH3)52+

+NH3 ⇌ Co(NH3)62+K6

Co2+ +

6NH3

b) Ag+

AgNH3+

Kf = K1K2K3K4K5K6.

+NH3 ⇌ AgNH3+

K1

+NH3 ⇌ Ag(NH3)2+

K2

+2NH3 ⇌ Ag(NH3)2+

Ag+ 82.

⇌ Co(NH3)62+

Kf = K1K2.

+ CN–

⇌ NiCN+

K1

NiCN+

+ CN–



Ni(CN)2

K2

Ni(CN)2

+ CN–



Ni(CN)3–

K3

Ni(CN)3– + CN–



Ni(CN)42–

K4

Ni2+



Ni(CN)42–

Kf = K1K2K3K4

a) Ni2+

+ 4CN– +

C2O42–



MnC2O4

K1

MnC2O4

+

C2O42–



Mn(C2O4)22–

K2

Mn2+

+

2C2O42–



Mn(C2O4)22–

Kf = K1K2

b) Mn2+

© 2007 Les Éditions CEC inc. Toute reproduction interdite

227

Chapitre 6 Applications de l’équilibre en milieu aqueux

83.

Mn2+



+ C2O42−

K1 = 7,9 × 103

MnC2O4

MnC2O4 + C2O42− ⇌ Mn(C2O4)22− K2 = 7,9 × 101 ____________________________________________________________ Mn2+(aq) + 2 C2O42−(aq)

84.

Fe (aq) + 6 CN (aq) K=

85.



3+





Mn(C2O4)22−(aq)

Fe(CN)6

Kf = K1K2 = 6,2 × 105

[Fe(CN)3-6 ] K= [Fe3+ ][CN - ]6

3−

1,5 × 10−3 = 1,0 × 1042 6 −40 (8,5 × 10 )(0,11)

Hg2+(aq) + 2I-(aq) → HgI2(s) (précipité orange) ; HgI2(s) + 2I-(aq) → HgI42-(aq) (ion complexe soluble).

86.

Ag+(aq) + Cl–(aq) → AgCl(s)

Précipité blanc

AgCl(s) + 2NH3(aq) ⇌ Ag(NH3)2+(aq) + Cl– Ag(NH3)2+ + Br– ⇌ AgBr(s) + 2NH3(aq) Précipité jaune pâle Ag(NH3)2+ est formé en présence de AgCl, mais pas en présence de AgBr : Kps(AgCl) > Kps(AgBr) AgBr(s) + 2S2O32–(aq) ⇌ Ag(S2O3)23–(aq) + Br–(aq) Le complexe Ag(S2O3)23– est formé en présence de AgBr alors que Ag(NH3)2+ ne l’est pas : Kf[Ag(S2O3)3–] > Kf[Ag(NH3)2+]

Ag(S2O3)23–(aq) + I–(aq) ⇌ AgI(s) + 2S2O32–(aq) Ag(S2O3)23– est formé en présence de AgBr, mais pas en présence de AgI : Kps(AgBr) > Kps(AgI).

Hg2+(aq)

87. Initiale Changement Équilibre

0,010 –0,010 ≈0

+

4I–(aq)



0,78 –0,040 0,74

HgI42–(aq) 0 mol/L +0,010 mol/L 0,010 mol/L

La réaction est complète puisque K est très grand.

© 2007 Les Éditions CEC inc. Toute reproduction interdite

Chapitre 6 Applications de l’équilibre en milieu aqueux

228

K = 1,0 × 1030 =

[HgI 2-4 ] 2+

=

_ 4

[Hg ][I ]

0,010 2+ 4 [Hg ](0,74)

[Hg2+] = 3,3 × 10–32 mol/L

88.

[X−]0 = 5,00 mol/L et [Cu+]0 = 1,0 × 10 −3 mol/L parce que des volumes égaux de chaque réactif sont mélangés. Étant donné que les valeurs de K sont beaucoup plus grande que 1, on suppose que la réaction sera complète pour former CuX32−; on résout alors un problème d’équilibre. Cu+

3 X−

+

Avant 1,0 × 10 −3 mol/L Après 0 Équilibre x

K=



5,00 mol/L 5,00 - 3( 10 −3 ) ≈ 5,00 5,00 + 3x

CuX32−

K = K1 × K2 × K3 = 1,0 × 109

0 1,0 × 10 −3 Réaction complète −3 1,0 × 10 - x

(1, 0 × 10−3 − x) 1, 0 × 10−3 9 = 1,0 × 10 ≈ , x = [Cu+] = 8,0 × 10 −15 mol/L 3 3 ( x) (5, 00 + 3 x) ( x) (5, 00) Approximation valide.

[CuX32−] = 1,0 × 10 −3 - 8,0 × 10 −15 = 1,0 × 10 −3 mol/L

K3 =

[CuX 32- ] (1,0 ×10-3 ) 3 = 1,0 × 10 = , [CuX2−] = 2,0 × 10 −7 mol/L [CuX -2 ](5,00) [CuX -2 ][X - ]

En résumé : [CuX32-] = 1,0 × 10 −3 mol/L [CuX2-] = 2,0 × 10 −7 mol/L [Cu2+] = 8,0 × 10 −15 mol/L 89.

a) Initiale Équilibre

AgI(s)

(réponse a) (réponse b) (réponse c)



s = solubility (mol/L)

Ag+(aq) + I−(aq) 0 s

0 s

Kps = 1,5 × 10 −16 = s2, s = 1,2 × 10 −8 mol/L

© 2007 Les Éditions CEC inc. Toute reproduction interdite

Kps = [Ag+][I−] = 1,5 × 10 −16

229

Chapitre 6 Applications de l’équilibre en milieu aqueux



AgI(s)

b)

Ag+ + I−

Kps = 1,5 × 10 −16

Ag+ + 2 NH3 ⇌ Ag(NH3)2+ Kf = 1,7 × 107 ___________________________________________________________________



AgI(s) + 2 NH3(aq) AgI(s)

+

Ag(NH3)2+(aq) + I−(aq)

2 NH3



K = Kps × Kf = 2,6 × 10 −9

Ag(NH3)2+ + I−

Initiale 3,0 mol/L 0 0 s mol/L de AgBr(s) se dissout pour atteindre l’équilibre = solubilité molaire Équilibre 3,0 - 2s s s +

K=

[Ag( NH 3 ) 2 ][I − ] s2 s2 −9 = = 2,6× 10 ≈ , s = 1,5 × 10 −4 mol/L [ NH 3 ] 2 (3.0 − 2s ) 2 (3.0) 2

Approximation valide. c) La présence de NH3 augmente la solubilité de AgI. Le NH3 ajouté élimine Ag+ de la solution en formant l’ion complexe Ag(NH3)2+. À mesure que Ag+ est enlevé, plus de AgI se dissout pour rétablir la concentration de Ag+.

AgBr(s)

90.

Ag+ + 2 S2O32−

⇌ Ag+ + Br-

Kps = 5,0 × 10 −13



Kf = 2,9 × 1013

Ag(S2O3)23−

AgBr(s) + 2 S2O32− ⇌ Ag(S2O3)23− + Br-

AgBr(s)

+

2 S2O32−



K = Kps × Kf = 14,5 (On garde C.S. suppl.)

Ag(S2O3)23− +

Br-

Initiale

0,500 mol/L 0 0 s mol/L AgBr(s) se dissout pour atteindre l’équilibre Changement -s -2s → +s +s Équilibre 0,500 - 2s s s

K=

s2 = 14,5; en prenant la racine carrée des deux membres : (0,500 − 2 s ) 2

s = 3,81, s = 1,91 - 7,62 s, s = 0,222 mol/L 0,500 − 2 s 1,00 L ×

0,222 mol AgBr 187,8 g AgBr = 41,7 g AgBr = 42 g AgBr × L mol AgBr

© 2007 Les Éditions CEC inc. Toute reproduction interdite

Chapitre 6 Applications de l’équilibre en milieu aqueux

230

91.

CuCl(s)

a)



Cu+

État initial Équilibre

+

0 s

Cl– 0 s

Kps = 1,2 × 10–6 = [Cu+][Cl–] = s2, s = 1,1 × 10–3 mol/L b) En présence de Cl–, 2 équilibres sont établis :

⇌ Cu+(aq) + Cl–(aq)

CuCl(s)

Cu+(aq) + 2Cl–(aq) ⇌ CuCl2–(aq) CuCl(s) + Cl–(aq)

État initial À l’équilibre

Kf = 8,7 × 104

⇌ CuCl2–(aq) CuCl(s)

+

Kps = 1,2 × 10–6

K = Kps × Kf = 0,10 Cl–(aq)



CuCl2–(aq)

0,10 mol/L 0,10 s

0 s

où s = solubilité de CuCl(s) en mol/L

K = 0,10 =

92.

93.

[CuCl-2 ] s = , s = 9,1 × 10–3 mol/L [Cl ] 0,10 - s

Éprouvette no 1 : Cl− ajouté réagit avec Ag+ pour former un précipité de chlorure d’argent. L’équation ionique nette est Ag+(aq) + Cl−(aq) → AgCl(s). Éprouvette no 2 : NH3 ajouté réagit avec les ions Ag+ pour former un ion complexe soluble, Ag(NH3)2+. À mesure que le complexe se forme, Ag+ est enlevé de la solution, ce qui cause la dissolution de AgCl(s). Quand suffisamment de NH3 est ajouté, tout le précipité de chlorure d’argent se dissout. L’équation est AgCl(s) + 2 NH3(aq) → Ag(NH3)2+(aq) + Cl−(aq). Éprouvette 3 : H+ ajouté réagit avec la base faible, NH3, pour former NH4+. À mesure que NH3 est enlevé de l’ion complexe Ag(NH3)2+, les ions Ag+ ions sont libérés dans la solution et peuvent réagir avec Cl− pour reformer AgCl(s). Les équations sont Ag(NH3)2+(aq) + 2 H+(aq) → Ag+(aq) + 2 NH4+(aq) et Ag+(aq) + Cl−(aq) → AgCl(s). Avec NH3, la dissolution a lieu selon l’équation Cu(OH)2(s) + 4NH3 ⇌ Cu(NH3)42+ + 2OH-. Avec HNO3, il y a neutralisation des ions OH- : Cu(OH)2(s) + 2H+ → Cu2+ + 2H2O. Tous les sels dont l’anion est une base deviennent plus solubles en solution acide. AgCH3COO(s) peut lui aussi être dissous par NH3 : AgCH3COO(s) + 2NH3 ⇌ Ag(NH3)2+ + CH3COO-, et par H+ : AgCH3COO(s) + H+ → Ag+ + CH3COOH. Pour sa part, AgCl(s) ne sera soluble que dans NH3, car l’ion Cl- n’est pas basique et ne peut donc réagir avec HNO3.

© 2007 Les Éditions CEC inc. Toute reproduction interdite

231

Chapitre 6 Applications de l’équilibre en milieu aqueux EXERCICES SUPPLÉMENTAIRES 94.

NH3 + H2O ⇌ NH4+ + OH– Kb =

[ NH +4 ] [ NH +4 ] , – log [OH–] = – log Kb + log [ NH3] [ NH3]

– log Kb = – log [OH–] – log

95.

[ NH +4] [OH _ ] [ NH3]

pOH = pKb + log

[ NH +4 ] [acide] ou pOH = pKb + log [base] [ NH3]

a) C6H5COOH



0,10 pH = pKa + log

C6H5COO–

[C6 H5 COO _ ] [C6 H5 COOH] 0,10 = 4,19 + log 1 = 4,19 0,10

C6H5COOH

b) Initiale



C6H5COO– +

H2O mol/L

(−0,10 x 20,0) 100

(+0,10 x 20,0) 100

mol/L

0,08

0,12

mol/L

0,12 = 4,19 + 0,18 = 4,37 0,8

C6H5COO– +

Kb =

OH–

0,10

pH = 4,19 + log

Initiale Changement Équilibre

+

0,10

Équilibre

c)

x mol/L

0,10

pH = –log (6,4 × 10–5) + log

Changement

H+

+

0,12 –x 0,12 – x

H2O



C6H5COOH + 0,08 +x 0,08 + x

OH–

+

≈0 mol/L +x mol/L x mol/L

[OH - ][C6 H 5COOH] (0,080 + x)(x) (0,080)(x) 1,0 × 10-14 = = ≈ (0,120 - x) 0,120 [C6 H 5COO- ] 6,4 × 10-15

© 2007 Les Éditions CEC inc. Toute reproduction interdite

Chapitre 6 Applications de l’équilibre en milieu aqueux

232

x = 1,34 × 10–10 ; l’approximation est valide. [OH–] = x = 2,35 × 10–10 mol/L, pOH = 9,63 pH = 4,37 d) Les réponses de b et c sont les mêmes.

Les deux équilibres impliquent les mêmes espèces. C6H5COOH et C6H5COO–, KaKb = 10–14 H+ et OH–, [H+] [OH–] = 10–14 96.

a) C2H5NH3+



pH = pKa + log

H+ + C2H5NH2

Ka =

Ke 1, 0 × 10−14 = = 1,8 × 10 −11 ; pKa = 10,74 −4 5, 6 × 10 Kb

[C2 H 5 NH 2 ] 0,10 = 10,74 + log = 10,74 - 0,30 = 10,44 + 0, 20 [C2 H 5 NH 3 ]

b) C2H5NH3+ + OH− ⇌ C2H5NH2; après que OH− 0,050 mol/L a complètement réagi (convertissant C2H5NH3+ en C2H5NH2), il y a toujours une solution tampon où [C2H5NH3+] = [C2H5NH2] = 0,15 mol/L. Ici pH = pKa + log 1,0 = 10,74 (pH = pKa). 97.

pH = pKa + log

[CH 3COO- ] [CH 3COO- ] , 4,00 = -log(1,8 × 10 −5 ) + log [CH 3COOH] [CH 3COOH]

[CH 3COO- ] = 0,18; c’est aussi égal au rapport en moles entre CH3COO− et CH3COOH. [CH 3COOH] Soit x = volume de CH3COOH 1,00 mol/L et y = volume de NaCH3COO 1,00 mol/L

x + y = 1,00 L, x = 1,00 – y x (1,0 mol/L) = mol CH3COOH; y (1,00 mol/L) = mol NaCH3COO = mol CH3COO− Donc,

y y = 0,18 ou = 0,18; en résolvant : y = 0,15 L donc x = 1,00 - 0,15 = x 1, 00 − y

0,85 L Il faut 850 mL de CH3COOH 1,00 mol/L et 150 mL de NaCH3COO 1,00 mol/L pour produire une solution tampon à pH = 4,00. 98.

Une solution tampon doit contenir à la fois un acide faible et sa base conjuguée. Elle peut être préparée: – en mélangeant les deux espèces conjuguées ; – en ajoutant un acide fort à la base faible, la base étant en excès ;

© 2007 Les Éditions CEC inc. Toute reproduction interdite

233

Chapitre 6 Applications de l’équilibre en milieu aqueux – en ajoutant une base forte à l’acide faible, l’acide faible étant en excès. Le pKa de l’acide faible utilisé doit être aussi voisin que possible du pH désiré. a) pH = 3,0 acide faible utilisé : HF (pKa = 3,14) : KF + HCl b) pH = 4,0 acide faible utilisé : acide benzoïque (pKa = 4,19) acide benzoïque + NaOH c) pH = 5,0 acide faible utilisé : acide acétique (pKa = 4,74) acide acétique + acétate de sodium ou acide acétique + NaOH ou acétate de sodium + HCl d) pH = 7,0 acide faible utilisé : HOCl (pKa = 7,46) : HOCl + NaOH e) pH = 9,0 acide faible utilisé : NH4+ (pKa = 9,26) chlorure d’ammonium (NH4Cl) + NaOH 99.

a) Le pH optimal pour un tampon est pH = pKa. Or le pKa de TRISH+ est d’environ 8 :

Kb = 1,19 × 10–6 ; Ka = Keau/Kb = 8,40 × 10–9 ; pKa = –log (8,40 × 10–9) = 8,076

b) pH = pKa + log

[TRIS] [TRISH + ]

[TRIS] +

[TRISH ]

[TRIS] [TRISH + ]

= 10–1,08 = 0,083 (à pH = 7,00)

9,00 = 8,076 + log

c)

, 7,00 = 8,076 + log

[TRIS] +

[TRISH ]

,

[TRIS] +

[TRISH ]

= 100,92 = 8,3 (à pH = 9,00)

50,0 g TRIS 1 mol × = 0,206 mol/L ≈ 0,21 mol/L = [TRIS] 121,14 g 2,0 L 65,0 g TRISHCl 1 mol × = 0,206 mol/L ≈ 0,21 mol/L = [TRISHCl] = [TRISH+] 157,60 g 2,0 L pH = pKa + log

[TRIS] +

[TRISH ]

= 8,076 + log

Quantité ajoutée de HCl : 0,50 × 10–3 L ×

0, 21 = 8,08 0, 21 12 mol = 6,0 × 10–3 mol H+ L

© 2007 Les Éditions CEC inc. Toute reproduction interdite

Chapitre 6 Applications de l’équilibre en milieu aqueux

234 TRIS

H+

+



État initial

0,21 mol/L

6,0 × 10-3 = 0,030 mol/L 0,2005

Réaction État final

–0,030 0,18

–0,030 0

pH = 8,076 + log

TRISH+ 0,21 mol/L



+0,030 0,24

0,18 = 7,95 0,24

100. a) CH3COOH + OH– ⇌ CH3COO–

Kéq =

[CH3COO- ] [H +] 1,8 - 10-5 K a, CH 3CO 2_ = 1,8 × 109 × = = -14 _ + [CH 3COOH] [OH ] [H ] 1,0 - 10 K eau

b) CH3CO2– + H+ ⇌ CH3CO2H Keq =

[CH 3COOH] 1 = = 5,6 × 104 + K a, CH3COOH [H ][CH 3COO ]

c) HCl + NaOH → NaCl + H2O

Réaction ionique nette : H+ + OH–



H2O ; Kéq =

1

K eau

= 1,0 × 1014

101. a) Comme tous les acides ont la même concentration initiale, la courbe de pH avec le pH le plus élevé à 0 mL de NaOH ajouté correspond au titrage de l’acide le plus faible. C’est la courbe f ; b) La courbe de pH avec le pH le plus bas à 0 mL de NaOH ajouté correspond au titrage de l’acide le plus fort. C’est la courbe a. Le meilleur point à examiner pour distinguer le titrage d’un acide fort du titrage d’un acide faible (si les concentrations initiales ne sont pas connues) est le pH au point d’équivalence. Si le pH = 7,00, l’acide titré est un acide fort ; si le pH est supérieur à 7,00, l’acide titré est un acide faible ; c) La courbe de pH qui représente le titrage d’un acide avec Ka = 1,0 × 10-6, aura un pH = -log(1 × 10-6) = 6,0 au point de demi-neutralisation. La courbe d a un pH ~6,0 à 25 mL de NaOH ajouté, de sorte que l’acide titré dans cette courbe de pH (courbe d) a une Ka ~1 × 10-6. 102. Dans la solution finale : [H+] = 10−2,15 = 7,1 × 10 −3 mol/L mmol HCl au départ = 500,0 mL × 0,200 mmol/mL = 100 mmol HCl

Quantité de HCl qui réagit avec NaOH = 1,50 × 10 −2 mmol/mL × V

© 2007 Les Éditions CEC inc. Toute reproduction interdite

235

Chapitre 6 Applications de l’équilibre en milieu aqueux

7,1 × 10-3 mmol mmol H + final 1,00 - 0,0150 V = = mL volume total 500,0 + V 3,6 + 7,1 × 10 −3 V = 100 - 1,50 × 10 −2 V, 2,21 × 10 −2 V = 100 - 3,6

V = 4,36 × 103 mL = 4,36 L = 4,4 L NaOH 103. HA + OH– → A– + H2O où HA = acide acétylsalicylique (en supposant que c’est un monoacide)

mmol HA en présence = 27,36 mL OH– ×

0,5106 mmol OH 1 mmol HA × mL OH mmol OH = 13,97 mmol HA

masse molaire de HA =

2,51g HA = 180 g/mol 13,97 × 10-3 mol HA

Après neutralisation complète de HA par OH–, on a 13,97 mmol de A–, lesquelles réagiront complètement avec H+ pour redonner HA.

0,5106 mmol H + mmol H ajouté = 13,68 mL × = 6,985 mmol H+ mL +

A– État initial Réaction État final

+

13,97 mmol –6,985 6,985 mmol

H+



HA

6,985 mmol –6,985 0



0 +6,985 6,985 mmol

Puisque [A–] = [HA], nous sommes au point d’équivalence où pH = pKa = 3,48. Donc Ka = 10–3,48 = 3,3 × 10–4 104. NaOH ajouté = 50,0 mL ×

0,500 mmol = 25,0 mmol NaOH mL

NaOH qui n’a pas réagi = 31,92 mL HCl ×

0,289 mmol 1 mmol NaOH × mL mmol HCl = 9,22 mmol

NaOH NaOH qui a réagi avec l’aspirine = 25,0 - 9,22 = 15,8 mmol NaOH

© 2007 Les Éditions CEC inc. Toute reproduction interdite

Chapitre 6 Applications de l’équilibre en milieu aqueux

236

15,8 mmol NaOH ×

Pureté =

1 mmol aspirine 180,2 mg = 1420 mg = 1,42 g aspirine × 2 mmol NaOH mmol

1,42 g × 100 = 99,5 % 1,427 g

Dans le cas présent, il s’agit du titrage d’une base forte par un acide fort, de sorte que le point d’équivalence sera à pH = 7,0. Cependant, l’ion acétate est présent et on veut que l’indicateur change de couleur avant que l’acétate commence à être titré. Donc, un indicateur qui change de couleur à un pH élevé serait meilleur. Le rouge de crésol semble être le meilleur choix d’après la figure 6.8. Son point de virage se produit à environ 9,0 et son changement de couleur est très facile à voir (contrairement à la phénolphtaléine et à d’autres indicateurs qui changent de couleur à pH ~9,0). Parce que les titrages base forteacide fort ont un grand changement de pH au point d’équivalence, le fait d’avoir un indicateur qui change de couleur à un pH supérieur à 7,0 n’ajoute pas d’erreur significative. 105. CH3COOH



H+ + CH3COO−; soit co = concentration initiale de CH3COOH

Selon un tableau de réaction normal d’un acide faible où x = [H+] :

[H + ][CH 3COO- ] [H + ]2 Ka = 1,8 × 10 = = c0 - [H + ] [CH 3COOH] −5

+

[H ] = 10

−2,68

(2,1 × 10-3 ) 2 = 2,1 × 10 mol/L; 1,8 × 10 = , c0 = 0,25 mol/L c0 - 2,1 × 10-3 −3

−5

25,0 mL × 0,25 mmol/mL = 6,3 mmol CH3COOH; il faut 6,3 mol KOH pour atteindre le point d’équivalence. 6,3 mmol KOH = VKOH × 0,0975 mmol/mL, VKOH = 65 mL 106. mol acide = 0,210 g ×

1 mol = 0,00109 mol 192 g

mol OH− ajouté = 0,0305 L ×

0,108 mol NaOH 1 mol OH = 0,00329 mol OH− × L mol NaOH

mol OH 0,00329 = 3,02 = mol acide 0,00109 C’est un triacide (H3A) parce que 3 mol de OH− sont nécessaires pour réagir avec 1 mol de l’acide, autrement dit, l’acide doit avoir 3 mol H+ dans sa formule pour réagir avec 3 mol OH-.

© 2007 Les Éditions CEC inc. Toute reproduction interdite

237

Chapitre 6 Applications de l’équilibre en milieu aqueux 107. 50,0 mL × 0,100 mol/L = 5,00 mmol NaOH au départ

à pH = 10,50, pOH = 3,50, [OH−] = 10−3,50 = 3,2 × 10 −4 mol/L mmol OH− qui reste = 3,2 × 10 −4 mmol/mL × 73,75 mL = 2,4 × 10 −2 mmol mmol OH− qui a réagi = 5,00 - 0,024 = 4,98 mmol Comme l’acide faible est un monoacide, 23,75 mL de la solution de l’acide faible contiennent 4,98 mmol HA. [HA]o =

4,98 mmol = 0,210 mol/L 23,75 mL

108. HA + OH− → A− + H2O; il faut 25,0 mL de NaOH 0,100 mol/L pour atteindre le point d’équivalence où mmol HA = mmol OH− = 25,0 mL (0,100 mol/L) = 2,50 mmol. Au point d’équivalence, du HCl est ajouté. Le H+ de l’acide fort réagit complètement avec la base la plus forte présente, A-.

H+

A−

+

Avant 13,0 mL × 0,100 mol/L Réaction -1,3 mmol Après 0



2,5 mmol -1,3 mmol 1,2 mmol

HA 0 +1,3 mmol 1,3 mmol

Il se forme une solution tampon après la réaction complète de H+. pH = pKa + log

⎛ 1,2 mmol/VT ⎞ [A − ] , 4,7 = pKa + log ⎜ ⎟ [HA] ⎝ 1,3 mmol/VT ⎠

Parce que le terme logarithmique est négatif [log(1,2/1,3) = -0,035)], la valeur de pKa de l’acide doit être supérieure à 4,7. Cu(OH)2 ⇌ Cu2+ + 2 OH−

109. a)

Cu2+ + 4 NH3 Cu(OH)2(s) + 4 NH3(aq)

⇌ Cu(NH3)42+

Kf = 1,0 × 1013

⇌ Cu(NH3)42+(aq) + 2 OH−(aq)

K = KpsKf = 1,6 × 10 −6

Cu(OH)2(s) + 4 NH3

b) Initiale

Équilibre

Kps = 1,6 × 10 −19



Cu(NH3)42+ +

2 OH−

K = 1,6 × 10 −6

5,0 mol/L 0 0,0095 mol/L s mol/L Cu(OH)2 se dissout pour atteindre l’équilibre 5,0 - 4s s 0,0095 + 2s

© 2007 Les Éditions CEC inc. Toute reproduction interdite

Chapitre 6 Applications de l’équilibre en milieu aqueux

238

[Cu(NH 3 ) 42+ ][OH - ]2 s (0,0095 + 2s ) 2 K = 1,6 × 10 = = [NH 3 ]4 (5,0 - 4s ) 4 −6

Si s est petite : 1,6 × 10 −6 =

s (0, 0095) 2 , s = 11 mol/L (5, 0) 4

Approximation non valide. Résolvons le problème par approximations successives.

scalc =

1, 6 × 10−6 (5.0 − 4sévaluée ) 4 , Les résultats de six essais sont : (0, 0095 + 2 sévaluée ) 2

sévaluée: 0,10, 0,050, 0,060, 0,055, 0,056 scalc:

1,6 × 10-2, 0,071, 0,049, 0,058, 0,056

Donc, la solubilité de Cu(OH)2 est 0,056 mol/L dans NH3 5,0 mol/L. Ba(OH)2(s)

110.



Initiale s = solubilité (mol/L) Équilibre

Ba2+(aq) + 2 OH−(aq) 0 s

Kps = [Ba2+][OH−]2 = 5,0 × 10 −3

~0 2s

Kps = 5,0 × 10 −3 = s(2s)2 = 4s3, s = 0,11 mol/L; approximation valide. [OH−] = 2s = 2(0,11) = 0,22 mol/L; pOH = 0,66, pH = 13,34 Sr(OH)2(s)



Sr2+(aq) + 2 OH−(aq)

s

Équilibre

Kps = [Sr2+][OH−]2 = 3,2 × 10 −4

2s

Kps = 3,2 × 10 −4 = 4s3, s = 0,043 mol/L; approximation valide. [OH−] = 2(0,043) = 0,086 mol/L; pOH = 1,07, pH = 12,93 Ca(OH)2(s) Équilibre



Ca2+(aq) + 2 OH−(aq)

s

Kps = [Ca2+][OH−]2 = 1,3 × 10 −6

2s

Kps = 1,3 × 10 −6 = 4s3, s = 6,9 × 10 −3 mol/L; approximation valide. [OH−] = 2(6,9 × 10 −3 ) = 1,4 × 10 −2 mol/L; pOH = 1,85, pH = 12,15

© 2007 Les Éditions CEC inc. Toute reproduction interdite

239

Chapitre 6 Applications de l’équilibre en milieu aqueux

Ca5(PO4)3OH(s)

111.



5Ca2+(aq)

Initiale Changement Équilibre

+ 3PO43–(aq) + OH–(aq)

0 +5s 5s

10–7 mol/L +s mol/L (10–7 + s) mol/L

0 +3s 3s

Kps = 6,8 × 10–37 = [Ca2+]5[PO43–]3[OH–] = (5s)5(3s)3(10–7 + s) ≈ (5s)5(3s)3s s = 3,1 × 10–5 ; l’approximation est valide. Solubilité : 3,1 × 10–5 mol Ca5(PO4)3OH/L Si [H+] augmente, [OH–] diminue et l’équilibre est déplacé vers la droite. La solubilité augmente lorsque le pH diminue. De plus, PO43– réagit aussi avec H+. Ca5(PO4)3F(s)



5Ca2+(aq) +

Initiale Changement Équilibre

3PO43–(aq) +

F–(aq)

0 +3s 3s

0 +s s

0 +5s 5s

mol/L mol/L mol/L

Kps = 1 × 10–60 = [Ca2+]5[PO43–]3[F–] = (5s)5(3s)3(s) = 84375s9 s = 6,9 × 10–8 Solubilité : 6,9 × 10–8 mol Ca5(PO4)3F/L Les ions fluorure de l’eau fluorée convertissent l’hydroxyapatite en fluorapatite moins soluble et donc moins sujette à la carie. 112. a) Initiale Équilibre

Pb(OH)2(s) ⇌ s = solubilité (mol/L)

Pb2+(aq) 0 s

+ 2 OH-(aq) 1,0 × 10 −7 mol/L provenant de l’eau 1,0 × 10 −7 + 2s

Kps = 1,2 × 10 −15 = [Pb2+] [OH−]2 = s(1,0 × 10 −7 + 2s)2 ≈ s(2s2) = 4s3 s = [Pb2+] = 6,7 × 10 −6 mol/L; l’approximation qui consiste à négliger OH- provenant de l’eau est valide selon la règle des 5 %. Pb(OH)2(s)

b) Initiale mol/L Équilibre



Pb2+(aq) 0

+

2 OH−(aq) 0,10 mol/L

pH = 13,00, [OH−] = 0,10

s mol/L Pb(OH)2(s) se dissout pour atteindre l’équilibre s 0,10 (solution tampon)

1,2 × 10 −15 = (s)(0,10)2, s = [Pb2+] = 1,2 × 10 −13 mol/L

© 2007 Les Éditions CEC inc. Toute reproduction interdite

Chapitre 6 Applications de l’équilibre en milieu aqueux

240

c) Il faut calculer la concentration de Pb2+ en équilibre avec EDTA4−. Étant donné que K pour la formation de PbEDTA2− est élevée, la réaction est complète, puis on résout un problème d’équilibre pour obtenir la concentration de Pb2+.

Pb2+

EDTA4−

+



PbEDTA2−

0,010 mol/L 0,050 mol/L 0 0,010 mol/L Pb2+réagit complètement (K élevée) Changement -0,010 -0,010 → +0,010 Équilibre 0 0,040 0,010 Nouvelle concentration initiale x mol/L PbEDTA2− se dissocie pour atteindre l’équilibre Équilibre x 0,040 + x 0,010 - x

K = 1,1 × 1018

Initiale

1,1 × 1018 =

Réaction complète

(0, 010 − x) (0, 010) ≈ , x = [Pb2+] = 2,3 × 10 −19 mol/L; ( x )(0, 040 + x) ( x )(0, 040)

approximation valide. Maintenant, calculons le quotient de solubilité pour Pb(OH)2 pour voir si la précipitation se produit. La concentration de OH− est 0,10 mol/L parce qu’il s’agit d’une solution tamponnée à pH = 13,00.

Q = [Pb2+]o[OH−]02 = (2,3 × 10 −19 )(0,10)2 = 2,3 × 10 −21 < Kps (1,2 × 10 −15 ) Pb(OH)2(s) ne se forme pas étant donné que Q est inférieur à Kps.

PROBLÈMES DÉFIS 113. mmol CH3CH2COOH présent au départ = 45,0 mL ×

0,750 mmol mL =33,8 mmol CH3CH2COOH

mmol CH3CH2COO− présent au départ = 55,0 mL ×

0,700 mmol mL = 38,5 mmol CH3CH2COOH−

Le pH initial du tampon est :

38,5 mmol [CH 3CH 2COO ] 100,0 mL = -log (1,3 × 10 −5 ) + log pH = pKa + log 33,8 mmol [CH 3CH 2COOH] 100,0 mL -

© 2007 Les Éditions CEC inc. Toute reproduction interdite

241

Chapitre 6 Applications de l’équilibre en milieu aqueux

pH = 4,89 + log

38,5 = 4,95 33,8

Note : parce que les composantes du tampon occupent le même volume de solution, on peut utiliser le rapport molaire (ou mmol) dans l’équation de Henderson-Hasselbalch pour trouver le pH au lieu d’utiliser le rapport de concentrations de [CH3CH2COO]/[CH3CH2COOH]. Le volume total s’annule toujours pour les solutions tampons. Quand NaOH est ajouté, le pH augmente et OH- ajouté convertit CH3CH2COOH en CH3CH2COO−. Le pH après l’addition des OH- augmente de 2,5 %, de sorte que le pH résultant est : 4,95 + 0,025 (4,95) = 5,07 À ce pH, il y a toujours une solution tampon et le rapport en mmol entre CH3CH2COO− et CH3CH2COOH est : pH = pKa + log

mmol CH 3CH 2 COOmmol CH 3CH 2 COO, 5,07 = 4,89 + log mmol CH 3CH 2 COOH mmol CH 3CH 2 COOH

mmol CH 3CH 2 COO= 100,18 = 1,5 mmol CH 3CH 2 COOH Soit x = mmol OH− ajouté pour augmenter le pH à 5,07. Étant donné que OH- réagit complètement avec CH3CH2COOH, le tableau de réaction pour le problème exprimé en mmol est : CH3CH2COOH Avant 33,8 mmol Changement -x Après 33,8 - x

+

OH−



CH3CH2COOH− + H2O

x mmol -x 0



38,5 mmol +x Réaction complète 38,5 + x

Trouvons x :

mmol CH 3CH 2 COO38,5 + x = 1,5 = , 1,5 (33,8 - x) = 38,5 + x 33,8 − x mmol CH 3CH 2 COOH x = 4,9 mmol OH−ajouté Le volume de NaOH nécessaire pour élever le pH de 2,5 % est : 4,9 mmol NaOH ×

1 mL = 49 mL 0,10 mmol NaOH

Il faut ajouter 49 mL de NaOH 0,10 mol/L pour augmenter le pH de 2,5 %.

© 2007 Les Éditions CEC inc. Toute reproduction interdite

Chapitre 6 Applications de l’équilibre en milieu aqueux

242

114. 0,400 mol/L × V NH3 = mol NH3 = mol NH4+ après réaction avec HCl au point d’équivalence.

0,400 × VNH3 mol NH +4 Au point d’équivalence : [NH4 ]o = = = 0,267 mol/L volume total 1,50 × VNH3 +

NH4+ Initiale Équilibre

Ka =



0,267 mol/L 0,267 – x

H+

+

NH3

0 x

0 x

-14 2 2 x x K eau = 1,0 × 10 = 5,6 × 10–10 = ≈ 1,8 × 10-15 0,267 − x 0,267 Kb

x = [H+] = 1,2 × 10–5 mol/L ; approximation valide ; pH = 4,92 115. Pour HOCl, Ka = 3,5 × 10–8 et pKa = –log (3,5 × 10–8) = 7,46 ; comme le pH est proche du pKa, il s’agit d’une solution tampon.

pH = pKa + log

[OCl_ ] [OCl_ ] [OCl_ ] , 8,00 = 7,46 + log , = 100,54 = 3,5 [HOCl] [HOCl] [HOCl]

Initialement, il y avait 1,00 L × 0,0500 mol/L = 0,0500 mol HOCl. L’ajout de OH– transforme HOCl en OCl–. Le nombre de moles total de HOCl et OCl– = 0,0500 mol.

nOCl- + nHOCl = 0,0500 et n OCl− = 3,5 nHOCl 4,5 n HOCl = 0,0500, n HOCl = 0,011 mol ; n − = 0,039 mol OCl Il faut ajouter 0,039 mol NaOH pour produire 0,039 mol OCl–. 0,039 mol OH– = V × 0,0100 mol/L, V = 3,9 L NaOH 116. 50,0 mL × 0,100 mol/L = 5,00 mmol H2SO4; 30,0 mL × 0,10 mol/L = 3,0 mmol HOCl

25,0 mL × 0,20 mol/L = 5,0 mmol NaOH; 25,0 mL × 0,10 mol/L = 2,5 mmol Ca(OH)2 = 5,00 mmol OH– 10,0 mL × 0,15 mol/L = 1,5 mmol KOH; au total, il y a 11,5 mmol OH–. OH– réagira d’abord avec l’acide le plus fort, H2SO4, lequel est un diacide. Pour H2SO4, –2 K a1 >> 1 et K a 2 = 1,2 × 10 . La réaction est : 10,0 mmol OH– + 5,00 mmol H2SO4 → 10,0 mmol H2O + 5,0 mmol SO42–

© 2007 Les Éditions CEC inc. Toute reproduction interdite

Chapitre 6 Applications de l’équilibre en milieu aqueux

243

Il reste donc 1,5 mmol de OH–, qui réagira avec l’acide le plus fort en présence, soit HOCl (3,0 mmol), et en transformera 1,5 mmol en 1,5 mmol de OCl–. Il en résulte une solution tampon où [HOCl] = [OCl–], les espèces majeures étant HOCl, OCl–, SO42– et H2O, plus des cations Na+ et K+ neutres. SO42– est une base très faible (Kb = 8,3 × 10–13). C’est donc l’équilibre HOCl ⇌ H+ + OCl– qui détermine le pH, lequel est égal au pKa de HOCl, puisque [HOCl] = [OCl–]. pH = 7,46 [H+] = Ka = 3,5 × 10–8 mol/L; pH = 7,46 117. La première courbe de titrage (de 0 à 100,0 mL) correspond au titrage de H2A par OH−. La réaction est H2A + OH− → HA− + H2O. Une fois que tout le H2A a réagi, le deuxième titrage (de 100,0 à 200,0 mL) correspond au titrage de HA− par OH−. La réaction est HA− + OH− → A2− + H2O. a) À 100,0 mL de NaOH, juste assez de OH- a été ajouté pour réagir complètement avec tout le H2A présent (mol OH− ajouté = mol H2A présent initialement). D’après la réaction équilibrée, le nombre de mol de HA- produit sera égal au nombre de mol de H2A présent initialement. Étant donné que le nombre de mol de HA− présent après l’addition de 100,0 mL de OH− est égal au nombre de mol de H2A présent initialement, il faut ajouter exectement 100,0 mL de NaOH de plus pour réagir avec tout le HA−. Le volume de NaOH ajouté pour atteindre le deuxième point d’équivalence est égal à 100,0 mL + 100,0 mL = 200,0 mL. b) H2A + OH− → HA− + H2O est la réaction qui se produit à l’addition de 0 à 100,0 mL de NaOH ajouté.

i) Il ne se produit aucune réaction, de sorte que H2A et H2O sont les principales espèces. ii) L’addition de OH- transforme H2A en HA−. Les principales espèces jusqu’à 100,0 mL de NaOH ajouté sont H2A, HA−, H2O et Na+. iii) À 100,0 mL de NaOH ajouté, mol de OH− = mol de H2A, de sorte que tout le H2A présent initialement a été transformé en HA−. Les espèces principales sont HA−, H2O et Na+. iv) Entre 100,0 et 200,0 mL de NaOH ajouté, les OH− transforment HA− en A2−. Les principales espèces sont HA−, A2−, H2O et Na+. v) Au deuxième point d’équivalence (200,0 mL), il y a juste assez de OH− ajouté pour transformer tout HA− en A2−. Les espèces principales sont A2−, H2O et Na+. vi) Après l’ajout de 200,0 mL de NaOH, il y a un excès de OH−. Les principales espèces sont OH−, A2−, H2O et Na+. c) 50,0 mL de NaOH ajouté correspond au premier point de demi-neutralisation. Exactement la moitié de H2A présent initialement a été transformé en sa base conjuguée HA−, de sorte que [H2A] = [HA−] dans cette solution tampon.

© 2007 Les Éditions CEC inc. Toute reproduction interdite

Chapitre 6 Applications de l’équilibre en milieu aqueux

244

H2A ⇌ HA− + H+

K a1 =

[HA − ][H + ] [H 2 A]

Quand [HA−] = [H2A], K a1 = [H+] ou pK a1 = pH. Here, pH = 4,0 so K a1 = 4,0 and K a1 = 10 −4.0 = 1 × 10 −4 . 150,0 mL de NaOH ajouté correspond au deuxième point de demi-neutralisation où [HA−] = [A2−] dans cette solution tampon. HA− ⇌ A2− + H+

Ka2 =

[A 2 − ][ H + ] [HA − ]

Où [A2−] = [HA−], alors K a 2 = [H+] ou pK a 2 = pH. Ici, pH = 8,0 de sorte que pK a 2 = 8,0 and K a 2 = 10-8,0 = 1 × 10 −8 . 118. a) Na+ est présent dans toutes les solutions. H+ de HCl ajouté réagit complètement avec CO32- pour le transformer en HCO3- (points A à C). Une fois que tout le CO32- a réagi (après le point C, le premier point d,équivalence), H+ réagit complètement avec la base présente, HCO3-(points C à E). Le point E représente le second point d’équivalence. Les principales espèces présentes à divers points après que H+ a réagi sont les suivantes.

A. B. C. D. E. F.

CO32−, H2O CO32−, HCO3−, H2O, Cl− HCO3−, H2O, Cl− HCO3−, CO2 (H2CO3), H2O, Cl− CO2 (H2CO3), H2O, Cl− H+ (excès), CO2 (H2CO3), H2O, Cl−

b) H2CO3



HCO3− + H+

K a1 = 4,3 × 10 −7

HCO3−



CO32− + H+

K a 2 = 5,6 × 10 −11

La première réaction de titrage qui se produit entre les points A à C est : H+ + CO32− → HCO3− Au point B, suffisamment de H+ a été ajouté pour transformer la moitié de CO32− en son conjugué. À ce point de demi-équivalence, [CO32−] = [HCO3−]. Pour cette solution tampon, pH = pK a 2 = -log (5,6 × 10 −11 ) = 10,25.

© 2007 Les Éditions CEC inc. Toute reproduction interdite

245

Chapitre 6 Applications de l’équilibre en milieu aqueux

La deuxième réaction de titrage qui se produit entre les points C à E est : H+ + HCO3− → H2CO3 Le point D est le deuxième point de demi-neutralisation où [HCO3−] = [H2CO3]. Ici, pH = pK a1 = -log (4,3 × 10 −7 ) = 6,37. 119. Un indicateur change de couleur à pH ≈ pKa ± 1. Chaque indicateur nous dit quelque chose du pH de la solution de HX :

indicateur

pH

bleu de bromophénol

≥ ~ 5,0

pourpre de bromocrésol

≤ ~ 5,0

vert de bromocrésol

pH ≈ pKa ≈ 4,8 ≈ 5,0

alizarine

≤ ~ 5,5

Le pH de la solution est donc à peu près 5,0. HX Initiale Équilibre

Ka =

Ka ≈

120.



1,0 mol/L 1,0 – x

H+

+

~0 x

X– 0 x

2 [H + ][X _ ] x ; comme le pH ~5,0, alors [H+] = x ≈ 1 × 10–5 mol/L = [HX] 1,0 - x

(1

× 10-5 )

2

1,0 - (1 ×10

-5

)

≈ 1 × 10–10

⇌ AgNH3+

K1 = 2,1 × 103

AgNH3+ + NH3



Ag(NH3)2+

K2 = 8,2 × 103

Ag+ + 2 NH3



Ag(NH3)2+

K = K1K2 = 1,7 × 107

Ag+ + NH3

Les concentrations initiales sont diminuées de moitié parce que des volumes égaux des deux solutions sont mélangées. La réaction est complète étant donné que K est élevée; on résout ensuite un problème d’équilibre.

© 2007 Les Éditions CEC inc. Toute reproduction interdite

Chapitre 6 Applications de l’équilibre en milieu aqueux

246

Ag+

+

Initiale 0,20 mol/L Changement 0 Équilibre x

K = 1,7 × 107 =

2 NH3



Ag(NH3)2+

2,0 mol/L 1,6 1,6 + 2x

0 0,20 0,20 - x

[Ag(NH 3 ) 2+ ] 0, 20 − x 0, 20 = ≈ , x = 4,6 × 10 −9 mol/L; + 2 2 [Ag ][NH 3 ] x (1,6 + 2x) x (1,6) 2 Approximation valide.

[Ag+] = x = 4,6 × 10 −9 mol/L; [NH3] = 1,6 mol/L; [Ag(NH3)2+] = 0,20 mol/L On utilise l’expression d’équilibre soit de K1, soit de K2 pour calculer [AgNH3+]. AgNH3+ + NH3 8,2 × 103 =

121. MX





Ag(NH3)2+

K2 = 8,2 × 103

[Ag(NH 3 ) 2+ ] 0,20 = , [AgNH3+] = 1,5 × 10 −5 mol/L + [AgNH 3 ][NH3 ] [AgNH3+ ](1,6)

M+ + X−;

ΔT =Kfm, m =

ΔT 0,028 o C = = 0,015 mol/kg Kf 1,86 o C ⋅ kg/mol

0,015 mol 1 kg × × 250 g = 0,00375 mol total de particules de soluté kg 1000 g 0,0375 mol = mol M+ + mol X−, mol M+ = mol X− = 0,0375/2 Étant donné que la masse volumique de la solution est 1,0 g/mL, 250 g = 250 mL de solution. [M+] =

(0,00375/2) mol M + (0,00375/2) mol X = 7,5 × 10 −3 mol/L, [X−] = 0,25 L 0,25 L = 7,5 × 10 −3 mol/L

Kps = [M+][X−] = (7,5 × 10 −3 ) 2 = 5,6 × 10 −5 SrF2(s)

122. a)



Sr2+(aq)

+ 2 F−(aq)

Initiale

0 0 s mol/L SrF2 se dissout pour atteindre l’équilibre Équilibre s 2s [Sr2+][F−]2 = Kps = 7,9 × 10 −10 = 4s3, s = 5,8 × 10 −4 mol/L

© 2007 Les Éditions CEC inc. Toute reproduction interdite

247

Chapitre 6 Applications de l’équilibre en milieu aqueux b) Supérieure, parce qu’une partie des F− réagit avec l’eau :

F− + H2O



HF + OH−

Kb =

Ke = 1,4 × 10 −11 K a (HF)

Cela diminue la concentration de F−, ce qui provoque la dissolution de plus de SrF2. c) SrF2(s)

⇌ Sr2+ + 2 F−

Kps = 7,9 × 10 −10 = [Sr2+][F−]2

Soit s = solubilité = [Sr2+], alors 2s = concentration totale de F−. Puisque F− est une base faible, une partie des F− est convertie en HF. Par conséquent : concentration totale de F− = 2s = [F−] + [HF]. HF ⇌H + F +

7,2 × 10 −2 =



[H + ][F- ] 1,0 × 10-2 [F- ] = Ka = 7,2 × 10 = (pH = 2,00 tampon) [HF] [HF] −4

[F − ] , [HF] = 14 [F−]; on résout : [HF]

[Sr2+] = s; 2s = [F−] + [HF] = [F−] + 14 [F−], 2s = 15 [F−], [F−] = 2s/15 2

⎛ 2s ⎞ 7,9 × 10 −10 = [Sr2+] [F−]2 = (s) ⎜ ⎟ , s = 3,5 × 10 −3 mol/L ⎝ 15 ⎠ M3X2(s)

123. Initiale Équilibre



s = solubilité (mol/L)

3 M2+(aq)

2 X3-(aq)

+

0 3s

Kps = [M2+]3[X3−]2

0 2s

Kps = (3s)3(2s)2 = 108s5; concentration totale des ions = 3s + 2s = 5s π = icRT, ic = concentration totale des ions =

π RT

=

2,64 × 10-2 atm 0,08206 L ⋅ atm/K ⋅ mol × 298 K = 1,08 × 10 −3 mol/L

5s = 1,08 × 10 −3 mol/L, s = 2,16 × 10 −4 mol/L; Kps = 108 s5 = 108(2,16 × 10 −4 )5

Kps = 5,08 × 10 −17

© 2007 Les Éditions CEC inc. Toute reproduction interdite

Chapitre 6 Applications de l’équilibre en milieu aqueux

248 PROBLÈMES D’INTÉGRATION 124. pH = pKa + log

⎛ 55,0 mL × 0,472 c / VT ⎞ [FC6 H 4COO- ] = 2,90 + log ⎜ ⎟ [FC6 H 4 COOH] ⎝ 75,0 mL × 0,275 c / VT ⎠

⎛ 26, 0 ⎞ ⎟ = 2,90 + 0,101 = 3,00 ⎝ 20, 6 ⎠

pH = 2,90 + log ⎜

125. M : [Xe]6s24f145d10; c’est le mercure, Hg. Étant donné que X− a 54 électrons, X a 53 protons et c’est l’iode, I. L’identité de A = Hg2I2.

1,98 g NaI × [I−]0 =

1 mol NaI 1 mol I× 149,9 g mol NaI = 0,0881 mol/L 0,150 L

Hg2I2(s) Initiale



Hg22+ +

2 I−

Kps = 4,5 × 10 −29

s = solubilité (mol/L) 0 s

Équilibre

0,0881 mol/L 0,0881 + 2s

Kps = 4,5 × 10 −29 = [Hg22+][I−]2 = s(0,0881 + 2s)2 ≈ s(0,0881)2 s = 5,8 × 10 −27 mol/L; approximation valide. 126. OH− ajouté provenant de la base forte réagit complètement avec l’acide le plus fort présent, HF. Pour déterminer le pH, examinons ce que contient la solution après que OH− a réagi complètement.

OH− ajouté = 38,7 g soln ×

1,50 g NaOH 1 mol NaOH 1 mol OH × × 100,0 g soln 40,00 g mol NaOH = 0,0145 mol

OH−

Pour la solution de HF 0,174 m, si on avait exactement 1 kg de H2O, alors la solution contiendrait 0,174 mol HF. 0,174 mol HF ×

20,01 g = 3,48 g HF mol HF

masse de solution = 1000,00 g H2O + 3,48 g HF = 1003,48 g volume de solution = 1003,48 g ×

1 mL = 912 mL 1,10 g

© 2007 Les Éditions CEC inc. Toute reproduction interdite

249

Chapitre 6 Applications de l’équilibre en milieu aqueux

mol HF = 250 mL ×

0,174 mol HF = 4,77 × 10 −2 mol HF 912 mL

OH− Initiale Changement Équilibre 0

+

F−

HF →

0,0145 mol -0,0145

0,0477 mol -0,0145 0,0332 mol

+

H2O

0 +0,0145 0,0145 mol

Après réaction, il se forme une solution tampon contenant HF, un acide faible et F−, sa base conjuguée. Soit VT = volume total de la solution. pH = pKa + log

⎛ 0, 0145 / VT ⎞ [F − ] = -log (7,2 × 10 −4 ) + log ⎜ ⎟ [HF] ⎝ 0, 0332 / VT ⎠

⎛ 0, 0145 ⎞ ⎟ = 3,14 + (-0,360), pH = 2,78 ⎝ 0, 0332 ⎠

pH = 3,14 + log ⎜

PROBLÈME DE SYNTHÈSE 127. a) Étant donné que

Ka1 >> K a , la quantité de H+ contribuée par la réaction de K a est 2

2

+

négligeable. [H ] fournie par la réaction de K a1 est 10 ⇌ H+

H2A Initiale Équilibre

+

HA−

−2,06

= 8,7 × 10

−3

mol/L H+.

K a1 = 5,90 × 10 −2

~0 0 [H2A]o [H2A]o = concentration initiale [H2A]o - x x x

K a1 = 5,90 × 10 −2 =

x2 (8,7 ×10-3 ) 2 = , [H2A]o = 1,0 × 10 −2 mol/L [H 2 A]0 - x [H 2 A]0 - 8,7 ×10-3

mol H2A présent initialement = 0,250 L ×

masse molaire de H2A =

1×10-2 mol H 2 A = 2,5 × 10 −3 mol H2A L

0,225 g H 2 A = 90 g/mol 2,5 ×10-3 mol H 2 A

© 2007 Les Éditions CEC inc. Toute reproduction interdite

Chapitre 6 Applications de l’équilibre en milieu aqueux

250

b) H2A + 2OH− → A2− + H2O; au deuxième point d’équivalence, OH− ajouté a transformé tout le H2A en A2−; donc, A2− est la principale espèce présente qui détermine le pH. Le nombre de mmol de A2− présent au point d’équivalence est égal au nombre de mmol de H2A présent initialement (2,5 mmol) et le nombre de mmol de OH− ajouté pour atteindre le deuxième point d’équivalence est 2(2,5 mmol) = 5,0 mmol OH− ajouté. La seule information qu’il faut connaître pour calculer la valeur de K a 2 est le volume de Ca(OH)2

ajouté pour atteindre le second point d’équivalence. Utilisons la valeur de Kps pour Ca(OH)2 pour aider à trouver le volume de Ca(OH)2 ajouté. Ca(OH)2(s)



Initiale s = solubilité (mol/L) Équilibre

Ca2+ + 2OH− 0 s

Kps = 1,3 × 10-6 = [Ca2+][OH−]2

~0 2s

Kps = 1,3 × 10 −6 = (s)(2s)2 = 4s3, s = 6,9 × 10 −3 mol/L de Ca(OH)2; approximation valide. Le volume de Ca(OH)2 requis pour fournir 5,0 mmol OH− (la quantité nécessaire de OH− pour atteindre le second point d’équivalence) est : 5,0 mmol OH− ×

1 mmol Ca(OH) 2 1 mL × -3 2 mmol OH 6,9 ×10 mmol Ca(OH) 2 = 362 mL = 360 mL Ca(OH)2

Au deuxième point d’équivalence, le volume total de la solution est : 250 mL + 360 mL = 610 mL On trouve alors K a 2 à l’aide des données de pH au second point d’équivalence. La seule

espèce présente qui influe sur le pH est la base faible, A2−, de sorte que le tableau de réaction pour le problème nécessite la réaction de Kb pour A2−. A2−

Ke 1, 0 × 10 = Ka2 Ka2 Initiale Équilibre

+ H2O

−14



HA−

2,5 mmol 610 mL 4,1 × 10 −3 mol/L- x

+

0

x

OH−

Kb =

0

x

1, 0 × 10−14 x2 = Kb = Ka2 4,1 × 10−3 − x D’après les données du problème, pH = 7,96, donc OH− = 10-6,04 = 9,1 × 10-7 mol/L = x.

© 2007 Les Éditions CEC inc. Toute reproduction interdite

Chapitre 6 Applications de l’équilibre en milieu aqueux

Kb =

251

1,0 ×10-14 (9,1×10-7 ) 2 = = 2,0 ×10-10 , K a 2 = 5,0 ×10-5 Ka2 4,1×10-3 - 9,1×10-7

Note: la quantité de OH− fournie par la base faible HA− est négligeable étant donné que la valeur de Kb pour A2− est au moins 1000 fois plus élevée que celle de Kb pour HA−.

© 2007 Les Éditions CEC inc. Toute reproduction interdite

CHAPITRE 7 THERMODYNAMIQUE CHIMIQUE QUESTIONS 7.

Les organismes vivants ont besoin d’une source extérieure d’énergie pour effectuer ces processus. Les plantes vertes utilisent l’énergie solaire pour produire du glucose à partir de dioxyde de carbone et de l’eau par photosynthèse. Dans le corps humain, l’énergie libérée par le métabolisme du glucose aide à la synthèse des protéines. Pour tous les processus combinés, ΔSuniv doit être supérieure à zéro (2e loi).

8.

Plus une substance est dispersée, plus le désordre est grand et son entropie est grande. Il faudra fournir de l’énergie pour nettoyer l’environnement. Il est donc avantageux de prévenir la contamination.

9.

Il semble que la somme des deux processus n’ait aucun changement net. Il n’en n’est pas ainsi. Selon la deuxième loi de la thermodynamique, ΔSuniv doit avoir augmenté même si en apparence il y a eu un processus cyclique.

10.

L’un des effets de l’entropie est l’introduction d’erreurs dans l’information transmise. Le but de l’information redondante est de fournir un moyen de contrôler l’exactitude de l’information transmise.

11.

À mesure que s’effectue un processus, ΔSuniv augmente ; ΔSuniv ne peut pas diminuer. Le temps, comme ΔSuniv, s’écoule dans une direction.

12.

Cette réaction est lente du point de vue cinétique, mais elle est favorisée d’un point de vue thermodynamique (ΔG < 0). La thermodynamique nous dit seulement si une réaction peut se produire. Pour répondre à la question à savoir si la réaction va se produire, on doit également prendre en considération la cinétique (vitesse de réaction). La lumière ultraviolette fournit l’énergie d’activation pour que cette réaction lente se produise.

13.

ΔSext = -ΔH/T ; ΔSext est déterminée par la chaleur (ΔH) qui entre ou sort d’un système. Si la chaleur est acheminée vers le milieu extérieur, les mouvements au hasard du milieu augmentent et l’entropie du milieu augmente. L’opposé est vrai lorsque la chaleur passe du milieu extérieur vers le système (une réaction endothermique). Bien que la force motrice décrite ici résulte réellement de la variation d’entropie du milieu, elle est souvent décrite en termes d’énergie. La nature tend vers l’énergie la plus faible possible.

14.

Remarquez que ces substances ne sont pas à l’état solide, mais en solution aqueuse; des molécules d’eau sont également présentes. Il y a une augmentation apparente de l’ordre quand ces ions sont placés dans l’eau comparé à leur état séparé. Les molécules d’eau hydratantes doivent être dans un arrangement très ordonné quand elles entourent ces anions.

© 2007 Les Éditions CEC inc. Toute reproduction interdite

Chapitre 7 Thermodynamique chimique

254

15.

16.

ΔG° = -RTlnK = ΔH° - TΔS° ; HX(aq) ⇌ H+(aq) + X-(aq) Ka réaction ; la valeur de Ka pour HF est inférieure à un, alors que pour les autres halogénures d’hydrogène, Ka > 1. En termes de ΔG°, HF doit avoir un ΔG°réaction positive, alors que les autres acides ont une ΔG°réaction < 0. La raison pour laquelle Ka change de signe quand on compare HF à HCl, à HBr et à HI, c’est l’entropie. ΔS pour la dissociation de HF est très élevée et négative. Il y a un degré élevé d’ordre quand les molécules d’eau s’associent (liaison hydrogène) avec les petits ions F-. L’entropie d’hydratation s’oppose fortement à la dissociation de HF dans l’eau, au point de dépasser l’énergie d’hydratation favorable, ce qui rend HF un acide faible. On peut déterminer ΔS° et ΔH° pour la réaction en utilisant les entropies standard et les enthalpies standard de l’annexe 4, puis l’équation ΔG° = ΔH° - TΔS°. On peut également utiliser les énergies libres standard de formation de l’annexe 4. Enfin, on peut utiliser la loi de Hess pour calculer ΔG°. Dans ce cas, les réactions ayant des valeurs de ΔG° connues sont modifiées pour déterminer ΔG° pour une réaction différente. Pour les températures différentes de 25 °C, on évalue ΔG° à l’aide de l’équation ΔG° = ΔH° - TΔS°. Il faut supposer que les valeurs de ΔH° et ΔS° déterminées à partir des données de l’annexe 4 sont indépendantes de la température. On utilise les mêmes valeurs de ΔH° et de ΔS° telles que déterminées quand T = 25 °C, puis on substitue la nouvelle température en Kelvin dans l’équation pour évaluer ΔG° à la nouvelle température.

17.

Le signe de ΔG nous permet de savoir si une réaction est spontanée ou non, quelles que soient les concentrations (à T et à P constantes). Lorsque ΔG < 0, la grandeur nous indique quelle quantité de travail peut, en théorie, être tirée de la réaction. Lorsque ΔG > 0, sa grandeur nous indique la quantité minimale de travail qui doit être fournie pour que la réaction ait lieu. ΔG° nous donne la même information seulement quand la concentration de tous les réactifs et produits sont dans des conditions standard (1 atm pour les gaz, 1 mol/L pour les solutés). ΔG° = -RTlnK : à partir de cette équation, on peut calculer K pour une réaction si ΔG° est connue à cette température. Pour déterminer K à une température différente de 25 °C, il faut connaître ΔG° à cette température. On suppose que ΔH° et ΔS° ne dépendent pas de la température, et on utilise l’équation ΔG° = ΔH° - TΔS° pour évaluer ΔG° à différentes températures. Pour K = 1, on veut ΔG° = 0, ce qui a lieu quand ΔH° = TΔS°. Encore une fois, on suppose que ΔH° et ΔS° ne dépendent pas de la température, puis on détermine T (=ΔH°/ΔS°). À cette température, K = 1 parce que ΔG° = 0. Cela ne fonctionne que pour les réactions où les signes de ΔH° et de ΔS° sont les mêmes (soit tous les deux positifs, soit négatifs).

EXERCICES Spontanéité, entropie et deuxième loi de la thermodynamique : énergie libre 18.

a, b et c. L’expérience commune montre que l’eau salée, l’eau colorée et la rouille se forment sans intervention extérieure. Cependant, il faut une telle intervention pour qu’une chambre devienne bien rangée.

© 2007 Les Éditions CEC inc. Toute reproduction interdite

Chapitre 7 Thermodynamique chimique

19.

255

Représentation des arrangements possibles : 2 kJ ___ 1 kJ ___ 0 kJ x x

___ x_ x_

x_ __ x_

___ xx_ ___

x_ x_ ___

xx _ xx _ ___

Etotale =0 kJ

1 kJ

2 kJ

2 kJ

3 kJ

4 kJ

L’énergie la plus probable est 2 kJ. 20.

2 kJ 1 kJ 0 kJ

AB

Etotale =

0 kJ

AB AB 2 kJ

4 kJ

B

A

B A

A B

A

B

1 kJ

1 kJ

2 kJ

2 kJ

B A

A_ B_ _

3 kJ

3 kJ

L’énergie la plus probable est 2 kJ. 21.

a) H2 à 100 °C et 0,5 atm ; si T plus élevée et P plus basse, le volume est plus grand, ce qui augmente l’entropie de position. b) N2 à TPN a le plus grand volume. c) H2O(l) est plus désordonné que H2O(s).

22.

Parmi les trois phases, (solide, liquide, gaz), les solides sont les plus ordonnés et les gaz, les plus désordonnés. Donc, a, b et f (fusion d’un solide, sublimation et ébullition) impliquent une augmentation de l’entropie du système étant donné que le passage d’un solide à un liquide ou d’un solide à un gaz ou d’un liquide à un gaz augmente le désordre. Dans le cas de la congélation (processus c), une substance passe de l’état liquide moins ordonnée à l’état solide plus ordonnée, ce qui entraîne une diminution de l’entropie. Le processus d (mélange) implique une augmentation du désordre (entropie) alors que la séparation augmente l’ordre (diminue l’entropie du système). Donc, parmi tous les processus, a, b, d et f provoquent une augmentation de l’entropie du système.

23.

a) Faire bouillir un liquide requiert un apport de chaleur. C’est un processus endothermique, ce qui diminue l’entropie de l’environnement (ΔSext est négatif). b) Ce processus étant exothermique, l’entropie de l’environnement augmente (ΔSext est positif).

24.

a) ΔSext =

-ΔH -890 kJ = –3,0 kJ/K = –3,0 × 103 J/K = T 298 K

b) ΔSext =

-ΔH 43 kJ = 0,14 kJ/K = 140 J/K = T 298 K

© 2007 Les Éditions CEC inc. Toute reproduction interdite

Chapitre 7 Thermodynamique chimique

256 25.

ΔG = ΔH – TΔS ; si ΔG < 0, la variation est spontanée. Lorsque ΔH < 0 et ΔS > 0, ΔG < 0 à toutes les températures. Lorsque ΔH > 0 et ΔS < 0, ΔG > 0 à toutes les températures. a) ΔG = 25 × 103 J – (300 K × 5,0

J ) = 24 × 103 J > 0, non spontanée K

b) ΔG = 25 × 103 J – (300 K × 100

c) ΔG = –10 × 103 J – (298 K × 5

J ) = –5 × 103 J < 0, spontanée K

J ) = –11,5 × 103 J < 0, spontanée K

Ce processus sera spontané à toutes les températures. d) ΔG = –10 × 103 J – (200 K × (–40 26.

J )) = –2 × 103 J < 0, spontané K

Voir le début de la réponse 25. Supposons que ΔH et ΔS ne dépendent pas de la température. a) Lorsque ΔH et ΔH sont toutes deux négatives, ΔG sera négative en dessous d’une température où le terme favorable ΔH prédomine. Lorsque ΔG = 0, ΔH = TΔS. On peut ainsi calculer T : T=

ΔH -25 000 J = 5,0 × 103 K = ΔS -5,0 J/K

À T < 5,0 × 103 K, ce processus sera spontané (ΔG < 0). b) Lorsque ΔH et ΔS sont toutes deux positives, ΔG est négative au-dessus de la température T où ΔG = 0 et ΔH = TΔS. T=

ΔH 25 000 J = = 5,0 × 103 K ΔS 5,0 J/K

À T > 5,0 × 10–3 K, ce processus sera spontané (ΔG < 0). c) Lorsque ΔH est positive et ΔS négative, le processus ne peut jamais être spontané, ΔG ne pouvant pas prendre une valeur négative. d) Lorsque ΔH est négative et ΔS positive, le processus est spontané à toute température.

© 2007 Les Éditions CEC inc. Toute reproduction interdite

Chapitre 7 Thermodynamique chimique 27.

Au point d’ébullition, ΔG = 0 de sorte que ΔH = TΔS. ΔS =

28.

257

ΔH 27,5 kJ/mol = = 8,93 × 10 −2 kJ/K·mol = 89,3 J/K·mol T (273 + 35) K

a) NH3(s) → NH3(l) ; ΔG = ΔH – TΔS = 5650 J/mol – 200 K (28,9 J/K⋅mol) ΔG = 5650 J/mol – 5780 J/mol = –130 J/mol Oui, NH3 fondra puisque ΔG < 0 à cette température. b) Au point de fusion, ΔG = 0 donc T =

29.

ΔH 5650 J/mol = = 196 K ΔS 28,9 J/K ⋅ mol

C2H5OH(l) → C2H5OH(g); au point d’ébullition, ΔG = 0 et ΔSuniv = 0. Pour le processus de vaporisation, ΔS est une valeur positive alors que ΔH est une valeur négative. Pour calculer ΔSsys, on détermine ΔSext à partir de ΔH et de la température, puis ΔSsys = -ΔSext pour un système à l’équilibre. ΔSext =

−ΔH 38,7 ×103 J/mol = = -110 J/K·mol T 351 K

ΔSsys = -ΔSext = -(-110) = 110 J/K·mol

Réactions chimiques : variations d’entropie et énergie libre 30.

a) Diminution du désordre ; ΔS° (–) b) Augmentation du désordre ; ΔS° (+) c) Diminution du désordre (Δn < 0) ; ΔS° (–) d) Augmentation du désordre (Δn > 0) ; ΔS° (+) Pour c et d, il faut tenir compte des réactifs et des produits gazeux. Quand il y a plus de produits gazeux que de réactifs gazeux (Δn > 0), alors le désordre augmente et ΔS° est positive.

31.

a) Saccharose – c’est une molécule plus grosse et plus complexe. b) H2O à 0 °C – à 0 K, l’entropie est nulle. c) H2S à 25 °C – un gaz a une entropie plus grande qu’un liquide.

© 2007 Les Éditions CEC inc. Toute reproduction interdite

Chapitre 7 Thermodynamique chimique

258 32.

a) H2(g) + 1/2 O2(g) ; (Δn < 0), donc ΔS° sera négative. ΔS° = ΣnpS°produits – = ΣnrS°réactifs ΔS° = 1 mol H2O(g)(189) J/K⋅mol) – [1 mol H2(g)(131) J/K⋅mol + 1/2 mol O2(g) (205 J/K⋅mol)] ΔS° = 189 J/K – 234 J/K = –45 J/K b) 3O2(g) → 2O3(g); Δn < 0, donc ΔS° sera négative. ΔS° = 2 mol(239 J/K⋅mol) – [3 mol(205 J/K>mol)] = –137 J/K c) N2(g) + O2(g) → 2NO(g) ; ici, Δn = 2 – 2 = 0. On ne peut facilement prédire le signe de ΔS°. ΔS° = 2(211) – (192 + 205) = 25 J/K

33.

a) H2(g) + 1/2 O2(g) → H2O(l) ; comme Δn des gaz est négatif, alors ΔS° sera négative. ΔS° = 1 mol H2O(l)(70 J/K⋅mol) – [1 mol H2(g)(131 J/K⋅mol) + 1/2 mol O2(g)(205 J/K⋅mol)] ΔS° = 70 J/K – 243 J/K = –164 J/K b) N2(g) + 3H2(g) → 2NH3(g) ; Δn < 0, alors ΔS° sera négative. ΔS° = 2(193) – [1(192) + 3(131)] = –199 J/K c) HCl(g) → H+(aq) + Cl–(aq) ; la valeur de ΔS° est fortement affectée par l’état gazeux, qui est plus désordonné que celui des ions en solution. ΔS° sera négative. ΔS° = 1 mol H+(0) + 1 mol Cl–(57 J/K⋅mol) – 1 mol HCl(187 J/K⋅mol) = –130 J/K

34. 35.

o o – 144 J/K = (2 mol) S AlBr – [2(28 J/K) + 3(152 J/K)], S AlBr = 184 J/K⋅mol 3 3

Orthorhombique → monoclinique ; ΔH est (+) et ΔG est (–) en haut de 95 °C, donc ΔS doit être positive. À 95 °C, ΔG = 0; ΔS =

36.

ΔH (0,30 × 103 J/mol) = = 0,82 J/K·mol T (95 + 273)K

L’enthalpie n’est pas favorable, alors ΔS doit être la force agissante du processus, c’est-àdire que ΔS est positive. S’il y a augmentation du désordre, alors l’enzyme originale est la plus ordonnée des deux structures.

© 2007 Les Éditions CEC inc. Toute reproduction interdite

Chapitre 7 Thermodynamique chimique 37.

259

a) Le bris de la liaison H–H est endothermique : ΔH est positive. Comme Δn > 0, ΔS est aussi positive. b) ΔG = ΔH – TΔS. Pour que cette réaction ait lieu, il faut que le terme TΔS prédomine, ce qui arrivera à des T élevées.

38.

La réaction est spontanée si ΔG < 0. Comme ΔG – ΔH – TΔS et que ΔH < 0 et ΔS > 0, ΔG sera toujours négative quelle que soit la température. Il n’y a donc pas de raison thermodynamique d’augmenter la température. Toutefois, la réaction sera plus rapide à température plus élevée.

39.

a)

CH4(g)

+



2O2(g)

CO2(g)

+

2H2O(g)

ΔH fo

–75

0

–393,5

–242

kJ/mol



186 –51

205 0

214 –394

189 –229

J/K⋅mol kJ/mol

ΔG

o f

–ΔH° = Σ ΔH f,o (prod) − Σ ΔH f,o (réactifs)

⎡⎛

kJ

⎞ ⎠

⎛ ⎝

kJ

⎞⎤ ⎠⎦

× 1 mol ⎟ + ⎜ -242 × 2 mol ⎟ ⎥ ΔH° = ⎢⎜ -393,5 mol mol ⎣⎝

⎡⎛ kJ ⎤ ⎞ - ⎢⎜ -75 × 1 mol ⎟ + 0⎥ = - 803 kJ mol ⎠ ⎣⎝ ⎦ ΔS° = ΣS° (prod) – ΣS° (réactifs)

⎡ (214 J × 1 mol) + (189 J × 2 mol) ⎤ ⎥⎦ K ⋅ mol ⎣

ΔS° = ⎢

⎡ (186 J × 1 mol) + (205 J × 2 mol) ⎤ - ⎢ ⎥⎦ = –4 J/K K ⋅ mol ⎣ ΔG° = ΣΔG°f (prod) – ΣΔG°f (réactifs)

⎡ (-394 kJ × 1 mol) + (-229 kJ × 2 mol) ⎤ ⎡ (-51 kJ × 1 mol) + (0) ⎤ ⎥⎦ ⎥⎦ - ⎢⎣ mol mol ⎣

ΔG° = ⎢

= –801 kJ On peut aussi calculer ΔG° = ΔH° – TΔS° = –803 × 103 J – (298 K) (–4 J/K) = –802 kJ

© 2007 Les Éditions CEC inc. Toute reproduction interdite

Chapitre 7 Thermodynamique chimique

260 6CO2(g)

b)

+

6H2 O(l)

→ C6H12O6(s) +

6O2(g)

ΔH fo

–393,5

–286

–1275

0

kJ/mol



214 –394

70 –237

212 –911

205 0

J/K⋅mol kJ/mol

ΔGfo ΔH° =

⎡ (-1275 kJ × 1mol) + (0) ⎤ ⎡ (-393,5 kJ × 6 mol) + (-286 kJ × 6 mol) ⎤ ⎢⎣ ⎥⎦ - ⎢⎣ ⎥⎦ mol mol = 2802 kJ ΔS° =

⎡ (212 J × 1 mol) + (205 J × 6 mol) ⎤ ⎡ (214 J × 6 mol) + (70 J × 6 mol) ⎤ ⎥⎦ ⎢⎣ ⎥⎦ - ⎢⎣ K ⋅ mol K ⋅ mol = –262 J/K

⎡ (-911 kJ × 1 mol) + (0) ⎤ ⎡ (-394 kJ × 6 mol) + (-237 kJ × 6 mol) ⎤ ⎥⎦ - ⎢⎣ ⎥⎦ mol mol ⎣

ΔG° = ⎢

= 2875 kJ P4O10(s)

c)

ΔH fo S°

+ 6H2O(l)



4H3PO4(s)

–2984

–286

–1279 kJ/mol

229

70

110 J/K⋅mol

⎡ (-1279 kJ × 4 mol) ⎤ ⎡ (-2984 kJ × 1mol) + (-286 kJ × 6 mol) ⎤ ⎥⎦ - ⎢⎣ ⎥⎦ mol mol ⎣

ΔH° = ⎢

= –416 kJ

⎡ (110 J × 4 mol) ⎤ ⎡ (229 J × 1 mol) + (70 J × 6 mol) ⎤ ⎥⎦ = –209 J/K K ⋅ mol K ⋅ mol ⎣ ⎦⎥ ⎣⎢

ΔS° = ⎢

ΔG° = ΔH° – TΔS° = –416 kJ – (298 K) (–0,209 kJ/K) = –354 kJ HCl(g)

d)

+

NH3(g)



NH4Cl(s)

ΔH fo

–92

–46

–314

kJ/mol



187

193

96

J/K⋅mol

© 2007 Les Éditions CEC inc. Toute reproduction interdite

Chapitre 7 Thermodynamique chimique

261

⎡ (-314 kJ × 1 mol) ⎤ ⎡ (-92 kJ × 1 mol) + (-46 kJ × 1 mol) ⎤ ⎥⎦ - ⎢⎣ ⎥⎦ = –176 kJ mol mol ⎣

ΔH° = ⎢

⎡ (96 J × 1 mol) ⎤ ⎡ (187 J × 1 mol) + (193 J × 1 mol) ⎤ ⎥⎦ = –284 J/K ⎥⎦ - ⎢⎣ K ⋅ mol K ⋅ mol ⎣

ΔS° = ⎢

– ΔG° = ΔH° – TΔS° = – 176 kJ – (298 K) (–0,284 kJ/K) = –91 kJ

40.

a) ΔH° = 2(-46 kJ) = -92 kJ; ΔS° = 2(193 J/K) - [3(131 J/K) + 192 J/K] = -199 J/K ΔG° = ΔH° - TΔS° = -92 kJ - 298 K(-0,199 kJ/K) = -33 kJ b) ΔG° est négative, de sorte que la réaction est spontanée dans des conditions standard. c) ΔG° = 0 quand T =

ΔH o -92 kJ = 460 K = o ΔS -0,199 kJ/K

À T < 460 K et à pression standard (1 atm), le terme ΔH° favorable prédomine et la réaction est spontanée (ΔG° < 0). 41.

ΔG° = – 58,03 kJ – (298 K)(– 0,1766 kJ/K) = – 5,40 kJ ΔG° = 0 = ΔH° – TΔS°, T =

ΔH o -58,03 kJ = = 328,6 K o ΔS -0,1766 kJ/K

ΔG° est négative en dessous de 328,6 K, là où le terme favorable ΔH° prédomine. 42.

H2O(l) → H2O(g); ΔG° = 0 au point d’ébullition de l’eau à 1 atm et à 100 °C. ΔH° = TΔS°, ΔS° =

ΔH o 40,6 ×103 J/mol = = 109 J/K·mol 373 K T

À 9 °C: ΔG° = ΔH° - TΔS° = 40,6 kJ/mol – (363 K)(0,109 kJ/K·mol) = 1,0 kJ/mol Comme prévu, ΔG° > 0 aux températures inférieures au point d’ébullition de l’eau à 1 atm (le processus est non spontané). À 110 °C: ΔG° = ΔH° - TΔS° = 40,6 kJ/mol – (383 K)(0,109 J/K·mol) = -1,1 kJ/mol Quand ΔG° < 0, le point d’ébullition de l’eau est spontané à 1 atm et à T > 100 °C (comme prévu).

© 2007 Les Éditions CEC inc. Toute reproduction interdite

Chapitre 7 Thermodynamique chimique

262 43.

CH4(g) + CO2(g) → CH3CO2H(l) ΔH° = –484 – [75 + (–393,5)] = –16 kJ ; ΔS = 160 – [186 + 214] = –240 J/K ΔG = ΔH° – TΔS = –16 kJ – (298 K)(–0,240 kJ/K) = 56 kJ Cette réaction n’est spontanée qu’à des températures inférieures à T = ΔH°/ΔS° = 67 K, ce qui n’est pas pratique pour deux raisons : les substances seront probablement solides, et la vitesse de réaction sera très lente à cette température très basse. CH3OH(g) + CO(g) → CH3CO2H(l) ΔH° = –484 – [110,5 + (–201)] = –173 kJ; ΔS = 160 – [198 + 240] = –278 J/K ΔG° = –173 kJ – (298 K)(–0,278 kJ/K) = –90 kJ Cette réaction aussi a une ΔH° favorable et une ΔS° défavorable. Elle est spontanée à des températures inférieures à T = ΔH°/ΔS° = 622 K. Contrairement à la première, on peut donc l’effectuer à des températures suffisamment élevées pour que la vitesse de réaction soit raisonnable.

44.

La réaction qui a le ΔG le plus négatif est la plus spontanée. C2H4(g) ΔGof

68

+

H2O(g)



–229

CH3CH2OH(l) –175 kJ/mol

⎡ (-175 kJ × 1 mol) ⎤ ⎡ (68 kJ × 1 mol) + (-229 kJ × 1 mol) ⎤ ⎥⎦ ⎥⎦ - ⎢⎣ mol mol ⎣

ΔG° = ⎢

ΔG° = (–175 kJ) – (–161 kJ) = –14 kJ (ΔG < 0) Il s’agit d’une réaction spontanée aux conditions standard. C2H6(g) ΔG°f

–32,9

+

H2O(g)



–229

CH3CH2OH(l) –175 0

+

H2(g)

kJ/mol

⎡ (-175 kJ × 1 mol) + 0 ⎤ ⎡ (-32,9 kJ × 1 mol) + (-229 kJ × 1 mol) ⎤ ⎥⎦ - ⎢⎣ ⎥⎦ mol mol ⎣

ΔG° = ⎢

ΔG° = (–175 kJ) – (–262 kJ) = 87 kJ (ΔG° > 0) Il s’agit d’une réaction non spontanée aux conditions standard.

© 2007 Les Éditions CEC inc. Toute reproduction interdite

Chapitre 7 Thermodynamique chimique 45.

CH4(g) → 2 H2(g) + C(s) 2 H2(g) + O2(g) → 2 H2O(l) C(s) + O2(g) → CO2(g) CH4(g) + 2 O2(g) → 2 H2O(l) + CO2(g)

46.

263 ΔG° = -(-51 kJ) ΔG° = -2(237 kJ) ΔG° = -394 kJ ΔG° = -817 kJ

ΔG° = Σnp ΔGf,o produits − Σnr ΔGf,o réactifs , –374 kJ = –1105 kJ – ΔGf,o SF4 ,

ΔGf,o SF4 = –731 kJ/mol

47.

ΔG° =

∑ n ΔG p

o f, produits

− ∑ nr ΔGf,o réactifs

ΔG° = [-57.37 kJ + (-68.85 kJ) + 3(-95.30 kJ)] – [3(0) + 2(-50.72 kJ)] = -310.68 kJ Pour une variation de température de 25 °C à ~20 °C (température ambiante), la grandeur de ΔG° ne variera pas beaucoup. Par conséquent, ΔG° sera une valeur négative à la température ambiante (~20 °C), donc la réaction sera spontanée. 48.

a) ΔG° = 2(-270 kJ) - 2(-502 kJ) = 464 kJ b) Puisque ΔG° est positive, cette réaction n’est pas spontanée dans des conditions standard à 298 K. c) ΔG° = ΔH° - TΔS°, ΔH° = ΔG° + TΔS° = 464 kJ + 298 K(0,179 kJ/K) = 517 kJ Il faut résoudre pour trouver la température quand ΔG° = 0 : ΔG° = 0 = ΔH° - TΔS°, T =

ΔH o 517 kJ = = 2890 K o 0,179 kJ/K ΔS

Cette réaction est spontanée dans des conditions standard (ΔG° < 0) quand T > 2890 K. Ici, le terme entropie favorable prédomine.

Énergie libre, pression et équilibre 49.

ΔG = ΔG° + RT ln Q ; pour cette réaction : ΔG = ΔG° + RT ln

P NO 2 × P O 2 P NO × P O3

ΔG° = 1 mol(52 kJ/mol) + 1 mol(0) – [1 mol(87 kJ/mol) + 1 mol(163 kJ/mol)] = –198 kJ

© 2007 Les Éditions CEC inc. Toute reproduction interdite

Chapitre 7 Thermodynamique chimique

264

ΔG = –198 kJ +

8,315 J/K⋅ mol (1,00 × 10-7 atm)(1,00 × 10-3atm) (298 K) ln (1,00 × 10-6 atm)(2,00 × 10-6 atm) 1000 J/kJ

ΔG = –198 kJ + 9,69 kJ = –188 kJ 50.

2H2S(g) + SO2(g) ⇌ 3S(s) + 2H2O(g) ΔG° = 3 (0) + 2 (–299) – [2(–34) + 1 (–300)] = –90 kJ 2

ΔG = ΔG° + RT ln

ΔG = –90 kJ +

P H2O 2 P H2S × P SO 2

⎡ ⎤ 8,315 J kJ (3 × 10-2 ) 2 × × 298 K × ⎢ ln -4 2 -2 ⎥ K ⋅ mol 1000 J ⎣ (1,0 × 10 ) (1,0 × 10 ) ⎦

ΔG = –90kJ + 39,7 kJ = 50 kJ 51.

ΔG = ΔG° + RT lnQ = ΔG° + RT ln

PN2O4 2 PNO 2

ΔG° = 1 mol(98 kJ/mol) - 2 mol(52 kJ/mol) = -6 kJ a) Ce sont des conditions standard, donc ΔG = ΔG° parce que Q = 1 et ln Q = 0. Étant donné que ΔG° est négative, la réaction directe est spontanée. La réaction se déplace vers la droite pour atteindre l’équilibre. b) ΔG = -6 × 103 J + 8.3145 J/K·mol (298 K) ln

0,50 (0, 21) 2

ΔG = -6 × 103 J + 6.0 × 103 J = 0 Puisque ΔG = 0, cette réaction est à l’équilibre (aucun déplacement). c) ΔG = -6 × 103 J + 8.3145 J/K·mol (298 K) ln

1, 6 (0, 29) 2

ΔG = -6 × 103 J + 7.3 × 103 J = 1.3 × 103 J = 1 × 103 J Puisque ΔG est positive, la réaction inverse est spontanée et la réaction se déplace vers la gauche pour atteindre l’équilibre.

© 2007 Les Éditions CEC inc. Toute reproduction interdite

Chapitre 7 Thermodynamique chimique 52.

265

À 25,0 °C : ΔG° = ΔH° − TΔS° = −58,03 × 103 J/mol − (298,2 K)(−176,6 J/K·mol) = −5,37 × 103 J/mol

⎛ ⎞ -ΔG o -(-5,37 × 103 J/mol) ΔG° = −RT ln K, ln K = = exp ⎜ ⎟ = 2,166 RT ⎝ (8,3145 J/K ⋅ mol)(298,2 K) ⎠ K = e2,166 = 8,72 À 100,0 °C : ΔG° = −58,03 × 103 J/mol − (373,2 K)(−176,6 J/K·mol) = 7,88 × 103 J/mol

-(7,88 ×103 J/mol) = −2,540, K = e-2,540 = 0,0789 ln K = (8,3145 J/K ⋅ mol)(373,2 K) Note : quand on détermine les exposants, on arrondit une fois que les calculs sont terminés. Cela aide à éliminer l’erreur due à l’arrondissement excessif. 53.

a) ΔH = 2 mol(–92 kJ/mol) – [1 mol(0) + 1 mol(0)] = –184 kJ ΔS° = 2 mol(187 J/K⋅mol) – [1 mol(131 J/K⋅mol) + 1 mol(223 J/K⋅mol)] = 20 J/K ΔG° = ΔH° – TΔS° = –184 × 103 J – 298 K (20 J/K) = –1,90 × 105 J = –190 kJ ΔG = –RT ln K, ln K =

ΔG° -(-1,90 × 105 J) = = 76,683, 8,315 J/K ⋅ mol (298 K) RT

K = e76,683 = 2,01 × 1033 b) À ces condition standard, ΔG = ΔG° = –190 kJ. Comme ΔG est négative, la réaction se déplacera spontanément vers la droite. 54.

Quand on additionne des réactions, les constantes d’équilibre sont multipliées l’une par l’autre pour déterminer la valeur de K pour la réaction finale. H2(g) +O2(g)

⇌ H2O2(g)

K = 2,3 × 106

H2O(g) ⇌ H2(g) + 1/2O2(g) K = (1,8 × 1037)−1/2 _______________________________________________________________________ H2O(g) + 1/2O2(g)

⇌ H2O2(g)

K = 2,3 × 106(1,8 × 10−37) −1/2 = 5,4 × 10 −13

ΔG° = −RT ln K = −8,3145 J/K·mol (600 K) ln(5,4 × 10 −13 ) = 1,4 × 105 J/mol = 140 kJ/mol

© 2007 Les Éditions CEC inc. Toute reproduction interdite

Chapitre 7 Thermodynamique chimique

266 55.

a) 1re étape

4NH3(g)

+



5O2(g)

4NO(g)

+

6H2O(g)

ΔH fo

–46

0

90

–242

kJ/mol



193 –17

205 0

211 87

189 –229

J/K⋅mol kJ/mol

ΔG

o f

⎡ (90 kJ × 4 mol) + (-242 kJ × 6 mol) ⎤ ⎥⎦ mol ⎣

ΔH° = ⎢

⎡ (-46 kJ × 4 mol) + 0 ⎤ -⎢ ⎥⎦ mol ⎣ = –908 kJ

ΔS° =

⎡ (211 kJ × 4 mol) + (189 J × 6 mol) ⎤ ⎢⎣ ⎥⎦ K ⋅ mol

⎡ (193 J × 4 mol) + (205 J × 5 mol) ⎤ -⎢ ⎥⎦ K ⋅ mol ⎣ = 181 J/K

⎡ (87 kJ × 4 mol) + (+229 kJ × 6 mol) ⎤ ⎥⎦ mol ⎣

ΔG° = ⎢

⎡ (-17 kJ × 4 mol) + (0) ⎤ -⎢ ⎥⎦ mol ⎣ = –958 kJ

ΔG° = –RT ln K ln K = –

ΔG ° -(-958 × 103 J) K ⋅ mol = × = 386 RT 298 K 8,315 J

ln K = 2,303 log K, log K = ln K/2,303 = 168 K = 10168 (un nombre extrêmement élevé)

2e étape 2NO(g)

+

O2(g)



2NO2(g)

ΔH fo

90

0

34

kJ/mol



211 87

205 0

240 52

J/K⋅mol kJ/mol

ΔG

o f

⎡ (34 kJ × 2 mol) ⎤ ⎥⎦ mol ⎣

ΔH° = ⎢

⎡ (90 kJ × 2 mol) + 0 ⎤ -⎢ ⎥⎦ = –112 kJ mol ⎣

© 2007 Les Éditions CEC inc. Toute reproduction interdite

Chapitre 7 Thermodynamique chimique

267

⎡ (240 J × 2 mol) ⎤ ⎥⎦ K ⋅ mol ⎣

⎡ (211 J × 2 mol) + (205 J × 1 mol) ⎤ -⎢ ⎥⎦ = –147 J/K K ⋅ mol ⎣

⎡ (52 kJ × 2 mol) ⎤ ⎥⎦ mol ⎣

⎡ (87 kJ × 2 mol) + 0 ⎤ -⎢ ⎥⎦ = –70 kJ mol ⎣

ΔS° = ⎢

ΔG° = ⎢

ΔG° = –RT ln K ln K =

ΔG° -(-70 × 103 J) K ⋅ mol = x = 28 RT 298 K 8,315 J

K = 1,4 × 1012

3e étape → 2HNO3(l) +

3NO2(g)

+ H2O(l)

ΔH fo

34

–286

–174

90



240 52

70 –237

156 –81

211 J/K⋅mol 87 kJ/mol

ΔG

o f

NO(g) kJ/mol

ΔH° =

⎡ (-174 kJ × 2 mol) + (90 kJ × 1 mol) ⎤ ⎢⎣ ⎥⎦ mol

⎡ (34 kJ × 3 mol) + (-286 kJ × 1 mol) ⎤ -⎢ ⎥⎦ mol ⎣ = –74 kJ

ΔS° =

⎡ (156 J × 2 mol) + (211 J × 1 mol) ⎤ ⎢⎣ ⎥⎦ K ⋅ mol

⎡ (240 J × 3 mol) + (70 J × 1 mol) ⎤ -⎢ ⎥⎦ K ⋅ mol ⎣ = – 267 J/K

ΔG° =

⎡ (-81 kJ × 2 mol) + (87 kJ × 1 mol) ⎤ ⎢⎣ ⎥⎦ mol

⎡ (52 kJ × 3 mol) + (-237 kJ × 1 mol) ⎤ -⎢ ⎥⎦ mol ⎣

ΔG° = (–75 kJ) – (–81 kJ) = 6 kJ ΔG° = –RT ln K

© 2007 Les Éditions CEC inc. Toute reproduction interdite

Chapitre 7 Thermodynamique chimique

268

ln K = –

ΔG° -6 × 103 J K ⋅ mol = × = –2,4 RT 298 K 8,315 J

K = 9,1 × 10–2 b) ΔG° = –RT ln K

T = 85 °C = 1098 K

ΔG° = ΔH° – TΔS° = – 908 kJ – (1098 K)(0,181 kJ/K) = – 1107 kJ

⎛ ⎞ -ΔG o -(-1,107 × 106 J) K = exp = exp ⎜ ⎟ RT ⎝ 8,3145 J/mol ⋅ (1098 K) ⎠ e121,258 = 4,589 × 1052 c) Pour l’étape 1, ΔH° < 0, ΔS° > 0

ΔG = ΔH – TΔS sera négative quelle que soit la température. La réaction est toujours spontanée et il n’y a pas de raison thermodynamique pour augmenter la température. Toutefois, la vitesse de réaction sera plus grande à température plus élevée.

56.

2 SO2(g) + O2(g) → 2 SO3(g); ΔG° = 2(−371 kJ) − [2(−300 kJ)] = −142 kJ ΔG° = −RT ln K, ln K =

-ΔG o -(-142,000 J) = 57.311 = RT 8,3145 J/K ⋅ mol (298 K)

K = e57,311 = 7,76 × 1024 K = 7,76 × 1024 =

2 PSO 3 2 PSO × PO2 2

=

(2, 0) 2 , PSO 2 = 1,0 × 10 −12 atm 2 PSO2 × (0,50)

D’après la valeur négative de ΔG°, cette réaction est spontanée dans des conditions standard. Il y a plus de molécules de réactifs gazeux que de produits gazeux, de sorte que ΔS° sera négative (non favorable). Par conséquent, cette réaction doit être exothermique (ΔH° < 0). Quand ΔH° et ΔS° sont toutes les deux négatives, la réaction est spontanée à des températures relativement basses où le terme favorable ΔH° prédomine.

© 2007 Les Éditions CEC inc. Toute reproduction interdite

Chapitre 7 Thermodynamique chimique 57.

269

Le graphique de lnK en fonction de 1/T donne une droite. Pour un processus endothermique, la pente est négative (pente = -ΔH°/R) (y = mx + b). En augmentant la température de T1 à T2 (1/T2 < 1/T1), on voit que lnK, et donc K, augmente. Si K augmente, c’est qu’une plus grande quantité de réactifs est transformée en produits, donc que la réaction se déplace vers la droite.

EXERCICES SUPPLÉMENTAIRES 58.

D’après l’annexe 4, S° = 198 J/K·mol pour CO(g) et S° = 27 J/K·mol pour Fe(s). Soit S ol = S° pour Fe(CO)5(l) et S og = S° pour Fe(CO)5(g). ΔS° = -677 J/K = 1 mol(S ol ) – [1 mol (27 J/K·mol) + 5 mol(198 J/K·mol] S ol = 340 J/K·mol ΔS° = 107 J/K = 1 mol (S og ) – 1 mol (340 J/K·mol) S og = S° pour Fe(CO)5(g) = 447 J/K·mol

59.

Lorsqu’un solide ionique se dissout, on s’attend à une augmentation du désordre du système, de sorte que ΔSsyst est positive. Puisque la température augmente à mesure que le solide se dissout, c’est un processus exothermique et ΔSext est positive (ΔSext = -ΔH/T). Comme le solide s’est dissous, le processus de dissolution est spontané, de sorte que ΔSuniv est positive.

60.

ΔS sera négative parce que 2 mol de réactrifs gazeux forment 1 mol de produit gazeux. Pour que ΔG soit négative, ΔH doit être négative (exothermique). Pour les réactions exothermiques, K diminue à mesure que T augmente. Par conséquent, le rapport de la pression partielle de PCl5 à la pression partielle de PCl3 diminuera quand T augmentera.

61.

ΔS (méthane) = 73,2 J/K·mol ; ΔS(hexane) = 84,5 J/K·mol ; Vmét = 9,19 L et Vhex = 28,1 L; l’hexane a le plus grand volume molaire au point d’ébullition, de sorte que l’hexane devra avoir l’entropie la plus élevée. À mesure que le volume de gaz augmente, le désordre de position augmente.

Au point d’ébullition, ΔG = 0 de sorte que ΔS =

ΔH vap T

; pour le méthane:

© 2007 Les Éditions CEC inc. Toute reproduction interdite

Chapitre 7 Thermodynamique chimique

270

ΔS =

8,20 × 103 J/mol = 73,2 J/mol•K 112 K

Pour l’hexane : ΔS =

Vmét =

28,9 × 103 J/mol = 84,5 J/mol·K 342 K

nRT 1,00 mol (0,08206) (112 K) nRT = = 9,19 L; Vhex = = R(342 K) = 28,1 L P P 1,00 atm

L’hexane a le volume molaire le plus grand au point d’ébullition de sorte que l’hexane devrait avoir l’entropie la plus élevée. À mesure que le volume d’un gaz augmente, le désordre de position augmente. 62.

solide I → solide II; l’équilibre s’établit quand ΔG = 0. ΔG = ΔH - TΔS, ΔH = TΔS, T = ΔH/ΔS =

63.

-743,1 J/mol = 43,7 K = -229,5°C -17,0 J/K ⋅ mol

a) ΔG° = –RT ln K

ΔG° = -

8,315 J × 298 K × ln 0,090 = 5,97 × 103 J = 5,97 kJ K ⋅ mol

b) H–OH + Cl–O–Cl → H–O–Cl + H–O–Cl

2 liens O–H + 2 liens O–Cl → 2 liens O–H + 2 liens O–Cl ΔH° ≈ 0 c) ΔG° = ΔH° – TΔS°

ΔS° = −

ΔG o - ΔH o 5,97 kJ - 0 kJ = –0,0200 = –20,0 J/K = T K 298 K

H2O(g)

d)

+

Cl2O(g)



2HOCl(g)

ΔH fo

–242

80,3

?

kJ/mol



198

266,1

?

J/K⋅mol

⎡ (-242 kJ × 1 mol) + (80,3 kJ × 1 mol) ⎤ ⎥⎦ = 0 mol ⎣

ΔH° = ⎡ ΔH f,oHOCl × 2 mol ⎤ – ⎢ ⎣ ⎦

ΔH f,o HOCl = –81 kJ/mol © 2007 Les Éditions CEC inc. Toute reproduction interdite

Chapitre 7 Thermodynamique chimique

271

⎡ (187 J × 1 mol) + (266,1 J × 1 mol) ⎤ ⎥⎦ = –20,0 J/K K ⋅ mol ⎣

o × 2 mol ⎤⎦ – ⎢ ΔS° = ⎡⎣ ΔS HOCl

S°HOCl = 216,5 J/K⋅mol

e) ln K500 = –

o ΔG500 RT

Si on suppose que ΔH et ΔS ne varient pas avec la température

kJ ⎞ ⎛ 20 J × ⎟ = 10,0 kJ 1000 J ⎠ ⎝ K

ΔG°500 = 0 – 500 K × ⎜ -

ln K500 = -

10,0 kJ 1000 J K ⋅ mol × × = –2,41 500 K kJ 8,315 J

K500 = 0,090

⎛ ( PHOCl ) 2 ⎞ ⎜ ( PCl O )( PH O ) ⎟⎟ 2 2 ⎝ ⎠

f) ΔG = ΔG° + RT ln ⎜

ΔG = 5,97 kJ +

8,315 J kJ (1,3 × 10-2 kPa) 2 × × 298 K ln K ⋅ mol 1000 J (2,4 kPa)(0,27 kPa)

ΔG = 5,97 kJ + (–20,4 kJ) = –14,5 kJ

64.

HgbO2 → Hgb + O2 ΔG° = - (-70 kJ) Hgb + CO → HgbCO ΔG° = -80 kJ ____________________________________________ ΔG° = -10 kJ HgbO2 + CO → HgbCO + O2

⎛ -ΔG o ⎞ ⎛ ⎞ -(-10 × 103 J) ΔG° = -RT ln K, K = exp ⎜ ⎟ = exp ⎜ ⎟ = 60 ⎝ RT ⎠ ⎝ 8,3145 J/K ⋅ mol (298 K) ⎠ 65.

Ba(NO3)2(s) ⇌ Ba2+(aq) + 2NO3–(aq) ; ΔG° = –561 + 2(–109) – (–797) = 18 kJ ΔG° = –RT ln Kps, ln Kps = -

ΔG o -18,000 J = –7,26 ; Kps = e–7,26 = 8,315 J/K ⋅ mol (298 K) RT = 7,0 × 10–4

© 2007 Les Éditions CEC inc. Toute reproduction interdite

Chapitre 7 Thermodynamique chimique

272

66.

H2O(l) ⇌ H+(aq) + OH–(aq) ; ΔG° = 79,9 kJ. ΔG = ΔG° + RT ln Q où Q = [H+][OH–]

kJ ⎛ 8,315 J ⎞ × × 298 K × ln(10-7 )(10-7 ) ⎟ = 0 ; 1000 J ⎝ K ⋅ mol ⎠

a) ΔG = 79,9 kJ + ⎜

système à l’équilibre

kJ ⎛ 8,315 J ⎞ × × 298 K × ln(10-5 )(10-9 ) ⎟ = 0 ; 1000 J ⎝ K ⋅ mol ⎠

b) ΔG = 79,9 kJ + ⎜

système à l’équilibre

kJ ⎛ 8,315 J ⎞ × × 298 K × ln(10-10 )(10-10 ) ⎟ = –34 kJ < 0 ; 1000 J ⎝ K ⋅ mol ⎠

c) ΔG = 79,9 kJ + ⎜

spontanée vers la droite

kJ ⎛ 8,315 J ⎞ × × 298 K × ln(10)(10-7 ) ⎟ = +45,7 kJ > 0 ; 1000 J ⎝ K ⋅ mol ⎠

d) ΔG = 79,9 kJ + ⎜

vers la gauche

kJ ⎛ 8,315 J ⎞ × × 298 K × ln(1,0)(1,0) ⎟ = 79,9 kJ > 0 ; 1000 J ⎝ K ⋅ mol ⎠

e) ΔG – 79,9 kJ + ⎜

vers la gauche Les résultats sont compatibles avec le principe de Le Chatelier. 67.

K+(sang) ⇌ K+(muscle) Si les deux concentrations sont égales à 1,0 mol/L, c’est l’équilibre, ΔG° est donc égal à 0. ΔG = ΔG° + RT ln Q ΔG = RT ln Q

© 2007 Les Éditions CEC inc. Toute reproduction interdite

Chapitre 7 Thermodynamique chimique

ΔG =

273

8,315 J 0,15 × 310 K × ln = 8,77 × 103 J/mol K ⋅ mol 0,005

ΔG = Wmax Wmax = 8,77 kJ/mol

Pour que les solutions restent neutres, des ions négatifs devront aussi être transportés vers le muscle ou d’autres ions positifs devront être transportés vers le sang. 68.

− (−30500 J) ⎞ ⎟ = 2,22 × 10–5 ⎝ 8,315 J/K⋅ mol × 298 K ⎠ ⎛

a) ΔG° = – RT ln K, K = exp (–ΔG°/RT) = exp ⎜ b) C6H12O6(s) + 6O2(g) → 6CO2(g) + 6H2O(l)

ΔG° = 6 mol(–394 kJ/mol) + 6 mol(–237 kJ/mol) – 1 mol(–911 kJ/mol) = –2875 kJ

2875 kJ 1 mol ATP x = 94,3 mol ATP ; mol glucose 30,05 kJ 94,3 molécules ATP/molécule glucose En fait, seulement 38 mol d’ATP sont produites par le métabolisme d’une mole de glucose. Une partie importante de ΔG° est utilisée pour d’autres processus. 69.

a) ΔG° = –RT ln K

ln K = −

ΔG o −14 000 J = = –5,65, K = e–5,65 = 3,5 × 10–3 RT 8,315 J/K⋅ mol (298 K)

b) Acide glutamique + NH3 → Glutamine + H2O

ATP + H2O → ADP + H2PO4–

ΔG° = 14 kJ ΔG° = –30,5 kJ

Acide glutamique + ATP + NH3 → Glutamine + ADP + H2PO4– ΔG = 14 – 30,5 = –17 kJ ln K = 70.

ΔG o −17 000 J = − = 6,86, K = e6,86 = 9,5 × 102 8,315 J/K⋅ mol (298 K) RT

ΔS est plus favorable pour la réaction 2 que pour la réaction 1, ce qui donne K2 > K1. Dans la réaction un, sept particules en solution forment une particule. Dans la réaction deux, quatre particules forment une particule ce qui cause une diminution de désordre plus faible que dans la réaction un.

© 2007 Les Éditions CEC inc. Toute reproduction interdite

Chapitre 7 Thermodynamique chimique

274 71.

ΔG° = -RT ln K; quand K = 1,00, ΔG° = 0 étant donné que ln 1,00 = 0. ΔG° = 0 = ΔH° TΔS° ΔH° = 3(-242 kJ) - [-826 kJ] = 100 kJ; ΔS° = [2(27 J/K) + 3(189 J/K)] – [90 J/K + 3(131 J/K)] = 138 J/K ΔH° = TΔS°, T =

ΔH o 100 kJ = = 725 K o ΔS 0,138 kJ/K

PROBLÈMES DÉFIS 72.

3 O2(g) ln K =



2 O3(g); ΔH° = 2(143 kJ) = 286 kJ; ΔG° = 2(163 kJ) = 326 kJ

−ΔG o -326 × 103 J = = -131,573, K = e −131.573 = 7.22 × 10 −58 RT (8,3145 J/K ⋅ mol)(298 K)

On a besoin de la valeur de K à 230 K. D’après la section 7.8 du manuel :

−ΔG o ΔS o + ln K = RT R Pour deux conditions de K et de T :

−ΔH o ⎛ 1 ⎞ ΔS −ΔH o ⎛ 1 ⎞ ΔS o ln K1 = ; ln K2 = ⎜ ⎟ + ⎜ ⎟ + R ⎝ T1 ⎠ R R ⎝ T2 ⎠ R On soustrait la première expression de la seconde :

ΔH o ⎛ 1 1 ⎞ ΔH o ⎛ 1 1 ⎞ K2 = ln K2 - ln K1 = ⎜ − ⎟ ou ln ⎜ − ⎟ R ⎝ T1 T2 ⎠ K1 R ⎝ T1 T2 ⎠ Soit K2 = 7,22 × 10 −58 , T2 = 298 K; K1 = K230, T1 = 230 K; ΔH° = 286 × 103 J ln

7, 22 × 10−58 286 × 103 ⎛ 1 1 ⎞ = − ⎜ ⎟ = 34,13 K 230 8,3145 ⎝ 230 298 ⎠

7, 22 × 10−58 34,13 =e = 6,6 × 1014, K230 = 1,1 × 10 −72 K 230 K230 = 1,1 × 10

−72

=

PO23 PO33

=

PO23 (1, 0 × 10−3 atm)3

, PO3 = 3,3 × 10 −41 atm

© 2007 Les Éditions CEC inc. Toute reproduction interdite

Chapitre 7 Thermodynamique chimique

275

Le volume occupé par une molécule d’ozone est : V=

nRT P

=

(1/6,022 × 1023 mol) (0,08206 L ⋅ atm/mol ⋅ K)(230 K) = 9,5 × 1017 L (3,3 × 10-41atm)

L’équilibre n’est probablement pas maintenue dans ces conditions. Quand il y a seulement deux molécules d’ozone dans un volume de 9,5 × 1017 L, la réaction n’est pas à l’équilibre. Dans ces conditions, Q > K et la réaction se déplace vers la gauche. Mais avec seulement deux molécules d’ozone dans un énorme volume, il est extrêmement improbable qu’elles vont entrer en collision l’une avec l’autre. Dans ces conditions, la concentration d’ozone n’est pas assez grande pour maintenir l’équilibre.

73.

Arrangement I et V :

S = k ln W; W = 1; S = k ln 1 = 0

Arrangement II et IV :

W = 4; S = k ln 4 = 1,38 × 10 −23 J/K ln 4, S = 1,91 × 10 −23 J/K

Arrangement III :

W = 6; S = k ln 6 = 2,47 × 10 −23 J/K

74.

Ea(dir) = G1 – G2 Ea(inv) = G1 – G3 ΔG° = G3 – G2 → G3 = ΔG° + G2 Ea(inv) = G1 – (ΔG° + G2) Ea(inv) = G1 – G2 – ΔG° Ea(inv) = Ea(dir) – ΔG° (1) kdir = Ae- Ea (dir) / RT → ln kdir = lnA – kinv = Ae- Ea (inv) / RT → ln kinv = lnA –

E a (dir) (2) RT E a (inv) (3) RT

© 2007 Les Éditions CEC inc. Toute reproduction interdite

Chapitre 7 Thermodynamique chimique

276

De (2), ln A = ln kdir +

E a (dir) RT

De (3), ln A = ln kinv +

E a (inv) RT

(2) = (3) ln kdir +

E a (dir) = ln k + E a (inv) (4) inv RT RT

(1) dans (4) ln kdir +

ln kdir – ln kinv = –

o E a (dir) = ln k + E a (dir) – ΔG inv RT RT RT

ΔG o RT

ΔG k dir =– RT k inv

o

ln

Comme ΔG° = –RT ln K, ln

ln 75.

k dir ⎛ ( − RT ln K ) ⎞ = ⎜− ⎟ RT ⎠ k inv ⎝

k dir k = ln K et dir = K k inv k inv

À l’équilibre :

⎛ 1,10 × 1013 molécules ⎞ ⎛ 0,08206 L ⋅ atm ⎞ ⎟ (298 K) ⎜ ⎟⎜ 6,022 × 1023 molécules/mol ⎠ ⎝ K ⋅ mol nRT ⎠ ⎝ PH2 = = V 1, 00 L = 4,47 × 10–10 atm La pression de H2 a diminué de 1,00 atm à 4,47 × 10–10 atm, ce qui indique que presque tout le H2 et le Br2 ont réagi pour former HBr. Donc, PHBr 2,00 atm en raison du rapport stœchiométrique et P Br 2 = P H 2 = 4,47 × 10–10 atm. K=

2 (2, 00) 2 P HBr = 2,00 × 10–19 = -10 2 × (4, 47 × 10 ) P H 2 P Br 2

ΔG° = –RT ln K = –(8,315 J/K⋅mol)(298 K) ln (2,00 × 10–19) = –1,0 × 105 J/mol

© 2007 Les Éditions CEC inc. Toute reproduction interdite

Chapitre 7 Thermodynamique chimique

ΔS° =

76.

277

ΔH o − ΔG o -103 800 J/mol - (-1,10 × 105 J/mol) = 20 J/K⋅mol = 298 K T

a) ΔG° = GB° – GA° = 11,718 – 8996 = 2722 J B

⎛ - ΔG o ⎞ ⎟ = exp ⎝ RT ⎠

K = exp ⎜

⎛ ⎞ −2722 J ⎜ ⎟ = 0,333 ⎝ (8,315 J/K⋅ mol)(298 K) ⎠

b) Puisque Q = 1,00 > K, la réaction se déplace vers la gauche.

A(g) Initiale Équilibre

1,00 atm 1,00 + x



B(g) 1,00 atm 1,00 – x

1,00 - x = 0,333, 1,00 – x = 0,333 + 0,333 x, x = 0,50 atm 1,00 + x PB = 1,00 – 0,50 = 0,50 atm; PA = 1,00 + 0,50 = 1,50 atm B

c) ΔG = ΔG° + RT ln Q = ΔG° + RT ln (PB/PA) B

ΔG = 2722 J + (8,3145)(298) ln (0,50/1,50) = 2722 J – 2722 J = 0

77.

Selon l’exercice 65, ln K =

ΔS ° - ΔH ° , R = 8,315 J/K⋅mol + RT R

Pour K à deux températures T2 et T2, l’équation est : ln

ln 10,0 =

o K 2 = ΔH ⎛ 1 - 1 ⎞ ⎟ R ⎜⎝ T 1 T2⎠ K1

ΔH o ⎛ 1 1 ΔH o ⎞ , 2,30 = (4,76 × 10–4) 8,315 ⎜⎝ 300,0 K 350,0 K ⎠⎟ 8,315

ΔH° = 4,02 × 104 J/mol = 40,2 kJ/mol 78.

Kp = P CO 2 ; pour empêcher Ag2CO3 de se décomposer, P CO 2 doit être plus grand que Kp.

Selon l’exercice 65, ln K =

− ΔH ° ΔS ° . Pour deux conditions de K et T, + RT R

l’équation est :

© 2007 Les Éditions CEC inc. Toute reproduction interdite

Chapitre 7 Thermodynamique chimique

278

ln K 2 = K1

ΔH o ⎛ 1 1 ⎞ − ⎜ ⎟ R ⎝ T1 T2⎠

Soit T1 = 25 °C = 298 K, K1 = 6,23 × 10–3 torr ; T2 = 110 °C = 383 K, K2 ?

79,14 ×103 J/mol ⎛ 1 1 ⎞ K2 ln = − ⎜ ⎟ −3 6,23 ×10 torr 8,315 J/K ⋅ mol ⎝ 298 K 383 K ⎠

ln

K2 = 7,1, 6,23 × 10−3

K2 = e7,1 = 1,2 × 103, K2 = 7,5 torr 6,23 × 10−3

Pour empêcher la décomposition de Ag2CO3, la pression partielle de CO2 doit être supérieure à 7,5 torr.

79.

L D’après les données du problème, χ CL6 H6 = χ CCl = 0,500. On a besoin des pressions de 4

vapeur pure (Po) pour calculer la pression de vapeur de la solution. À l’aide des données thermodynamiques : C6H6(l) ⇌ C6H6(g)

K = PC6 H6 = PCo6 H6

à 25 °C

o ΔGréaction = ΔGf,o C6 H6 (g ) − ΔGf,o C6 H6 (l ) = 129,66 kJ/mol - 124,50 kJ/mol = 5,16 kJ/mol

ΔG° = -RT lnK, lnK =

−ΔG o -5,16 × 103 J/mol = exp = -2,08 (8,3145 J/K ⋅ mol)(298 K) RT

K = PCo6 H6 = e −2,08 = 0,125 atm o Pour CCl4: ΔGréaction = ΔGf,o CCl4 (g ) − ΔGf,o CCl4 (l ) = -60,59 kJ/mol - (-65,21 kJ/mol)

= 4,62 kJ/mol

⎛ -ΔG o ⎝ RT

o K = PCCl = exp ⎜ 4

⎞ ⎛ ⎞ -4620 J/mol ⎟ = exp ⎜ ⎟ = 0,155 atm ⎝ 8,3145 J/K ⋅ mol × 298 K ⎠ ⎠

o PC6 H6 = χ CL6 H6 PCo6 H6 = 0,500 (0,125 atm) = 0,0625 atm; PCCl = 0,500 (0,155 atm) 4

= 0,0775 atm

© 2007 Les Éditions CEC inc. Toute reproduction interdite

Chapitre 7 Thermodynamique chimique

χ CV H = 6

6

PC6 H6 Ptot

=

279

0,0625 atm 0, 0625 = = 0,446 0,0625 atm + 0,0775 atm 0,1400

V χ CCl = 1,000 - 0,446 = 0,554 4

80.

À l’aide des données thermodynamique on calcule le point d’ébullition du solvant.

ΔH 33,90 ×103 J/mol = 353,3 K Au point d’ébullition : ΔG = 0 = ΔH - TΔS, T = = ΔS 95,95 J/K ⋅ mol ΔT = Kbm, (355,4 K -353,3 K) = 2,5 K·kg/mol (m), m =

masse de solvant = 150 mL ×

0,879 g 1 kg = 0,132 kg × mL 1000 g

masse de soluté = 0,132 kg solvant ×

81.

2,1 = 0,84 mol/kg 2,5

0,84 mol solute 142 g = 15,7 g = 16 g soluté × kg solvent mol

ΔSext = -ΔH/T = -qP/T q = perte de chaleur par l’eau chaude = moles × capacité calorifique molaire × ΔT q = 1,00 × 103 g H2O ×

ΔSext =

1 mol H 2 O 75,4 J × × (298,2 – 363,2) = -2,72 × 105 J 18,02 g K ⋅ mol

-(-2,72 ×105 J) = 912 J/K 298,2 K HX

82. Initiale Équilibre



0,10 mol/L 0,10 – x

H+

+

~0 x

X−

Ka =

[H + ][X − ] [HX]

0 x

D’après les données du problème, x = [H+] = 10−5,83 = 1,5 × 10 −6 ; Ka =

(1,5 × 10−6 ) 2 = 2,3 × 10−11 0,10 − 1,5 × 10−6

ΔGo = -RTln(K) = -8,3145 J/K·mol (298 K) ln(2,3 × 10 −11 ) = 6,1 × 104 J/mol = 61 kJ/mol

© 2007 Les Éditions CEC inc. Toute reproduction interdite

Chapitre 7 Thermodynamique chimique

280

83.

NaCl(s)



Na+(aq) + Cl−(aq)

K = Kps = [Na+][Cl−]

ΔGo = [(-262 kJ) + (-131 kJ)] – (-384 kJ) = -9 kJ = -9000 J



⎤ -(-9000 J) ⎥ = 38 = 40 ⎣ 8,3145 J/K ⋅ mol× 298 K ⎦

ΔGo = = -RTln(Kps), Kps = exp ⎢

NaCl(s)



Initiale s = solubilité (mol/L) Équilibre

Na+(aq) + Cl−(aq) 0 s

Kps = 40

0 s

Kps = 40 = s(s), s = (40)1/2 = 6.3 = 6 mol/L = [Cl−]

PROBLÈMES D’INTÉGRATION 84.

Étant donné que la pression partielle de C(g) a diminué, le changement net qui se produit dans cette réaction afin d’atteindre l’équilibre c’est la conversion de produits en réactifs. A(g) Initiale Changement Équilibre

0,100 atm +x 0,100 + x

+

2 B(g)



0,100 atm +2x ← 0,100 + 2x

C(g) 0,100 atm -x 0,100 – x

D’après les données du problème, PC = 0,040 atm = 0,100 – x, x = 0,060 atm Les pressions partielles à l’équilibre sont : PA = 0,100 + x = 0,100 + 0,060 = 0,160 atm, PB = 0,100 + 2((0,60) = 0,220 atm, et PC = 0,040 atm B

K=

0, 040 = 5,2 0,160(0, 220) 2

ΔGo = -RTln(K) = -8,3145 J/K·mol (298 K) ln5,2 = -4,1 × 103 J/mol = -4,1 kJ/mol

85.

ΔGo = ΔHo - TΔSo = -28,0 × 103 J – 298 K(-175 J/K) = 24,200 J

-ΔG o -24 000 J = ΔG = -RTln(K), ln(K) = = -9,767 RT 8,3145 J/K ⋅ mol × 298 K o

K = e-9,767 = 5,73 × 10 −5

© 2007 Les Éditions CEC inc. Toute reproduction interdite

Chapitre 7 Thermodynamique chimique

B +

H2O



Initiale 0,125 mol/L Changement -x Équilibre 0,125 - x Kb = 5,73 × 10 −5 =

281

BH+ 0 +x x

+

OH−

K = Kb = 5,73 × 10 −5

~0 +x x 2

2

x x [BH + ][OH − ] = ≈ , x = [OH−] = 2,68 × 10 −3 mol/L [ B] 0,125 − x 0,125

pH = -log(2,68 × 10 −3 ) = 2,572; pOH = 14,000 – 2,572 = 11,428; Approximation valide

PROBLÈME DE SYNTHÈSE 86.

a) ΔS° est négative parce qu’il y a une diminution du nombre de moles de gaz. b) Étant donné que ΔS° est négative, ΔH° doit être négative pour que la réaction soit spontanée à certaines températures. Par conséquent, ΔSext est positive. c) Ni(s) + 4 CO(g)



Ni(CO)4(g)

ΔH° = -607 - [4(-110.5)] = -165 kJ; ΔS° = 417 - [4(198) + (30)] = -405 J/K d) ΔG° = 0 = ΔH° - TΔS°, T =

ΔH o -165 ×103 J = = 407 K ou 134 °C ΔS o -405 J/K

e) T = 50°C + 273 = 323 K o ΔG 323 = -165 kJ - (323 K)(-0,405 kJ/K) = -34 kJ

ln(K) =

-ΔG o -(-34 000 J) = = 12,66, K = e12,66 = 3,1 × 105 RT 8,3145 J/K ⋅ mol (323 K)

f) T = 227 °C + 273 = 500 K o ΔG 500 = -165 kJ - (500 K)( -0,405 kJ/K) = 38 kJ

ln(K) =

-38 000 J = -9,14, K = e-9,14 = 1,1 × 10 −4 (8,3145 J/K ⋅ mol )(500 K)

g) Le changement de température provoque le changement de la valeur de la constante d’équilibre qui passe d’une valeur élevée favorisant la formation de Ni(CO)4 à une valeur faible favorisant la décomposition de Ni(CO)4 en Ni pur et en CO. C’est exactement ce qu’il faut pour purifier un échantillon de nickel.

© 2007 Les Éditions CEC inc. Toute reproduction interdite

Chapitre 7 Thermodynamique chimique

282 h) Ni(CO)4(l) ⇌ Ni(CO)4(g)

K = PNi(CO)4

À 42 °C (le point d’ébullition) : ΔG° = 0 = ΔH° - TΔS° ΔS ° =

ΔH o 29,0 ×103 J = = 92,1 J/K T 315 K

o À 152 °C : ΔG 152 = ΔH° - TΔS° = 29,0 × 103 J - 425 K (92,1 J/K) = -10 100 J

ΔG° = -RTln(K), ln(K) =

-(-10 100 J) = 2,858, Kp = e2,858 = 17,4 8,3145 J/K ⋅ mol(425 K)

On peut atteindre une pression maximum de 17,4 atm avant que Ni(CO)4(g) se liquéfie.

© 2007 Les Éditions CEC inc. Toute reproduction interdite

CHAPITRE 8 ÉLECTROCHIMIE RÉVISION DES RÉACTIONS D’OXYDORÉDUCTION 13.

Oxydation : augmentation du nombre d’oxydation, perte d’électrons. Réduction : diminution du nombre d’oxydation, gain d’électrons.

14.

Voir le tableau 1.2 du chapitre 1 pour les règles d’attribution des nombres d’oxydation. a) H (+1), O (–2), N (+5)

b) Cl (–1), Cu (+2)

c) O (0)

d) H (+1), O (–1)

e) Mg (+2), O (–2), S (+6)

f) Ag (0)

g) Pb (+2), O (–2), S (+6)

h) O (–2), Pb (+4)

i) Na (+1), O (–2), C (+3)

j) O (–2), C (+4)

k) (NH4)2Ce(SO4)3 contient des ions NH4+ et SO42–. Cela donne à Ce une charge 4+. H (+1), N (–3), Ce (+4), S (+6), O (–2) l) O (–2), Cr (+3) 15.

Les réactions a, b et c sont des réactions d’oxydoréduction. Rédox?

Agent oxydant

Agent réducteur

Espèce oxydée

Espèce réduite

a) oui

H2O

CH4

CH4(C)

H2O(H)

b) oui

AgNO3

Cu

Cu

AgNO3(Ag)

c) oui

HCl

Zn

Zn

HCl(H)

d) non; il n’y a aucune variation des nombres d’oxydation

© 2007 Les Éditions CEC inc. Toute reproduction interdite

Chapitre 8 Électrochimie

284 16.

Voir la section 1.10 pour les règles d’équilibrage des réactions rédox. a) Cr → Cr3+ + 3e–

NO3– → NO 4 H+ + NO3– → NO + 2H2O – 3e + 4H+ + NO3– → NO + 2H2O

Cr → Cr3+ + 3e– 3e + 4H + NO3– → NO + 2H2O –

+

4H+(aq) + NO3–(aq) + Cr(s) → Cr3+(aq) + NO(g) + 2H2O(l) b) (Al → Al3+ + 3e–) × 5

MnO4– → Mn2+ 8H + MnO4– → Mn2+ + 4H2O (5e– + 8H+ + MnO4– → Mn2+ + 4H2O) × 3 +

5Al → 5Al3+ + 15e– 15e– + 24H+ + 3MnO4– → 3Mn2+ + 12H2O 24H+(aq) + 3MnO4–(aq) + 5Al(s) → 5Al3+(aq) + 3Mn2+(aq) + 12H2O(l) c) (Ce4+ + e– → Ce3+) × 6

CH3OH → CO2 H2O + CH3OH → CO2 + 6H+ H2O + CH3OH → CO2 + 6H+ + 6e–

6Ce4+ + 6e– → 6Ce3+ H2O + CH3OH → CO2 + 6H+ + 6e– H2O(l) + CH3OH(aq) + 6Ce4+(aq) → 6Ce3+(aq) + CO2(g) + 6H+(aq) d)

PO33– → PO43– (H2O + PO33– → PO43– + 2H+ + 2e–) × 3 MnO4– → MnO2 (3e– + 4H+ + MnO4– → MnO2 + 2H2O) × 2 3H2O + 3PO33– → 3PO43– + 6H+ + 6e– 6e– + 8H+ + 2MnO4– → 2MnO2 + 4H2O 2H+ + 2MnO4– + 3PO33– → 3PO43– + 2MnO2 + H2O Convertissons à la solution basique en ajoutant 2OH– de chaque côté. 2H+ + 2OH– → du côté des réactifs. Simplifions enfin les H2O : H2O(l) + 2MnO4–(aq) + 3PO33–(aq) → 3PO43–(aq) + 2MnO2(s) + 2OH–(aq)

© 2007 Les Éditions CEC inc. Toute reproduction interdite

Chapitre 8 Électrochimie

e)

285 OCl– → Cl– 2e + 2H + OCl– → Cl– + H2O

Mg → Mg(OH)2 2H2O + Mg → Mg(OH)2 + 2H+ + 2e–



+

2H2O + Mg → Mg(OH)2 + 2H+ + 2e– 2e + 2H+ + OCl– → Cl– + H2O –

OCl–(aq) + H2O(l) + Mg(s) → Mg(OH)2(s) + Cl–(s) L’équation finale ne contenant pas de H+, l’opération est terminée. f)

H2CO → HCO3– 2H2O + H2CO → HCO3– + 5H+ + 4e–

Ag(NH3)2+ → Ag + 2NH3 (e + Ag(NH3)2+ → Ag + 2NH3) × 4 –

2H2O + H2CO → HCO3– + 5H+ + 4e– 4e + 4Ag(NH3)2+ → 4Ag + 8NH3 –

4Ag(NH3)2+ + 2H2O + H2CO → HCO3– + 5H+ + 4Ag + 8NH3 Ajoutons 5OH– de chaque côté (5H+ + 5OH– → 5H2O), puis simplifions les H2O. 5OH–(aq) + 4Ag(NH3)2+(aq) + H2CO(aq) → HCO3–(aq) + 3H2O(l) + 4Ag(s) + 8NH3(aq)

QUESTIONS 17.

Le magnésium est un métal alcalino-terreux ; Mg s’oxyde en Mg2+. L’état d’oxydation de l’hydrogène dans HCl est +1. Pour être réduit, l’état d’oxydation de H doit diminuer. Le choix évident pour l’hydrogène produit est H2(g), où l’hydrogène a un état d’oxydation de zéro. L’équation équilibrée est Mg(s) + 2HCl(aq) → MgCl2(aq) + H2(g). Étant donné qu’il y a deux atomes H dans l’équation équilibrée, les atomes H gagnent au total deux électrons. Par conséquent, deux électrons sont transférés dans l’équation équilibrée. Lorsque les électrons sont transférés directement de Mg à H+, aucun travail n’est produit. Afin de pouvoir exploiter cette réaction pour effectuer un travail utile, on doit faire passer le courant d’électrons dans un fil conducteur. On peut y arriver en construisant une pile électrochimique qui sépare la réaction de réduction de la réaction d’oxydation, afin de faire passer le courant d’électrons dans un fil conducteur pour produire un voltage.

18.

Les piles électrochimiques utilisent des réactions redox pour produire un voltage. La o positive quand on traite les demisolution consiste à avoir une valeur globale de ε pile réactions. Pour deux demi-réactions quelconque, la demi-réaction ayant le potentiel de réduction le plus positif sera toujours la réaction à la cathode. Pour les potentiels négatifs, ce sera la demi-réaction ayant le potentiel standard de réduction le plus près de zéro. La o demi-réaction qui reste (celle avec le ε réd ) le plus négatif) est inversée et devient la demio ). Cette combinaison donne toujours un potentiel standard de réaction à l’anode (εox = - ε réd pile global positif qui peut être utilisé pour faire fonctionner une pile.

© 2007 Les Éditions CEC inc. Toute reproduction interdite

Chapitre 8 Électrochimie

286 19.

20.

Une propriété extensive dépend directement de la quantité de substance. La variation d’énergie libre pour une réaction dépend si 1 mol de produit est formée ou 2 mol de produits sont formées ou 1 million de mol de produits sont formées. L’équation qui relie ΔG à ε est ΔG = -nFε. C’est le terme n qui convertit la propriété intensive ε en propriété extensive ΔG. C’est avec le terme n qui est le nombre de moles d’électrons transférés dans l’équation équilibrée que ΔG est associée. o ε = ε pile -

0, 0591 log Q n

Dans une pile de concentration, les composants de l’anode et de la cathode sont les mêmes, o donc, ε pile = 0 pour une pile de concentration. Quelle que soit la demi-réaction choisie, la demi-réaction opposée se produit dans l’autre compartiment. La force motrice qui produit un voltage est le terme -log Q dans l’équation de Nernst. Q est déterminé par la concentration des ions dans les compartiments de l’anode et de la cathode. Plus la différence des concentrations est grande, plus le voltage produit est grand. Par conséquent, la force motrice pour les piles de concentration est la différence de la concentration des ions entre les compartiments de la cathode et de l’anode. Quand les concentrations des ions sont égales, Q = 1 et log Q = 0, et aucun voltage n’est produit. 21.

Quand on utilise une batterie d’appoint pour faire démarrer une auto, le danger est la possibilité que l’électrolyse de H2O(l) ait lieu. Lorsque H2O(l) est électrolysée, les produits forment un mélange gazeux explosif de H2(g) et de O2(g). Une étincelle produite durant le démarrage peut enflammer H2(g) et O2(g). Fixer le fil de masse loin de la batterie diminue le risque d’une étincelle près de la batterie où H2(g) et O2(g) peuvent s’accumuler.

22.

Les métaux se corrodent parce qu’ils s’oxydent facilement. Si l’on se reporte au tableau 8.1, la plupart des métaux sont associés à des potentiels standard de réduction négatifs. Cela signifie que les réactions inverses, les demi-réactions d’oxydation, ont des potentiels d’oxydation positifs, ce qui indique qu’ils s’oxydent très facilement. Un autre point important c’est que la réduction de O2 (qui est le réactif dans les processus d’oxydation) o o possède une ε réd plus positive que celle de la plupart des métaux (pour O2, ε réd = 0,40 V). Cela signifie que si O2 est couplé avec la plupart des métaux, la réaction sera spontanée o > 0, de sorte que la corrosion a lieu. puisque ε pile Les métaux nobles (Ag, Au et Pt) ont tous des potentiels de réduction standard plus grand que celui de O2. Par conséquent, O2 n’est pas capable d’oxyder ces métaux dans des conditions standard. Note : les potentiels de réduction standard pour Pt → Pt2+ + 2 e− n’est pas dans le tableau 8.1. Comme on s’y attend, son potentiel de réduction est plus grand que celui de O2. ( ε Pto = 1,19 V).

23.

Il faut connaître la nature du métal afin de savoir quelle masse molaire utiliser. Il faut connaître l’état d’oxydation de l’ion métallique dans le sel, de façon à pouvoir déterminer le nombre de moles d’électrons transférés. Enfin, il faut connaître la quantité de courant et le temps pendant lequel le courant passe dans la cellule électrolytique.

© 2007 Les Éditions CEC inc. Toute reproduction interdite

Chapitre 8 Électrochimie 24.

287

L’aluminium existe naturellement sous forme d’oxyde. Il a une affinité plus grande pour l’oxygène de sorte qu’il est extrêmement difficile de réduire les ions Al3+ de l’oxyde en métal pur. Une façon possible consiste à essayer de dissoudre l’oxyde d’aluminium dans l’eau afin de libérer les ions. Même si les ions aluminium passaient en solution, l’eau serait réduite préférentiellement dans une cellule électrolytique. Une autre méthode pour mobiliser les ions consiste à faire fondre l’oxyde d’aluminium. Ce n’est pas pratique à cause du point de fusion très élevé de l’oxyde d’aluminium. La découverte clé a été de trouver un solvant qui ne serait pas réduit plus facilement que les ions Al3+(comme l’eau qui est plus facilement réduite). Le solvant découvert par Hall et Héroult (séparément) fut Na3AlF6. Un mélange de Al2O3 et de Na3AlF6 possède un point de fusion beaucoup plus bas que celui de Al2O3 pur. Par conséquent, la mobilité de l’ion Al3+ est plus facile à atteindre, ce qui rend possible la réaduction de Al3+ en Al.

EXERCICES Piles électrochimiques, potentiels de pile, potentiels standards d’électrode et énergie libre 25.

Voir la figure 8.3 pour la structure typique d’une pile électrochimique. Le compartiment de l’anode contient les composés et les ions participant à la demi-réaction d’oxydation, et celui de la cathode, ceux de la demi-réaction de réduction. Les électrons vont de l’anode à la cathode ; les cations se déplacent vers la cathode et les anions, vers l’anode. a) Pour obtenir la réaction fournie, il faut qu’il y ait réduction de Cl2 en Cl-, et oxydation de Cr3+ en Cr2O72- ; 7H2O(l) + 2Cr3+(aq) + 3Cl2(g) → Cr2O72-(aq) + 6Cl-(aq) + 14H+(aq) ; cathode : électrode de Pt, solution de Cl- dans laquelle on fait barboter Cl2(g) ; anode : électrode de Pt, solution contenant Cr3+, H+ et Cr2O72- ; b) Cu2+(aq) + Mg(s) → Cu(s) + Mg2+(aq) ; cathode : électrode de Cu, solution de Cu2+ ; anode : électrode de Mg, solution de Mg2+.

26.

(Voir le diagramme de la question 25.) a) IO3– + Fe2+ ⇌ Fe3+ + I2 Compartiment anodique Fe2+ → Fe3+ Fe2+ → Fe3+ + 1e– Anode : métal inerte : platine. Solution : Fe2+, Fe3+ Compartiment cathodique IO3– → I2 12H+ + 2IO3– + 10e– → I2 + 6H2O Cathode : métal inerte : platine Solution : IO3–, I2, milieu acide

© 2007 Les Éditions CEC inc. Toute reproduction interdite

Chapitre 8 Électrochimie

288 10 [Fe2+ → Fe3+ + 1e–] 12H+ +2IO3– + 10e– → I2 + 6H2O 12H+ + 2IO3– + 10Fe2+ → I2+ 10Fe3+ + 6H2O b) Zn + Ag+ → Zn2+ + Ag Compartiment anodique Zn → Zn2+ Zn → Zn2+ + 2e– Anode : Zn Solution : Zn2+ Compartiment cathodique Ag+ → Ag Ag+ + 1e– → Ag Cathode : Ag Solution : Ag+ Zn → Zn2+ + 2e– 2[Ag+ + 1e– → Ag] Zn + 2Ag+ → Zn2+ + 2Ag 27.

28.

29.

Pour déterminer εpile, il faut additionner le potentiel standard de réduction au potentiel standard d’oxydation (εºpile = εºréd + εºox). Cependant, il faut se souvenir que εºox = –εºréd et que ces valeurs ne sont pas mutlipliées par les coefficients utilisés pour équilibrer la réaction globale. 25a)

o o o = ε Cl + ε Cr ε pile → Cl

25b)

o o = ε Cu ε pile

26a)

o o o ε pile = ε IO → I + ε Fe

26b)

o o = ε Ag ε pile

-

2

2+

3

→ Cu

2

+

→ Ag

3+

→ Cr2 O72-

= 1,36 V + (–1,33 V) = 0,03 V

o + ε Mg = 0,34 V + 2,37 V = 2,71 V → Mg 2+

2+

→ Fe3+

= 1,20 V + (–0,77 V) = 0,43 V

o + ε Zn = 0,80 V + 0,76 V = 1,56 V → Zn 2+

Voir l’exercice 25 pour la description générique d’une pile aux conditions standard. a) Compartiment anodique Br– → Br2 2Br– → Br2 + 2e– Anode : platine Solution : Br2, Br–

© 2007 Les Éditions CEC inc. Toute reproduction interdite

Chapitre 8 Électrochimie

289

Compartiment cathodique Cl2 → Cl– Cl2 + 2e– → 2Cl– Cathode : platine Solution : Cl2 qui barbote dans une solution contenant Cl–. 2Br– → Br2 + 2e– –εº = –1,09 V Cl2 – 2e– → 2Cl– εº = 1,36 V – – Cl2 + 2Br → Br2 + Cl εºpile = 1,36 + (–1,09) = 0,27 V b) Compartiment anodique Mn2+ → MnO4– Mn2+ → 4H2O → MnO4– + 8H+ + 5e– Anode : platine Solution : Mn2+, MnO4–, milieu acide Compartiment cathodique IO4– → IO3– IO4– + 2H+ + 2e– → IO3– + H2O Cathode : platine Solution : IO4–, IO3–, milieu acide 2[Mn2+ + 4H2O → MnO4– + 8H+ + 5e–] –εº = 1,51 V – + – – 5[IO4 + 2H + 2e → IO3 + H2O] εº = 1,60 V 2Mn2+ + 5IO4– + 3H2O → 2MnO4– + 5IO3– + 6H+ εºpile = 1,60 + (–1,51) = 0,09 V 30.

a) Compartiment anodique Al → Al3+ Al → Al3+ + 3e– Anode : Al Solution : Al3+ Compartiment cathodique Ni2+ → Ni Ni2+ + 2e– → Ni Cathode : Ni Solution : Ni2+ –2 [Al → Al3+ + 3e–] –εº = 1,66 V 3 [Ni2+ + 2e– → Ni] εº = –0,23 V 2+ 3+ 2Al + 3Ni → 3Ni + 2Al εºpile = 1,66 + (–0,23) = 1,43 V

© 2007 Les Éditions CEC inc. Toute reproduction interdite

Chapitre 8 Électrochimie

290 b) Compartiment anodique Fe2+ → Fe3+ Fe2+ → Fe3+ + 1e– Anode : platine Solution : Fe2+, Fe3+ Compartiment cathodique Co3+ → Co2+ Co3+ + 1e– → Co2+ Cathode : platine Solution : Co2+, Co3+ Fe2+ → Fe3+ + 1e– –εº = –0,77 V 3+ – 2+ Co + 1e → Co εº = 1,95 V Fe2+ + Co3+ → Co2+ + Fe3+ εºpile = 1,95 + (–0,77) = 1,18 V 31.

Dans la représentation schématique d’une pile, on place l’anode en premier et la cathode en dernier. Une ligne double verticale sépare les deux compartiments et, dans chaque compartiment, une ligne simple indique un changement de phase. Il faut aussi indiquer la concentration des solutés et la pression des gaz. 25a) Pt⏐Cr3+ (1,0 mol/L), H+ (1,0 mol/L), Cr2O72- (1,0 mol/L)⏐⏐Cl2 (101,3 kPa)⏐Cl- (1,0 mol/L)⏐Pt ; 25b) Mg⏐Mg2+ (1,0 mol/L)⏐⏐Cu2+ (1,0 mol/L)⏐Cu ; 29a) Pt⏐Br- (1,0 mol/L), Br2 (1,0 mol/L)⏐⏐Cl2 (101,3 kPa)⏐Cl- (1,0 mol/L)⏐Pt ; 29b) Pt⏐Mn2+ (1,0 mol/L), MnO4- (1,0 mol/L), H+ (1,0 mol/L)⏐⏐IO4- (1,0 mol/L), IO3- (1,0 mol/L), H+ (1,0 mol/L)⏐Pt.

32.

26a) Pt⏐Fe2+ (1,0 mol/L), Fe3+ (1,0 mol/L)⏐⏐IO3– (1,0 mol/L), I2 (1,0 mol/L), H+ (1,0 mol/L)⏐Pt 26b) Zn⏐Zn2+ (1,0 mol/L)⏐⏐Ag+ (1,0 mol/L)⏐Ag 30a) Al⏐Al3+ (1,0 mol/L)⏐⏐Ni2+ (1,0 mol/L)⏐Ni 30b) Pt⏐Fe2+ (1,0 mol/L), Fe3+ (1,0 mol/L)⏐⏐Co3+ (1,0 mol/L), Co2+ (1,0 mol/L)⏐Pt

33.

Repérez les demi-réactions pertinentes au tableau 8.1 et imaginez quelle combinaison donnera un potentiel standard de pile positif. Dans tous les cas, le compartiment anodique contient l’espèce qui a le plus petit potentiel de réduction standard. Pour la partie a, l’anode est le compartiment du cuivre, et dans la partie b, l’anode est le compartiment de cadmium.

© 2007 Les Éditions CEC inc. Toute reproduction interdite

Chapitre 8 Électrochimie

a)

b)

291

Au3+ + 3 e− → Au εº = 1,50 V + 2+ − (Cu → Cu + e ) × 3 -εº = -0,16 V ________________________________________________________ o Au3+(aq) + 3 Cu+(aq) → Au(s) + 3 Cu2+(aq) ε pile = 1,34 V o ε pile = 1,00 V

(VO2+ + 2 H+ + e− → VO2+ + H2O) × 2

o −ε pile = 0,40 V

Cd → Cd2+ + 2e-

___________________________________________________________________ o 2 VO2+(aq) + 4 H+(aq) + Cd(s) → 2 VO2+(aq) + 2 H2O(l) + Cd2+(aq) ε pile = 1,40 V

34.

a)

(5e– + 8H+ + MnO4– → Mn2+ + 4H2O) × 2 (2I– → I2+ 2e–) × 5 16H+ + 2MnO4– + 10I– → 5I2 + 2Mn2+ + 8H2O

b)

(5e– + 8H+ + MnO4– → Mn2+ + 4H2O) × 2 (2F– → F2 + 2e–) × 5 16H+ + 2MnO4– + 10F– → 5F2 + 2Mn2+ + 8H2O

Cl2 + 2e– → 2Cl– (ClO2– → ClO2 + e–) × 2

35.

2ClO2–(aq) + Cl2(g) → 2ClO2(aq) + 2Cl–(aq)

ε° = 1,51 V –ε° = –0,54 V o = 0,97 V ε pile

Spontanée

ε° = 1,51 V –ε° = –2,87 V o = –1,36 V Non spontanée ε pile

ε° = 1,36 V –ε° = –0,954 V εºpile = 0,41 V = 0,41 J/C

ΔG° = –nFεºpile = –(2 mol e–)(96 485 C/mol e–)(0,41 J/C) = –7,91 × 104 J = –79 kJ

36.

a)

(4H+ + NO3– + 3e– → NO + 2H2O) × 2 (Mn → Mn2+ + 2e–) × 3 3Mn(s) + 8H+(aq) + 2NO3–(aq) → 2NO(g) + 4H2O(l) + 3Mn2+(aq) 5 × (2e– + 2H+ + IO4– → IO3– + H2O) 2 × (Mn2+ + 4H2O → MnO4– + 8H+ + 5e–)

ε° = 0,96 V –ε° = 1,18 V εºpile = 2,14 V ε° = 1,60 V –ε° = –1,51 V

5IO4–(aq) + 2Mn2+(aq) + 3H2O(l) → 5IO3–(aq) + 2MnO4–(aq) + 6H+(aq) εºpile = 0,09 V b) Oxydation par l’acide nitrique : ΔG° = –nFεºpile = –(6 mol e–)(96 485 C/mol e–)(2,14 J/C) = –1,24 × 106 J = –1240 kJ

© 2007 Les Éditions CEC inc. Toute reproduction interdite

Chapitre 8 Électrochimie

292 Oxydation par le periodate :

ΔG° = –(10 mol e–)(96 485 C/mol e–)(0,09 J/C)(1 kJ/1000 J) = –90 kJ 37.

Pour ces piles aux conditions standard, Wmax = ∆G = ∆Gº = –nFεºpile. 33a) Wmax = –(2 mol e–)(96 485 C/mol e–)(0,31 J/C) = –6,0 × 104 J = –60 kJ 33b) Wmax = –(6 mol e–)(96 485 C/mol e–)(0,47 J/C) = –2,7 × 105 J = –270 kJ

38.

CH3OH(l) + 3/2O2(g) → CO2(g) + 2H2O(l)

ΔG° = 2(–237) + (–394) – [–166] = –702 kJ

Les demi-réactions équilibrées sont : H2O + CH3OH → CO2 + 6H+ + 6e– et O2 + 4H+ + 4e– → 2H2O Pour la réaction de 3/2 mol O2, il y a transfert de 6 mol e–. ΔG° = –nFεº, ε° =

39.

-ΔG° -(-702 000 J) = 1,21 J/C = 1,21 V = nF (6 mol e _ )(96 485 C/mol e _ )

Fe2+ + 2e– → Fe

ε° = –0,44 V = –0,44 J/C

ΔG° = –nFεº = –(2 mol e–)(96 485 C/mol e–)(–0,44 J/C)(1 kJ/1000 J) = 85 kJ 85 kJ = 0 – [ΔGf,o Fe2+ + 0] , ΔGf,o Fe2+ = –85 kJ On peut calculer ΔGf,o Fe3+ de deux façons, soit par la demi-réaction Fe3+ + e– → Fe2+ εº = 0,77 V ΔG° = –(1 mol e–)(96 485 C/mol e–)(0,77 J/C) = –74,300 J = –74 kJ Fe2+ → Fe3+ + e– Fe → Fe2+ + 2e– Fe → Fe3+ + 3e–

ΔG° = 74 kJ ΔG° = –85 kJ ΔG° = –11 kJ, ΔGf,o Fe3+ = –11 kJ/mol

soit par la demi-réaction : Fe3+ + 3e– → Fe

εº = –0,036 V

ΔG° = –(3 mol e–)(96 485 C/mol e–)(–0,036 J/C) = 10 400 J ≈ 10 kJ 10 kJ = 0 – [ΔGf,0 Fe3+ + 0] , ΔGf,o Fe3+ = –10 kJ/mol Les erreurs d’arrondissement expliquent la différence de 1 kJ.

© 2007 Les Éditions CEC inc. Toute reproduction interdite

Chapitre 8 Électrochimie

40.

293

Forme générale : oxydant + ne– → réducteur Plus la valeur de εº est élevée, plus l’oxydant est puissant. MnO4– > Cl2 > Cr2O72– > Fe3+ > Fe2+ > Mg2+ εº(V)

41.

1,68

1,36

1,33

0,77

–0,44

(ordre décroissant)

–2,37

Forme générale : oxydant + ne– → réducteur Plus la valeur de εº est faible, plus le réducteur est puissant.

–εº(V) 42.

Li

> Zn

–3,05

–0,76

a) 2H+ + 2e– → H2

> H2 0

> Fe2+ > Cr3+ > F–

(ordre décroissant)

+0,77 +1,33 +2,87

ε° = 0,00 V ; Cu → Cu2+ + 2e–

–ε° = –0,34 V

εºpile = –0,34 V ; non, H+ ne peut oxyder Cu en Cu2+ aux conditions standard (εºpile < 0). b) 2H+ + 2e– → H2

ε° = 0,00 V ; Mg → Mg2+ + 2e–

–ε° = 2,37 V

εºpile = 2,37 V ; oui, H+ peut oxyder Mg en Mg2+ aux conditions standard (εºpile > 0). c) Fe3+ + e– → Fe2+

ε° = 0,77 V ; 2I– → I2 + 2e–

–ε° = –0,54 V

εºpile = 0,77 – 0,54 = 0,23 V ; oui, Fe3+ peut oxyder I– en I2. d) Fe3+ + e– → Fe2+

ε° = 0,77 V ; 2Br– → Br2 + 2e–

–ε° = –1,09 V

εºpile = 0,77 – 1,09 = –0,32 V ; non, Fe3+ ne peut oxyder Br– en Br2. 43.

Br2 + 2e– → 2Br– 2H+ + 2e– → H2 Cd2+ + 2e– → Cd

ε° = 1,09 V ε° = 0,00 V ε° = –0,40 V

La3+ + 3e– → La Ca2+ + 2e– → Ca

ε° = –2,37 V ε° = –2,76 V

a) Br2 est le meilleur oxydant (plus la valeur de ε° est grande) ; b) Ca est le meilleur réducteur (plus la valeur de -ε° est grande); c) MnO4- + 8H+ + 5e- → Mn2+ + 4H2O; ε° = 1,51 V ; le permanganate peut oxyder Br-, H2, Cd, La et Ca aux conditions standard. Quand MnO4- est couplé avec des réactifs, la valeur de ε° est positive; d) Zn → Zn2+ + 2e- ; -ε° = 0,76 V ; le zinc peut réduire Br2, H+ et Cd+2 puisque ε°pile > 0. 44.

a) Pour oxyder Br–, il faut ε° > 1,09 V (2Br– + 2e– → Br2, ε° = 1,09 V). Pour ne pas oxyder Cl–, il faut ε° < 1,36 V (2Cl– + 2e– → Cl2, ε° = 1,36 V). Les réactifs suivants conviendraient : Cr2O72–, O2, MnO2 et IO3– ; couplés à Br–, ils donnent ε°pile > 0 ; couplés à Cl–, ils donnent ε°pile < 0.

© 2007 Les Éditions CEC inc. Toute reproduction interdite

Chapitre 8 Électrochimie

294

b) Mn → Mn2+ + 2e– ; –ε° = 1,18 V ; Ni → Ni2+ + 2e–, –ε° = 0,23 V. Tout agent oxydant avec –0,23 V > ε° > – 1,18 V conviendra ; par exemple, PbSO4, Cd2+, Fe2+, Cr3+, Zn2+ et H2O peuvent oxyder Mn, mais non Ni aux conditions standard. 45.

a) Pour réduire Fe3+, mais non Fe2+, le réducteur doit avoir un potentiel standard d’oxydation (ε°ox = -ε°) entre -0,77 V et 0,44 V. Les réducteurs sont à droite dans les équations du tableau 8.1. Ceux qui conviendraient pour cette tâche sont donc H2O2, MnO42-, I-, Cu, OH-, Hg + Cl-, Ag + Cl-, H2SO3, Cu+, H2, Fe, Pb, Sn, Ni, Pb + SO42- et Cd ; b) pour réduire Ag+ en Ag (ε° = 0,80 V), mais non O2 en H2O2 (ε° = 0,68 V) ; aux conditions standard, seul Fe2+ conviendrait selon le tableau 8.1.

46.

ClO− + H2O + 2 e− → 2 OH− + Cl− ε° = 0,90 V 2 NH3 + 2 OH− → N2H4 + 2 H2O + 2 e− -ε° = 0,10 V __________________________________________________________ o ClO−(aq) + 2 NH3(aq) → Cl−(aq) + N2H4(aq) + H2O(l) = 1,00 V ε pile o Étant donné que la valeur de ε pile est positive pour cette réaction, dans des conditions

standard, ClO− peut oxyder spontanément NH3 en N2H4 plutôt toxique. 47.

Tl3+ + 2 e− → Tl+ ε° = 1,25 V − − − 3 I → I3 + 2 e -ε° = -0,55 V ______________________________________ o Tl3+ + 3 I− → Tl+ + I3− ε pile = 0,70 V En solution, Tl3+ peut oxyder I- en I3-. Par conséquent, on s’attend à ce que ce soit le triiodure de thallium(I).

Équation de Nernst 48.

H2O2 + 2H+ + 2e– → 2H2O (Ag → Ag+ + e–) × 2 H2O2(aq) + 2H+(aq) + 2Ag(s) → 2H2O(l) + 2Ag+(aq)

ε° = 1,78 V –ε° = –0,80 V ε°pile = 0,98 V

a) Toute variation dans les concentrations augmentant la tendance vers la droite de cette réaction en augmentera aussi la valeur de εpile. Par ailleurs, tout changement favorisant la réaction inverse diminuera la valeur de εpile par rapport à la valeur de ε°pile. Ici, l’augmentation de 1,0 mol/L à 2,0 mol/L des concentrations de H2O2 et H+ favorise la réaction directe selon le principe de Le Chatelier ; alors, la valeur de εpile sera plus élevée que ε°pile. b) Ici, l’augmentation de 1,0 mol/L à 2,0 mol/L du produit Ag+ et la diminution de 1,0 mol/L à 1,0 × 10–7 mol/L du réactif H+ favorisent tous deux la réaction inverse ; donc, εpile sera plus faible que ε°pile.

© 2007 Les Éditions CEC inc. Toute reproduction interdite

Chapitre 8 Électrochimie

295

49.

Les concentrations de Fe2+ sont maintenant de 0,01 mol/L et de 1 × 10-7 mol/L dans les deux compartiments. La tendance pour cette pile sera d’égaliser les [Fe2+] dans les compartiments. Cela se fera si le compartiment avec Fe2+ 1 × 10-7 mol/L devient l’anode (Fe → Fe2+ + 2e-) et l’autre, la cathode (Fe2+ + 2e- → Fe). Le flux d’électrons partira donc du compartiment de droite (Fe2+ 1 × 10-7 mol/L) vers le compartiment de gauche.

50.

Pour les piles à concentrations, la force motrice de la réaction est la différence de concentration des ions entre les compartiments cathodique et anodique. Pour équilibrer ces concentrations, l’anode est toujours le compartiment où la concentration est la plus faible. Cathode : Anode :

Ag+(x mol/L) + e– → Ag Ag → Ag+ (y mol/L) + e–

ε° = 0,80 V –ε° = –0,80 V

Ag+(cathode, x mol/L) → Ag+ (anode, y mol/L) ε°pile = 0,00 V

[Ag+ ]anode 0,0592 - 0,0592 εpile = ε°pile – log Q = log n 1 [Ag+ ]cathode a) Puisque les concentrations dans les deux compartiments correspondent aux conditions standard (1,0 mol/L), alors εpile = ε°pile = 0 V. Il n’y a pas de réaction. Pour qu’une pile à concentration produise un voltage, les concentrations doivent être différentes. b) [Ag+]cathode = 2,0 mol/L, [Ag+]anode = 1,0 mol/L ; le flux d’électrons se faisant toujours de l’anode vers la cathode, il se dirigera donc vers la droite dans le diagramme.

[Ag+ ]anode -0,0592 - 0,0592 1,0 εpile = log = log = 0,018 V + 2,0 n 1 [Ag ]cathode c) Cathode = Ag+ 1,0 mol/L ; anode = Ag+ 0,10 mol/L ; les électrons se déplacent vers la gauche dans le diagramme. + [Ag ]anode -0,0592 0,10 - 0,0592 εpile = log = log = 0,059 V + n 1,0 1 [Ag ]cathode

d) Cathode = Ag+ 1,0 mol/L ; anode = Ag+ 4,0 × 10–5 mol/L ; les électrons se déplacent vers la gauche. εpile =

-0,0592 4,0 × 10-5 log = 0,26 V 1 1,0

e) Puisque les concentrations des ions sont identiques, alors log [Ag+]anode/[Ag+]cathode = log(1,0) = 0 et εpile = 0. Il n’y a pas de déplacement d’électrons.

© 2007 Les Éditions CEC inc. Toute reproduction interdite

Chapitre 8 Électrochimie

296 51.

n = 2 pour cette réaction (Pb → Pb2 dans PbSO4). ε = ε° –

1 0,0592 0,0592 1 log = 2,04 V – log 2 + 2 - 2 2 2 [H ] [HSO 4 ] (4,5) (4,5) 2

ε = 2,04 V + 0,077 V = 2,12 V 52.

Cu2+ + 2e– → Cu Zn → Zn2+ + 2e–

ε° = 0,34 V –ε° = 0,76 V

Cu2+(aq) + Zn(s) → Zn2+(aq) + Cu(s)

ε°pile = 1,10 V

[ Zn 2+] 0, 0592 0,0592 log Q = 1,10 V – log n 2 [Cu 2+] 0,0592 1,20 log = 1,10 V – 0,0052 V = 1,09 V εpile = 1,10 V – 0,80 2 εpile = ε°pile –

53.

Cu2+(aq) + H2(g) → 2H+(aq) + Cu(s)

ε°pile = 0,34 V – 0,00V = 0,34 V et n = 2

Comme PH2 = 1,0 atm et [H+] = 1,0 mol/L : εpile = ε°pile –

a) εpile = 0,34 V –

0,0592 1 log 2 [Cu 2+]

0,0592 1 log = 0,34 V – 0,11 = 0,23 V 2 2,5 ×10-4

b) 0,195 V = 0,34 V –

0,0592 1 1 log , log = 4,90, [Cu2+] = 10–4,90 2+ 2 [Cu ] [Cu 2+] = 1,3 × 10–5 mol/L

54.

Cu2+(aq) + H2(g) → 2H+(aq) + Cu(s)

ε°pile = 0,34 V – 0,00 V = 0,34 V et n = 2

Utilisons l’expression de Kps pour calculer [Cu2+]. Cu(OH)2(s) État initial Réaction À l’équilibre



0,1 –s s

Cu2+

+

+s 0,1

2OH–(aq) mol/L +2s mol/L +2s mol/L

Kps = 1,6 × 10–19 = [Cu2+][OH–]2 = (s)(0,2 + 2s)2 → (s)(0,1)2 s = 1,6 × 10–17; approximation valide.

© 2007 Les Éditions CEC inc. Toute reproduction interdite

Chapitre 8 Électrochimie

297

[Cu2+] = s = 1,6 × 10–17 mol/L

0,0592 1 log = 0,34 – 0,50 = –0,16 V 2 1,6 × 10-17

ε = 0,34 –

La réaction dans la pile est inversée : Cu + 2H+ → H2 + Cu2+. 55.

o = -0,23 V + 1,66 V = 1,43 V; n = 6 3 Ni2+(aq) + 2 Al(s) → 2 Al3+(aq) + 3 Ni(s) ε pile

o ε pile = ε pile -

0,0591 [Al3+ ]2 0, 0591 [Al3+ ]2 log log , 1,82 V = 1,43 V n [Ni 2+ ]3 6 (1, 0)3

log [Al3+]2 = -39,59, [Al3+]2 = 10 −39.59 , [Al3+] = 1,6 × 10 −20 mol/L Al(OH)3(s)



Al3+(aq) + 3 OH−(aq) Kps = [Al3+][OH−]3;

D’après les données du problème, [OH−] = 1,0 × 10 −4 mol/L. Kps = (1,6 × 10 −20 ) (1,0 × 10 −4 )3 = 1,6 × 10 −32 56.

Cathode: Anode:

M2+ + 2e− → M(s) M(s) → M2+ + 2e−

M2+ (cathode) → M2+ (anode)

εpile = 0,44 V = 0,00 V -

log [M2+]anode = -

ε° = -0,31 V - ε° = 0,31 V o ε pile = 0,00 V

[M 2+ ]anode [M 2+ ]anode 0,0591 0,0591 log log , 0,44 = 2 1,0 2 [M 2+ ]cathode

2(0, 44) = -14,89, [M2+]anode = 1,3 × 10 −15 mol/L 0, 0591

Étant donné qu’il y avait initialement des nombres égaux de moles de SO42− et de M2+, [M2+] = [SO42−] à l’équilibre. Kps = [M2+][SO42-] = (1,3 × 10-15)2 = 1,7 × 10-30 57.

a) Ag+ (x mol/L, anode) → Ag+ (0,10 mol/L, cathode); pour l’électrode d’argent, ε° = 0,00 (comme c’est toujours le cas pour les piles de concentration) et n = 1. ε = 0,76 V = 0,00 -

[Ag + ]anode 0,0591 log 1 [Ag + ]cathode

© 2007 Les Éditions CEC inc. Toute reproduction interdite

Chapitre 8 Électrochimie

298

[Ag + ]anode [Ag + ]anode , 0,76 = -0,0591 log = 10−12,86 , [Ag+]anode = 1,4 × 10 −14 mol/L 0,10 0,10 b) Ag+(aq) + 2 S2O32−(aq) K=

58.

⇌ Ag(S2O3)23−(aq)

[Ag(S2 O3 )3-2 ] 1, 0 × 10−3 = = 2,9 × 1013 + 2- 2 −14 2 1, 4 × 10 (0, 050) [Ag ][S2O3 ]

Voir les exercices 25 et 29 pour les réactions équilibrées et les potentiels standard d’électrodes. Les réaction équilibrées sont nécessaires pour déterminer n, le nombre de moles transférées. 25a) 7 H2O + 2 Cr3+ + 3 Cl2 → Cr2O72− + 6 Cl− + 14 H+

o = 0,03 V = 0,03 J/C ε pile

o ΔG° = − nF ε pile = -(6 mol e−)(96,485 C/mol e−)(0,03 J/C) = -1,7 × 104 J = -20 kJ

o ε pile = ε pile −

o ε pile =

0, 0591 log Q : à l’équilibre, ε°pile = 0 et Q = K, donc : n

0, 0591 nε o 6(0, 03) log K, log K = = = 3,05, K = 103,05 = 1 × 103 0, 0591 0, 0591 n

Note : quand on détermine les exposants, on arrondit au nombre de chiffres significatifs approprié une fois les calculs terminés afin d’éliminer les erreurs d’arrondissement. 25b) ΔG° = -(2 mol e−)(96 485 C/mol e−)(2,71 J/C) = -5,23 × 105 J = -523 kJ log K =

2(2, 71) = 91,709, K = 5,12 × 1091 0, 0591

29a) ΔG° = -(2 mol e−)(96 485 C/mol −)(0,27 J/C) = -5,21 × 104 J = -52 kJ log K =

2(0, 27) = 9,14, K = 1,4 × 109 0, 0591

29b) ΔG° = - (10 mol e−)(96 485 C/mol e−)(0,09 J/C) = -8,7 × 104 J = -90 kJ log K =

10(0, 09) = 15,23, K = 2 × 1015 0, 0591

© 2007 Les Éditions CEC inc. Toute reproduction interdite

Chapitre 8 Électrochimie 59.

299

Cu2+ + 2 e− → Cu ε° = 0,34 V 2+ − Fe → Fe + 2 e -ε° = 0,44 V _______________________________________ o Fe + Cu2+ → Cu + Fe2+ ε pile = 0,78 V

[Fe 2+ ] , donc trouvons K pour déterminer le rapport ionique à Pour cette réaction, K = [Cu 2+ ] l’équilibre.

0, 0591 2(0, 78) [Fe 2+ ] ε° = log K, log K = = 26,40, K = = 1026,40 = 2,5 × 1026 2+ [Cu ] 0, 0591 n 60.

Ni2+ + 2 e− → Ni ε° = -0,23 V 2+ − Sn → Sn + 2 e -ε° = 0,14 V _______________________________________ o ε pile = -0,09 V Ni+2+ + Sn → Sn2+ + Ni On calcule le rapport ionique quand cette réaction est à l’équilibre (ε°pile = 0). ε° =

0, 0591 0, 0591 [Sn 2 + ] log K, -0,09 = log [ Ni 2+ ] 2 n

[Sn 2 + ] = 102( −0,09) / 0,0591 = 9 × 10 −4 2+ [ Ni ] Cette réaction est à l’équilibre quand le rapport [Sn2+]/[Ni2+] est égal à 9 × 10 −4 . Une réaction se déplace vers la droite (c’est ce qu’on veut afin de produire un voltage) quand Q < Kps. Le rapport minimal nécessaire pour que cette réaction soit spontanée est un rapport [Sn2+]/[Ni2+] tout juste inférieur à 9 × 10 −4 . εpile est positif seulement lorsque ce rapport est inférieur à 9 × 10 −4 . 61.

a) Réaction possible : I2(s) + 2 Cl−(aq) → 2 I−(aq) + Cl2(g) o ε pile = 0,54 V - 1,36 V = -0,82 V o < 0; aucune Cette réaction n’est pas spontanée aux conditions standard parce que ε pile

réaction n’a lieu. b) Réaction possible : Cl2(g) + 2 I−(aq) → I2(s) + 2 Cl−(aq)

o = 0,82 V; cette réaction ε pile

o est spontanée aux conditions standard parce que ε pile > 0. Cette réaction a lieu.

Cl2(g) + 2 I−(aq) → I2(s) + 2 Cl−(aq)

o ε pile = 0,82 V = 0,82 J/C

o = -(2 mol e-)(96 485 C/mol e-)(0,82 J/C) = -1,6 × 105 J = -160 kJ ΔG° = − nF ε pile

© 2007 Les Éditions CEC inc. Toute reproduction interdite

Chapitre 8 Électrochimie

300

ε° =

0, 0591 nε o 2(0,82) log K, log K = = = 27,75, K = 1027,75 = 5,6 × 1027 0, 0591 0, 0591 n

o c) Réaction possible: 2 Ag(s) + Cu2+(aq) → Cu(s) + 2 Ag+(aq) ε pile = -0,46 V;

aucune réaction n’a lieu. d) Fe2+ peut être oxydé ou réduit. Les autres espèces présentes sont H+, SO42−, H2O et O2 de l’air. Seul O2 en présence de H+ possède un potentiel de réduction standard assez élevé o > 0). Toutes les autres combinaisons, pour oxyder Fe2+ en Fe3+ (ce qui donne ε pile

incluant la réduction possible de Fe2+, donnent des potentiels de pile négatifs. La réaction spontanée est : o = 1,23 - 0,77 = 0,46 V Fe2+(aq) + 4 H+(aq) + O2(g) → 4 Fe3+(aq) + 2 H2O(l) ε pile o = -(4 mol e−)(96,485 C/mol e−)(0,46 J/C)(1 kJ/1000 J) = -180 kJ ΔG° = − nF ε pile

log K =

62.

4(0, 46) = 31,13, K = 1,3 × 1031 0, 0591

a) Cu+ + e– → Cu Cu+ → Cu2+ + e–

2Cu+(aq) → Cu2+(aq) + Cu(s)

ε° = 0,52 V –ε° = –0,16 V ε°pile = 0,36 V; spontanée

ΔG° = nFε°pile = –(1 mol e–)(96 485 C/mol e)(0,36 J/C) = –34,700 J = –35 kJ ε°pile =

0,0592 nε o 1(0,36) = log K, log K = = 6,08, K = 106,08 = 1,2 × 106 0,0592 0,0592 n

b) Fe2+ + 2e– → Fe (Fe2+ → Fe3+ + e–) × 2

3Fe2+(aq) → 2Fe3+(aq) + Fe(s)

ε° = –0,44 V –ε° = –0,77 V ε°pile = –1,21 V ; non spontanée

c) HClO2 + 2H+ + 2e– → HClO + H2O HClO2 + H2O → ClO3– + 3H+ + 2e–

2HClO2(aq) → ClO3–(aq) + H+(aq) + HClO(aq)

ε° = 1,65 V –ε° = –1,21 V ε°pile = 0,44 V ; spontanée

ΔG° = –nFε°pile = –(2 mol e–)(96 485 C/mol e–)(0,44 J/C) = –84 900 J = –85 kJ

© 2007 Les Éditions CEC inc. Toute reproduction interdite

Chapitre 8 Électrochimie

log K =

63.

301

nε o 2(0,44) = = 14,86, K = 7,2 × 1014 0,0592 0,0592 Au3+ + 3e– → Au (Tl → Tl+ + e–) × 3

a)

Au3+(aq) + 3Tl(s) → Au(s) + 3Tl+(aq)

ε° = 1,50 V –ε° = 0,34 V ε°pile = 1,84 V

b) ΔG° = –nFε°pile = –(3 mol e–)(96 485 C/mol e–)(1,84 J/C) = –5,33 × 105 J = –533 kJ

nε o 3(1,84) = = 93,243, K = 1093,243 = 1,75 × 1093 log K = 0,0592 0,0592 (1,0 × 10-4 ) 3 0,0592 0,0592 [Tl+ ]3 c) εpile = 1,84 V – log = 1,84 – log 1,0 × 10-2 [Au3+] 3 3 εpile = 1,84 – (–0,20) = 2,04 V 64.

2+

Co

(Cr2+ → Cr3+ + e−) × 2 + 2 e− → Co

2 Cr2+ + Co2+ → 2 Cr3+ + Co o ε pile =

ε = ε° -

0, 0591 0, 0591 log K = log (2,79 × 107) = 0,220 V 2 n 0,0591 [Cr 3+ ]2 0, 0591 (2, 0) 2 log log = 0,220 V = 0,151 V n [Cr 2+ ]2 [Co 2+ ] 2 (0,30) 2 (0, 20)

ΔG = -nFε = -(2 mol e−)(96 485 C/mol e−)(0,151 J/C) = -2,91 × 104 J = -29,1 kJ 65.

La réaction de Kps est : FeS(s) ⇌ Fe2+(aq) + S2−(aq) K = Kps. On additionne les équations o données de façon à obtenir la réaction de Kps. On peut alors utiliser la valeur de ε pile pour la réaction afin de déterminer Kps. ε° = -1,01 V FeS + 2 e− → Fe + S2− -ε° = 0,44 V Fe → Fe2+ + 2 e− ____________________________________________ o ε pile = -0,57 V Fe(s) → Fe2+(aq) + S2−(aq)

log Kps =

nε o 2(−0,57) = = -19,29, Kps = 10−19,29 = 5,1 × 10 −20 0, 0591 0, 0591

© 2007 Les Éditions CEC inc. Toute reproduction interdite

Chapitre 8 Électrochimie

302 Al3+ + 3 e− → Al Al + 6 F− → AlF63- + 3 e−

66.

Al3+(aq) + 6 F−(aq) → AlF63−(aq)

log K =

67.

ε° = -1,66 V -ε° = 2,07 V o ε pile = 0,41 V

K=?

nε o 3(0, 41) = = 20,81, K = 1020,81 = 6,5 × 1020 0, 0591 0, 0591

NO3− est un ion spectateur. C’est Ag+ qui réagit avec Zn. ε° = 0,80 V (Ag+ + e− → Ag) × 2 -ε° = 0,76 V Zn → Zn2+ + 2 e− _________________________________________ o 2 Ag+ + Zn → 2 Ag + Zn2+ ε pile = 1,56 V

log K =

68.

nε 2(1,56) = , K = 1052,792 = 6,19 × 1052 0, 0591 0, 0591

CuI + e– → Cu + I– Cu → Cu+ + e– CuI(s) → Cu+(aq) + I–(aq)

ε°CuI = ? –ε° = –0,52 V ε°pile = ε°CuI – 0,52 V

Pour cette réaction globale, K = Kps = 1,1 × 10–12 : ε°pile =

0,0592 0,0592 log Kps = log (1,1 × 10–12) = –0,71 V n 1

ε°pile = –0,71 V = ε°CuI – 0,52, ε°CuI = –0,19 V

Électrolyse 69.

a) Al3+ + 3e– → Al (Al : 27 g/mol) t=

3 mol e _ 96 485 C 1000 g Al 1 1A ⋅ s × × 1,0 kg Al × × × _ 27 g Al mol e kg Al 100,0 A 1C

t = 1,1 × 105 s b) Ni2+ + 2e– → Ni (Ni : 58,7 g/mol) t=

2 mol e _ 96 485 C 1 1A ⋅ s × × 1,0 g Ni × × = 33 s _ 58,7 g Ni mol e 100,0 A C

© 2007 Les Éditions CEC inc. Toute reproduction interdite

Chapitre 8 Électrochimie

303

c) Ag+ + 1e– → Ag t= 70.

1 mol e _ 96 485 C 1 1A ⋅ s × × 5,0 mol Ag × × = 4,8 × 103 s _ mol Ag mol e 100,0 A C

L’état d’oxydation du bismuth dans BiO+ est +3 parce que l’oxygène a un état d’oxydation de -2 dans cet ion. Par conséquent, 3 moles d’électrons sont nécessaires pour réduire le bismuth dans BiO+ en Bi(s). 10,0 g Bi ×

71.

Q=

1 mol Bi 3 mol e96 485 C 1s × × × = 554 s = 9,23 min 209,0 g Bi mol Bi mol e 25,0 C

15 A 3600 s C ×1h × × = 5,4 × 104 C h h 1 A⋅ s

a) Co2+ + 2e– → Co

masse Co =

1 mol Co 58,93 g Co mol e _ × × = 5,4 × 104 C = 16 g _ 2 mol e mol Co 96 485 C

b) Hb4+ + 4e– → Hf

masse HF =

1 mol Hf 178,5 g Hf mol e _ × × = 5,4 × 104 C = 25 g 4 mol e _ mol Hf 96 485 C

c) 2I– → I2 + 2e– (à l’anode)

masse I2 =

1 mol I 2 254 g I 2 mol e _ × × = 5,4 × 104 C = 71 g 2 mol e _ mol I2 96 485 C

d) Cr6+ + 6e– → Cr

1 mol Cr 52 g Cr mol e _ × × = 5,4 × 104 C = 4,8 g masse Cr = _ 6 mol e mol Cr 96 485 C 72.

Al est dans l’état d’oxydation +3 dans Al2O3, de sorte que 3 mol e− sont nécessaires pour convertir Al3+ en Al(s). 2,00 h ×

60 min 60 s 1,00 ×106 C 1 mol e1 mol Al 26,98 g Al × × × × × h min s 96 485 C 3 mol emol Al = 6,71 × 105 g

© 2007 Les Éditions CEC inc. Toute reproduction interdite

Chapitre 8 Électrochimie

304

73.

74,1 s ×

2,00 C 1 mol e1 mol M × × = 5,12 × 10 −4 mol M où M = métal inconnu s 96 485 C 3 mol e-

Masse molaire =

74.

0,107 g M 209 g = ; l’élément est le bismuth. -4 5,12 ×10 mol M mol

F2 est produit à l’anode: 2 F- → F2 + 2 e2,00 h ×

60 min 60 s 10,0 C 1 mol e× × × = 0,746 mol e− h min s 96 485 C

0,746 mol e− ×

1 mol F2 nRT = 0,373 mol F2; PV = nRT, V = P 2 mol e

(0,373 mol) (0,08206 L ⋅ atm/K ⋅ mol) (298 K) = 9,12 L F2 1,00 atm K est produit à la cathode : K+ + e− → K

0,746 mol e− ×

75.

1 mol K 39,10 g K × = 29,2 g K mol emol K

150 × 103 g C 6 H 8 N 2 1h 1 min 1 mol C 6 H 8 N 2 × × × h 60 min 60 s 108,14 g C 6 H 8 N 2 ×

76.

2,30 min ×

2 mol e _ 96 485 C × = 7,44 × 104 C/s = 7,44 × 104 A _ mol C 6 H 8 N 2 mol e

60 s 2,00 C 1 mol e _ 1 mol Ag × × = 138 s ; 138 s × s 96 485 C mol e _ min = 2,86 × 10–3 mol Ag

[Ag+] = 2,86 × 10–3 mol Ag+/0,250 L = 1,14 × 10–2 mol/L

77.

0,50 L × 0,010 mol Pt4+/L = 5,0 × 10–3 mol Pt4+ Pour plaquer 99 % du Pt4+ , il faut produire 0,99 × 5,0 × 10–3 mol Pt, 0,99 × 5,0 × 10–3 mol Pt ×

4 mol e _ 96485 C 1s × × = 480 s _ mol Pt mol e 4,00 C

© 2007 Les Éditions CEC inc. Toute reproduction interdite

Chapitre 8 Électrochimie

78.

Au3+ + 3 e− → Au Ag+ + e− → Ag 2 H2O + 2e− → H2 + 2 OH−

305 Ni2+ + 2 e− → Ni Cd2+ + 2 e− → Cd

ε° = 1,50 V ε° = 0,80 V

ε° = -0,23 V ε° = -0,40 V

ε° = -0,83 V

Au(s) se déposera le premier étant donné qu’il a le potentiel de réduction le plus positif, suivi de Ag(s), suivi de Ni(s) et enfin Cd(s) se déposera en dernier étant donné qu’il a le potentiel de réduction le plus négatif des métaux énumérés. L’eau ne nuira pas au processus de placage. 79.

Pour que Pd commence à se déposer : ε = 0,62 −

0,0591 [Cl- ]4 0, 0591 (1, 0) 4 log log = 0,62 − 2 0, 020 2 [PdCl2-4 ]

ε = 0,62 V − 0,050 V = 0,57 V

Lorsque 99 % du Pd s’est déposé, [PdCl4−] =

0, 020 = 0,00020 mol/L. 100

0, 0591 (1, 0) 4 log ε = 0,62 − = 0,62 V − 0,11V = 0,51 V 2 2, 0 × 10−4 Pour que Pt commence à se déposer : ε = 0,73 V −

0, 0591 (1, 0) 4 log = 0,73 − 0,050 2 0, 020 = 0,68 V

Lorsque 99 % de Pt s’est déposé : ε = 0,73 −

0, 0591 (1, 0) 4 log = 0,73 − 0,11 2 2, 0 × 10−4 = 0,62 V

Pour que Ir commence à se déposer : ε = 0,77 V −

0, 0591 (1, 0) 4 log = 0,77 − 0,033 3 0, 020 = 0,74 V

Lorsque 99 % de Ir s’est déposé : ε = 0,77 −

0, 0591 (1, 0) 4 log = 0,77 − 0,073 3 2, 0 × 10−4 = 0,70 V

© 2007 Les Éditions CEC inc. Toute reproduction interdite

Chapitre 8 Électrochimie

306

Oui, puisque la gamme de potentiels pour précipiter chaque métal ne se superpose pas, il serait possible de séparer les trois métaux. Le potentiel exact qu’il faut appliquer dépend de la réaction d’oxydation. L’ordre dans lequel les métaux précipitent est : Ir(s) en premier, suivi de Pt(s) et enfin Pd(s), à mesure que le potentiel est graduellement augmenté. 80.

La réduction a lieu à la cathode, l’oxydation à l’anode. Il faut d’abord déterminer la nature des espèces présentes, puis consulter le tableau 8.1 pour les valeurs de potentiels de réduction/oxydation. La réaction à la cathode sera celle ayant le potentiel de réduction le plus positif et la réaction à l’anode sera celle ayant le potentiel d’oxydation le plus positif. a) Expèces présentes : K+ et F– ; selon le tableau 8.1, K+ peut être réduit en K, et F– peut être oxydé en F2. Les réactions sont :

Cathode : K+ + e– → K Anode : 2F– → F2 + 2e–

ε° = –292 V –ε° = –2,87 V

b) Espèces présentes : Cu2+ et Cl– ; Cu2+ peut être réduit et Cl– peut être oxydé. Les réactions sont :

Cathode : Cu2+ + 2e– → Cu Anode : 2Cl– → Cl2 + 2e–

ε° = 0,34 V –ε° = –1,36 V

c) Espèces présentes : Mg2+ et I– ; Mg2+ peut être réduit et I– peut être oxydé. Les réactions sont :

Cathode : Mg2+ + 2e– → Mg Anode : 2I– → I2 + 2e–

ε° = –2,37 V –ε° = –0,54 V

EXERCICES SUPPLÉMENTAIRES 81.

Hg2Cl2 + 2e– → 2Hg + 2Cl–sat, → = 0,242 V. Supposons que l’autre demi-pile est aux conditions standard. Si ε° de l’autre demi-pile est plus grande que 0,242 V, alors elle agira comme cathode. Dans le cas inverse, c’est l’électrode au calomel qui agira comme cathode. a) Cu2+ + 2e– → Cu ε° = 0,34 V : cathode, calomel : anode εpile = 0,34 + (–0,242) = 0,10 V b) Fe3+ + 1e– → Fe ε° = 0,77 V : cathode, calomel : anode εpile = 0,77 + (–0,242) = 0,53 V c) AgCl + 1e– → Ag + Cl– ε° = 0,22 V < 0,242 V Ag + Cl– → AgCl + 1 e– –ε° = –0,22 V : anode, calomel : cathode εpile = 0,242 + (–0,22) = 0,02 V

© 2007 Les Éditions CEC inc. Toute reproduction interdite

Chapitre 8 Électrochimie

307

d) Al3+ + 3e– → Al ε = –1,66 V < 0,242 V Al → Al3+ + 3e– –ε° = 1,66 V : anode, calomel : cathode εpile = 1,66 + 0,242 = 1,90 V e) Ni2+ + 2e– → Ni ε° = –0,23 V < 0,242 V Ni → Ni2+ + 2e– –ε° = +0,23 V : anode, calomel : cathode εpile = 0,23 + 0,242 = 0,47 V 82.

H+ n’est pas un oxydant assez puissant pour oxyder Pt. Pt → Pt2+ + 2e– –ε° = –1,188 V 2H+ + 2e– → H2 ε° = 0 Pt + 2H+ → Pt2+ + H2 ε° = 0 + (–1,188) = –1,188 V < 0, réaction non spontanée Même en présence de Cl–, H+ n’est pas assez puissant. Pt + 4Cl– → PtCl42– + 2e– –ε° = –0,755 V 2H+ + 2e– → H2 ε° = 0 Pt + 4Cl– + 2H+ → H2 + PtCl42– ε° = –0,755 V, réaction non spontanée NO3– n’est pas un oxydant assez puissant pour oxyder Pt. 3 [Pt → Pt2+ + 2e–] –ε° = –1,188 V 2 [NO3– + 4H+ + 3e– → NO + 2H2O] ε° = 0,96 V 3Pt + 2NO3– + 8H+ → 3Pt2+ + 2NO + 4H2O ε° = 0,96 + (–1,188) = –0,228 réaction non spontanée Toutefois, en présence de Cl–, NO3– peut oxyder Pt. 3 [Pt + 4Cl– → PtCl42– + 2e–] –ε° = 0,755 V 2 [NO3– + 4H+ + 3e– → NO + 2H2O] ε° = +0,96 V 3Pt + 2NO3– + 12Cl– + 8H+ → 3PtCl42– + 2NO + 4H2O ε° = 0,96 + (–0,755) ε° = +0,20V Il s’agit d’une réaction spontanée.

83.

2 Ag+(aq) + Cu(s) → Cu2+(aq) + 2 Ag(s)

o ε pile = 0,80 - 0,34 V = 0,46 V; une pile

électrochimique produit un voltage pendant que la réaction directe a lieu. Toute modification qui augmente la tendance de la réaction directe à se produire augmentera le potentiel de la pile, tandis qu’une modification qui diminue la tendance de la réaction directe à avoir lieu diminuera le potentiel de la pile. a) L’ajout de Cu2+ (un ion produit) diminue la tendance de la réaction directe à avoir lieu, ce qui diminue le potentiel de la pile. b) L’ajout de NH3 enlève Cu2+ sous la forme de Cu(NH3)42+. L’élimination d’un produit augmente la tendance de la réaction directe à se produire, ce qui augmente le potentiel de la pile.

© 2007 Les Éditions CEC inc. Toute reproduction interdite

Chapitre 8 Électrochimie

308

c) L’ajout de Cl− enlève Ag+ sous la forme deAgCl(s). L’élimination d’un ion réactif diminue la tendance de la réaction directe à se produire, ce qui diminue le potentiel de la pile.

[Cu 2+ ]o d) Q1 = ; quand le volume de la solution est doublé, chaque concentration diminue [Ag + ]o2 de moitié. Q2 =

1/2[Cu 2+ ]o [Cu 2+ ]o = = 2 Ql (1/2[Ag + ]o ) 2 [Ag + ]o2

Le quotient réactionnel double lorsque les concentrations diminuent de moitié. Étant donné que les réactions sont spontanées quand Q < K et parce que Q augmente lorsque le volume de la solution double, la réaction est plus près de l’équilibre, ce qui diminue le potentiel de la réaction. e) Étant donné que Ag(s) n’est pas un réactif dans cette réaction spontanée et parce que les solides n’apparaissent pas dans les expressions des quotients réactionnels, remplacer l’électrode d’argent par une électrode de platine n’aura aucun effet sur le potentiel de la pile. 84.

(Al3+ + 3 e− → Al) × 2 ε° = -1,66 V 2+ − (M → M + 2 e ) × 3 -ε° = ? _____________________________________________________________ o 3 M(s) + 2 Al3+(aq) → 2 Al(s) + 3 M2+(aq) = -ε° - 1,66 V ε pile o o o , -411 × 103 J = -(6 mol e−)(96 485 C/mol e−) ε pile , ε pile = 0,71 V ΔG° = − nF ε pile o ε pile = -ε° - 1,66 V = 0,71 V, -ε° = 2,37 ou ε° = -2,37

D’après le tableau 8.1, le potentiel de réduction pour Mg2+ + 2 e- → Mg est -2,37 V, ce qui correspond aux données. Par conséquent, le métal est le magnésium. 85.

a) ΔG° = ∑ np ΔGf,o produits − ∑ nr ΔGf,o réactifs = 2(-480) + 3(86) - [3(-40)] = -582 kJ

D’après les nombres d’oxydation, n = 6. ΔG° = -nFε°, ε° =

-ΔG o -(-582 000 J) = 6(96 485) C nF = 1,01 V

log K =

nε o 6(1, 01) = = 102,538, K = 10102,538 = 3,45 × 10102 0, 0591 0, 0591

© 2007 Les Éditions CEC inc. Toute reproduction interdite

Chapitre 8 Électrochimie

309

2 e− + Ag2S → 2 Ag + S2−) × 3

b)

o ε Ag S =? 2

(Al → Al3+ + 3 e−) × 2 -ε° = 1,66 V ______________________________________________________________________ o o 3 Ag2S(s) + 2 Al(s) → 6 Ag(s) + 3 S2−(aq) + 2 Al3+(aq) ε pile = 1,01 V = ε Ag + 1,66V 2S o ε Ag S = 1,01 V - 1,66 V = -0,65 V 2

86.

Zn → Zn2+ + 2e– –ε° = 0,76 V ; Fe → Fe2+ + 2e– –ε° = 0,44 V Le zinc est plus facile à oxyder que le fer. En sa présence, Fe ne peut être oxydé, ce qui protégera la coque du Monitor.

87.

L’aluminium exposé à O2 forme un oxyde (Al2O3) qui adhère très bien à la surface du métal et empêche l’oxydation de se poursuivre pour les atomes Al qui ne sont pas en surface. Quand on plonge l’aluminium dans HCl, HCl attaque d’abord cet oxyde : Al2O3 + 6HCl → AlCl3 + 3H2O ; puis les autres atomes Al sont alors mis en contact avec H+ et facilement oxydés : 2Al + 6H+ → 2Al3+ + 3H2(g). Bouillonnement = dégagement de H2(g).

88.

Seul l’énoncé e est vrai. Les métaux reliés qui sont plus facilement oxydés que le fer sont appelés métaux sacrificiels. Pour l’énoncé a, la corrosion est un processus spontané, comme les processus employés pour fabriquer des piles électrolytiques. Pour l’énoncé b, la corrosion de l’acier est l’oxydation du fer couplée à la réduction de l’oxygène. Pour l’énoncé c, les autos rouillent plus facilement dans les régions humides parce que l’eau est un réactif dans la demi-réaction de réduction de même qu’un milieu pour la migration des ions (un pont électrolytique, en quelque sorte). Pour l’énoncé d, l’application de sel sur les routes ajoute des ions au processus de corrosion, ce qui augmente la conductivité de la solution aqueuse et, conséquemment, accélère la corrosion.

89.

Soit l’agent oxydant le plus fort combiné à l’agent réducteur le plus fort du tableau 8.1 : F2 + 2 e− → 2 F− ε° = 2,87 V -ε° = 3,05 V (Li → Li+ + e−) × 2 _______________________________________________ o F2(g) + 2 Li(s) → 2 Li+(aq) + 2 F− (aq) ε pile = 5,92 V La demande est impossible. L’agent oxydant le plus fort et l’agent réducteur le plus fort lorsqu’ils sont combinés donnent une valeur de ε°pile d’environ 6 V.

90.

2 H2(g) + O2(g) →2 H2O(l); l’oxygène passe de l’état d’oxydation zéro à -2 dans H2O. Puisque qu’il y a deux moles de O dans la réation équilibrée, n = 4 moles d’électrons sont transférées. o a) ε pile =

0, 0591 0, 0591 o log K = log (1,28 × 1083), ε pile = 1,23 V 4 n

o = - (4 mol e−)(96 485 C/mol e−)(1,23 J/C) = -4,75 × 105 J = -475 kJ ΔG° = − nF ε pile

© 2007 Les Éditions CEC inc. Toute reproduction interdite

Chapitre 8 Électrochimie

310

b) Puisque les moles de gaz diminuent à mesure que les réactifs sont convertis en produits, ΔS° sera négative (non favorable). Puisque la valeur de ΔG° est négative, ΔH° doit être négative pour surmonter la ΔS° non favorable (ΔG° = ΔH° - TΔS°). c) ΔG = Wmax = ΔH - TΔS. Puisque ΔS est négative, à mesure que T augmente, ΔG devient plus positive (plus près de zéro). Par conséquent, Wmax diminuera à mesure que T augmente. 91.

O2 + 2H2O + 4e– → 4OH– (H2 + 2OH– → 2H2O + 2e–) × 2 2H2(g) + O2(g) → 2H2O(l)

ε° = 0,40 V –ε° = 0,83 V ε°pile = 1,23 V = 1,23 J/C

Aux conditions standard, Wmax = ∆Gº pour la production de 2 mol H2O. ΔG° = –nFε°pile = –(4 mol e–)(96485 C/mol e–)(1,23 J/C) = –475,000 J = –475 kJ Pour 1,00 × 103 g H2O, Wmax est : 1,00 × 103 g H2O ×

− 475 kJ 1 mol H 2 O × = –13,200 kJ = Wmax 18,02 g H 2 O 2 mol H 2 O

La quantité de travail ne peut jamais dépasser la variation d’énergie libre. En fait, elle est toujours inférieure à ∆G, parce qu’il se perd toujours de l’énergie dans un processus réel. Cependant, une pile à combustible convertit l’énergie chimique directement en électricité, ce qui est mieux que de mettre les réactifs en contact direct et d’exploiter la chaleur dégagée pour créer de l’électricité. Désavantage principal : les piles à combustible sont très coûteuses à fabriquer. 92.

Le cadmium passe de l’état d’oxydation zéro à +2 dans Cd(OH)2. Étant donné qu’il y a une mole de Cd dans la réaction équilibrée, n = 2 moles d’électrons sont transférées. Aux conditions standard : Wmax = ΔG° = -nFε°, Wmax = -(2 mol e−)(96 485 C/mol e−)(1,10 J/C) = -2,12 × 105 J = -212 kJ

93.

(CO + O2− → CO2 + 2 e−) × 2 O2 + 4 e− → 2 O2− ___________________________ 2 CO + O2 → 2 CO2 ΔG = -nFε, ε =

−ΔG o -(-380 ×103 J) = 0,98 V = (4 mol e- )(96 485 C/mol e- ) nF

© 2007 Les Éditions CEC inc. Toute reproduction interdite

Chapitre 8 Électrochimie

94.

311

C2H5OH(l) + 3O2(g) → 2CO2(g) + 3H2O(l) ; dans C2H5OH, C a l’état d’oxydation –2 et dans CO2, l’état +4. Chaque C perd donc 6e–, ce qui fait 12e– par molécule C2H5OH. Wmax = –1,32 × 106 J = ΔG = –nFε°pile, ε°pile =

ε°pile =

− ΔG nF

-(-1,32 × 106 J) = 1,14 J/C = 1,14 V (12 mole _ )(96 485 C/mol e _ )

95.

L’état d’oxydation de l’or dans Au(CN)2− est +1. Chaque mole d’or produite nécessite 1 mole d’électrons gagnés (+1 → 0). Le seul réactif contenant de l’oxygène est H2O. Chaque mole d’oxygène passe de l’état d’oxydation -2 → 0 quand H2O est converti en O2. Une mole O2 contient 2 moles O, de sorte que 4 moles d’électrons sont perdues quand 1 mole O2 est formée. Afin d’équilibrer les électrons, il faut 4,00 moles d’or pour chaque mole O2 produites ou 0,250 moles O2 pour chaque 1,00 mole d’or formée.

96.

Dans l’électrolyse du chlorure de sodium aqueux, H2O est réduite de préférence à Na+, et Cl− est oxydé de préférence à H2O. La réaction à l’anode est 2 Cl− → Cl2 + 2 e−, et la réaction à la cathode est 2 H2O + 2 e− → H2 + 2 OH−. La réaction globale est : 2 H2O(l) + 2 Cl−(aq) → Cl2(g) + H2(g) + 2 OH−(aq). D’après le rapport molaire 1:1 entre Cl2 et H2 dans la réaction globale équilibrée, si 257 L de Cl2(g) sont produits, alors 257 L de H2(g) seront également produits parce que les moles et les volumes de gaz sont directement proportionnels à T et P constantes. (voir le chapitre 4 de Chimie générale).

97.

mol e− = 50,0 min ×

60 s 2,50 C 1 mol e× × = 7,77 × 10 −2 mol e− min s 96 485 C

mol Ru = 2,618 g Ru ×

1 mol Ru = 2,590 × 10 −2 mol Ru 101,1 g Ru

mol e7,77 ×10-2 mol e= = 3,00; la charge sur les ions ruthénium est 3+. mol Ru 2,590 ×10-2 mol Ru (Ru3+ + 3 e− → Ru) 98.

15 kWh =

15000 J ⋅ h 60 s 60 min × × = 5,4 × 107 J ou 5,4 × 104 kJ s min h

(procédé Hall-Héroult) Pour fondre, 1,0 kg Al nécessite : 1,0 × 103 g Al ×

1 mol Al 10,7 kJ × = 4,0 × 102 kJ 26,98 g mol Al

© 2007 Les Éditions CEC inc. Toute reproduction interdite

Chapitre 8 Électrochimie

312

Il est rentable de recycler Al en faisant fondre le métal parce qu’en théorie, il faut moins de 1 % de l’énergie requise pour produire la même quantité de Al par le procédé Hall-Héroult.

PROBLÈMES DÉFIS 99.

ε° = TΔS/nF - ΔH°/nF ; si on met en graphique ε° en fonction de T, cela devrait donner une droite de pente ∆S°/nF et d’ordonnée à l’origine –∆H/nF. Une demi-pile de référence doit être simple et son potentiel doit varier aussi peu que possible en fonction de la concentration et de la température.

100. a) On peut calculer ∆G° à partir de ∆G° = ∆H° – T∆S°, pour ε° à partir de ∆G° = –nFε°, on peut aussi utiliser l’équation dérivée dans l’exercice précédent (n = 2). o = ε -20

TΔS o - ΔH o (253 K)(263,5 J/K) + 315,9 × 103 J = = 1,98 J/C = 1,98 V (2 mol e _ )(96 485 C/mol e _ ) nF

o b) ε -20 = ε -20 −

RT RT 1 ln Q = 1,98 V – ln + 2 nF nF [H ] [HSO-4 ]2

ε −20 = 1,98 V –

(8,315 J/K⋅ mol)(253 K) 1 ln = 1,98 V + 0,066 V 2 _ _ (2 mol e )(96 485 C/mol e ) (4,5 ) (4,5 )2 = 2,05 V

c) Selon l’exercice 51, ε = 2,12 V à 25 °C. Lorsque la température diminue, ε pile diminue aussi. Comme l’huile du moteur devient plus visqueuse à basse température, cela ajoute à la difficulté de démarrer un moteur par temps froid. La combinaison de ces deux effets explique que les batteries faillissent plus souvent à la tâche par temps froid que par temps chaud. 101.

(Ag+ + e− → Ag) × 2 ε° = 0,80 V -ε° = -(-0,13) Pb → Pb2+ + 2 e− _______________________________________ o = 0,93 V 2 Ag+ + Pb → 2 Ag + Pb2+ ε pile

0, 0591 [Pb 2+ ] 0, 0591 (1,8) log log ε = ε° , 0,83 V = 0,93 V + 2 n [Ag ] [Ag + ]2 n

log

(1,8) 0,10(2) (1,8) = = 3,4, = 103,4, [Ag+] = 0,027 mol/L + 2 [Ag ] 0, 0591 [Ag + ]2 Ag2SO4(s)

Initiale Equilibre



s = solubilité (mol/L)

2 Ag+(aq) + SO42-(aq) Kps = [Ag+]2[SO42-] 0 2s

© 2007 Les Éditions CEC inc. Toute reproduction interdite

0 s

Chapitre 8 Électrochimie

313

D’après les données du problème : 2s = 0,027 mol/L, s = 0,027/2 Kps = (2s)2(s) = (0,027)2(0,027/2) = 9,8 × 10 −6 102. a) Zn(s) + Cu2+(aq) → Zn2+(aq) + Cu(s)

ε°pile = 1,10 V ;

εpile = 1,10 V –

0,0592 [ 2 +] log Zn 2 + 2 [Cu ]

εpile = 1,10 V –

0,0592 0,10 = 1,10 V + 0,041 V = 1,14 V log 2 2,50

b) 10,0 h ×

60 min 60 s 10,0 C 1 mol e _ 1 mol Cu × × × × h min s 96485 C 2 mol e _ = 1,87 mol Cu produit

[Cu2+] a diminué de 1,87 mol/L et [Zn2+] a augmenté de 1,87 mol/L. [Cu2+] = 2,50 – 1,87 = 0,63 mol/L ; [Zn2+] = 0,10 + 1,87 = 1,97 mol/L εpile = 1,10 V –

0,0592 1,97 = 1,10 V – 0,015 V = 1,09 V log 2 0,63

c) 1,87 mol Zn oxydé ×

65,38 g Zn = 122 g Zn ; mol Zn

masse de l’électrode = 200 – 122 = 78 g Zn 1,87 mol Cu formé ×

63,55 g Cu = 119 g Cu ; mol Cu

masse de l’électrode = 200 + 119 = 319 g Cu d) Cette pile pourrait s’arrêter pour l’une ou l’autre de ces raisons :

1. 2. 3.

Tout le zinc a été oxydé. Tout le Cu2+ a été utilisé. L’équilibre est atteint et εpile = 0.

On avait au début 2,50 mol Cu2+ et 200 g Zn ×

1 mol Zn = 3,06 mol Zn. 65,38 g Zn

© 2007 Les Éditions CEC inc. Toute reproduction interdite

Chapitre 8 Électrochimie

314

Cu2+ est donc le réactif limitant. Pour le faire réagir complètement, cela requiert : 2,50 mol Cu2+ ×

2 mol e _ 96 485 C 1s 1h × × × = 13,4 h. 2+ _ mol Cu mol e 10,0 C 3600 s

Pour atteindre l’équilibre : ε = 0 = 1,10 V –

[ 2+] 0,0592 log Zn 2+ 2 [Cu ]

[ Zn 2+] = K = 102(1,10)/0,0592 = 1,45 × 1037 2+ [Cu ] Cette constante est tellement élevée que pratiquement tout le Cu2+ sera épuisé pour atteindre l’équilibre. Donc la pile fonctionnera pendant 13,4 h.

103.

2 H+ + 2 e− → H2 ε° = 0,000 V 2+ − Fe → Fe + 2 e -ε° = -(-0,440V) ___________________________________________________ o ε pile = 0,440 V 2 H+(aq) + Fe(s) → H2(g) + Fe2+(aq)

ε pile = ε

o pile

PH2 ×[Fe 2+ ] 0, 0591 − log Q, où n = 2 et Q = [H + ]2 n

Pour déterminer Ka de l’acide faible, on utilise d’abord les données électrochimiques pour déterminer la concentration de H+ dans la demi-pile contenant l’acide faible. 0,333 V = 0,440 V −

0,0591 1,00 (1,00 ×10-3 ) log 2 [H + ]2

0,107(2) 1,00 ×10-3 1,00 ×10-3 = log , = 103,621 = 4,18 × 103, 0,0591 [H + ]2 [H + ]2 [H+] = 4,89 × 10 −4 mol/L On trouve ensuite la valeur de Ka de l’acide faible HA à l’aide du tableau de réaction normal pour un problème d’acide faible. HA Initiale Équilibre Ka =



1,00 mol/L 1,00 - x

H+ +

A−

~0 x

0 x

Ka =

[H + ][A - ] [HA]

(4,89 × 10−4 ) 2 x2 où x = [H+] = 4,89 × 10 −4 mol/L, Ka = = 2,39 × 10 −7 −4 1, 00 − 4,89 × 10 1, 00 − x

© 2007 Les Éditions CEC inc. Toute reproduction interdite

Chapitre 8 Électrochimie

315

104. a)

b) Au3+(aq) + 3Fe2+(aq) → 3Fe3+(aq) + Au(s) ε°pile = ε°pile = –

ε°pile = 1,50 – 0,77 = 0,73 V

0,0592 [Fe3+ ]3 0,0592 log Q = 0,73 V – log n 3 [Au3+] [Fe2+ ]3

Puisque [Fe3+] = [Fe2+] = 1,0 mol/L, 0,31 V = 0,73 V –

1 0,0592 log [Au3+ ] 3

3(−0, 42) 1 , log [Au3+] = –21,28, [Au3+] = 10–21,28 = 5,2 × 10–22 mol/L = − log 3+ 0, 0592 [Au ] Au3+ + 4 Cl– ⇌ AuCl4–; comme [Au3+] est très petit, on peut supposer que [AuCl4–] ≈ [Au3+]o ≈ 1,0 mol/L K=

[AuCl−4 ] 3+

_ 4

[Au ][Cl ]

=

1, 0 (5, 2 × 10 )(0,10) −22

4

= 1,9 × 1025 ; supposition valide.

105. a) D’après le tableau 8.1 : 2 H2O + 2 e- → H2 + 2 OH- ε° = -0,83 V o ε pile = ε Ho O − ε Zro = -0,83 V + 2,36 V = 1,53 V 2

Oui, la réduction de H2O en H2 par Zr est spontanée dans les conditions standard parce que ε°pile > 0; b)

(2 H2O + 2 e− → H2 + 2 OH−) × 2 Zr + 4 OH− → ZrO2·H2O + H2O + 4 e− ____________________________________ 3 H2O(l) + Zr(s) → 2 H2(g) + ZrO2·H2O(s)

c) ΔG° = -nFε° = -(4 mol e−)(96 485 C/mol e−)(1,53 J/C) = -5,90 × 105 J = -590 kJ

© 2007 Les Éditions CEC inc. Toute reproduction interdite

Chapitre 8 Électrochimie

316

ε = ε° -

ε° =

0, 0591 log Q; à l’équilibre, ε = 0 et Q = K. n

0, 0591 4(1,53) log K, log K = = 104; K ≈ 10104 0, 0591 n

d) 1,00 × 103 kg Zr ×

2 mol H 2 1000 g 1 mol Zr = 2,19 × 104 mol H2 × × kg 91,22 g Zr mol Zr

2,19 × 104 mol H2 ×

V=

nRT = P

2,016 H 2 = 4,42 × 104 g H2 mol H 2

2,19 ×104 mol ×

0,08206 L ⋅ atm × 1273 K mol ⋅ K = 2,3 × 106 L H2 1,0 atm

e) Probablement oui; en évacuant H2, moins de radioactivité dans l’ensemble a été libérée que si H2 avait explosé à l’intérieur du réacteur (comme il est arrivé à Tchernobyl). Ni l’une, ni l’autre des possibilités n’est réjouissante, mais évacuer l’hydrogène radioactif est la possibilité la moins désagréable. 106. a)

(Ag+ + e− → Ag) × 2 ε° = 0,80 V Cu → Cu2+ + 2 e−ε° = −0,34 V ________________________________________________ o 2 Ag+(aq) + Cu(s) → 2 Ag(s) + Cu2+(aq) ε pile = 0,46 V o ε pile = ε pile −

0, 0591 [Cu 2+ ] log Q où n = 2 et Q = [Ag + ]2 n

Pour calculer εpile, il faut utiliser les données de Kps pour déterminer [Ag+]. AgCl(s) Initiale Équilibre



s = solubilité (mol/L)

Ag+(aq) + Cl−(aq) 0 s

Kps = 1,6 × 10 −10

0 s

Kps = 1,6 × 10 −10 = [Ag+][Cl−] = s2, s = [Ag+] = 1,3 × 10 −5 mol/L εpile = 0,46 V -

0, 0591 2, 0 log = 0,46 V - 0,30 = 0,16 V 2 (1,3 × 10−5 ) 2

b) Cu2+(aq) + 4 NH3(aq)



Cu(NH4)42+(aq)

K = 1,0 × 1013 =

© 2007 Les Éditions CEC inc. Toute reproduction interdite

[Cu ( NH 3 ) 24+ ] [Cu 2+ ][ NH 3 ]4

Chapitre 8 Électrochimie

317

Étant donné que K pour la formation de Cu(NH3)42+ est très grande, la réaction directe est dominante. À l’équilibre, presque tout le Cu2+ 2,0 mol/L réagit pour former Cu(NH3)42+ 2,0 mol/L. Cette réaction nécessite NH3 8,0 mol/L pour réagir avec tout le Cu2+ de l’équation équilibrée. Par conséquent, le nombre de mol NH3 ajouté à 1,0 L de solution sera supérieur à 8,0 mol étant donné qu’une certaine quantité de NH3 doit être présente à l’équilibre. Afin de calculer combien de NH3 est présent à l’équilibre, il faut utiliser les données électrochimiques pour déterminer la concentration de Cu2. o ε pile = ε pile −

log

0, 0591 0, 0591 [Cu 2+ ] log Q , 0,52 V = 0,46 V log n 2 (1,3 ×10-5 ) 2

[Cu 2+ ] -0,06(2) [Cu 2+ ] = = -2,03, = 10-2,03 = 9,3 ×10-3 -5 2 -5 2 (1,3 × 10 ) 0,0591 (1,3 × 10 )

[Cu2+] = 1,6 × 10 −12 = 2 × 10 −12 mol/L Note : l’hypothèse selon laquelle le Cu2+ 2,0 mol/L réagit presque complètement est excellente car il ne reste que 2 × 10 −12 mol/L de Cu2+ après cette réaction. On peut maintenant trouver [NH3] à l’équilibre. K = 1,0 × 1013 =

[Cu(NH 3 ) 42+ ] (2, 0) = , [NH3] = 0,6 mol/L 2+ 4 (2 × 10−12 )[NH 3 ]4 [Cu ][NH 3 ]

Étant donné qu’il y a 1,0 L de solution, il reste 0,6 mol NH3 à l’équilibre. Le nombre de mol total de NH3 ajouté est 0,6 mol plus les 8,0 mol NH3 nécessaires pour former Cu(NH3)42+ 2,9 mol/L. Par conséquent, 8,0 + 0,6 = 8,6 mol NH3 ont été ajoutées. 107. On peut additionner ensemble les potentiels de réduction standard seulement lorsque les électrons dans la demi-réaction de réduction s’annulent exactement avec ceux de la demiréaction d’oxydation. On résout ce problème en appliquant l’équation ΔGo = -nFε° aux demi-réactions. ΔG of pour e− = 0.

M3+ + 3 e− → M

ΔGo = -nFε° = -3(96 485)(-0,10) = 2,9 × 104 J

Puisque M et e− ont ΔGfo = 0, alors 2,9 × 104 J = - ΔGf,o M3+ , ΔGf,o M3+ = -2,9 × 104 J M2+ + 2 e− → M

ΔGo = -nFε° = -2(96 485)(-0,50) = 9,6 × 104 J

9,6 × 104 J = - ΔGf,o M2+ , ΔGf,o M2+ = -9,6 × 104 J M3+ + e− → M2+ ε° =

ΔGo = -9,6 × 104 J – (-2,9 × 104 J) = -6,7 × 104 J

−ΔG o −(−6, 7 × 104 ) = = 0,69 V pour M3+ + e− → M2+ (1)(96 485) nF

© 2007 Les Éditions CEC inc. Toute reproduction interdite

Chapitre 8 Électrochimie

318 108. 2 Ag+ + Ni → Ni2+ + Ag; la pile est « à plat » à l’équilibre. o εpile = 0,80 V + 0,23 V = 1,03 V

0, 0591 log K; on trouve : K = 7,18 × 1034 2

0 = 1,03 V -

K est très grande. Laissons la réaction directe se produire complètement.

2Ag+ + Ni → Ni2+ + 2 Ag Avant Après

1,0 mol/L 0 mol/L

K = [Ni2+]/[Ag+]2 = 7,18 × 1034

1,0 mol/L 1,5 mol/L

Laissons maintenant la réaction retourner à l’équilibre. 2Ag+ + Ni Initiale Changement Équilibre 2x

0 +2x

K = 7,18 × 1034 =





Ni2+ + Ag 1,5 mol/L -x 1,5 - x

1,5 − x 1,5 ≈ ; 2 (2 x) (2 x) 2

On trouve : x = 2,3 × 10 −18 mol/L. Approximation valide. [Ag+] = 2x = 4,6 × 10 −18 mol/L; [Ni2+] = 1,5 – 2,3 × 10 −18 mol/L = 1,5 mol/L 109. a) Ag2CrO4(s) + 2 e− → 2 Ag(s) + CrO42−(aq)

Hg2Cl2 + 2 e− → 2 Hg + 2 Cl−

ε° = 0,446 V εECS = 0,242 V

ECS = électrode au calomel saturée ECS est la demi-réaction d’oxydation avec εpile = 0,446 - 0,242 = 0,204 V. ΔG = -nFεpile = -2(96 485)(0,204)J = -3,94 × 104 J = -39,4 kJ b) Dans ECS, on suppose que toutes les concentrations sont constantes. Par conséquent, seulement CrO42− apparaît dans l’expression de Q et il apparaît au numérateur puisque CrO42− est produit dans la demi-réaction de réduction. Pour calculer εpile à des concentrations non standard de CrO42−, on utilise l’équation suivante. o εpile = εpile -

0, 0591 0,0591 log [CrO42−] = 0,204 V log [CrO42-] 2 2

© 2007 Les Éditions CEC inc. Toute reproduction interdite

Chapitre 8 Électrochimie

c) εpile = 0,204 -

319

0, 0591 log (1,00 × 10 −5 ) = 0,204 V - (-0,148 V) = 0,352 V 2

d) 0,504 V = 0,204 V - (0,0591/2) log [CrO42−]

log [CrO42−] = -10,152, [CrO42−] = 10−10,152 = 7,05 × 10 −11 mol/L

ε o = 0,446 V - ε o = -0,80 V

e) Ag2CrO4 + 2 e− → 2 Ag + CrO42−

(Ag → Ag+ + e−) × 2 ________________________________________________________________ o εpile = -0,35 V K = Kps = ? Ag2CrO4(s) → 2 Ag+(aq) + CrO42−(aq) o εpile =

0, 0591 (−0,35)(2) log Kps, log Kps = = -11,84, Kps = 10 −11.84 = 1,4 × 10 −12 0,0591 n

110. a) ε° = 0 (pile de concentration); ε = 0 -

⎛1,0 × 10−4 ⎞ 0,0591 log ⎜ ⎟ , ε = 0,12 V 2 ⎝ 1,00 ⎠

b) Cu2+ + 4 NH3 ⇌ Cu(NH3)42+ Kglobale = K1·K2·K3·K4 = 1,0 × 1013

Puisque K >>1, la réaction est complète et on résout le problème d’équilibre. Cu2+

+

4 NH3

Avant 1,0 × 10 −4 mol/L Après 0



2,0 mol/L 2,0

Cu(NH3)42+

K = 1,0 × 1013

0 1,0 × 10 −4

Maintenant, on laisse la réaction atteindre l’équilibre. Cu2+ Initiale Équilibre

0 +x

K = 1,0 × 1013 =

+ 4 NH3 2,0 mol/L 2,0 + 4x



Cu(NH3)42+ 1,0 × 10 −4 mol/L 1,0 × 10 −4 - x

(1, 0 × 10−4 − x) 1, 0 × 10−4 ≈ = 1,0 × 1013, 4 4 x(2, 0 + 4 x) x(2, 0)

x = [Cu2+] = 6,3 × 10 −19 mol/L

Approximation valide.

À cette concentration de Cu2+, le potentiel de pile est : ε=0-

⎛ 6,3 × 10−19 ⎞ 0,0591 log ⎜ ⎟ , ε = 0,54 V 2 ⎝ 1,00 ⎠

© 2007 Les Éditions CEC inc. Toute reproduction interdite

Chapitre 8 Électrochimie

320 3 × (e− + 2 H+ + NO3− → NO2 + H2O)

111. a)

2 H2O + NO → NO3− + 4 H+ + 3 e− 2 H+(aq) + 2 NO3−(aq) + NO(g) → 3 NO2(g) + H2O(l)

ε o = 0,775 V - ε o = −0,957 V o εpile = −0,182 V K = ?

3(−0,182) nε o log K = = = −9,239, K = 10−9,239 = 5,77 × 10 −10 0, 0591 0, 0591 b) Soit C = concentration de HNO3 = [H+] = [NO3−]

5,77 × 10

−10

=

P 3NO2 PNO×[H+ ]2[NO3- ]2

=

P 3NO2 PNO × C 4

Si 0,20 mol % NO2 et Ptot = 1,00 atm :

PNO2 =

0,20 mol NO 2 × 1,00 atm = 2,0 × 10 −3 atm; PNO = 1,00 - 0,0020 = 1,00 atm 100 mol total

5,77 × 10

−10

(2, 0 × 10−3 )3 , C = 1,9 mol/L HNO3 = (1, 00) C 4

PROBLÈMES D’INTÉGRATION 112. a) (In+ + e− → In) × 2 ε° = -0,126 V In+ → In3+ + 2 e− -ε° = 0,444 V ________________________________________ o 3 In+ → In3+ + 2 In εpile = 0,318

log K =

nε o 2(0,318) = =10,761, K = 1010,761 = 5,77 × 1010 0, 0591 0, 0591

b) ΔGo = -nFε° = -(2 mol e−)(96 485 C/mol e−) (0,318 J/C) = -6,14 × 105 J = -61,4 kJ o ΔGréaction = -61,4 kJ = [2(0) + 1(-97,9 kJ] – 3 ΔGf,o In + , ΔGf,o In + = -12,2 kJ/mol

o 113. εpile = 0,400 V – 0,240 V = 0,160 V; ε = ε° -

0,180 = 0,160 -

0, 0591 log Q n

0, 0591 0,120 log (9,32 × 10 −3 ), 0,020 = , n=6 n n

© 2007 Les Éditions CEC inc. Toute reproduction interdite

Chapitre 8 Électrochimie

321

Six moles d’électrons sont transférées dans la réaction globale équilibrée. Il faut maintenant trouver comment on peut avoir 6 mol e− dans l’équation globale équilibrée. Les deux possibilités consistent à avoir des ion de charges 1+ et 6+ ou 2+ et 3+; seulement ces deux combinaisons donnent 6 quand des multiples communs sont déterminés en additionnant la demi-réaction de réduction à la demi-réaction d’oxydation. Puisque N forme des ions de charge 2+, M doit former des ions de charges 3+. On peut alors déterminer la réaction globale de la pile. (M3+ + 3 e− → M) × 2 ε° = 0,400 V (N → N2+ + 2 e−) × 3 -ε° = -0,240 V _____________________________________________ o εpile = 0,160 V 2 M3+ + 3 N → 3 N2+ + 2 M Q = 9,32 × 10 −3 =

[N 2+ ]3o (0,10)3 = , [M3+] = 0,33 mol/L 3+ 2 3+ 2 [M ] [M ]o

Wmax = ΔG = -nFε = -6(96 485)(0,180) = -1,04 × 105 J = -104 kJ

La quantité maximale de travail que peut produire cette pile est 104 kJ. 114. Dans le nitrate de chrome(III) [Cr(NO3)3], le chrome est dans l’état d’oxydation +3.

1 mol Cr 3mol e- 96 485C 1,15 g Cr × = 6,40 × 103 C de charge × × 52,00 g mol Cr mol e Pour la pile à Os, 6,40 × 103 C de charge a traversé la pile. 3,15 g Os ×

1 mol Os 1 mol e= 0,0166 mol Os; 6,40 × 103 C × = 0,0663 mol e− 190,2 g 96 485 C

mol e0, 0663 = = 3,99 ≈ 4 mol Os 0, 0166 Ce sel est composé d’ions Os4+ et NO3−. Le composé est Os(NO3)4, le nitrate d’osmium(IV). Pour la troisième pile, on identifie X en déterminant sa masse molaire. Deux moles d’électrons sont transférées quand X2+ est réduit en X. masse molaire =

2,11 g X = 63,6 g/mol 1 mol e1 mol X 3 6,40 ×10 C × × 96 485 C 2 mol e-

C’est le cuivre, Cu. La configuration électronique est : [Ar]4s13d10.

© 2007 Les Éditions CEC inc. Toute reproduction interdite

Chapitre 8 Électrochimie

322 PROBLÈMES DE SYNTHÈSE 115. a)

Cu2+ + 2 e− → Cu ε° = 0,34 V − ε° = 1,20 V V → V2+ + 2 e− _____________________________________________________ o Cu2+(aq) + V(s) → Cu(s) + V2+(aq) εpile = 1,54 V o εpile = εpile

0, 0591 [V2+ ] [V2+ ] log Q où n = 2 et Q = = n [Cu2+ ] 1,00 mol/L

Pour déterminer εpile, il faut connaître [V2+] initiale, qui peut être déterminée à partir des données du point stoechiométrique. Au point stoechiométrique, mol H2EDTA2− ajouté = mol V2+ présent initialement. mol V2+ présent initialement = 0,5000 L ×

0,0800 mol H 2 EDTA 21 mol V 2+ × L mol H 2 EDTA 2= 0,0400 mol V2+

[V2+]o =

0,0400 mol V 2+ = 0,0400 mol/L 1,00 L

εpile = 1,54 V -

0, 0591 0, 0400 = 1,54 V - (-0,0413 V) = 1,58 V log 2 1, 00

b) On utilise les données électrochimiques pour trouver [V2+] à l’équilibre. o εpile = εpile -

0,0591 [V 2+ ] 0,0591 [V 2+ ] log , 1,98 V = 1,54 V log n [Cu 2+ ] 2 1,00 mol/L

[V2+] = 10− (0,44)(2) / 0,0591 = 1,3 × 10 −15 mol/L H2EDTA2-(aq) + V2+(aq) ⇌ VEDTA2−(aq) + 2 H+(aq)

K=

[ VEDTA 2 − ][H + ]2 [H 2 EDTA 2 − ][V 2 + ]

Dans cette réaction de titrage, des nombres égaux de moles de V2+ et de H2EDTA2− réagissent au point stoechiométrique. Par conséquent, des nombres égaux de moles des deux réactifs doivent être présents à l’équilibre, de sorte que [H2EDTA2−] = [V2+] = 1,3 × 10 −15 mol/L. De plus, puisque [V2+] à l’équilibre est très petite comparée à la concentration initiale de 0,0400 mol/L, la réaction est presque complète. Le nombre de moles de VEDTA2− produites est égal au nombre de moles de V2+ qui ont réagi (= 0,0400 mol). À l’équilibre, [VEDTA2−] = 0,0400 mol/(1,00 L + 0,5000 L) = 0,0267

© 2007 Les Éditions CEC inc. Toute reproduction interdite

Chapitre 8 Électrochimie

323

mol/L. Enfin, étant donné qu’il y a une solution tampon, on peut supposer que le pH ne change pas de sorte que [H+] = 10−10,00 = 1,0 × 10 −10 mol/L. On calcule K pour la réaction : −10 2

K=

(0, 0267)(1, 0 × 10 ) [ VEDTA 2 − ][H + ]2 = = 1,6 × 108 −15 −15 2− 2+ [H 2 EDTA ][V ] (1,3 × 10 )(1,3 × 10 )

c) Au point de demi-neutralisation, 250,0 mL de H2EDTA2− ont été ajoutés à 1,00 L de V2+ 0,0400 mol/L. Exactement la moitié de 0,0400 mol de V2+ présent initialement a été convertie en VEDTA2−. Par conséquent, il reste 0,0200 mol de V2+ dans 1,00 + 0,2500 = 1,25 L de solution.

0, 0591 [V 2+ ] 0, 0591 (0, 0200 /1, 25) log εpile = 1,54 V = 1,54 log 2+ [Cu ] n 2 1, 00 εpile = 1,54 - (-0,0531) = 1,59 V 116. On commence par attribuer un potentiel de réduction de 0,00 V. Par exemple, supposons que :

B2+ + 2 e− → B

ε° = 0,00 V

D’après les données, lorsque B/B2+ et E/E2+ sont ensemble dans une pile, ε° = 0,81 V. E2+ + 2 e− → E doit avoir un potentiel de -0,81 V ou 0,81 V étant donné que E peut participer soit à la demi-réaction de réduction, soit à le demi-réaction d’oxydation. On choisit arbitrairement a un potentiel de -0,81 V pour E. En fixant le potentiel de réduction à -0,81 pour E et 0,00 pour B, on obtient le tableau des potentiels suivant. B2+ + 2 e− E2+ + 2 e− D2+ + 2 e− C2+ + 2 e− A2+ + 2 e−

→B →E →D →C →A

0,00 V -0,81 V 0,19 V -0,94 V -0,53 V

Du plus grand au plus petit : 0,19 V D2+ + 2 e− → D 2+ − B +2e →B 0,00 V -0,53 V A2+ + 2 e− → A -0,81 V E2+ + 2 e− → E 2+ − C +2e →C -0,94 V A2+ + 2 e− → A est au milieu. On fixe sa valeur à 0,00 V. On obtient : D2+ + 2 e− → D 0,72 V 0,53 V B2+ + 2 e-− → B A2+ + 2 e-− → A 0,00 V © 2007 Les Éditions CEC inc. Toute reproduction interdite

Chapitre 8 Électrochimie

324 E2+ + 2 e- → E C2+ + 2 e− → C

-0,28 V -0,41 V

Évidemment, étant donné qu’on aurait pu supposer que le potentiel de réduction de E soit 0,81 V au lieu de -0,81 V, on peut également obtenir : C2+ + 2 e− → C E2+ + 2 e− → E A2+ + 2 e-− → A B2+ + 2 e− → B D2+ + 2 e− → D

0,41 V 0,28 V 0,00 V -0,53 V -0,72 V

Une façon de déterminer quelle table est correcte consiste à ajouter le métal C à une solution contenant D2+ et le métal D contenant C2+. Si D précipite, le premier tableau est correct. Si C précipite, le deuxième tableau est correct.

© 2007 Les Éditions CEC inc. Toute reproduction interdite

LISTE DES TABLEAUX DU MANUEL

Règles relatives à la solubilité des sels dans l’eau Règles d’attribution des degrés d’oxydation Différents types de solutions Masse molaire, masse équivalente et relation entre la concentration molaire volumique et la normalité de plusieurs acides et bases Paramètres pour divers genres de solutés et de solvants Résumé des comportements des différents types de solutions Constantes ébullioscopiques molales (kéb) et constantes cryoscopiques molales (kcong) pour plusieurs solvant Valeurs théorique et expérimentale du facteur de van’t Hoff pour différentes solutions contenant 0,05 mol d’électrolyte par kg de solvant Types de colloïdes Variation de la concentration des réactifs et des produits en fonction du temps pour la réaction 2NO2(g) → 2NO(g) + O2(g) (à 300 °C) Variation de la vitesse moyenne (en mol/L · s) de la réaction de décomposition du dioxyde d’azote en fonction du temps Variation de la concentration de N2O5 en fonction du temps pour la réaction 2N2O5 (solution) → 4 NO2 (solution) + O2(g) (à 45 °C) Vitesses initiales obtenues à l’aide de trois expériences pour la réaction NH4+ (aq) + NO2-(aq) → N2(g) + 2 H2O(l) Étude de la réaction BrO3- (aq) + 5Br- (aq) + 6H+ (aq) → 3Br2(l) + 3H2O(l) Résultats de quatre expériences Résumé de la cinétique des réactions de type A → produits, d’ordre 0, 1 ou 2 par rapport à [A] Exemples d’étapes élémentaires Résultats de trois expériences relatives à la réaction N2(g) + 3H2(g) ⇌ 2NH3(g) Variation en fonction de la température et de la pression totale du pourcentage massique de NH3 à l’équilibre dans un mélange de N2, de H2 et de NH3 Variation de la valeur de K en fonction de la température pour la réaction de synthèse de l’ammoniac Déplacements de la position d’équilibre pour la réaction 58kJ + N2O4(g) ⇌ 2NO2(g) Différentes façons de décrire la force d’un acide Valeurs de Ka pour quelques monoacides courants

18 30 53

55 60 71 73

81 83

97

98

102

104

105

116 118 152

170 176 176 191 192

Valeurs de Kb pour quelques bases faibles courantes Constantes de dissociation successives de plusieurs polyacides courants Évaluation qualitative du pH de solutions de sels dont le cation et l’anion ont des propriétés acides ou basiques Propriétés acide-base de différents types de sels Force de la liaison et acidité des halogénures Constantes d’acidité de plusieurs oxacides Électronégativité de X et constantes d’acidité de quelques oxacides Trois théories relatives aux acides et aux bases Variations du rapport [CH3CO2-]/[CH3CO2H] quand on ajoute 0,01 mol de H- à 1,0 L de chacune des deux solutions Résumé des résultats du titrage de 100,0 mL d’une solution de NH3 0,050 mol/L par une solution de HCl 0,10 mol/L Valeurs choisies du pH près du point d’équivalence du titrage de 100,0 mL d’une solution de HCl 0,10 mol/L par une solution de NaOH 0,10 mol/L Valeurs de Kps à 25 °C pour des solides ioniques courants Solubilités calculées du CuS, du Ag2S et du Bi2S3, à 25 °C Micro-états correspondant à un arrangement (état particulier) Probabilités que toutes les molécules s’amassent dans le ballon de gauche en fonction du nombre total de molécules Influence de ΔSsyst et de ΔSext sur le signe de ΔSuniv Valeurs de ΔSuniv et de ΔG° pour le processus H2O(s) → H2O(l) à - 10, 0 °C et 10 °C Différentes combinaisons possibles de ΔH et de ΔS pour un processus donné et leur influence sur la spontanéité du processus en fonction de la température Relations qualitatives entre la variation d’énergie libre standard et la constante d’équilibre pour une réaction donnée Potentiels standard d’électrode (de réduction)à 25 °C (298 K) pour de nombreuses demi-réactions courantes Quelques ions dont on peut déterminer la concentration à l’aide d’électrodes ioniques spécifiques Variation du prix de l’aluminium depuis plus d’un siècle

211 215

223 224 225 226 226 227

258

274

280 282 286 317

317 323 324

325

339 360

371 386

© 2007 Les Éditions CEC inc. Toute reproduction interdite

LA CHIMIE SUR INTERNET Voici quelques sites susceptibles de vous intéresser sur Internet. Cette liste ne constitue qu’un point de départ pour vous et vos élèves. N’hésitez pas à nous faire part de vos découvertes.



Site du cégep de Drummondville http://www.cdrummond.qc.ca/



Site du cégep de Saint-Laurent http://www.cegep-st-laurent.qc.ca/



Nist WebBook http://webbook.nist.gov/chemistry/



The Chemplace http://www.chemplace.com/



Web Elements http://www.webelements.com/

© 2007 Les Éditions CEC inc. Toute reproduction interdite

8101, boul. Métropolitain Est, Anjou (Québec) Canada H1J 1J9 Téléphone : 514-351-6010 • Télécopieur : 514- 351-3534